Sei sulla pagina 1di 170

Somatórios e Produtórios – Teoria e exercícios

Por Ricardo L. Bertolucci Filho.

Sumário:
Capítulo 0 – Noções gerais e revisão:
I – Visão geral;
II – Indexadores (Limites Inferiores e Superiores);
III – O Princípio da Indução Finita (P.I.F);
IV – Exercícios gerais.

Capítulo 1 – Somatórios:
I – Introdução e notação Sigma (Σ);
II – Definição intuitiva de somatório;
III – Definição formal de somatório;
IV – Propriedades;
V – Frações Parciais e a Soma Telescópica;
VI – Introdução às Séries e o Limite de uma soma;
VII – Exercícios resolvidos;
VIII – Exercícios propostos.

Capítulo 2 – Produtórios:
I – Introdução e notação Pi (∏);
II – Definição intuitiva de produtório;
III – Definição formal de produtório;
IV – Propriedades;
V – Logaritmos (revisão e propriedades) e o Produto Telescópico;
VI – Exercícios resolvidos;
VII – Exercícios propostos.

Capítulo 3 – Exercícios de aprofundamento, gabarito e resoluções.

* Para uma melhor compreensão, os exercícios serão divididos em três categorias


principais e uma adicional, da seguinte maneira:
⊕ − Fácil;
⊕⊕ − Intermediário;
⊕⊕⊕ − Difícil;
† − Desafio em nível olímpico ou pré-universitário.
Capítulo 0 – Noções gerais e revisão

I. Visão geral
Os conceitos de Somatório e Produtório, como veremos a seguir, são relativamente
modernos, algo que fora intensivamente difundido com o decorrer dos avanços da
construção do formalismo e rigor matemático, como nos adventos do Cálculo
Diferencial e Integral, no aprimoramento da Álgebra, Análise, Combinátoria, etc.
Porém, a ideia por trás destes conceitos é algo intrínseco à matemática pois, antes
mesmo de nos aprofundarmos nesta ciência, somos introduzidos a conceitos
“primitivos” como o conceito de soma, subtração, multiplicação e divisão.
Ao nos iniciarmos na Matemática, somos guiados por noções básicas de abstração
numérica, como por exemplo:
5 1
1 + 1 = 2, 2 − 1 = 1, 3.3 = 9 e = .
10 2
Tais noções são o “coração” dos conceitos de Somatório e Produtório que serão, mais
adiante, apresentados.
Tomemos o seguinte problema:
“1. Ao entrar na universidade, Silas terá cinco disciplinas na parte da manhã, duas na
parte da tarde e uma na parte da noite.
Quantas serão as disciplinas de Silas?”
Este problema explora noções triviais da adição numérica, uma vez que a totalidade de
disciplinas do curso de Silas é tal que:
Sendo 𝑛 o número total de disciplinas de Silas, temos que:
𝑛 = 5 + 2 + 1 = 8 disciplinas.
Agora, vejamos o seguinte problema:
“2. Morgana, para comparecer à festa de sua formatura, dispõe de quatro pares de
sapatos, dois vestidos, três colares e dois brincos.
De quantas maneiras possíveis Morgana poderá se vestir para ir à festa?”
Este problema explora a noção de contagem, da Combinátoria, uma vez que a
quantidade de maneiras com as quais Morgana poderá se vestir é tal que:
Sendo 𝑛 o número total de maneiras possíveis para Morgana se vestir, temos que:
𝑛 = 4.2.3.2 = 48 maneiras possíveis.
Dito isto, a familiarização com os futuros conceitos de Somatório e Produtório virá de
maneira direta e rápida.

II. Indexadores (Limites Inferiores e Superiores)


A noção de indexador é, também, algo de fácil compreensão, pois nada mais são do
que a quantidade inicial com a qual estaremos trabalhando (limite inferior) e a
quantidade final com a qual também estaremos trabalhando (limite superior).
As definições Limite Inferior e Limite Superior também são intuitivas, pois o corpo de
nossa operação (seja esta soma ou multiplicação) é composto de duas quantidades:
1 – A quantidade inicial a ser trabalhada;
2 – A quantidade final a ser trabalhada.
Por exemplo, seja o problema:
“1. Qual é a soma dos seis primeiros números naturais?”
O problema nos pede para encontrar a soma dos seis primeiros números naturais, o
qual, traduzido para a linguagem matemática, se torna:
1 + 2 + 3 + 4 + 5 + 6 = S, S = 21.
Neste problema, temos que nosso limite inferior é o número 1, pois é o primeiro
número natural a ser somado, e o número 6 é nosso limite superior, pois é o último
número natural a ser somado.
“2. Encontre o produto dos 5 primeiros números ímpares menores do que 17 e maiores
do que 3.”
O problema nos pede o produto dos cinco primeiros ímpares menores que 17 e
maiores que 3, ou seja:
Seja P o produto pedido, P = 5.7.9.11.13.15, pois 5 > 3 e 15 < 17.
Neste caso, 5 é o nosso limite inferior, pois é o primeiro número ímpar maior do que 3
e menor do que 17 e 15 é o nosso limite superior, pois é o último ímpar que é maior do
que 3 e menor do que 17.

III. O Princípio da Indução Finita (P.I.F)


O denominado Princípio da Indução Finita é um método de demonstração matemática
de extrema importância, pois dispensa esforço e algebrismo desnecessários, além de
propiciar uma elegante demonstração matemática.
O conceito de demonstração matemática é, também, algo intrínseco a esta, pois, como
o próprio nome sugere, é a ideia de provar a validade de teoremas, lemas, fórmulas,
etc.
Alguns dos métodos mais conhecidos da prova matemática são:
Prova por indução (a qual veremos a seguir);
Prova por redução ao absurdo;
Prova direta, e etc.
O Princípio da Indução Finita nos possibilita, de maneira rápida e concisa, a resolução
de problemas como:
1
1. Prove que 1² + 2² + ⋯ + 𝑛2 = 𝑛(𝑛 + 1)(2𝑛 + 1).
6
𝑛
𝑛 1
2. Prove que (1 + 2 + 3 + ⋯ + 𝑛) = ( 𝑛(𝑛 + 1)) .
2
3. Sejam 𝑎 e 𝑏 números reais e 𝑛 um natural qualquer, prove que:
𝑎𝑛 − 𝑏 𝑛 = (𝑎 − 𝑏)(𝑎𝑛−1 + 𝑎𝑛−2 𝑏 + ⋯ + 𝑏 𝑛−2 𝑎 + 𝑏 𝑛−1 ).
4. Prove que, para todo 𝑛 natural, vale que 𝑥 𝑛 − 1 = (𝑥 − 1)(𝑥 𝑛−1 + 𝑥 𝑛−2 + ⋯ + 1).
Em geral, numa situação que nos pede algo como:
Prove que 𝑓(𝑎) = 𝑓(𝑏), a ferramenta a ser utilizada é o P.I.F.
Seja 𝑓(𝑥) um lema qualquer a ser provado, o Princípio da Indução Finita (P.I.F) é
composto de três etapas:
1. (Verificação) – Verificar a validade de 𝒇(𝒙) para 𝒙 = 𝟏, ou seja, calcular 𝒇(𝟏);
2. (Hipótese de indução) – Supor a validade de 𝒇(𝒙) para um número 𝒌 qualquer,
ou seja, supomos que para este número k qualquer, o lema é válido.
3. (Demonstração ou conclusão) – Constatar que, se para o número 𝒌 o lema é
válido, então este também é válido para o número 𝒌 + 𝟏 e, consequentemente, é
válida para qualquer outro número.

Façamos alguns exemplos:


1
1. Prove que 1² + 2² + ⋯ + 𝑛2 = 𝑛(𝑛 + 1)(2𝑛 + 1).
6
Passo 1 – Verificar a validade para 𝒏 = 𝟏:
1 6
1² = (1 + 1)(2.1 + 1) = = 1, ✓.
6 6
Passo 2 – Supor a validade para um número natural 𝒌 qualquer:
1
1² + 2² + ⋯ + 𝑘 2 = 𝑘(𝑘 + 1)(2𝑘 + 1), ✓.
6
Passo 3 – Provar que para o natural 𝒌 + 𝟏, a relação também é válida:
1
1² + 2² + ⋯ + 𝑘 2 + (𝑘 + 1)2 = (𝑘 + 1)(𝑘 + 1 + 1)(2(𝑘 + 1) + 1);
6
1
1² + 2² + ⋯ + 𝑘 2 + (𝑘 + 1) = (𝑘 + 1)(𝑘 + 2)(2𝑘 + 2 + 1);
2
6
2 2 1
1² + 2² + ⋯ + 𝑘 + (𝑘 + 1) = (𝑘 + 1)(𝑘 + 2)(2𝑘 + 3);
6
1
1² + 2² + ⋯ + 𝑘 2 + (𝑘 + 1)2 = (𝑘 + 1)(2𝑘 2 + 3𝑘 + 4𝑘 + 6);
6
1
𝟏² + 𝟐² + ⋯ + 𝒌 + (𝑘 + 1) = (𝑘 + 1)(2𝑘 2 + 7𝑘 + 6);
𝟐 2
6
Temos, no primeiro membro (lado esquerdo) da igualdade, a soma 𝟏² + 𝟐² + ⋯ + 𝒌𝟐 ,
𝟏
a qual é igual a 𝒌(𝒌 + 𝟏)(𝟐𝒌 + 𝟏), assim:
𝟔
1
1² + 2² + ⋯ + 𝑘 + (𝑘 + 1)2 = (𝑘 + 1)(2𝑘 2 + 7𝑘 + 6);
2
6
1 1
𝑘(𝑘 + 1)(2𝑘 + 1) + (𝑘 + 1)2 = (𝑘 + 1)(2𝑘 2 + 7𝑘 + 6);
6 6
𝑘(𝑘+1)(2𝑘+1) 1
+ (𝑘 + 1)2 = (𝑘 + 1)(2𝑘 2 + 7𝑘 + 6);
6 6
𝑘(𝑘+1)(2𝑘+1)+6(𝑘+1)2 1
= (𝑘 + 1)(2𝑘 2 + 7𝑘 + 6);
6 6
Coloquemos o termo (𝑘 + 1) em evidência:
(𝑘+1)(𝑘(2𝑘+1)+6(𝑘+1)) 1
= (𝑘 + 1)(2𝑘 2 + 7𝑘 + 6);
6 6
(𝑘+1)(2𝑘 2 +𝑘+6𝑘+6) 1
= (𝑘 + 1)(2𝑘 2 + 7𝑘 + 6);
6 6
1 1
(𝑘 + 1)(2𝑘 + 7𝑘 + 6) = (𝑘 + 1)(2𝑘 2 + 7𝑘 + 6), ✓.
2
6 6
O primeiro membro da igualdade é igual ao segundo (lado esquerdo igual ao direito),
portanto, a igualdade foi provada para 𝑘 + 1 e, por conseguinte, demonstramos a
validade da igualdade para qualquer número natural.
É comum, ao final de uma demonstração, terminarmos com as iniciais C.Q.D (como
queríamos demonstrar) ou Q.E.D (Quod Erat Demonstrandum – latim para “como
queríamos demonstrar”).

1
2. Prove que 1 + 2 + ⋯ + 𝑛 = 𝑛(𝑛 + 1).
2
Passo 1 – Verificar a validade para 𝒏 = 𝟏:
1
1 = (1 + 1) = 1, ✓.
2
Passo 2 – Supor a validade para um número natural 𝒌 qualquer:
1
1 + 2 + ⋯ + 𝑘 = 𝑘(𝑘 + 1), ✓.
2
Passo 3 – Provar que para o natural 𝒌 + 𝟏, a relação também é válida:
1
1 + 2 + ⋯ + 𝑘 + (𝑘 + 1) = (𝑘 + 1)(𝑘 + 1 + 1);
2
1
𝟏 + 𝟐 + ⋯ + 𝒌 + (𝑘 + 1) = (𝑘 + 1)(𝑘 + 2);
2
1
Temos que 𝟏 + 𝟐 + ⋯ + 𝒌 = 𝑘(𝑘 + 1):
2
1 1
𝑘(𝑘 + 1) + (𝑘 + 1) = (𝑘 + 1)(𝑘 + 2);
2 2
Colocando o termo (𝑘 + 1) em evidência, obtemos:
1 1
(𝑘 + 1) ( 𝑘 + 1) = (𝑘 + 1)(𝑘 + 2);
2 2
𝑘+2 1
(𝑘 + 1) ( ) = 2 (𝑘 + 1)(𝑘 + 2);
2
1 1
(𝑘 + 1) ( (𝑘 + 2)) = (𝑘 + 1)(𝑘 + 2);
2 2
1 1
(𝑘 + 1)(𝑘 + 2) = (𝑘 + 1)(𝑘 + 2), ✓.
2 2
Q.E.D.

3. Prove que, para todo 𝑚 natural, o produto:


𝑚(𝑚 + 1)(𝑚 + 2)(𝑚 + 3) é divisível por 12, ou seja, prove que o produto de quatro
números naturais consecutivos é divisível por 12.
Passo 1 – Verificar a validade para 𝒎 = 𝟏:
1(1 + 1)(1 + 2)(1 + 3) = 2.3.4 = 24, 24 é divisível por 12, ✓.
Passo 2 – Supor a validade para um número natural 𝒌 qualquer:
𝑘(𝑘 + 1)(𝑘 + 2)(𝑘 + 3) é divisível por 12, onde 𝑘 é um natural qualquer, ✓.
Passo 3 – Provar que para o natural 𝒌 + 𝟏, a relação também é válida:
(𝑘 + 1)(𝑘 + 1 + 1)(𝑘 + 1 + 2)(𝑘 + 1 + 3) = (𝑘 + 1)(𝑘 + 2)(𝑘 + 3)(𝑘 + 4)
= (𝑘 2 + 2𝑘 + 𝑘 + 2)(𝑘 2 + 4𝑘 + 3𝑘 + 12) = (𝑘 2 + 3𝑘 + 2)(𝑘 2 + 7𝑘 + 12);
𝑘 4 + 7𝑘 3 + 12𝑘 2 + 3𝑘 3 + 21𝑘 2 + 36𝑘 + 2𝑘 2 + 14𝑘 + 24;
Chamemos 𝑥 = 𝑘 4 + 7𝑘 3 + 12𝑘 2 + 3𝑘 3 + 21𝑘 2 + 36𝑘 + 2𝑘 2 + 14𝑘 + 24 para
facilitar os cálculos.
Temos que 𝑥 = 𝑘 4 + 10𝑘 3 + 35𝑘 2 + 50𝑘 + 24;
𝑥 𝑘 4 +10𝑘 3 +35𝑘 2 +50𝑘+24
= ;
12 12
𝑥 𝑘 4 +10𝑘 3 +35𝑘 2 +50𝑘 24
= + ;
12 12 12
Segundo nossa hipótese de indução (passo 2), o número 𝑘 4 + 10𝑘 3 + 35𝑘 2 + 50𝑘 é
divisível por 12 e 24 também é, ✓.
Logo, demonstramos que o produto de quatro naturais consecutivos é divisível por 12.
Q.E.D.

IV. Exercícios gerais


⊕ − 1. Prove que, para todo natural 𝑛, a soma
2𝑛 + 1 + 2𝑛 + 3 é um número par.

⊕ − 2. Mostre que a soma de 4 números ímpares é um número par.

⊕⊕ − 3. Mostre que a soma de 2𝑘 números ímpares, onde 𝑘 é um número natural, é


um número par.

⊕⊕⊕ − 4. Seja 𝑟 um número real, 𝑟 > 0.


1−𝑟 𝑛
Prove que 1 + 𝑟 + 𝑟 2 + ⋯ + 𝑟 𝑛 = − .
1−𝑟
* Sugestão: Soma dos termos de uma progressão geométrica.

⊕⊕⊕ − 5. Mostre que 2 + 4 + 6 + ⋯ + 2𝑛 = 𝑛(𝑛 + 1).

⊕⊕ − 6. Mostre que 1 + 3 + 5 + ⋯ + (2𝑛 − 1) = 𝑛2 .

⊕⊕⊕ − 7. (SIMULADO ITA) Mostre que para cada inteiro 𝑛, com 𝑛 ≥ 0, o inteiro
9𝑛 − 1 é divisível por 8.

⊕⊕ − 8. 𝑎) Demonstre a desigualdade das médias de Cauchy:


Sendo 𝑎 e 𝑏 números reais, vale que
𝑎 + 𝑏 ≥ 2√𝑎𝑏.
1
𝑏) Demonstre a desigualdade da soma de um número real 𝑎 com seu inverso :
𝑎
1
𝑎 + ≥ 2.
𝑎
2
3 3 3 1
⊕⊕ − 9. Prove que 1 + 2 + ⋯ + 𝑛 = ( 𝑛(𝑛 + 1)) .
2
Capítulo 1 – Somatórios

I. Introdução e notação Sigma (𝚺)


Como já dito anteriormente, o conceito de soma é algo primitivo na aritmética.
Podemos dizer que a noção de Somatório é a evolução natural deste conceito, pois nos
possibilita, de maneira prática e formal, um vasto ferramental teórico, o qual está
presente nas mais diversas áreas, como na álgebra, na computação, no Cálculo, etc.
A ideia fundamental por trás do Somatório é a compactação de uma soma por meio de
uma notação denominada Notação Sigma: ∑.
Esta notação foi introduzida pelo matemático Joseph Fourier, em 1829.
A letra grega Sigma corresponde, no alfabeto grego, a nossa letra S.
Seja:
𝒏

S = ∑ 𝑎𝑘 = 𝑎1 + 𝑎2 + ⋯ + 𝑎𝑛 .
𝒌=𝟏
As três partes principais que compõem o Somatório são:
1 – O argumento do somatório nos indica o que irá ser somado, no caso acima,
todos os números 𝒂𝒌 .
2 – O somatório se inicia em 𝒌 = 𝟏 (limite inferior).
3 – O somatório termina em 𝒌 = 𝒏 (limite superior).
Dito isto, vejamos a definição intuitiva de Somatório.

II. Definição intuitiva de somatório


Consideremos o seguinte problema:
“1. Calcule a soma dos 50 primeiros números naturais.”
O problema nos pede a seguinte soma:
S = 1 + 2 + ⋯ + 50.
Da maneira convencional, é uma soma relativamente grande, porém, utilizando a
notação de somatório, obtemos:
S = 1 + 2 + ⋯ + 50;
50

S = ∑ 𝑘 = 1 + 2 + ⋯ + 50.
𝑘=1
O número 1 é o limite inferior, pois é o primeiro número a ser somado e o número 50 é
o limite superior, pois é o último número a ser somado.
A letra 𝑘 é o argumento do somatório, que irá assumir os valores de 1 até 50.
“2. Coloque na notação de somatório a soma dos logaritmos, na base 10, de 10 a 20.”
O problema nos pede para colocarmos na notação sigma a soma dos logaritmos de 10
até 20 na base 10, de tal maneira:
log10 10 + log10 11 + log10 12 + ⋯ + log10 20.
Nosso índice 𝑘 irá variar de 10 a 20, logo:
20

log10 10 + log10 11 + log10 12 + ⋯ + log10 20 = ∑ log10 𝑘 .


10
“3. Coloque na notação sigma a soma dos senos dos arcos de 15° até 30°.”
O problema nos pede para colocarmos na notação sigma a soma dos senos dos arcos
de 15° até 30°:
⟹ sen 15° + sen 16° + sen 17° + ⋯ + sen 30°.
O índice 𝑘 irá variar de 15° até 30°, assim:
30°

sen 15° + sen 16° + sen 17° + ⋯ + sen 30° = ∑ sen 𝑘 .


15°
“4. A soma S = 2 + 4 + 6 + ⋯ + 2𝑛, quando colocada na notação de somatório, é
equivalente a?”
Temos a seguinte soma S = 2 + 4 + 6 + ⋯ + 2𝑛.
É fácil perceber que é a soma dos primeiros 𝑛 primeiros números pares.
Um número par qualquer pode ser escrito da seguinte maneira:
par = 2𝑘, onde 𝑘 pode assumir qualquer valor inteiro.
Dito isto, temos:
𝑛

2 + 4 + 6 + ⋯ + 2𝑛 = ∑ 2𝑘.
1
O argumento 2𝑘, para 𝑘 = 1, torna-se: 2.1 = 2.
Para 𝑘 = 2 ⟹ 2.2 = 4.
𝑘 = 3 ⟹ 2.3 = 6.
𝑘 = 4 ⟹ 2.4 = 8.
𝑘 = 𝑛 ⟹ 2. 𝑛 = 2𝑛.
“5. Coloque na forma de somatório a seguinte soma:
S = 1 + √2 + √3 + √4 + ⋯ + √𝑛.”
Temos a soma das raízes dos 𝑛 primeiros números naturais, ou seja:
𝑛

S = ∑ √𝑘 .
1
“6. Coloque na forma de somatório a seguinte soma:
𝜋 𝜋 𝜋
S = 𝜋 + + + ⋯ + .”
2 3 𝑛
𝜋
Temos a soma das frações de numerador 𝜋 e denominador na forma , no qual o
𝑘
número 𝑘 varia de 1 até 𝑛, ou seja:
𝑛
𝜋
S=∑ .
1𝑘
“7. Coloque na forma de somatório a seguinte soma:
S = arc tg(0) + arc tg(1) + arc tg(2) + ⋯ + arc tg(𝑛).”
Temos a soma dos arcos cuja tangente varia de 0 até 𝑛, ou seja:
𝑛

S = ∑ arc tg(𝑘) .
0
“8. Coloque na forma de somatório:
S = (11) + (21) + (31) + ⋯ + (𝑛1).”
Temos a soma dos 𝑛 números binomiais localizados na primeira coluna do Triângulo
de Pascal, ou seja:
𝑛
𝑘
S = ∑( ).
1
1
Ou seja, o grande feito desta notação é a simplificação de operações de soma
relativamente grandes.
Tal simplificação também garante o uso de certas propriedades, as quais serão vistas
no item IV.

III. Definição formal de somatório


Ampliando a noção de Somatório, façamos sua definição formal.
Seja A ⊂ ℝ, e ℤ ⊂ A, com 𝑎 e 𝑏 ∈ ℤ, tal que:
𝑓: A → ℝ ⟹ ∑𝑏𝑎 𝑓(𝑘) = 𝑓(𝑎) + ⋯ + 𝑓(𝑏).
Sendo 𝑛 o número de termos da soma, este é calculado, segundo o termo geral de uma
progressão aritmética de razão 1 (𝑎𝑛 = 𝑎1 + (𝑛 − 1)), da seguinte maneira:
𝑏 = 𝑎 + (𝑛 − 1); 𝑛 = 𝑏 − 𝑎 + 1.
Deste modo, temos que:
𝑎

∑ 𝑓(𝑘) = 𝑓(𝑎)
𝑘=𝑎
𝑏 𝑏−1

∑ 𝑓(𝑘) = 𝑓(𝑏) + ∑ 𝑓(𝑘)


{𝑘=𝑎 𝑖=𝑎
Estas definições nos serão úteis na resolução de inúmeros problemas, os quais podem
pedir para demonstrarmos propriedades, lemas, calcularmos somas com mais de 𝑛
termos, etc.
As propriedades, de importância crucial para esta teoria, serão discutidas e
demonstradas a seguir, no item IV, com exceção da Propriedade Telescópica, a qual,
por se tratar de uma das mais importantes propriedades desta teoria, com inúmeras
aplicações em diversas áreas da Matemática, será discutida de maneira intensa
(incluindo suas aplicações no Cálculo das Séries, progressões, etc, e demonstrada no
item V da teoria e, no último capítulo, será introduzido o conceito de
Série Telescópica, estendendo este para o caso no qual o limite superior é o próprio
infinito.
IV. Propriedades
Discutiremos, agora, as propriedades recorrentes dos somatórios.
Tais propriedades são de absoluta importância para esta teoria, pois poupam esforço
matemático desnecessário.
Tais propriedades são:
P1. – Somatório de uma constante (linearidade);
P2. – Somatório de uma constante por uma variável;
P3. – Somatório de uma soma ou uma diferença (∑𝒃𝒌=𝒂(𝒇(𝒌) ± 𝒈(𝒌));
P4. – Soma vazia e propriedade recursiva.
Existem outras propriedades, como o somatório duplo, que fogem do escopo desta
teoria e, portanto, não serão discutidas.
Agora, estudaremos cada uma destas propriedades.

P1. – Somatório de uma constante:


“O somatório de uma constante é igual ao produto da constante pelo número de
termos do somatório.”
Em linguagem matemática, o caso geral (𝑎 ≠ 1) do somatório de uma constante é tal
que:
𝑏

∑ 𝛼 = (𝑏 − 𝑎 + 1)𝛼 .
𝑘=𝑎
Onde 𝛼 é uma constante real qualquer não nula, 𝛼 ∈ ℝ∗ , e (𝑏 − 𝑎 + 1) é o número de
termos a serem somados.
No caso no qual o somatório se inicia no limite inferior igual a 1, vale que:
𝑛

∑ 𝛼 = 𝑛𝛼.
𝑘=1
Demonstração:
Demonstraremos ambos os casos, a começar pelo caso particular do limite inferior ser
igual a 1:
𝑛

∑ 𝛼 = 𝛼 + 𝛼 + ⋯ + 𝛼.
𝑘=1
Temos que 𝑛 = 1 + (𝑛𝑡 − 1) ∴ 𝑛𝑡 = 𝑛;
Assim,
𝑛

∑ 𝛼 = 𝑛𝛼.
𝑘=1
Agora, o caso geral:
𝑏

∑ 𝛼 = 𝛼 + 𝛼 + ⋯ + 𝛼.
𝑘=𝑎
O número de termos 𝑛 deste somatório é dado por 𝑏 = 𝑎 + (𝑛 − 1)
(novamente o termo geral de uma progressão aritmética de razão 1),
𝑛 = 𝑏 − 𝑎 + 1.
Assim:
𝑏

∑ 𝛼 = 𝑛. 𝛼 = (𝑏 − 𝑎 + 1)𝛼.
𝑘=𝑎

Agora, vejamos alguns exemplos:


“1. Calcule o valor da soma:
∑25
𝑘=1 5.”
Temos que o número de termos desta soma é dado por 25 = 1 + (𝑛 − 1), 𝑛 = 25.
Assim:
25

∑ 5 = 25.5 = 125.
𝑘=1
“2. Calcule: ∑13
𝑘=5 4.”
Temos que o número de termos desta soma é dado por 13 = 5 + (𝑛 − 1), 𝑛 = 9.
Assim:
13

∑ 4 = 9.4 = 36.
𝑘=5
“3. Calcule o valor de 𝑥 em:
𝑥 = ∑25 3 9
3 4 − ∑1 7 + ∑2 5.”
Calculando cada uma das somas separadamente, temos:
Definindo ∑25 3 9
3 4 = 𝒂, ∑1 7 = 𝒃 e ∑2 5 = 𝒄, temos que 𝑥 = 𝑎 + 𝑏 + 𝑐 e, sejam:
𝒏𝒂 (número de termos da soma 𝒂); dado por 25 = 3 + (𝑛𝑎 − 1),
𝑛𝑎 = 22.
Assim, 𝑎 = 22.4 = 88.
𝒏𝒃 (número de termos da soma 𝒃); dado por 3 = 1 + (𝑛 − 1),
𝑛𝑏 = 3.
Assim, 𝑏 = 3.7 = 21.
𝒏𝒄 (número de termos da soma 𝒄); dado por 9 = 2 + (𝑛 − 1),
𝑛𝑐 = 8.
Assim, 𝑐 = 8.5 = 40.
Por fim, 𝑥 = 𝑎 + 𝑏 + 𝑐 = 88 − 21 + 40 = 100.
“4. (CHINA JÚNIOR) – Se 𝛼, 𝛽 e 𝛾 são constantes reais não nulas tais que
∑𝑚2 𝛼 = 7, ∑4
𝑚−1
𝛽 = 14 e ∑𝑚−33 𝛾 = 18, calcule o valor de 𝛼 + 𝛽 + 𝛾,
sabendo-se que a soma dos números de termos das três somas é igual a 20.”
O problema nos pede a soma das constantes 𝛼, 𝛽 e 𝛾.
Sejam 𝑛1 , 𝑛2 e 𝑛3 os números de termos da primeira, segunda e terceira soma.
Temos que:
𝑛1 : 𝑚 = 2 + (𝑛1 − 1), 𝑛1 = 𝑚 − 1.
{𝑛2 : 𝑚 − 1 = 4 + (𝑛2 − 1), 𝑛2 = 𝑚 − 4.
𝑛3 : 𝑚 − 3 = 3 + (𝑛3 − 1), 𝑛3 = 𝑚 − 5.
O problema nos informa que 𝑛1 + 𝑛2 + 𝑛3 = 20;
3𝑚 − 10 = 20; 𝑚 = 10.
Assim, como a primeira sola vale 7, a segunda 14 e a terceira 18, temos que:
7
7 = 𝑛1 . 𝛼; 7 = (𝑚 − 1)𝛼; 𝛼 = .
𝑚−1
14
14 = 𝑛2 . 𝛽; 14 = (𝑚 − 4)𝛽; 𝛽 = .
𝑚−4
18
18 = 𝑛3 . 𝛾; 18 = (𝑚 − 5)𝛾; 𝛾 = .
𝑚−5
7 14 18
𝛼+𝛽+𝛾 = + + ;
𝑚−1 𝑚−4 𝑚−5
(𝑚−4)(𝑚−5)7+(𝑚−1)(𝑚−5)14+(𝑚−1)(𝑚−4)18
𝛼+𝛽+𝛾 = (𝑚−1)(𝑚−4)(𝑚−5)
;
(10−4)(10−5).7+(10−1)(10−5).14+(10−1)(10−4).18
𝛼+𝛽+𝛾 = (10−1)(10−4)(10−5)
;
6.5.7+9.5.14+9.6.18 302
𝛼+𝛽+𝛾 = = .
9.6.5 45

P2. – Somatório de uma constante por uma variável:


“O somatório de uma constante por uma variável é o produto da constante pelo
somatório da variável.”
Em linguagem matemática, temos que:
𝑏 𝑏

∑ 𝛼. 𝑓(𝑘) = 𝛼 [∑ 𝑓(𝑘)] .
𝑎 𝑎
Demonstração:
A demonstração desta propriedade é deveras simples, basta expandir o somatório e
colocar em evidência a constante multiplicativa 𝛼:
𝑏

∑ 𝛼. 𝑓(𝑘) = 𝛼. 𝑓(𝑎) + 𝛼. 𝑓(𝑎 + 1) + 𝛼. 𝑓(𝑎 + 2) + ⋯ + 𝛼. 𝑓(𝑏)


𝑎
𝑏

= 𝛼(𝑓(𝑎) + 𝑓(𝑎 + 1) + 𝑓(𝑎 + 2) + ⋯ + 𝑓(𝑏)) = 𝛼 ∑ 𝑓(𝑘) .


𝑎

Agora, façamos uns exemplos:


9
1
“1. Calcule ∑ (𝑘 2 + 5𝑘).”
55
Temos que:
1 1 1
∑95 (𝑘 2 + 5𝑘) = (∑95 𝑘 2 + 5𝑘) = 5 (52 + 25 + 62 + 30 + 72 + 35 + 82 + 40 + 92 + 45) = 86.
5 5
(Veremos que a próxima propriedade nos poupará tais operações.)
“2. Calcule ∑31 4𝑘.”
Temos que:
3 3

∑ 4𝑘 = 4 ∑ 𝑘 = 4(1 + 2 + 3) = 24.
1 1
1
“3. Calcule ∑3𝑗=1 √(𝑗 2 + 5𝑗).”
√5
Temos que:
3 3
1 1 1
∑ (√𝑗 2 + 5𝑗) = ∑ √𝑗 2 + 5𝑗 = (√6 + √14 + √24)
𝑗=1
√5 √5 𝑗=1 √5
√6 + √14 + √24 √5 √30 + √70 + √120 3√30 + √70
= . = = .
√5 √5 5 5

P3. – Somatório de uma adição ou uma diferença (propriedade aditiva):


“O somatório de uma adição ou de uma diferença é a adição ou diferença dos
somatórios.”
Matematicamente:
𝑏 𝑏 𝑏

∑(𝑓(𝑘) + 𝑔(𝑘)) = ∑ 𝑓(𝑘) + ∑ 𝑔(𝑘) .


𝑎 𝑎 𝑎
É necessária a compreensão da reversão da propriedade, ou seja:
𝒃
Passar de ∑𝒃𝒂 𝒇(𝒌) + ∑𝒃𝒂 𝒈(𝒌) para ∑𝒂(𝒇(𝒌) + 𝒈(𝒌)).
Demonstração:
A demonstração desta propriedade também é relativamente simples, pois:
𝑏

∑(𝑓(𝑘) + 𝑔(𝑘))
𝑎
= (𝑓(𝑎) + 𝑔(𝑎)) + (𝑓(𝑎 + 1) + 𝑔(𝑎 + 1)) + (𝑓(𝑎 + 2) + 𝑔(𝑎 + 2)) + ⋯ + (𝑓(𝑏) + 𝑔(𝑏))
= (𝒇(𝒂) + 𝒇(𝒂 + 𝟏) + 𝒇(𝒂 + 𝟐) + ⋯ + 𝒇(𝒃)) + (𝒈(𝒂) + 𝒈(𝒂 + 𝟏) + 𝒈(𝒂 + 𝟐) + ⋯ + 𝒈(𝒃))
𝒃 𝒃

= ∑ 𝒇(𝒌) + ∑ 𝒈(𝒌) .
𝒂 𝒂

1
“𝟏. Calcule ∑95 (𝑘 2 + 5𝑘).”
5
Esta questão já fora resolvida na propriedade 2, porém, mostraremos como a aplicação
da propriedade 3 facilita em sua resolução.
Da P2, temos:
9 9
1 1
∑ (𝑘 2 + 5𝑘) = (∑(𝑘 2 + 5𝑘)) .
5 5
5 5
Aplicando, agora, a P3:
9 9 9
1 1
(∑(𝑘 2 + 5𝑘)) = (∑ 𝑘 2 + ∑ 5𝑘) .
5 5
5 5 5
Observe, ainda, que podemos aplicar a P2 novamente em ∑95 5𝑘.
Obtemos, então:
9 9 9 9
1 1
(∑ 𝑘 2 + ∑ 5𝑘) = (∑ 𝑘 2 + 5 ∑ 𝑘) .
5 5
5 5 5 5
9 9
1
= ∑ 𝑘2 + ∑ 𝑘 .
5
5 5
1
= (52 + 62 + 72 + 82 + 92 ) + 5 + 6 + 7 + 8 + 9 = 86.
5
Insistimos que é necessária a compreensão da reversão da propriedade, ou seja:
De ∑𝑏𝑎 𝑓(𝑘) + ∑𝑏𝑎 𝑔(𝑘), realizar a passagem para ∑𝑏𝑎(𝑓(𝑘) + 𝑔(𝑘)).

P4. – Soma vazia e propriedade recursiva (ou de recorrência):


1 – “Uma soma é dita vazia, se e somente se, esta soma é nula.”
Matematicamente:
𝑏

∑ 𝑓(𝑘) = 0 ⟺ 𝑓(𝑎) + 𝑓(𝑎 + 1) + ⋯ + 𝑓(𝑏 − 1) + 𝑓(𝑏) = 0.


𝑘=𝑎

Antes de entrarmos na demonstração da Soma Vazia propriamente dita,


demonstraremos a recursividade de um somatório.
Define-se recursividade como o uso de definições prévias para a construção de
definições posteriores, em função destas.
Por exemplo, seja a sequência:
𝑎𝑛 : ℕ − {0} → ℝ | 𝑎𝑛 = 𝑎𝑛−1 − 𝑎𝑛−2 .
Tomando 𝑛 = 5, temos que:
𝑎5 = 𝑎4 − 𝑎3 ;
Tomando 𝑛 = 6:
𝑎6 = 𝑎5 − 𝑎4 ;
Ou seja, podemos definir qualquer termo da sequência (obedecendo as restrições do
domínio da mesma) a partir de seus termos anteriores.
A definição recursiva de um somatório segue o mesmo princípio demonstrado acima
nas sequências numéricas, pois utilizamos um número qualquer (este número deve
pertencer ao domínio da função argumento da soma) para “perturbar” a soma
em um determinado momento ao nosso bel-prazer e, então, utilizamos seu
sucessor para a finalizarmos.
Considerando os números 𝒂 (limite inferior) e 𝒃 (limite superior), seja A*1 o conjunto
no qual os argumentos 𝑘 da soma estão definidos.
Seja, também, o conjunto dos números reais como o conjunto que contém os
equivalentes 𝑓(𝑘).
Seja 𝑝 um número real qualquer, tal que 𝑝 pertence a A, e 𝒇(𝒑) pertence aos reais.
Temos, então, que:
Seja 𝑝 ∈ A ⊂ ℝ, tal que ∃ 𝑓(𝑝) ∈ ℝ.
Se as condições acima são satisfeitas, vale que:
𝒃 𝒑 𝒃

∑ 𝒇(𝒌) = ∑ 𝒇(𝒌) + ∑ 𝒇(𝒌) .


𝒌=𝒂 𝒌=𝒂 𝒌=𝒑+𝟏
A seguir, proporemos a demonstração da propriedade acima.
Demonstração da propriedade recursiva:
Dado que:
𝑏 𝑝 𝑏

∑ 𝑓(𝑘) = ∑ 𝑓(𝑘) + ∑ 𝑓(𝑘);


𝑎 𝑎 𝑝+1
Vale que:
𝑓(𝑎) + 𝑓(𝑎 + 1) + ⋯ + 𝑓(𝑏) = 𝑓(𝑎) + 𝑓(𝑎 + 1) + ⋯ + 𝑓(𝑝) + 𝑓(𝑝 + 1) + 𝑓(𝑝 + 2) + ⋯ + 𝑓(𝑏);
𝑓(𝑎) + 𝑓(𝑎 + 1) + ⋯ + 𝑓(𝑏) = 𝑓(𝑎) + 𝑓(𝑎 + 1) + ⋯ + 𝑓(𝑏) + 𝑓(𝑝) + 𝑓(𝑝 + 1) + ⋯;
𝑏 𝑏 𝑥

∑ 𝑓(𝑘) = ∑ 𝑓(𝑘) + ∑ 𝑓(𝑘).


𝑎 𝑎 𝑝
Como o término da soma dos 𝑝 é indeterminado (não sabemos onde esta termina),
tomaremos este como a letra 𝑥.
Assim, obtemos:
𝑥

∑ 𝑓(𝑘) = 0.
𝑝
Ou seja, se esta soma é nula, ela não interfere na primeira.
Assim, demonstramos a propriedade recursiva de maneira intuitiva.
(A demonstração da propriedade recursiva é semelhante a demonstração da
propriedade da soma vazia.)

Demonstração da propriedade da soma vazia:


Sabemos que:
𝑏 𝑝 𝑏

∑ 𝑓(𝑘) = ∑ 𝑓(𝑘) + ∑ 𝑓(𝑘) .


𝑎 𝑎 𝑝+1
Tendo isto em mente, podemos tomar qualquer valor de 𝒑.
Tomemos, então, 𝑝 = 𝑎 − 1.

1
A é o conjunto, ou domínio de todos os argumentos 𝒌 que terão um equivalente 𝒇(𝒌).
Assim:
𝒃 𝑎−1 𝒃

∑ 𝒇(𝒌) = ∑ 𝑓(𝑘) + ∑ 𝒇(𝒌).


𝒂 𝑎 𝒂
Deste modo, verificamos que
𝑎−1

∑ 𝑓(𝑘) = 0.
𝑎
Esta soma é denominada soma vazia.

“1. – Se:
𝑚

∑(2𝑘 + 5) = 72 ;
1
𝑚
Obtenha o valor de .”
2𝑚−6
Típico exercício de somatórios.
Para resolvê-lo, devemos ter em mente que:
𝑚 𝑚 𝑚

∑(2𝑘 + 5) = ∑ 2𝑘 + ∑ 5.
1 1 1
𝑚 𝑚

∑ 2𝑘 + ∑ 5 = 72;
1 1
𝑚 𝑚

2 ∑ 𝑘 + ∑ 5 = 72;
1 1
2(1 + 2 + 3 + ⋯ + 𝑚) + 5𝑚 = 72;
𝟏
𝟐 ( (𝟏 + 𝒎)𝒎) + 5𝑚 = 72;
𝟐
* Expandimos a primeira soma e aplicamos a soma de uma progressão aritmética de razão 1.
𝑚(1 + 𝑚) + 5𝑚 = 72;
𝑚(1 + 𝑚 + 5) = 72;
𝑚(𝑚 + 6) = 72;
𝑚² + 6𝑚 = 72;
2
𝑚² + 6𝑚 = 72;
2
𝑚² + 6𝑚 + 9 = 72 + 9;
(𝑚 + 3)2 = 81;
𝑚 + 3 = ± 9;
𝒎 = 𝟔 ou 𝑚 = − 12.
Como 𝑚 deve ser um número inteiro maior do que zero, temos que 𝑚 = 6.
Assim:
𝑚 6
= = 𝟏.
2𝑚−6 12−6
“2. – Obtenha o valor de:
(𝑥 − 𝑎) ∑𝑛0(𝑛𝑘) 𝑥 𝑛−𝑘 (− 𝑎𝑘 ).”
Aplicação direta do conceito de Binômio de Newton. Temos que:
𝑛
𝑛
(𝑥 − 𝑎) ∑ ( ) 𝑥 𝑛−𝑘 (− 𝑎𝑘 ) = (𝑥 − 𝑎)(𝑥 − 𝑎)𝑛 = (𝑥 − 𝑎)𝑛+1 .
𝑘
0
“3. Se 𝑎 é uma constante não nula, obtenha o valor da soma S, sabendo-se que:
1 𝑛
S−𝑎 = ∑1 (𝑘 + 𝑘 2 ).”
𝑎+𝑎2 +⋯+𝑎𝑛
1
Se 𝑎 é uma constante, então a parcela também é.
𝑎+𝑎2 +⋯+𝑎𝑛
1
Assim, tomemos = 𝑣.
𝑎+𝑎2 +⋯+𝑎𝑛
𝑛

S − 𝑎 = 𝑣 ∑(𝑘 + 𝑘 2 ) ;
1
𝑛

S = 𝑣 ∑(𝑘 + 𝑘 2 ) + 𝑎;
1
𝑛 𝑛

S = 𝑣 (∑ 𝑘 + ∑ 𝑘²) + 𝑎.
1 1
De acordo com nossa teoria a respeito do Princípio da Indução Finita,
demonstramos que:
1:
𝒏
𝟏
∑ 𝒌² = 𝟏² + 𝟐² + ⋯ + 𝒏𝟐 = 𝒏(𝒏 + 𝟏)(𝟐𝒏 + 𝟏);
𝟔
𝟏
2:
𝒏
𝟏
∑𝒌 = 𝟏 + 𝟐 + ⋯+ 𝒏 = 𝒏(𝒏 + 𝟏).
𝟐
𝟏
Obtemos, então:
1 1
S = 𝑣 ( 𝑛(𝑛 + 1) + 𝑛(𝑛 + 1)(2𝑛 + 1)) + 𝑎;
2 6
1
S = 𝑣 ( (𝑛 + 1)(𝑛 + 2)) + 𝑎;
3
𝑣 3𝑎 1
S = (𝑛 + 1)(𝑛 + 2) + = (𝑣(𝑛 + 1)(𝑛 + 2) + 3𝑎);
3 3 3
1 1
S = (( 2
3 𝑎+𝑎 +⋯+𝑎𝑛
) (𝑛 + 1)(𝑛 + 2) + 3𝑎);
1 (𝑛+1)(𝑛+2)
S= ( 2 + 3𝑎).
3 𝑎+𝑎 +⋯+𝑎𝑛

No próximo item, proporemos a definição da Propriedade Telescópica e teceremos


comentários a respeito de sua importância na teoria dos somatórios, além de
estudarmos as Frações Parciais, também de grande importância para a teoria.
V. Frações Parciais e a Soma Telescópica
1. – Apêndice sobre Frações Parciais
O conceito de Fração Parcial é, também, algo de fácil compreensão.
Tal conceito ganha força com o advento do Cálculo, nas derivadas e nas integrais
polinomiais do tipo
𝑝(𝑥)
𝑟(𝑥) = , 𝑔(𝑥) ≠ 0.
𝑔(𝑥)
Tal ferramenta é muito útil em problemas de polinômios, e de crucial importância na
vindoura Soma Telescópica.
𝑓(𝑥)
A ideia por trás deste conceito é transformar a fração em:
𝑔(𝑥)
𝐀𝟏 𝐀𝟐 𝐀𝒏
+ + ⋯+ ,
𝒈(𝒙) 𝒈(𝒙) 𝒈(𝒙)
onde A1 , A2 ,..., A𝑛 são constantes reais não nulas.
O caso que será abordado mais comumente por nossa teoria é do tipo:
𝑝(𝑥)
𝑟(𝑥) = (𝑥−𝑎 , onde o denominador deverá ser um polinômio cujas raízes
1 )(𝑥−𝑎2 )…(𝑥−𝑎𝑛 )
deverão ser diferentes de 𝑎1 , 𝑎2 ,..., 𝑎𝑛 , e o grau do numerador deverá ser menor
que o grau do denominador. ← Importante!
Será abordado com mais frequência o caso no qual o denominador é um
polinômio quadrático do tipo
𝒈(𝒙) = (𝒙 − 𝒂𝟏 )(𝒙 − 𝒂𝟐 ).
O método de obtenção das constantes segue os seguintes passos:
1. – Multiplicar os membros da igualdade por (𝒙 − 𝒂𝒌 );
2. – Simplificar os equivalentes;
3. – Escolher as constantes 𝐀𝟏 , 𝐀𝟐 , ..., 𝐀𝒏 de modo que a escolhe de uma ocasione
na anulação das outras, seguindo as condições de existência.
4. – Propor as igualdades de forma a obter todas as outras constantes.

Façamos um exemplo:
4 A B
“1. Se (𝑥−2)(𝑥−3) = + , onde A e B são constantes reais, qual o valor de
𝑥−2 𝑥−3
A + B?”
A 𝑥−3 B 𝑥−2
Multiplicando por e por , obtemos:
𝑥−2 𝑥−3 𝑥−3 𝑥−2
4 A 𝑥−3 B 𝑥−2
(𝑥−2)(𝑥−3)
= ( )+ ( );
𝑥−2 𝑥−3 𝑥−3 𝑥−2
4 A(𝑥−3)+B(𝑥−2)
(𝑥−2)(𝑥−3)
= ;
(𝑥−2)(𝑥−3)
Multiplicamos ambos os lados pelo fator (𝑥 − 2)(𝑥 − 3) para eliminá-lo dos dois
denominadores, obtendo:
4 = A(𝑥 − 3) + B(𝑥 − 2).
Notemos que, quando ainda havia este fator como denominador, a condição de
existência deste era 𝒙 ≠ 𝟐 e 𝒙 ≠ 𝟑, porém, ao eliminarmos estes denominadores, a
variável 𝑥 poderá assumir qualquer valor em 4 = A(𝑥 − 3) + B(𝑥 − 2), incluindo
𝒙 = 𝟐 e 𝒙 = 𝟑.
Deste modo, para obtermos qualquer uma das constantes acima, devemos tomar um
valor de 𝑥 de tal modo que esta escolha afete na outra parcela e, consequentemente,
promova sua anulação.
Primeiramente, vamos encontrar o valor da constante A.
Temos a igualdade 𝟒 = 𝐀(𝒙 − 𝟑) + 𝐁(𝒙 − 𝟐).
Para encontrarmos o valor de A, deveremos tomar 𝒙 = 𝟐 para que a parcela
𝐁(𝒙 − 𝟐) se anule.
Dito isto, temos, para 𝑥 = 2:
4 = A(2 − 3) + 𝐁(𝟐 − 𝟐).
4 = A(−1) ⇒ 4 = − A ∴ 𝐀 = − 𝟒.
Obtendo o valor de A, devemos tomar o valor de 𝒙 para o qual a parcela
multiplicativa de A se anule e, no caso, 𝑥 = 3.
Para 𝑥 = 3:
4 = 𝐀(𝟑 − 𝟑) + B(3 − 2);
4 = B.
Assim, obtemos o valor das constantes A e B.
4 A B
(𝑥−2)(𝑥−3)
= + ;
𝑥−2 𝑥−3
4 4 4
(𝑥−2)(𝑥−3)
=− + ;
𝑥−2 𝑥−3
𝟒 𝟒 𝟒
= − .
(𝒙 − 𝟐)(𝒙 − 𝟑) 𝒙 − 𝟑 𝒙 − 𝟐
O valor da soma pedida é 𝐀 + 𝐁 = − 𝟒 + 𝟒 = 𝟎.
“ 2. Obtenha o valor das constantes 𝛼 e 𝛽, sabendo que:
𝑥−5 𝛼 𝛽
= + .”
𝑥 2 −5𝑥+6 𝑥−2 𝑥−3
Tirando o M.M.C do lado esquerdo da igualdade, temos:
𝑥−5 𝛼(𝑥−3)+𝛽(𝑥−2)
= .
𝑥 2 −5𝑥+6 (𝑥−2)(𝑥−3)
Do teorema da decomposição de polinômios, temos que o polinômio
𝒙𝟐 − 𝟓𝒙 + 𝟔 é equivalente a (𝒙 − 𝟐)(𝒙 − 𝟑).
Logo, multiplicando ambos os lados da igualdade pelo fator (𝑥 − 2)(𝑥 − 3):
𝒙 − 𝟓 = 𝜶(𝒙 − 𝟑) + 𝜷(𝒙 − 𝟐).
Para obtermos o valor da constante 𝜶, tomemos 𝒙 = 𝟐, para que eliminemos a
parcela 𝜷(𝒙 − 𝟐).
2 − 5 = 𝛼(2 − 3) + 𝛽(2 − 2);
− 3 = − 𝑎; 𝛼 = 3.
Tendo o valor da constante 𝛼, tomemos, agora, 𝒙 = 𝟑, para que anulemos a parcela
𝜶(𝒙 − 𝟑).
3 − 5 = 𝛼(3 − 3) + 𝛽(3 − 2);
− 2 = 𝛽.
Então, temos que ; 𝛼 = 3 e 𝛽 = − 2.
A fração dada, em formato de Fração Parcial é:
𝑥−5 3 2
= − .
𝑥 2 −5𝑥+6 𝑥−2 𝑥−3
Fica como curiosidade que, no Cálculo, o método das Frações Parciais é aplicado
exaustivamente em integrais do tipo
𝑓(𝑥)
∫ 𝑑𝑥 .
𝑔(𝑥)
Além de ser uma ferramenta útil na resolução de equações diferenciais como, por
exemplo:
𝑑2𝑥 𝑑𝑥 𝑑𝑡
= + .
𝑑𝑡 2 (𝑥 − 5)(𝑥 − 3) 5 ln 𝑥

Exercícios gerais – Frações Parciais


⊕ − 1. Seja:
𝑥−1 A B
= + .
𝑥²−4 𝑥+2 𝑥−2
1
Obtenha o valor de (A − B).
A+B

⊕⊕ − 2. Dado que:
𝑥+1 A B
= − , prove que:
𝑥²−6𝑥 6(𝑥−6) 6𝑥
|B − A| > 4.

⊕⊕⊕ − 3. Coloque em Frações Parciais:


3
a) ;
𝑥 2 −9
(𝑥−𝑎)
𝑏) ;
(𝑥−𝑏)(𝑥−𝑐)
𝑥 2 −5𝑥+6
c) (𝑥−1)(𝑥−6);
𝑥 3 −5𝑥+6
𝑑) ;
𝑥 2 −5𝑥
* Releia a teoria.
𝑥+1
e) ;
𝑥 2 +2𝑥+1
(𝑥−𝑎)
𝑓) .
(𝑥−𝑏)(𝑥−𝑐)(𝑥−𝑑)

⊕⊕ − 4.
8 𝐴 𝐵 𝐶
Dado que = + + , obtenha o valor da soma:
𝑥 3 −4𝑥 𝑥 𝑥−2 𝑥+2
1 1 1
+ + .
1+A 1+B 1+C
2. − A Soma Telescópica
Podemos afirmar, com certa prudência, que nossa teoria acerca dos somatórios
converge para este ponto.
A denominada Propriedade Telescópica, ou simplesmente Soma Telescópica é uma
técnica de absoluta importância pois promove uma simplificação inteligente e, em
alguns casos, extremamente necessária.
Quando adentramos na disciplina do Cálculo Diferencial e Integral, mais precisamente
no estudo das Séries Matemáticas, Nos deparamos com problemas como:

1
S𝑛 = ∑ .
𝑘(𝑘 + 1)
0
Esta soma recebe o nome de Série, pois seu limite superior é infinito e, se tivéssemos
que calcular o “valor exato” desta Série, tal operação nos levaria muito tempo.
Por ser uma Série, seu valor cresce indefinidamente a medida que nos aproximamos
do infinito, não sendo correto falar em um valor exato para esta, e sim em um limite
para o qual esta se aproxima (ou converge), quando mais nos aproximamos do infinito.
Ou seja:
𝐥𝐢𝐦 𝐒𝒏 .
𝒏→∞
Portanto, a aplicação de métodos algébricos comuns como o uso das propriedades dos
somatórios, simplificações e mudanças de variável não nos levariam a lugar algum
pois, para chegarmos a um valor concreto para o qual esta série converge, teríamos
que utilizar métodos estudados no Cálculo, como o Teste de Convergência por
Integral, Limite da Soma, Critério de D’Alembert, Testes de Abel e Dirichlet,
entre outros, os quais fogem absolutamente do escopo desta teoria.
Assim, a ferramenta correta a ser aplicada é a Propriedade Telescópica.

Definição intuitiva da Propriedade Telescópica – Somatórios:


Consideremos uma sequência 𝑎𝑛 tal que seus termos são dados por:
𝒂𝒏 = 𝒂𝒌+𝟏 − 𝒂𝒌 .
A Soma Telescópica, em resumo, é a soma de uma sequência de termos deste tipo, que
pode ser escrita da seguinte forma:
Stelescópica = (𝒂𝟐 − 𝑎1 ) + (𝒂𝟑 − 𝒂𝟐 ) + (𝒂𝟒 − 𝒂𝟑 ) + ⋯ + (𝒂𝒏 − 𝒂𝒏−𝟏 ) + (𝑎𝑛+1 − 𝒂𝒏 ).
Observe que esta soma pode ser simplificada em:
𝐒𝐭𝐞𝐥𝐞𝐬𝐜ó𝐩𝐢𝐜𝐚 = 𝒂𝒏+𝟏 − 𝒂𝟏 .
Ou seja, a dita Soma Telescópica é uma soma que pode ser resumida em apenas
dois termos.

Definição formal da Propriedade Telescópica – Somatórios:


Consideremos uma sequência qualquer 𝑎𝑛 de números reais tal que:
𝑎𝑛 : ℤ∗ → ℝ∗ , 𝑎𝑛 = 𝑎𝑘+1 − 𝑎𝑘 .
Seja S𝑛 = 𝑎1 + 𝑎2 + ⋯ + 𝑎𝑛 .
Definamos um operador algébrico ∆ denominado Operador Diferença tal que:
∆ 𝒂𝒏 = 𝒂𝒌+𝟏 − 𝒂𝒌 .
O qual aplicado a soma S𝑛 resulta em:
∆ S𝑘 = (𝑎2 − 𝑎1 ) + (𝑎3 − 𝑎2 ) + ⋯ + (𝑎𝑛 − 𝑎𝑛−1 ) + (𝑎𝑛+1 − 𝑎𝑛 );
∆ 𝐒𝒌 = 𝒂𝒏+𝟏 − 𝒂𝟏.
Na notação de somatório, temos que, se aplicada a definição de Soma Telescópica:
𝑛

S𝑘 = ∑ 𝑎𝑘 .
𝑘=1
𝒏

⟹ ∆ 𝐒𝒌 = ∑ ∆ 𝒂𝒌 = 𝒂𝒏+𝟏 − 𝒂𝟏 .
𝒌=𝟏
Ou, de maneira equivalente:
𝑏

S𝑘 = ∑ 𝑓(𝑘) .
𝑎
𝒃 𝒃

⟹ ∆ 𝐒𝒌 = ∑ ∆𝒇(𝒌) = ∑(𝒇(𝒌 + 𝟏) − 𝒇(𝒌)) = 𝒇(𝒃 + 𝟏) − 𝒇(𝒂) .


𝒂 𝒂
Demonstração:
Temos que:
𝑏

∆ S𝑘 = ∑(𝑓(𝑘 + 1) − 𝑓(𝑘))
𝑎
= (𝑓(𝑎 + 1) − 𝑓(𝑎)) + (𝑓(𝑎 + 2) − 𝑓(𝑎 + 1)) + ⋯ + (𝑓(𝑏 + 1) − 𝑓(𝑏)).
Todos os termos se anulam, com exceção de 𝑓(𝑎) e 𝑓(𝑏 + 1).
Obtemos, então:
𝒃

∆ 𝐒𝒌 = ∑(𝒇(𝒌 + 𝟏) − 𝒇(𝒌)) = 𝒇(𝒃 + 𝟏) − 𝒇(𝒂).


𝒂
Este resultado é comumente utilizado em provas olímpicas, vestibulares mais
tradicionais como o ITA e o IME, no Cálculo, Álgebra Linear, etc.
O estudo desta propriedade é feito de maneira rápida nas universidades, portanto é
interessante um conhecimento prévio desta.
Fica, também, como curiosidade, que a denominação Telescópica se dá em razão do
antigos telescópios, ou lunetas, utilizados na época das grandes navegações, o qual,
quando fechado, se resumia em duas partes:
O cabo do telescópio e seu visor.
Portanto, como a soma acima se resume a apenas duas parcelas, os antigos
matemáticos a batizaram de Soma Telescópica.
Para conseguirmos aplicar este conceito, é necessário manipular a estrutura de uma
soma a fim de chegar em 𝑓(𝑘 + 1) − 𝑓(𝑘) e, como já estudamos as Frações Parciais,
estas nos serão de extrema utilidade no cômputo de uma Soma Telescópica.
“1. (ÍNDIA) – Calcule o valor da soma:
𝑛
2
∑ − (𝑘+1)(𝑘+2).”
𝑘=2
Temos a seguinte soma:
𝑛
2
∑− .
(𝑘 + 1)(𝑘 + 2)
𝑘=2
Primeiramente, devemos utilizar a decomposição em Frações Parciais para transformar
2 A B
a função (𝑘+1)(𝑘+2) em + , sendo A e B constantes reais.
𝑘+1 𝑘+2
Assim, temos:
2 A B
(𝑘+1)(𝑘+2)
= + ;
𝑘+1 k+2
2 A(𝑘+2)+B(𝑘+1)
(𝑘+1)(𝑘+2)
= (𝑘+1)(𝑘+2)
;
2 = A(𝑘 + 2) + B(𝑘 + 1);
Para 𝑘 = − 1, temos:
2 = A(− 1 + 2) + B(− 1 + 1);
2 = A.
Agora, para obtermos B, seja 𝑘 = − 2:
2 = A(− 2 + 2) + B(− 2 + 1);
2 = − B;
B = − 2.
Então:
2 2 −2 2 2
= + = − .
(𝑘 + 1)(𝑘 + 2) 𝑘 + 1 𝑘 + 2 𝑘 + 1 𝑘 + 2
Então, a soma pode ser escrita como:
𝑛 𝑛 𝑛
2 2 2 2 2
∑− = ∑−( − ) = ∑( − ).
(𝑘 + 1)(𝑘 + 2) 𝑘+1 𝑘+2 𝑘+2 𝑘+1
𝑘=2 𝑘=2 𝑘=2
Observe que, simplesmente por utilizarmos a decomposição em frações parciais, já
2
obtivemos o padrão 𝑓(𝑘 + 1) − 𝑓(𝑘), onde 𝑓(𝑘) = .
𝑘
Porém, antes, façamos uma breve expansão desta soma para estudarmos seu padrão.
𝑛
2 2 𝟐 2 𝟐 𝟐 𝟐 𝟐
∑( − )=( − )+( − )+( − )
𝑘+2 𝑘+1 𝟐+𝟐 2+1 𝟑+𝟐 𝟑+𝟏 𝟒+𝟐 𝟒+𝟏
𝑘=2
𝟐 𝟐 𝟐 𝟐 𝟐 𝟐 2 𝟐
+( − )+( − ) + ⋯+ ( − )+( − ).
𝟓+𝟐 𝟓+𝟏 𝟔+𝟐 𝟔+𝟏 𝒏+𝟏 𝒏 𝑛+2 𝒏+𝟏
Percebamos que, os fatores em negrito irão todos se cancelar, sobrando apenas os
𝟐 𝟐
fatores – e .
𝟐+𝟏 𝒏+𝟐
Assim, nossa soma se torna:
𝒏
𝟐 𝟐 𝟐 𝟐
∑ (𝒌+𝟐 − 𝒌+𝟏) = 𝒏+𝟐 − 𝟑.
𝒌=𝟐
Veja que, em somas mais fáceis, não é necessário a aplicação da definição de
Soma Telescópica 𝒇(𝒃 + 𝟏) − 𝒇(𝒂) pois, ao fazermos a expansão de seus termos, só
restarão dois termos.
Porém, esta também pode ser calculada pela definição:
𝑛
2 2
∑( − ).
𝑘+2 𝑘+1
𝑘=2
Temos que os limites são 2 e 𝑛, assim:
𝑛
2 2 2
∑ (𝑘+2 − 𝑘+1) = 𝑓(𝑛 + 1) − 𝑓(2), onde 𝑓(𝑘) = 𝑘+1.
𝑘=2
Então:
𝑛
2 2 𝟐 𝟐
∑( − )= − .
𝑘+2 𝑘+1 𝒏+𝟐 𝟑
𝑘=2
“2. (UEPG) – Seja a soma:
7 8 9 𝑛+1
S = log ( ) + log ( ) + log ( ) + ⋯ + log ( ), onde 𝑛 é um número natural.
6 7 8 𝑛
Qual o valor de 𝑛 para o qual S = 1?”
Primeiramente, passemos a soma S para a notação sigma:
𝑛
7 8 9 𝑛 𝑘+1
S = log ( ) + log ( ) + log ( ) + ⋯ + log ( ) = ∑ log ( ).
6 7 8 𝑛+1 𝑘
6
Agora, a chave deste exercício é o uso das propriedades dos logaritmos,
principalmente:
𝒂
𝐥𝐨𝐠 𝒄 𝒂 − 𝐥𝐨𝐠 𝒄 𝒃 = 𝐥𝐨𝐠 𝒄 ( ), onde (𝒂, 𝒃) > 𝟎, 𝒄 ≠ 𝟏 e 𝒄 > 𝟎 e 𝐥𝐨𝐠 𝒙 = 𝐥𝐨𝐠 𝟏𝟎 𝒙.
𝒃
Então:
𝑘+1
log ( ) = log(𝑘 + 1) − log 𝑘.
𝑘
Assim:
𝑛 𝑛
𝑘+1
∑ log ( ) = ∑ log(𝑘 + 1) − log 𝑘.
𝑘
6 6
Observe que obtivemos o formato da soma telescópica, assim:
𝒃

∑(𝒇(𝒌 + 𝟏) − 𝒇(𝒌)) = 𝒇(𝒃 + 𝟏) − 𝒇(𝒂).


𝒂
𝑛

∑ log(𝑘 + 1) − log 𝑘 = log(𝑛 + 1) − log 6 .


6
O exercício nos pede o valor de 𝑛 para o qual se verifica S = 1.
Assim:
𝑛+1
log(𝑛 + 1) − log 6 = 1; log ( ) = 1;
6
𝑛+1
Da definição de logaritmo: 10 = ⟹ 𝒏 = 𝟓𝟗.
6
“3. (TAIWAN) –
Sabendo-se que 𝑚 é do tipo:
𝑎
𝑚 = − , onde 𝑎 e 𝑏 são números reais, 𝑏 ≠ 0 e que
𝑏
𝑚+5
3
∑ = 2016.
𝑘=1 (3𝑘+4)(3𝑘+7)
Obtenha o valor de:
𝑎 + 𝑏.”
3
Expandindo (3𝑘+4)(3𝑘+7) em frações parciais:
3 A B
(3𝑘+4)(3𝑘+7)
= + ;
3𝑘+4 3𝑘+7
3 A(3𝑘+7)+B(3𝑘+4)
(3𝑘+4)(3𝑘+7)
= (3𝑘+4)(3𝑘+7)
;
3 = A(3𝑘 + 7) + B(3𝑘 + 4).
𝟒
Para obtermos o valor de A, devemos tomar 𝒌 = − , pois:
𝟑
𝟒
B(3𝑘 + 4) = 0, 3𝑘 + 4 = 0, 𝒌 = − .
𝟑
Assim:
4 4
3 = A (3 (− ) + 7) + B (3 (− ) + 4);
3 3
3 = A(− 4 + 7); A = 𝟏.
𝟕
Para obtermos B, devemos tomar 𝒌 = − , pois:
𝟑
𝟕
A(3𝑘 + 7) = 0, 3𝑘 + 7 = 0, 𝒌 = − .
𝟑
Assim:
7 7
3 = A (3 (− ) + 7) + B (3 (− ) + 4);
3 3
3 = B(− 7 + 4); B = − 𝟏.
3
Assim, a fração (3𝑘+4)(3𝑘+7), quando expandida em frações parciais, é tal que:
𝟑 𝟏 (−𝟏) 𝟏 𝟏
= + = − .
(𝟑𝒌 + 𝟒)(𝟑𝒌 + 𝟕) 𝟑𝒌 + 𝟒 𝟑𝒌 + 𝟕 𝟑𝒌 + 𝟒 𝟑𝒌 + 𝟕
Então:
𝑚+5 𝑚+5
3 𝟏 𝟏
∑ = ∑( − ).
(3𝑘 + 4)(3𝑘 + 7) 𝟑𝒌 + 𝟒 𝟑𝒌 + 𝟕
𝑘=1 𝑘=1
Observe que, para obtermos o padrão 𝒇(𝒌 + 𝟏) − 𝒇(𝒌), devemos
manipular algebricamente o fator 𝟑𝒌 + 𝟕.
𝟏
Consideremos 𝒇(𝒌) = , observe que:
𝟑𝒌+𝟒
𝟏 𝟏 𝟏
= = = 𝒇(𝒌 + 𝟏).
𝟑𝒌 + 𝟕 𝟑𝒌 + 𝟒 + 𝟑 𝟑(𝒌 + 𝟏) + 𝟒
Assim:
𝒎+𝟓
𝟏 𝟏 𝒎+𝟓 1
∑ (𝟑𝒌+𝟒 − 𝟑𝒌+𝟕) = ∑𝒌=𝟏 (𝒇(𝒌) − 𝒇(𝒌 + 𝟏)), onde 𝑓(𝑘) = 3𝑘+4.
𝒌=𝟏
Note que obtivemos 𝒇(𝒌) − 𝒇(𝒌 + 𝟏), o oposto de 𝒇(𝒌 + 𝟏) − 𝒇(𝒌).
Então, de acordo com a P2 (vista no item IV), devemos fatorar a constante
multiplicativa – 𝟏, para que obtenhamos o padrão 𝒇(𝒌 + 𝟏) − 𝒇(𝒌) e, então,
apliquemos a Propriedade Telescópica.
Assim:
𝑚+5 𝑚+5
1 1 1 1
∑( − )=−∑( − ).
3𝑘 + 4 3𝑘 + 7 3𝑘 + 7 3𝑘 + 4
𝑘=1 𝑘=1
Mas, temos que:
𝑚+5
1 1
−∑ (3𝑘+7 − 3𝑘+4) = 2016.
𝑘=1
Lembrando que a Propriedade Telescópica é tal que:
𝒃

∑(𝒇(𝒌 + 𝟏) − 𝒇(𝒌)) = 𝒇(𝒃 + 𝟏) − 𝒇(𝒂).


𝒌=𝒂
Aplicando-a, enfim:
𝑚+5
1 1 𝑚+5
−∑ (3𝑘+7 − 3𝑘+4) = − ∑𝑘=1 (𝑓(𝑘 + 1) − 𝑓(𝑘)) = 2016 ;
𝑘=1
1
onde 𝑓(𝑘) = .
3𝑘+4
Então:
𝑚+5
1 1 1 1
−∑( − )=− ( − ) = 2016;
3𝑘 + 7 3𝑘 + 4 3(𝑚 + 5 + 1) (3.1 + 4)
𝑘=1
1 1
−( − ) = 2016;
3(𝑚+6)+4 7
1 1
− = 2016;
7 3𝑚+18+4
1 1
− = 2016;
7 3𝑚+22
𝟏𝟎𝟑𝟒𝟖𝟑
𝒎=− .
𝟏𝟒𝟏𝟏𝟏
𝑎 103483
Se 𝑚 = − = − , então 𝒂 + 𝒃 = 𝟏𝟎𝟑𝟒𝟖𝟑 + 𝟏𝟒𝟏𝟏𝟏 = 𝟏𝟏𝟕𝟓𝟗𝟒.
𝑏 14111
“4. Calcule:
2016
1
∑ .”
1 𝑘(𝑘+1)
1 A B
= + ;
𝑘(𝑘+1) 𝑘 𝑘+1
1 A(𝑘+1)+B𝑘
= ;
𝑘(𝑘+1) 𝑘(𝑘+1)
1 = A(𝑘 + 1) + B𝑘;
P/ 𝑘 = 0;
1 = A(0 + 1);
A = 1.
P/ 𝑘 = − 1;
1 = A(− 1 + 1) + B(− 1);
B = − 1.
1 1 (− 1) 1 1
= + = − ;
𝑘(𝑘+1) 𝑘 𝑘+1 𝑘 𝑘+1
2016 2016
1 1 1
∑ = ∑( − ).
𝑘(𝑘 + 1) 𝑘 𝑘+1
𝑘=1 𝑘=1
Lembrando que:
𝒃

∑(𝒇(𝒌 + 𝟏) − 𝒇(𝒌)) = 𝒇(𝒃 + 𝟏) − 𝒇(𝒂).


𝒌=𝒂
2016 2016
1 1 1 1
∑( − )=−∑( − )
𝑘 𝑘+1 𝑘+1 𝑘
𝑘=1 𝑘=1
1
= − (𝒇(𝒌 + 𝟏) − 𝒇(𝒌)), onde 𝑓(𝑘) = .
𝑘
Assim:
2016
1 1 1 1 1 𝟐𝟎𝟏𝟔
−∑( − )=− ( − )=1− = .
𝑘+1 𝑘 2016 + 1 1 2017 𝟐𝟎𝟏𝟕
𝑘=1
“5. (BRILLIANT) – Calcule o valor da soma:
3 5 7 81
S= + + + ⋯+ .”
1.2.3 2.3.4 3.4.5 40.41.42
Primeiramente, em notação sigma, perceba o padrão:
Temos, no numerador, um número ímpar da forma 2𝑘 + 1, onde 𝑘 = [1, 2, 3, ..., 40],
pois:
𝑘 = 1 ⇒ 𝟐. 𝟏 + 𝟏 = 𝟑;
𝑘 = 2 ⇒ 𝟐. 𝟐 + 𝟏 = 𝟓;
𝑘 = 3 ⇒ 𝟐. 𝟑 + 𝟏 = 𝟕;
...
𝑘 = 40 ⇒ 𝟐. 𝟒𝟎 + 𝟏 = 𝟖𝟏.
No denominador, temos o produto de três números na forma 𝑘(𝑘 + 1)(𝑘 + 2), onde
𝑘 = [1, 2, 3, ..., 40], pois:
𝑘 = 1 ⇒ 𝟏(𝟏 + 𝟏)(𝟏 + 𝟐) = 𝟏. 𝟐. 𝟑;
𝑘 = 2 ⇒ 𝟐(𝟐 + 𝟏)(𝟐 + 𝟐) = 𝟐. 𝟑. 𝟒;
𝑘 = 3 ⇒ 𝟑(𝟑 + 𝟏)(𝟑 + 𝟐) = 𝟑. 𝟒. 𝟓;
...
𝑘 = 40 ⇒ 𝟒𝟎(𝟒𝟎 + 𝟏)(𝟒𝟎 + 𝟐) = 𝟒𝟎. 𝟒𝟏. 𝟒𝟐.
Assim, a soma S é equivalente a:
𝟒𝟎
3 5 7 81 𝟐𝒌 + 𝟏
S= + + + ⋯+ =∑ .
1.2.3 2.3.4 3.4.5 40.41.42 𝒌(𝒌 + 𝟏)(𝒌 + 𝟐)
𝟏
É importante notar que, pela propriedade das frações:
𝒂+𝒃 𝒂 𝒃
= + .
𝒄 𝒄 𝒄
Assim:
𝟒𝟎 𝟒𝟎
𝟐𝒌 + 𝟏 𝒌+𝒌+𝟏
∑ =∑
𝒌(𝒌 + 𝟏)(𝒌 + 𝟐) 𝒌(𝒌 + 𝟏)(𝒌 + 𝟐)
𝟏 𝟏
𝟒𝟎
𝒌 𝒌+𝟏
= ∑[ + ]
𝒌(𝒌 + 𝟏)(𝒌 + 𝟐) 𝒌(𝒌 + 𝟏)(𝒌 + 𝟐)
𝟏
𝟒𝟎 𝟒𝟎 𝟒𝟎
𝟏 𝟏 𝟏 𝟏
= ∑[ + ]=∑ +∑ .
(𝒌 + 𝟏)(𝒌 + 𝟐) 𝒌(𝒌 + 𝟐) (𝒌 + 𝟏)(𝒌 + 𝟐) 𝒌(𝒌 + 𝟐)
𝟏 𝟏 𝟏
A seguir, faremos a decomposição em Frações Parciais de ambas as funções,
1 1
(𝑘+1)(𝑘+2)
e .
𝑘(𝑘+2)
Omitiremos este passo pois, subentende-se que você, leitor, já está familiarizado
com este conceito.
Assim, temos que, em Frações Parciais:
𝟏 𝟏 𝟏
(𝒌+𝟏)(𝒌+𝟐)
= −
𝒌+𝟏 𝒌+𝟐
{ 𝟏 𝟏 𝟏 ;
= −
𝒌(𝒌+𝟐) 𝟐𝒌 𝟐(𝒌+𝟐)
Então:
𝟒𝟎 𝟒𝟎 𝟒𝟎 𝟒𝟎
𝟏 𝟏 𝟏 𝟏 𝟏 𝟏
∑ +∑ = ∑( − ) + ∑( − ).
(𝒌 + 𝟏)(𝒌 + 𝟐) 𝒌(𝒌 + 𝟐) 𝒌+𝟏 𝒌+𝟐 𝟐𝒌 𝟐(𝒌 + 𝟐)
𝟏 𝟏 𝟏 𝟏
Veja que já obtivemos o padrão para o uso da Propriedade Telescópica:
𝒃
∑𝒂(𝒇(𝒌 + 𝟏) − 𝒇(𝒌)) = 𝒇(𝒃 + 𝟏) − 𝒇(𝒂).
Observe que:
𝒃

∑(𝒇(𝒌 + 𝟏) − 𝒇(𝒌)) = 𝒇(𝒃 + 𝟏) − 𝒇(𝒂)


𝒂
𝒃

⇒ − ∑(𝒇(𝒌 + 𝟏) − 𝒇(𝒌)) = 𝒇(𝒂) − 𝒇( 𝒃 + 𝟏)


𝒂
𝒃

⇒ ∑(𝒇(𝒌) − 𝒇(𝒌 + 𝟏)) = 𝒇(𝒂) − 𝒇(𝒃 + 𝟏) .


𝒂
Então:
𝟒𝟎
𝟏 𝟏 1
∑ (𝒌+𝟏 − 𝒌+𝟐) = 𝒇(𝟏) − 𝒇(𝟒𝟎 + 𝟏), onde 𝑓(𝑘) = 𝑘+1.
𝟏
Assim, a primeira soma é:
𝟒𝟎
𝟏 𝟏 𝟏 𝟏 𝟏 𝟏
∑ (𝒌+𝟏 − 𝒌+𝟐) = 𝒇(𝟏) − 𝒇(𝟒𝟏) = 𝟏+𝟏 − 𝟏+𝟒𝟏 = 𝟐 − 𝟒𝟐.
𝟏
A segunda soma é tal que:
𝟒𝟎
𝟏 𝟏
∑( − ).
𝟐𝒌 𝟐(𝒌 + 𝟐)
𝟏
Para obtermos o padrão da Soma Telescópica, temos que somar e subtrair o fator
1
, da seguinte maneira:
2(𝑘+1)
𝟒𝟎 𝟒𝟎
𝟏 𝟏 𝟏 𝟏 𝟏 𝟏
∑( − ) = ∑ [( − )+( − )] .
𝟐𝒌 𝟐(𝒌 + 𝟐) 𝟐𝒌 𝟐(𝒌 + 𝟏) 𝟐(𝒌 + 𝟏) 𝟐(𝒌 + 𝟐)
𝟏 𝟏
Assim, utilizando a P3 (somatório de uma adição), obtemos:
𝟒𝟎
𝟏 𝟏 𝟏 𝟏
∑ [( − )+( − )]
𝟐𝒌 𝟐(𝒌 + 𝟏) 𝟐(𝒌 + 𝟏) 𝟐(𝒌 + 𝟐)
𝟏
𝟒𝟎 𝟒𝟎
𝟏 𝟏 𝟏 𝟏
= ∑( − ) + ∑( − ).
𝟐𝒌 𝟐(𝒌 + 𝟏) 𝟐(𝒌 + 𝟏) 𝟐(𝒌 + 𝟐)
𝟏 𝟏
Observe que já obtivemos o padrão da Soma Telescópica, assim:
40
𝟏 𝟏 1
∑ (𝟐𝒌 − 𝟐(𝒌+𝟏)) = 𝒇(𝟏) − 𝒇(𝟒𝟎 + 𝟏), onde 𝑓(𝑘) = 2𝑘.
1
Deste modo:
40
𝟏 𝟏 1 1
∑( − )= − .
𝟐𝒌 𝟐(𝒌 + 𝟏) 2 82
1
E, para a segunda parcela:
𝟒𝟎
𝟏 𝟏 1
∑ (𝟐(𝒌+𝟏) − 𝟐(𝒌+𝟐)) = 𝒇(𝟏) − 𝒇(𝟒𝟎 + 𝟏), onde 𝑓(𝑘) = 2(𝑘+1).
𝟏
Igualmente:
𝟒𝟎
𝟏 𝟏 𝟏 𝟏 𝟏 𝟏
= ∑( − )= − = − .
𝟐(𝒌 + 𝟏) 𝟐(𝒌 + 𝟐) 𝟐(𝟐) 𝟐(𝟒𝟏 + 𝟏) 𝟒 𝟖𝟒
𝟏
Finalmente, temos que a soma pedida é:
40 𝟒𝟎 𝟒𝟎
2𝑘 + 1 𝟏 𝟏 𝟏 𝟏
∑ = ∑( − ) + ∑( − )
𝑘(𝑘 + 1)(𝑘 + 2) 𝒌+𝟏 𝒌+𝟐 𝟐𝒌 𝟐(𝒌 + 𝟐)
1 𝟏 𝟏
𝟒𝟎 𝟒𝟎 𝟒𝟎
𝟏 𝟏 𝟏 𝟏 𝟏 𝟏
= ∑( − ) + ∑( − )+ ∑( − )
𝒌+𝟏 𝒌+𝟐 𝟐𝒌 𝟐(𝒌 + 𝟏) 𝟐(𝒌 + 𝟏) 𝟐(𝒌 + 𝟐)
𝟏 𝟏 𝟏
1 1 1 1 1 1 𝟑𝟒𝟓
=( − )+ ( − )+ ( − )= .
2 42 2 82 4 84 𝟐𝟖𝟕
Exercício de altíssimo nível, pois exige um grande conhecimento algébrico, como, por
exemplo, o conhecimento das Frações Parciais,
propriedades operatórias dos somatórios, manipulação numérica e a Propriedade
Telescópica, além de uma enorme carga aritmética.
Podemos afirmar que, apenas por meio dos métodos convencionais da álgebra dos
somatórios, este exemplo é impraticável, fazendo do conhecimento da
Propriedade Telescópica algo imprescindível.
Este exercício é um exemplo típico de atividade em nível olímpico.
“6. (PUTNAM – EUA/CANADÁ) – Prove que:
1 1 1
+ + ⋯+ = 9.”
1+√2 √2+√3 √99+√100
Veja que esta soma é tal que:
99 99
1 1 1 1 1
+ +⋯+ =∑ =∑ .
1 + √2 √2 + √3 √99 + √100 𝑘=1
√𝑘 + √𝑘 + 1 𝑘=1
√𝑘 + 1 + √𝑘
Multiplicando numerador e denominador por (√𝑘 + 1 − √𝑘) (raízes conjugadas),
obtemos:
99 99 99
1 √𝑘 + 1 − √𝑘
√𝑘 + 1 − √𝑘
∑ . =∑ = ∑ √𝑘 + 1 − √𝑘.
√ 𝑘 + 1 + √𝑘 √ 𝑘 + 1 − √𝑘 𝑘=1 𝑘 + 1 − 𝑘
𝑘=1 𝑘=1
Observe que já obtivemos o padrão 𝑓(𝑘 + 1) − 𝑓(𝑘) para a aplicação da Soma
Telescópica.
Então, temos que:
𝒃

∑(𝒇(𝒌 + 𝟏) − 𝒇(𝒌)) = 𝒇(𝒃 + 𝟏) − 𝒇(𝒂).


𝒂
Assim:
99
∑ √𝑘 + 1 − √𝑘 = 𝑓(100) − 𝑓(1), onde 𝑓(𝑘) = √𝑘.
𝑘=1
Então,
99
∑ √𝑘 + 1 − √𝑘 = √100 − √1 = 10 − 1 = 𝟗.
𝑘=1
Q.E.D.

“7. (BRILLIANT – ADAPTADA) – Dada a função 𝑓(𝑘) = 4𝑘 2 − 1,


1 1 1 1
obtenha o valor de + + + ⋯+ .”
𝑓(1) 𝑓(2) 𝑓(3) 𝑓(100)
Nos foi fornecida a função 𝑓(𝑘) = 4𝑘 2 − 1.
1 1 1
Seja S = + + ⋯+ .
𝑓(1) 𝑓(2) 𝑓(100)
Temos que:
100 100 100
1 1 1
S=∑ =∑ =∑ .
1 𝑓(𝑘) 1 4𝑘 2 −1 1 (2𝑘+1)(2𝑘−1)
Expandindo o denominador em Frações Parciais (este passo será omitido, novamente),
obtemos:
1 1 1
(2𝑘+1)(2𝑘−1)
= − .
2(2𝑘−1) 2(2𝑘+1)
1 1 1 1
− = − .
2(2𝑘−1) 2(2𝑘+1) 4𝑘−2 4𝑘+2
Considerando a função 𝑔(𝑘) = 4𝑘 − 2, temos que 𝑔(𝑘 + 1) = 4(𝑘 + 1) − 2
= 4𝑘 + 4 − 2 = 4𝑘 + 2.
Ou seja, já obtivemos o padrão para o uso da Propriedade Telescópica.
Assim:
100 100 100
1 1 1 1 1
∑ = ∑( − ) = ∑( − ).
(2𝑘 + 1)(2𝑘 − 1) 2(2𝑘 − 1) 2(2𝑘 + 1) 4𝑘 − 2 4𝑘 + 2
1 1 1
Enfim:
𝟏𝟎𝟎
𝟏 𝟏 𝟏 𝟏 𝟏 𝟏 𝟏𝟎𝟎
∑( − )= − = − = .
𝟒𝒌 − 𝟐 𝟒𝒌 + 𝟐 𝟒. 𝟏 − 𝟐 𝟒. 𝟏𝟎𝟏 − 𝟐 𝟐 𝟒𝟎𝟐 𝟐𝟎𝟏
𝟏
“8. (EUA) – Obtenha o valor de:
2016
1
∑ ln (1 + ) . "
𝑘
1
∗ 𝐥𝐧 𝒙 = 𝐥𝐨𝐠 𝒆 𝒙.
𝟏 𝒌+𝟏
Temos que 𝐥𝐧 (𝟏 + ) = 𝐥𝐧 ( ) = 𝐥𝐧(𝒌 + 𝟏) − 𝐥𝐧(𝒌).
𝒌 𝒌
Assim:
2016 2016
1
∑ ln (1 + ) = ∑ (ln(𝑘 + 1) − ln(𝑘)).
𝑘
1 1
Como já obtivemos o padrão da Soma Telescópica: 𝑓(𝑘 + 1) − 𝑓(𝑘), onde
𝑓(𝑘) = ln 𝑘, basta utilizarmos a propriedade:
𝑏

∑(𝑓(𝑘 + 1) − 𝑓(𝑘)) = 𝑓(𝑏 + 1) − 𝑓(𝑎).


𝑎
Então:
2016

∑ (ln(𝑘 + 1) − ln(𝑘)) = 𝐥𝐧(𝟐𝟎𝟏𝟔 + 𝟏) − 𝐥𝐧 𝟏 = ln(2017) − 0 = 𝐥𝐧(𝟐𝟎𝟏𝟕) .


1
“9. Encontre o valor da soma abaixo:
1 2 3 1
+ + +⋯+ .”
1+12 +14 1+22 +24 1+32 +34 1+502 +504
Veja que, no denominador, temos o número 1 em todas as parcelas da soma.
Assim, este é uma constante e todos os outros números são variáveis.
Então:
50
1 2 3 50 𝑘
+ + +⋯+ =∑ .
1+12 +14 1+22 +2 4 1+32 +3 4 1+502 +504 2
𝑘=1 1+𝑘 +𝑘
4
Façamos a seguinte manipulação algébrica:
50 50 50
𝑘 𝑘 𝑘
∑ = ∑ = ∑
1 + 𝑘2 + 𝑘4 1 + 𝑘2 + 𝑘4 + 𝑘2 − 𝑘2 1 + 2𝑘 2 + 𝑘 4 − 𝑘 2
𝑘=1 𝑘=1 𝑘=1
50 50
𝑘 𝑘
=∑ = ∑
(1 + 𝑘 2 )2 − 𝑘² (1 + 𝑘 2 + 𝑘)(1 + 𝑘 2 − 𝑘)
1 1
50
𝑘
=∑ 2 2
.
1 (𝑘 +𝑘+1)(𝑘 −𝑘+1)
𝑘
A função (𝑘 2 , quando decomposta em Frações Parciais, se torna:
+𝑘+1)(𝑘 2 −𝑘+1)
𝒌 𝟏−𝒌 𝒌+𝟏
(𝒌𝟐 +𝒌+𝟏)(𝒌𝟐 −𝒌+𝟏)
= − .
𝟐(𝒌𝟐 −𝒌+𝟏) 𝟐(𝒌𝟐 +𝒌+𝟏)
Assim:
50 50
𝑘 1−𝑘 𝑘+1
∑ 2 = ∑ [ − ]
(𝑘 + 𝑘 + 1)(𝑘 2 − 𝑘 + 1) 2(𝑘 2 − 𝑘 + 1) 2(𝑘 2 + 𝑘 + 1)
1 1
50 50
1 1−𝑘 1 𝑘+1 1 1−𝑘 𝑘+1
= ∑[ + ] = ∑ [ + ]
2 (𝑘 2 − 𝑘 + 1) 2 (𝑘 2 + 𝑘 + 1) 2 𝑘² − 𝑘 + 1 𝑘² + 𝑘 + 1
1 1
50
1 1−𝑘 𝑘+1 𝟏 𝟏
= ∑ [ + ] = (𝟏 − ).
2 𝑘²−2𝑘+1+𝑘
1 𝑘²+2𝑘+1−𝑘 𝟐 𝟓𝟎.𝟓𝟎−𝟏
“10. (OBM) – A soma
1 1 1 1
+ + + ⋯+
1.4 4.7 7.10 2008.2011
𝑝
pode ser escrita sob a forma de fração irredutível .
𝑞
Obtenha o valor de 𝑝 + 𝑞.”
Veja que:
670
1 1 1 1 1
+ + +⋯+ =∑
1.4 4.7 7.10 2008.2011 (3𝑘 − 2)(3𝑘 + 1)
1
670
1 1
=∑ (3(3𝑘−2) − 3(3𝑘+1)).
𝑘=1
1
Sendo a função 𝑓(𝑘) = , temos que esta soma é tal que:
9𝑘−6
670
1 1 1 1 𝟐𝟕𝟎
∑ (3(3𝑘−2) − 3(3𝑘+1)) = 𝑓(1) − 𝑓(670) = 3 − 9.670−6 = 𝟐𝟎𝟏𝟏.
𝑘=1
𝒑 + 𝒒 = 𝟐𝟕𝟎 + 𝟐𝟎𝟏𝟏 = 𝟐𝟔𝟖𝟏.

“11. (OBM – NÍVEL UNIVERSITÁRIO) – Calcule o valor da soma:


1 1 1 1
+ + +⋯+ .”
1.4 4.7 7.10 2998.3001
Insistimos no fato de que exercícios deste tipo estão intimamente relacionados com
os conceitos de sequência, principalmente com as Progressões Aritméticas.
Pois, para determinarmos sua recorrência, seus limites, principalmente o limite
superior, é necessário o uso das fórmulas da teoria das Progressões Aritméticas, as
quais se encontram no apêndice do capítulo 3.
Dito isto, temos a seguinte soma:
1 1 1 1
+ + +⋯+ .
𝟏. 𝟒 𝟒. 𝟕 𝟕. 𝟏𝟎 𝟐𝟗𝟗𝟖. 𝟑𝟎𝟎𝟏
Perceba que, nos denominadores, temos as seguintes progressões aritméticas, ambas
de razão 3:
I: (𝟏, 𝟒, 𝟕,…, 𝟐𝟗𝟗𝟖);
II: (𝟒, 𝟕, 𝟏𝟎,…, 𝟑𝟎𝟎𝟏).
O termo geral da primeira P.A é:
I: 𝒂𝒏 = 𝒂𝟏 + (𝒏𝒂 − 𝟏)𝒓 ⟹ 𝑎𝑛 = 1 + (𝑛𝑎 − 1)3.
𝑎𝑛 = 1 + 3𝑛𝑎 − 3 = 𝟑𝒏𝒂 − 𝟐.
O termo geral da segunda P.A é:
II: 𝒃𝒏 = 𝒃𝟏 + (𝒏𝒃 − 𝟏)𝒓 ⟹ 𝑏𝑛 = 4 + (𝑏 − 1)3.
𝑏𝑛 = 4 + 3𝑛𝑏 − 3 = 𝟑𝒏𝒃 + 𝟏.
Agora, é necessário verificar se o número de termos das duas progressões são iguais:
I: Temos que o último termo da primeira progressão é 2998 ⟹ 𝒂𝒏 = 𝟑𝒏𝒂 − 𝟐;
2998 = 3𝑛𝑎 − 2;
𝑛𝑎 = 1000.
II: O último termo da segunda progressão é 3001 ⟹ 𝒃𝒏 = 𝟑𝒏𝒃 + 𝟏;
3001 = 3𝑛𝑏 + 1;
𝑛𝑏 = 1000.
𝒏𝒂 = 𝒏𝒃 = 𝒏.
Como verificamos que o número de termos é igual para as duas sequências, finalmente
temos que:
𝟏𝟎𝟎𝟎
𝟏 𝟏 𝟏 𝟏 𝟏
+ + + ⋯+ = ∑ .
𝟏. 𝟒 𝟒. 𝟕 𝟕. 𝟏𝟎 𝟐𝟗𝟗𝟖. 𝟑𝟎𝟎𝟏 (𝟑𝒏 − 𝟐)(𝟑𝒏 + 𝟏)
𝒏=𝟏
𝟏
Assim, expandindo (𝟑𝒏−𝟐)(𝟑𝒏+𝟏) em frações parciais, obtemos:
𝟏 𝟏 𝟏
= − .
(𝟑𝒏 − 𝟐)(𝟑𝒏 + 𝟏) 𝟑(𝟑𝒏 − 𝟐) 𝟑(𝟑𝒏 + 𝟏)
Assim:
𝟏𝟎𝟎𝟎 𝟏𝟎𝟎𝟎
𝟏 𝟏 𝟏
∑ = ∑( − ).
(𝟑𝒏 − 𝟐)(𝟑𝒏 + 𝟏) 𝟑(𝟑𝒏 − 𝟐) 𝟑(𝟑𝒏 + 𝟏)
𝒏=𝟏 𝒏=𝟏
1
Seja 𝑓(𝑛) = , percebamos que:
3(3𝑛−2)
𝟏 𝟏 𝟏
𝒇(𝒏 + 𝟏) = = = .
𝟑(𝟑(𝒏 + 𝟏) − 𝟐) 𝟑(𝟑𝒏 + 𝟑 − 𝟐) 𝟑(𝟑𝒏 + 𝟏)
Assim, obtivemos o padrão para a aplicação da Propriedade Telescópica.
𝟏𝟎𝟎𝟎
𝟏 𝟏
∑ (𝟑(𝟑𝒏−𝟐) − 𝟑(𝟑𝒏+𝟏)) = 𝒇(𝒏) − 𝒇(𝒏 + 𝟏) = 𝒇(𝟏) − 𝒇(𝟏𝟎𝟎𝟎 + 𝟏)
𝒏=𝟏
1
= 𝒇(𝟏) − 𝒇(𝟏𝟎𝟎𝟏), onde 𝑓(𝑛) = .
3(3𝑛−2)
𝟏𝟎𝟎𝟎
𝟏 𝟏 𝟏 𝟏
∑( − )= −
𝟑(𝟑𝒏 − 𝟐) 𝟑(𝟑𝒏 + 𝟏) 𝟑(𝟑. 𝟏 − 𝟐) 𝟑(𝟑. 𝟏𝟎𝟎𝟏 − 𝟐)
𝒏=𝟏
𝟏 𝟏 𝟏𝟎𝟎𝟎
−= = .
𝟑 𝟑(𝟑𝟎𝟎𝟏) 𝟑𝟎𝟎𝟏
“12. (IIT – ÍNDIA) – Calcule o valor de:
1 1 1
+ + ⋯+ .”
2√1+√2 3√2+2√3 100√99+99√100
Perceba que:
1 1 1
+ + ⋯+
2√1 + √2 3√2 + 2√3 100√99 + 99√100
99
1
=∑ .
1
(𝑘 + 1)√𝑘 + 𝑘√𝑘 + 1
99 99
1 1
∑ =∑
1
(𝑘 + 1)√𝑘 + 𝑘√𝑘 + 1 1
√𝑘 + 1√𝑘 + 1√𝑘 + √𝑘√𝑘√𝑘 + 1
* Suporemos que você, leitor, está familiarizado com as propriedades da
radiciação.
99 99
1 1 √𝑘 + 1 − √𝑘
=∑ =∑ .
1
√𝑘√𝑘 + 1(√𝑘 + 1 + √𝑘) 1
√𝑘√𝑘 + 1(√𝑘 + 1 + √𝑘) √𝑘 + 1 − √𝑘
99 99
√𝑘 + 1 − √𝑘 √𝑘 + 1 − √𝑘
=∑ =∑
1
√𝑘√𝑘 + 1(𝑘 + 1 − 𝑘) 1
√𝑘√𝑘 + 1
99 99
√𝑘 + 1 √𝑘 1 1
= ∑( − ) = ∑( − ).
1
√𝑘√𝑘 + 1 √𝑘√𝑘 + 1 1
√𝑘 √ 𝑘 + 1
Já obtivemos o padrão para a aplicação da Propriedade Telescópica.
1
Sendo 𝑓(𝑘) = , temos que:
√𝑘
𝒃

∑(𝒇(𝒌) − 𝒇(𝒌 + 𝟏)) = 𝒇(𝒂) − 𝒇(𝒃 + 𝟏) .


𝒂
Assim:
𝟗𝟗
𝟏 𝟏 𝟏 𝟏 𝟏 𝟗
∑( − ) = 𝒇(𝟏) − 𝒇(𝟏𝟎𝟎) = − =𝟏− = .
√𝒌 √𝒌 + 𝟏 √𝟏 √𝟏𝟎𝟎 𝟏𝟎 𝟏𝟎
𝟏

O último item deste capítulo de Somatórios contém os exercícios propostos a serem feitos, com um
enfoque maior nas propriedades e, principalmente, na Propriedade Telescópica e no conceito de
limite de uma soma, ou soma dos infinitos termos de uma progressão geométrica (Série Geométrica).
A seguir faremos uma introdução às Séries, porém sem o emprego das rigorosas definições de
Limites e Integrais, as quais permeiam o estudo das mesmas.
Faremos uma definição intuitiva de Limite, sem nos aprofundarmos nesta, e apresentaremos, de
maneira intuitiva, o conceito de Série, com um foco especial nas Séries Geométricas.

VI. Introdução às Séries e o Limite de uma soma


1. Noção intuitiva de Série Matemática
Consideremos um antigo exemplo, que data da Grécia Pré-Socrática.
O filósofo Zeno, famoso por sua formulação rigorosa dos denominados Paradoxos,
intrigou a comunidade grega ao apresentar o seguinte problema:
“O herói Aquiles é desafiado por uma tartaruga a uma corrida pelo território grego.
Por ser obviamente mais lenta que o mítico guerreiro Aquiles, a tartaruga começará a
corrida um trecho à frente deste e, além disto, Aquiles percorrerá sempre metade da
distância percorrida pela tartaruga, da seguinte maneira:
𝑑
A tartaruga percorre uma distância 𝑑, então Aquiles percorrerá uma distância .
2
𝑑 𝑑
A tartaruga percorre uma distância , então Aquiles percorrerá uma distância .
2 4
𝑑 𝑑
A tartaruga percorre uma distância , então Aquiles percorrerá uma distância .
4 8
...
𝑑 𝑑
A tartaruga percorre uma distância , então Aquiles percorrerá uma distância , onde
2𝑛 4𝑛
𝑛 é um natural não nulo.
E assim por diante, num processo infinito.”
A comunidade grega, estarrecida com o paradoxo proposto por Zeno, concluiu que
Aquiles nunca alcançará a tartaruga, pois quando este passa por uma posição
hipotética A, a tartaruga se encontra na posição B, e quando Aquiles chega em B, a
tartaruga está em C, e assim por diante, até o infinito.
Atualmente, este problema conta com inúmeras propostas de solução, matemáticas,
físicas (a citar a proposta extremamente rigorosa da Mecânica Quântica) e etc.
O maior avanço com respeito a este problema ocorre com o advento dos limites, os
quais estabelecem que, a soma das distâncias percorridas por Aquiles é tal que:
1 1 1 1 1 1 1 1 1 1 1
S𝑛 = + + + + + + + + + + ⋯+ + ⋯ = 𝟏.
2 4 8 16 32 64 128 256 512 1024 2𝑛
Ou seja, o limite desta soma, quando o número de termos desta se aproxima do
infinito, é igual a 1.
Empregando a linguagem matemática, temos que 𝐥𝐢𝐦𝒏→∞ 𝐒𝒏 = 𝟏.
Isto significa que, em algum momento, Aquiles alcançará a tartaruga, pois percorre as
distâncias de tal forma que:
𝟏 𝟏 𝟏 𝟏 𝟏 𝟏 𝟏 𝟏
𝒅 + 𝒅 + 𝒅 + ⋯+ 𝒅 + ⋯ = 𝒅( + + + ⋯+ 𝒅 + ⋯ ) = 𝒅. 𝐒𝒏 = 𝒅.
𝟐 𝟒 𝟖 𝟐𝒏 𝟐 𝟒 𝟖 𝟐𝒏
E assim, Aquiles percorrerá a mesma distância 𝑑 que a tartaruga, consequentemente a
alcançando.
Este paradoxo é uma interessante introdução às Séries.
2. Definição de Série Matemática
Utilizamos o nome Série para nos referirmos a uma soma cujo limite superior é o
infinito.
Como no exemplo acima, a Série em questão é:
∞ ∞
𝟏
∑ ( 𝒌 ) = ∑ 𝟐−𝒌 .
𝟐
𝟏 𝟏
Também são exemplos de Séries:

1. ∑0 5−𝑘 ;
1
2. ∑∞
0 𝑘!;
1
3. ∑∞
1 .
√𝑘
Em geral, uma Série é uma soma do tipo:

S𝑛 = ∑ 𝒇(𝒌) .
𝒂
Uma importante consideração a respeito das Séries é o Critério de Convergência.
I. Dizemos que uma Série é convergente, se e somente se, existe um único número 𝒑
para o qual esta tende (se aproxima de maneira infinitesimal, porém nunca
chegando, de fato, a este valor).
Matematicamente:
𝐒𝒏 é convergente ⇔ ∃ 𝒑: 𝐥𝐢𝐦𝒏→∞ 𝐒𝒏 = 𝒑.
II. Dizemos que uma Série é divergente, se e somente se, não existe um número 𝒑
para o qual esta tende.
Matematicamente:
𝐒𝒏 é divergente ⇔ ∄ 𝒑: 𝐥𝐢𝐦𝒏→∞ 𝐒𝒏 = 𝒑.
É claro que, para efeitos de simplificação, omitimos os detalhes mais rigorosos para a
classificação das Séries, os quais serão estudados na disciplina de Cálculo, nas
universidades.
Os tipos de Séries com os quais iremos nos preocupar nesta teoria são as denominadas
Séries Geométricas, que, como o próprio nome sugere, são Séries relacionadas às
Progressões Geométricas.
Assumiremos que você, caro leitor, está familiarizado com o conceito geral de uma
Progressão Geométrica, principalmente com a soma dos termos desta, pois não iremos
nos preocupar com a demonstração da mesma.
Aprendemos que a soma 𝑆𝑛 de uma progressão geométrica de 𝑛 termos (sendo,
portanto, finita) é dada pela fórmula:
𝒂𝟏 (𝒒𝒏 −𝟏)
𝐒𝒏 = , onde 𝒒 é a razão da progressão geométrica.
𝒒−𝟏
Como frisamos anteriormente, esta é a fórmula da soma dos 𝑛 termos de uma
progressão geométrica, com 𝒏 ∈ [𝟏, ∞[, ou 𝑛 ≪ ∞.
No caso de uma Série Geométrica, devemos ter que |𝒒| < 𝟏, para que esta seja
convergente.
Atenção leitor, utilizaremos a noção de Limite para demonstrarmos a fórmula de uma
Série Geométrica, portanto, não se preocupe com este conceito, pois ficará mais claro
com o decorrer da teoria.
Consideremos a seguinte P.G: (𝑎1 , 𝑎2 , 𝑎3 , ..., 𝑎𝑛 , ...) cujos termos vão até o infinito.
Só faz sentido falarmos na soma desta P.G, se e somente se, sua razão tiver módulo
menor que um, pois:
Sabemos que a soma desta P.G é dada por
𝒂𝟏 (𝒒𝒏 −𝟏)
𝐒𝒏 = , porém, se esta P.G contém infinitos termos, esta não é a fórmula a
𝒒−𝟏
ser aplicada para o cômputo da mesma.
A fórmula a ser aplicada é obtida se fizermos sua razão se aproximar cada vez mais de
zero e, para isso, devemos fazer o expoente 𝑛 tender a infinito, da seguinte maneira:
𝒂𝟏 (𝒒𝒏 − 𝟏)
𝐥𝐢𝐦 𝐒𝒏 = 𝐥𝐢𝐦 ( ).
𝒏→∞ 𝒏→∞ 𝒒−𝟏
Se a razão 𝑞 for um número cujo módulo é maior do que um, esta soma será
estritamente divergente.
Porém, como já explicitamos que |𝑞| < 1, faz sentido utilizarmos o limite tendendo ao
infinito pois, um número muito próximo de zero, quando elevado a uma potência
muito grande, tende a zero.
Por exemplo, o número 0,00000001 = 10−8 , quando elevado a um número 𝑛
muito grande, por exemplo, para 𝑛 = 1099 , se torna:
1 1099 𝟏
(10 −8 )1099
=( 8) = 𝟏𝟎𝟗𝟗
.
10
(𝟏𝟎𝟖 )
É fácil perceber que o denominador da fração obtida é muito próximo de zero, então
dizemos que ele tende a zero.
Dito isto, a demonstração da fórmula de uma Série Geométrica:
𝒂𝟏 (𝒒𝒏 −𝟏)
𝐥𝐢𝐦𝒏→∞ (
𝒒−𝟏
) , |𝒒| < 𝟏;
𝒂𝟏 (𝒒𝒏 − 𝟏) 𝒂𝟏 (𝟎 − 𝟏) 𝒂𝟏
𝐥𝐢𝐦 ( ) = 𝐥𝐢𝐦 ( )= .
𝒏→∞ 𝒒−𝟏 𝒏→∞ 𝒒−𝟏 𝟏−𝒒
Portanto, a fórmula de uma Série Geométrica de razão 𝑞, cujo módulo é menor do que
um, é:

𝒂𝟏
𝐒𝒏 = ∑ 𝒂 𝒏 = .
𝟏−𝒒
𝟏
Nossa teoria acerca das Séries termina aqui, pois, os outros tipos de Séries, como as
Séries de Fourier, Séries de Taylor – MacLaurin e etc, fogem do escopo desta.
No próximo subitem, veremos a definição do Limite de uma soma, lembrando que
será apenas uma definição intuitiva, propondo uma melhor compreensão sobre a ideia
de Limite, sem nos aprofundarmos neste.
3. Intuição de Limite
Imaginemos a função 𝑓(𝑥) = 4𝑥 − 3.
O comportamento desta função, quando 𝑥 → 1 pode ser definido como o limite desta
função quando seu valor 𝑥 tende ao número 1.
Matematicamente:
𝐥𝐢𝐦(𝟒𝒙 − 𝟑) .
𝒙→𝟏
Quando tomamos 𝑥 = 0,9, temos que:
𝒇(𝟎, 𝟗) = 𝟒(𝟎, 𝟗) − 𝟑 = 𝟎, 𝟔.
Quando tomamos 𝑥 = 0,99, temos que:
𝒇(𝟎, 𝟗𝟗) = 𝟒(𝟎, 𝟗𝟗) − 𝟑 = 𝟎, 𝟗𝟔.
Quando tomamos 𝑥 = 0,999, temos que:
𝒇(𝟎, 𝟗𝟗𝟗) = 𝟒(𝟎, 𝟗𝟗𝟗) − 𝟑 = 𝟎, 𝟗𝟗𝟔.
Quando tomamos 𝑥 = 0,9999, temos que:
𝒇(𝟎, 𝟗𝟗𝟗𝟗) = 𝟒(𝟎, 𝟗𝟗𝟗𝟗) − 𝟑 = 𝟎, 𝟗𝟗𝟗𝟔.
Percebemos que, conforme o argumento 𝒙 se aproxima de 𝟏, o valor de 𝒇(𝒙) se
aproxima do valor 1.
Ou seja, dizemos que o limite para o qual a função 𝒇(𝒙) = 𝟒𝒙 − 𝟑 tende quando 𝒙
se aproxima, de maneira infinitesimal, do número 1, é 1:
𝐥𝐢𝐦(𝟒𝒙 − 𝟑) = 𝟏.
𝒙→𝟏
Desta forma, o conceito de Limite pode ser entendido como um valor L para o qual
uma função 𝒇(𝒙) qualquer tende quando seu argumento 𝒙 se aproxima de um 𝒂
qualquer, pertencente ao domínio de 𝒇, sendo 𝑓 uma função contínua.
E, quando estas condições são satisfeitas, dizemos que o limite existe e é tal que:
𝐥𝐢𝐦 𝒇(𝒙) = 𝐋.
𝒙→𝒂
A seguir, veremos este conceito aplicado às somas, também de maneira bem intuitiva
e de fácil entendimento.

4. Limite de uma soma


Consideremos a seguinte soma:
𝑏

S = ∑ 𝑓(𝑘).
𝑘=𝑎
Dizemos que a soma S possui como limite um número L, quando 𝑘 se aproxima de um
número 𝑝, se e somente se:
𝐥𝐢𝐦 𝐒 = 𝐋.
𝒌→𝒑
De todo o conteúdo tratado neste capítulo VI, o conceito mais importante e que será cobrado nos
exercícios é o conceito de Soma dos infinitos termos de uma Progressão Geométrica, ou
simplesmente Limite de uma Série Geométrica.
Os outros conceitos apresentados neste capítulo, como a intuição de limite e o limite de uma soma
não serão tratados com profundidade, com exceção da Série Telescópica, a qual será apresentada
num apêndice no capítulo 3.
VII. Exercícios resolvidos
1. Notação Sigma
⊕ − 1. Passe para a notação Sigma as seguintes somas:
a) 1 + 4 + 9 + ⋯ + 𝑛²;
1 1 1 1
b) 1 + + + + ⋯ + ;
2 3 4 𝑛
c) 0 + 0 + 0 + ⋯;
𝜋 𝜋 𝜋
d) 𝑡𝑔 𝜋 + 𝑡𝑔 + 𝑡𝑔 + ⋯ + 𝑡𝑔 ;
3 9 3𝑛
e) ln 1 + ln 2 + ln 3 + ⋯ + ln(𝑛 − 1);
f) 1 + 4 + 16 + ⋯ + 4𝑛 .

R:
Temos que:
𝑛
a) 1 + 4 + 9 + ⋯ + 𝑛2 = ∑1 𝑘²;
𝑛
1 1 1 1 1
b) 1 + + + + ⋯ + = ∑ ;
2 3 4 𝑛 1𝑘
c) 0 + 0 + 0 + ⋯ = ∑𝑛0 0;
𝑛
𝜋 𝜋 𝜋 𝜋
d) 𝑡𝑔 𝜋 + 𝑡𝑔 + 𝑡𝑔 + ⋯ + 𝑡𝑔 = ∑ 𝑡𝑔 𝑘 ;
3 9 3𝑛
0 3

e) ln 1 + ln 2 + ln 3 + ⋯ + ln(𝑛 − 1) = ∑𝑛−1
1 ln 𝑘;
𝑛
f) 1 + 4 + 16 + ⋯ + 4𝑛 = ∑0 4𝑘 .

⊕⊕ − 2. Calcule o número de termos das seguintes somas:


a) ∑𝑚−8
7 𝑎, 𝑎 ∈ ℝ∗+ ;
b) ∑128
1 3;
c) ∑17
1 (3 + 𝑘𝑖), 𝑖 = √− 1.

R:
a) Temos a soma da constante 𝑎.
Do termo geral das progressões aritméticas, temos que:
𝑎𝑛 = 𝑎1 + (𝑛 − 1)𝑟 ⇒ 𝑚 − 8 = 7 + (𝑛 − 1);
𝑛 = 𝑚 − 14 termos.
b) Temos a soma da constante 3.
128 = 1 + (𝑛 − 1);
𝑛 = 128 termos.
c) Temos a seguinte soma:
(3 + 𝑖) + (3 + 2𝑖) + (3 + 3𝑖) + ⋯ + (3 + 17𝑖).
É fácil concluir que o número de termos 𝑛 é dado por:
3 + 17𝑖 = (3 + 𝑖) + (𝑛 − 1)𝑖;
3 + 17𝑖 = 3 + 𝑖 + 𝑛. 𝑖 − 𝑖; 𝟑 + 𝟏𝟕𝒊 = 𝟑 + 𝒏. 𝒊.
Da igualdade dos números complexos, temos que dois números complexos 𝑧 e 𝑤 são
iguais se, e somente se, Re(𝑧) = Re(𝑤) e Im(𝑧) = Im(𝑤).
Assim, temos os complexos 3 + 17𝑖 e 3 + 𝑛. 𝑖.
3 + 17𝑖 = 3 + 𝑛. 𝑖;
3 = 3; Re(3 + 17𝑖) = Re(3 + 𝑛. 𝑖);
𝑛 = 17; Im(3 + 17𝑖) = Im(3 + 𝑛. 𝑖);
Logo, 𝒏 = 𝟏𝟕.

⊕⊕⊕ − 3. Mostre que:


𝑛−1
1
𝑎) ∑ (𝑎 + 𝑏 + 𝑐)𝑘 = (𝑎 + 𝑏 + 𝑐) (𝑛2 − 𝑛), onde 𝑎, 𝑏 e 𝑐 são constantes reais
𝑘=1 2
não nulas.
𝑚+𝑛
𝑘 (𝑚+𝑛)(𝑚+𝑛+1)
𝑏) ∑ = , com 𝑎, 𝑏 e 𝑐 são constantes reais não nulas.
𝑘=1 𝑎+𝑏+𝑐 2(𝑎+𝑏+𝑐)

R:
𝑎) Sendo 𝑎, 𝑏 e 𝑐 constantes, vale a P2:
𝑛−1 𝑛−1

∑ (𝑎 + 𝑏 + 𝑐)𝑘 = (𝑎 + 𝑏 + 𝑐) ∑ 𝑘 .
𝑘=1 𝑘=1
Obtemos a seguinte soma:
𝑛−1

(𝑎 + 𝑏 + 𝑐) ∑ 𝑘 = (𝑎 + 𝑏 + 𝑐)(1 + 2 + ⋯ + (𝑛 − 1)).
𝑘=1
Da soma dos 𝑛 primeiros naturais (soma de uma P.A com 𝑛 termos e razão 1), temos
que:
(𝑎 + 𝑏 + 𝑐)(1 + 2 + ⋯ + (𝑛 − 1)) = (𝑎 + 𝑏 + 𝑐)(1 + 2 + ⋯ + (𝑛 − 1) + 𝑛 − 𝑛)
1
= (𝑎 + 𝑏 + 𝑐)((1 + 2 + ⋯ + 𝑛) − 𝑛) = (𝑎 + 𝑏 + 𝑐) ( 𝑛(𝑛 + 1) − 𝑛)
2
𝑛(𝑛 + 1) − 2𝑛 𝑛2 + 𝑛 − 2𝑛
= (𝑎 + 𝑏 + 𝑐) ( ) = (𝑎 + 𝑏 + 𝑐) ( )
2 2
𝑛2 −𝑛 1
= (𝑎 + 𝑏 + 𝑐) (
2
) = 2 (𝑎 + 𝑏 + 𝑐)(𝑛2 − 𝑛), Q.E.D.

𝑏) Novamente, por 𝑎, 𝑏 e 𝑐 serem constantes, vale a P2:


𝑚+𝑛 𝑚+𝑛
𝑘 1
∑ = ∑ 𝑘.
𝑎+𝑏+𝑐 𝑎+𝑏+𝑐
𝑘=1 𝑘=1
Assim, obtemos a relação:
𝑚+𝑛
1 1
∑𝑘= (1 + 2 + ⋯ + (𝑚 + 𝑛)) .
𝑎+𝑏+𝑐 𝑎+𝑏+𝑐
𝑘=1
Temos a soma de uma progressão aritmética de razão 1, primeiro termo igual a 1 e
último termo igual a (𝑚 + 𝑛).
Seja 𝑥 o número de termos desta P.A.
Assim:
𝑎𝑛 = 𝑎1 + (𝑛 − 1)𝑟 ⟹ (𝑚 + 𝑛) = 1 + (𝑥 − 1); 𝑥 = (𝑚 + 𝑛).
1
Assim, pela fórmula dos termos de uma P.A: S𝑛 = 𝑛(𝑎1 + 𝑎𝑛 ), temos:
2
1
1 + 2 + ⋯ + (𝑚 + 𝑛) = (𝑚 + 𝑛)(1 + (𝑚 + 𝑛)).
2
Então:
1
1 (𝑚 + 𝑛)(1 + (𝑚 + 𝑛))
(1 + 2 + ⋯ + (𝑚 + 𝑛)) = 2
𝑎+𝑏+𝑐 𝑎+𝑏+𝑐
(𝑚+𝑛)(𝑚+𝑛+1)
= , Q.E.D.
2(𝑎+𝑏+𝑐)

2. Propriedades e Soma Telescópica

⊕ − 4. Obtenha o valor das somas:


5
1
𝑎) ∑ (𝑘 2 + 𝑘);
1 √3
𝑏) ∑15
1 2;
4

𝑐) 5 + ∑ 𝑘 .
1

R:
5 5 5 5
1 1 1
𝑎) ∑ (𝑘 2 + 𝑘) = ∑(𝑘² + 𝑘) = (∑ 𝑘 2 + ∑ 𝑘 )
1
√3 √3 1
√3 1 1
1 70
= ((12 + 22 + 32 + 42 + 52 ) + (1 + 2 + 3 + 4 + 5)) = .
√3 √3
𝑏)15 = 1 + (𝑛 − 1); 𝑛 = 15.
15

∑ 2 = 15.2 = 30.
1
4

𝑐) 5 + ∑ 𝑘 = 5 + (1 + 2 + 3 + 4) = 15.
1

⊕⊕⊕ − 5. Obtenha o valor da soma:


1 1 1 1 1
+ + + +⋯+ .
6.11 11.16 16.21 21.26 1001.1006
R:
Temos a seguinte soma:
1 1 1 1 1
+ + + + ⋯+ .
𝟔. 𝟏𝟏 𝟏𝟏. 𝟏𝟔 𝟏𝟔. 𝟐𝟏 𝟐𝟏. 𝟐𝟔 𝟏𝟎𝟎𝟏. 𝟏𝟎𝟎𝟔
Observe que os números marcados em preto formam esta sequência I:
I: (6, 11, 16, 21, ..., 1001), a qual é uma progressão aritmética de primeiro termo
igual a 6, e razão igual a 5.
Enquanto que os números marcados em vermelho formam esta sequência II:
II: (11, 16, 21, 26, ..., 1006), a qual é uma progressão aritmética de primeiro termo
igual a 11, e razão igual a 5.
Temos que o termo geral da primeira sequência é dado por:
I: 𝑎𝑛 = 6 + (𝑛𝑎 − 1)5 ⟹ 𝑎𝑛 = 6 + 5𝑛𝑎 − 5 = 𝟓𝒏𝒂 + 𝟏.
O termo geral da segunda sequência é dado por:
II: 𝑏𝑛 = 11 + (𝑛𝑏 − 1)5 ⟹ 𝑏𝑛 = 11 + 5𝑛𝑏 − 5 = 𝟓𝒏𝒃 + 𝟔.
O número de termos da sequência I é dado por:
𝑎𝑛 = 𝑎1 + (𝑛 − 1)𝑟 ⟹ 1001 = 6 + (𝒏𝒂 − 1)5.
𝒏𝒂 = 200.
O número de termos da sequência II é dado por:
𝑎𝑛 = 𝑎1 + (𝑛 − 1)𝑟 ⟹ 1006 = 11 + (𝒏𝒃 − 1)5.
𝒏𝒃 = 200.
Com base nos dados obtidos acima, podemos afirmar que esta soma é tal que:
𝟐𝟎𝟎
1 1 1 1 1 𝟏
+ + + +⋯+ =∑ .
6.11 11.16 16.21 21.26 1001.1006 (𝟓𝒏 + 𝟏)(𝟓𝒏 + 𝟔)
𝒏=𝟏
1
Expandindo o argumento (5𝑛+1)(5𝑛+6) em frações parciais, obtemos:
1 𝟏 𝟏
= − .
(5𝑛 + 1)(5𝑛 + 6) 𝟓(𝟓𝒏 + 𝟏) 𝟓(𝟓𝒏 + 𝟔)
Então:
200 𝟐𝟎𝟎
1 𝟏 𝟏
∑ = ∑( − ).
(5𝑛 + 1)(5𝑛 + 6) 𝟓(𝟓𝒏 + 𝟏) 𝟓(𝟓𝒏 + 𝟔)
𝑛=1 𝒏=𝟏
1
Temos que, da P2, podemos fatorar a constante , pois:
5
𝟐𝟎𝟎 𝟐𝟎𝟎
𝟏 𝟏 𝟏 𝟏 𝟏 𝟏
∑( − )= ∑( − )
𝟓(𝟓𝒏 + 𝟏) 𝟓(𝟓𝒏 + 𝟔) 𝟓 (𝟓𝒏 + 𝟏) 𝟓 (𝟓𝒏 + 𝟔)
𝒏=𝟏 𝒏=𝟏
𝟐𝟎𝟎 𝟐𝟎𝟎
𝟏 𝟏 𝟏 𝟏 𝟏 𝟏
= ∑[ ( − )] = ∑ ( − ).
𝟓 𝟓𝒏 + 𝟏 𝟓𝒏 + 𝟔 𝟓 𝟓𝒏 + 𝟏 𝟓𝒏 + 𝟔
𝒏=𝟏 𝒏=𝟏
Observemos que, se considerarmos a função 𝑓(𝑛) = 5𝑛 + 1, e calcularmos 𝑓(𝑛 + 1),
obteremos o padrão da Soma Telescópica, pois:
Seja 𝑓(𝑛) = 5𝑛 + 1, calculando 𝒇(𝒏 + 𝟏) ⟹ 𝑓(𝑛 + 1) = 5(𝑛 + 1) + 1 = 𝟓𝒏 + 𝟔.
Então:
𝟐𝟎𝟎 𝟐𝟎𝟎
𝟏 𝟏 𝟏 𝟏
∑( − ) = ∑(𝒇(𝒏) − 𝒇(𝒏 + 𝟏)).
𝟓 𝟓𝒏 + 𝟏 𝟓𝒏 + 𝟔 𝟓
𝒏=𝟏 𝒏=𝟏
Da Propriedade Telescópica, temos que:
𝒃

∑(𝒇(𝒌) − 𝒇(𝒌 + 𝟏)) = 𝒇(𝒂) − 𝒇(𝒃 + 𝟏) .


𝒂
Assim:
𝟐𝟎𝟎 𝟐𝟎𝟎
𝟏 𝟏 𝟏 𝟏 𝟏
𝟓
∑ (𝟓𝒏+𝟏 − 𝟓𝒏+𝟔) = 𝟓 ∑ (𝒇(𝒏) − 𝒇(𝒏 + 𝟏)) = 𝟓 (𝒇(𝟏) − 𝒇(𝟐𝟎𝟎 + 𝟏)),
𝒏=𝟏 𝒏=𝟏
onde 𝑓(𝑛) = 5𝑛 + 1.
Enfim,
𝟐𝟎𝟎
𝟏 𝟏 𝟏 𝟏 𝟏 𝟏 𝟏 𝟓𝟎
∑(𝒇(𝒏) − 𝒇(𝒏 + 𝟏)) = ( − )= ( − )= .
𝟓 𝟓 𝟓. 𝟏 + 𝟏 𝟓. 𝟐𝟎𝟏 + 𝟏 𝟓 𝟔 𝟏𝟎𝟎𝟔 𝟏𝟓𝟎𝟗
𝒏=𝟏

⊕⊕ − 6. Demonstre que:
360°
361°
𝑎) ∑ (cos 𝑘 − cos(𝑘 + 1)) = 2𝑠𝑒𝑛2 ( );
0° 2
1000
1 999
𝑏) ∑ = ;
2 𝑘(𝑘−1) 1000

𝑐) ∑700
1 4 = 2800;
𝑚
𝑑) ∑1 (𝑘² + 𝑘 + 1) + ∑3𝑚+1(𝑘 2 + 𝑘 + 1), 𝑚 ∈ ℕ > 3 = 23.

R:
𝑎) Aplicação direta da Propriedade Telescópica.
Temos, então, que:
360°

∑ (cos 𝑘 − cos(𝑘 + 1)) = cos(0°) − cos(361°) .



Das fórmulas de Prostaferese, temos que:
𝒂+𝒃 𝒂−𝒃
𝐜𝐨𝐬 𝒂 − 𝐜𝐨𝐬 𝒃 = − 𝟐𝒔𝒆𝒏 ( ) 𝒔𝒆𝒏 ( ).
𝟐 𝟐
Assim:
0° + 361° 0° − 361°
cos(0°) − cos(361°) = − 2𝑠𝑒𝑛 ( ) 𝑠𝑒𝑛 ( )
2 2
𝟑𝟔𝟏° 𝟑𝟔𝟏°
= − 𝟐𝒔𝒆𝒏 ( ) 𝒔𝒆𝒏 (− ).
𝟐 𝟐
Da paridade da função seno, sabemos que:
𝒔𝒆𝒏(− 𝒙) = − 𝒔𝒆𝒏 𝒙.
Então:
𝟑𝟔𝟏° 𝟑𝟔𝟏° 𝟑𝟔𝟏° 𝟑𝟔𝟏° 𝟑𝟔𝟏°
= − 𝟐𝒔𝒆𝒏 (
𝟐
) 𝒔𝒆𝒏 (− 𝟐
) = − 𝟐𝒔𝒆𝒏 ( 𝟐
) (− 𝒔𝒆𝒏 ( 𝟐
)) = 𝟐𝒔𝒆𝒏𝟐 ( 𝟐
).
1000 1000
1 1 1
𝑏) ∑ =∑( − ).
𝑘(𝑘 − 1) 𝑘−1 𝑘
2 2
1
Aplicando a Propriedade Telescópica e considerando a função 𝑓(𝑘) = , obtemos:
𝑘−1
1000
1 1 1 1 𝟗𝟗𝟗
∑( − )=1− =1− = .
𝑘−1 𝑘 1001 − 1 1000 𝟏𝟎𝟎𝟎
2
700

𝑐) ∑ 4 = 4.700 = 2800.
1
𝑑) Fora demonstrado no item sobre as propriedades, que:
𝒑 𝒃 𝒃

∑ 𝒇(𝒌) + ∑ 𝒇(𝒌) = ∑ 𝒇(𝒌).


𝒂 𝒑+𝟏 𝒂
Assim:
𝑚 3 3

∑(𝑘² + 𝑘 + 1) + ∑ (𝑘 2 + 𝑘 + 1) = ∑(𝑘 2 + 𝑘 + 1) = 3 + 7 + 13 = 𝟐𝟑.


1 𝑚+1 1

3. Séries Geométricas

⊕⊕⊕ − 7. Se:
1 1 1 1 1 1 𝑝
+ + + + ⋯+ 𝑛
+ 𝑛
+ ⋯ = , obtenha o resto da divisão de 𝑝 por 𝑝 + 𝑞
3 4 9 16 3 4 𝑞
sabendo-se que esta soma é infinita.

R:
Veja que:
𝟏 𝟏 𝟏 𝟏 𝟏 𝟏
+ + + + ⋯+ 𝒏 + 𝒏 + ⋯
𝟑 𝟒 𝟗 𝟏𝟔 𝟑 𝟒
𝟏 𝟏 𝟏 𝟏 𝟏 𝟏
= ( + + ⋯+ 𝒏 + ⋯) + ( + + ⋯ 𝒏 + ⋯)
𝟑 𝟗 𝟑 𝟒 𝟏𝟔 𝟒
∞ ∞
𝟏 𝟏
= ∑ 𝒌+∑ 𝒌.
𝟑 𝟒
𝒌=𝟏 𝒌=𝟏

1 1 1
Sabemos que a razão da P.G ( , , ... ) é < 1, logo, a soma infinita de seus termos
3 9 3
(ou o limite para o qual a soma de seus termos converge, quando o número de termos
tende ao infinito) é dada por:
1
𝑎1 𝟏
lim S𝑛 = = 3 = .
𝑛→∞ 1−𝑞 1−1 𝟐
3

1 1 1
Do mesmo modo, a razão da PG ( , , ... ) é < 1, e o limite da soma dos termos
4 16 4
desta P.G infinita é dado por:
1
𝑎1 𝟏
S𝑛 = = 4 = .
1−𝑞 1−1 𝟑
4
1 1 𝑝 𝑝 5
Assim, como + = , temos que = .
2 3 𝑞 𝑞 6
Então, 𝑝 = 5 e 𝑞 = 6.
O resto da divisão de 5 por 11 é 5.

⊕⊕ − 8. Se |𝑥| < 1, resolva a equação, considerando o conjunto dos números reais:


1 + 𝑥² + 𝑥 4 + ⋯ + 𝑥 2𝑛 + ⋯ 1
= .
𝑥 + 𝑥³ + 𝑥 5 + ⋯ + 𝑥 2𝑛+1 + ⋯ 𝑥 + 𝑥 2

R:
Como nos foi garantido que |𝑥| < 1, temos que ambas as somas infinitas, do
numerador e denominador, são convergentes.
Assim:
1
1 + 𝑥² + 𝑥 4 + ⋯ + 𝑥 2𝑛 + ⋯ = .
1−𝑥²
𝑥
e 𝑥 + 𝑥³ + 𝑥 5 + ⋯ + 𝑥 2𝑛+1 + ⋯ = .
1−𝑥²
Então, temos que:
1
1
2
= 1 −𝑥 𝑥² .
𝑥+𝑥
1 − 𝑥²
1 1 1 − 𝑥² 1 1
= . ⟹ = .
𝑥 + 𝑥2 1 − 𝑥2 𝑥 𝑥 + 𝑥2 𝑥
𝑥 = 𝑥 + 𝑥², 𝑥 = 0.
1
Como 𝑥 = 0, substituindo na razão inicial, teremos uma indeterminação do tipo .
0
Portanto, não existe solução para este problema.
S = Ø.

⊕⊕ −𝟗. Se |𝑥| < 1, resolva a equação, considerando o conjunto dos números reais:

∑ 𝑥 𝑘+1 = 5.
0
R: Faça você.
VIII. Exercícios propostos
⊕⊕ − 1. Passe para a forma de somatório as seguintes somas:
𝑎) 𝑥 + 𝑥² + 𝑥³ + ⋯ + 𝑥 𝑛−1 ;
𝑏) 𝑎𝑟𝑐 𝑠𝑒𝑛(𝑥) + 𝑎𝑟𝑐 𝑠𝑒𝑛(𝑥 + 1) + 𝑎𝑟𝑐 𝑠𝑒𝑛(𝑥 + 2) + ⋯ + 𝑎𝑟𝑐 𝑠𝑒𝑛(𝑥 + 𝑛);
1 1 1 1
𝑐) + + + ⋯+ ;
1+1 1+2 1+3 1+(𝑛−1)
1 1 1
𝑑) 1. log 𝑎 1 + log 𝑎 2 + log 𝑎 3 + ⋯ + log 𝑎 𝑛, 𝑎 > 1.
2 3 𝑛
𝑛+𝑚
𝑒)1 + 𝑒 + 𝑒² + ⋯ + 𝑒 ;
1 1 1
𝑓) + +⋯+ .
5 10 5𝑘

⊕⊕⊕ − 2. Calcule o valor das seguintes somas:


4
𝑎) ∑𝑘=1(𝑘 3 − 5𝑘);
5
𝑏) ∑ √6 + √3;
𝑘=1
5
𝑐) ∑ √𝑘 − √𝑘 − 1;
𝑘=3
3
𝑑) ∑𝑘=1 𝑘 𝑘 ;
4
1
𝑒) ∑ ( (𝑘² + 4𝑘));
𝑘=1 √5
6
1
𝑓) ∑ (𝑥 2−3) .
𝑘=3

4 𝑛
log1⁄2 √32
⊕⊕⊕ − 3. (ITA) A soma ∑ 𝑛+2
é igual a:
𝑛=1 log1⁄2 8
8
A( ) .
9
14
B( ) .
15
15
C( ) .
16
17
D( ) .
18
E ( ) 1.

⊕⊕⊕ − 4. Prove que:


6° 1° 9° 𝜋
1 4√2 cos ( ) cos(1°) 𝑠𝑒𝑛 ( + )
𝑎) ∑ (𝑠𝑒𝑛𝜃 + 𝑐𝑜𝑠𝜃) = 2 2 4
𝑚 𝑚
𝜃=3°
𝑛
𝑘+5 𝑛² + 11𝑛
𝑏) ∑ =
(𝑎 + 𝑏 + 𝑐) 2(𝑎 + 𝑏 + 𝑐)
𝑘=1
⊕⊕ − 5. Calcule:
1 1 1 1
S= + + + ⋯+ .
√2 + √1 √3 + √2 √4 + √3 √81 + √80

⊕⊕⊕ − 𝟔. (BRILLIANT) Obtenha o valor da soma:


1 1 1
+ + ⋯+ .
√4 + √7 √7 + √10 √397 + √400

⊕⊕⊕ − 7. Dada a função 𝑓(𝑥) = 4𝑥² − 1, obtenha o valor de:


1 1 1 1
+ + ⋯+ + .
𝑓(1) 𝑓(2) 𝑓(99) 𝑓(100)

⊕⊕ − 8. Dado o polinômio 𝑝, na variável 𝑥, expresso por:


3
𝑝(𝑥) = ∑1(𝑎 − 𝑘)𝑥 𝑘 , cujas raízes são os números 𝛼, β e γ, onde 𝑎 é uma constante
real não nula, verifique que a expressão 𝛼² + β² + 𝛾² é tal que:
𝑎2 − 4𝑎 + 2
𝛼² + β² + 𝛾² = − 2 .
𝑎 − 6𝑎 + 9

⊕⊕⊕ − 9. Obtenha o valor das somas a seguir utilizando a Propriedade Telescópica:


60
3
𝑎) ∑ ;
𝑘=1 (3𝑘−1)(3𝑘+2)
40
5
𝑏) ∑ ;
𝑘=3 𝑘(𝑘−2)
25
1
𝑐) ∑ ;
𝑘=1 4𝑘(4𝑘+4)
𝑑) ∑500
𝑘=2(ln(𝑘 + 2) − ln(𝑘)); * 𝐥𝐧 𝒙 = 𝐥𝐨𝐠 𝒆 𝒙.
49
𝑒) ∑ (√𝑘 − √𝑘 − 2);
𝑘=2
35
5
𝑓) ∑ − ;
𝑘=2 𝑘(𝑘−1)

𝑔) ∑90°
0° (cos(𝑘 + 1) − cos 𝑘);
437
ℎ) ∑1 𝑒 𝑘−1 − 𝑒 𝑘 ;
𝑛−1 𝑘 𝑘−1
𝑖) ∑ [(√𝑎) − (√𝑎) ], 𝑎 ∈ ℝ ∖ {0}.
1

3
⊕ − 10. Se a soma ∑𝑘=0(𝑘 + 𝑖(𝑘 + 1)) tem como resultado o número complexo
𝑧𝑟 = 𝑚 + (𝑚 + 4)𝑖, verifique que 𝑚, 𝑚 + 2 e 𝑚 + 4 estão numa P.A e determine sua
razão.

𝑝
⊕⊕ − 11. Obtenha 𝑝 tal que: ∑1 (4𝑘 + 5) = 225.
⊕⊕ −𝟏𝟐. Resolva a seguinte equação, no âmbito dos números complexos:
1 + (𝑥 − 2) + (𝑥 − 2)2 + ⋯ + (𝑥 − 2)𝑛 + ⋯ = 7.

⊕⊕⊕ − 13. Obtenha o valor da soma:



1
∑ [ (7−𝑘 − 3−𝑘 )] .
0
√5

⊕⊕⊕ − 14. (ITA 2015) Prove que:



1 √2
∑ = .
𝑛=0
(√2 − 1)√2𝑛 3 − 2√2

⊕⊕⊕ − 15. Calcule:


100
𝑎) ∑𝑖=0 (4 + 3𝑖);
3
𝑏) ∑ (5 + √4𝑘 );
𝑘=0
50
𝑐) ∑𝑘=1[ln(𝑘 + 3) − ln(𝑘 + 2)];
R:
50
∑𝑘=1[ln(𝑘 + 3) − ln(𝑘 + 2)] = 𝑓(50 + 1) − 𝑓(1), onde 𝑓(𝑘) = ln(𝑘 + 2).
𝟓𝟑
𝑓(50 + 1) − 𝑓(1) = ln(53) − ln(3) = 𝐥𝐧 ( ).
𝟑
100
1
𝑑) ∑ .
(4𝑘² − 1)
𝑘=2

⊕⊕⊕ − 16. Prove que:


𝑛
1
𝑎) ∑ = √𝑛 + 1 − 1, ∀ 𝑘 ∈ ℕ;
𝑘=1 √𝑘+√𝑘+1
1 1 1
𝑏) + + ⋯+ = 11;
1+√2 √2+√3 √143+√144
1 1 1
𝑐) + + ⋯+ = 14.
1+√2 √2+√3 √224+√225

⊕⊕⊕ − 17. Dada as Séries Geométricas abaixo, obtenha lim𝑛→∞ S𝑛 .


1 1 1
𝑎) 1 + + + ⋯ + + ⋯;
2 4 2𝑛
3 9 27
𝑏) 1 + + + + ⋯;
4 16 64
1 1
𝑐) 1 + + 2 + ⋯;
√3√5 (√3√5)
1 1 1
𝑑) + 2 + 3 + ⋯;
1−5√7 (1−5√7) (1−5√7)
2 3
8 8 8
𝑒)1 + +( ) +( ) +⋯
9√7 − 13 9√7 − 13 9√7 − 13
⊕⊕⊕ − 18. Calcule as somas:
1 1 1 1
𝑎) + + + ⋯+ ;
2.9 9.16 16.23 1395.1402
1 1 1 1
𝑏) + + + ⋯+ ;
3.7 7.11 11.15 1739.1743
1 1 1 1
𝑐) + + +⋯ .
3.5 5.7 7.9 2001.2003

⊕⊕ − 19. Calcule o número de termos das somas:


𝑛+𝑚
𝑎) ∑2 (5𝑘 − 4);
R:
𝑎𝑛 = 𝑎1 + (𝑛𝑠 − 1)𝑟;
5(𝑛 + 𝑚) − 4 = 6 + (𝑛𝑠 − 1)5;
5(𝑛 + 𝑚) − 4 = 6 + 5𝑛𝑠 − 5;
5𝑛𝑠 = 5(𝑛 + 𝑚) − 5;
𝒏𝒔 = 𝒏 + 𝒎 − 𝟏.
𝑏) ∑𝑛−7
1 𝑎, 𝑎 ∈ ℝ∗+ ;
19
𝑐) ∑1 (3𝑘 − 2);
𝑑) ∑79
4 (6𝑘 − 3);
7
𝑒) ∑3 2𝑘 ;
R:
𝑎𝑛 = 𝑎1 𝑞 𝑛−1 ;
27 = 2³. 2𝑛−1 ;
2𝑛−1 = 27−3 ;
𝑛 − 1 = 4;
𝑛 = 5.
𝑛−9
𝑓) ∑3 5𝑘 ;
𝑝+𝑞
𝑔) ∑𝑘=𝑝(𝑎𝑘 + 𝑎𝑘−1 ).
R:
𝑝+𝑞 𝑝+𝑞 𝑝+𝑞
∑𝑘=𝑝(𝑎𝑘 + 𝑎𝑘−1 ) = ∑𝑝 𝑎 𝑘 + ∑𝑝 𝑎𝑘−1 ;
I: 𝑎𝑝+𝑞 = 𝑎𝑝 . 𝑎𝑛1−1 ;
𝑎𝑛1−1 = 𝑎𝑞 ;
𝑛1 = 𝑞 + 1.
𝑝+𝑞
II: 𝑎 = 𝑎𝑝−1 . 𝑎𝑛2−1 ;
𝑎𝑝+𝑞 = 𝑎𝑛2+𝑝−2 ;
𝑝 + 𝑞 = 𝑛2 + 𝑝 − 2;
𝑛2 = 𝑞 + 2.
𝒏𝒕 = 𝒏𝟏 + 𝒏𝟐 = 𝒒 + 𝟏 + 𝒒 + 𝟐 = 𝟐𝒒 + 𝟑 termos.
Capítulo 2 – Produtórios

I. Introdução e notação Pi (∏)


Assim como a notação Sigma dos Somatórios é uma evolução natural do conceito de
adição, a noção de Produtório exerce a mesma função, porém, para com as
multiplicações.
A notação de Produtório não é tão comum como a notação de Somatório, porém
desempenha um papel crucial em áreas como a Estatística, Probabilidade,
Computação, Matemática Discreta, em vertentes da Física, na própria Álgebra e etc.
A ideia subjacente a este conceito é a compactação de um produto por meio de uma
elegante notação denominada Notação Pi: Π.
Seja:
𝒏

P = ∏ 𝑎𝑘 = 𝑎1 . 𝑎2 . … . 𝑎𝑛 .
𝒌=𝟏
Os três principais componentes do Produtório são:
1 – O argumento do produtório nos indicia o que irá ser multiplicado, no caso
acima, todos os números 𝒂𝒌 .
2 – O produtório se inicia em 𝒌 = 𝟏 (limite inferior).
3 – produtório termina em 𝒌 = 𝒏 (limite superior).
Dito isto, passemos para a definição intuitiva de Produtório.

II. Definição intuitiva de produtório


Consideremos o seguinte problema:
“1. Calcule o produto dos 15 primeiros números naturais .”
O problema nos pede o seguinte produto:
P = 1.2. … .15.
Veja que, se comparados aos somatórios, os produtórios crescem numa taxa superior,
pois ocorre um Crescimento Fatorial dos termos do mesmo.
Transpondo o produto acima para a notação de produtório, obtemos:
P = 1.2. … .15;
15

P = ∏ 𝑘 = 1.2. … .15.
𝑘=1
O argumento do produtório se inicia em 𝑘 = 1, sendo este o limite inferior e o
primeiro número a ser multiplicado, e se encerra em 𝑘 = 15, sendo este o limite
superior e o último número a ser multiplicado.
A letra 𝑘 (argumento do produtório) assumirá os valores de 1 até 50.
“2. Coloque na notação de produtório o produto das tangentes dos arcos de 45° a 57°.”
O problema nos pede para colocarmos na notação pi o produto das tangentes de 45°
até 57°, de tal maneira:
𝑡𝑔 45°. 𝑡𝑔 46°. 𝑡𝑔 47°. … . 𝑡𝑔 57°.
O índice 𝑘 deste produto irá variar de 45° até 57°, logo:
57°

𝑡𝑔 45°. 𝑡𝑔 46°. 𝑡𝑔 47°. … . 𝑡𝑔 57° = ∏ 𝑡𝑔 𝑘 .


45°
“3. Passe para a notação pi o produto dos logaritmos naturais de 30 a 50.”
O problema nos pede para colocarmos na notação pi o produto dos logaritmos naturais
(base 𝑒) de 30 a 50:
ln 30. ln 31. ln 32. … ln 50.
O índice 𝑘 irá variar de 30 a 50, assim:
50

ln 30. ln 31. ln 32. … . ln 50 = ∏ ln 𝑘 .


30
“4. Seja o produto P = 2.4.6. … .2𝑛.
Passe P para a notação de produtório.”
Temos o produto P = 2.4.6. … .2𝑛, que é o produto dos 𝑛 primeiros números pares da
forma 2𝑘, onde 𝑘 assume qualquer valor inteiro maior do que zero.
Dito isto:
1

P = 2.4.6. … .2𝑛 = ∏ 2𝑘 .
𝑘=1
“5. Passe para a forma de produtório o seguinte produto:
P = √3. √5. √7. … . √2𝑛 + 1.”
Temos o produto das raízes dos 𝑛 primeiros números ímpares da forma 2𝑘 + 1.
Assim:
𝑛

P = √3. √5. √7. … . √2𝑛 + 1 = ∏ √2𝑘 + 1 .


𝑘=1
“6. Expanda o seguinte produtório:
5
1
∏ .”
1 𝑘𝜋
5
1
Temos o produtório ∏ , cujo índice 𝑘 varia de 1 (limite inferior) até 5
1 𝑘𝜋
(limite superior). Assim:
5
1 𝟏 𝟏 𝟏 𝟏 𝟏 𝟏
∏ = . . . . = .
𝑘𝜋 𝝅 𝟐𝝅 𝟑𝝅 𝟒𝝅 𝟓𝝅 𝟏𝟐𝟎𝝅𝟓
1
“7. Transponha o seguinte produto para a forma de produtório:
𝑎. 𝑎². 𝑎³. … . 𝑎𝑛 , onde 𝑎 é uma constante não nula e maior do que um.”
Temos o produto das potências de uma constante, potências estas variando de 1 até 𝑛.
Assim:
𝑛

𝑎. 𝑎². 𝑎³. … . 𝑎𝑛 = ∏ 𝑎𝑘 .
1
𝜋 𝜋 𝜋
“8. Coloque o produto cos ( + 1) . cos ( + 2) . … . cos ( + (𝑛 − 1)) na notação de
5 5 5
produtório.”
𝑛−1
𝜋 𝜋 𝜋 𝜋
Temos que cos ( + 1) . cos ( + 2) . … . cos ( + (𝑛 − 1)) = ∏ cos ( + 𝑘).
5 5 5 𝑘=1 5

Novamente, o grande feito desta notação é promover a compactação de produtos


extensos.
Assim como os somatórios, os produtórios também possuem propriedades operatórias,
as quais serão enunciadas e demonstradas no item IV.
A seguir, no item III, proporemos a construção formal de produtório.

III. Definição formal de produtório


Seja A ⊂ ℝ, e seja ℤ o conjunto dos números inteiros, tal que ℤ ⊂ A.
Sejam 𝑎 e 𝑏 ∈ ℤ tais que a função 𝑓: A → ℝ:
𝑏
∏𝑎 𝑓(𝑘) = 𝑓(𝑎). 𝑓(𝑎 + 1). … . 𝑓(𝑏 − 1). 𝑓(𝑏).
Para que seja satisfeito o exposto acima, é necessário que:
𝑎

∏ 𝑓(𝑘) = 𝑓(𝑎)
𝑘=𝑎
𝑏 𝑏−1

∏ 𝑓(𝑘) = 𝑓(𝑏). ∏ 𝑓(𝑘)


{ 𝑘=𝑎 𝑘=𝑎
Novamente, estas definições nos serão úteis na resolução de vários problemas que
envolvem o conceito de produtório, principalmente demonstrações.
A seguir, no item IV, discutiremos e demonstraremos as propriedades operatórias dos
produtórios, com exceção do Produto Telescópico, o qual, em alusão à Soma
Telescópica, é de crucial importância para esta teoria e será discutido de maneira
profunda no item V, que será inteiramente dedicado a este.

IV. Propriedades
Neste item, como fora feito no capítulo de somatórios, discutiremos as propriedades
recorrentes dos produtórios.
Novamente, as propriedades que serão aqui apresentadas são de absoluta importância
para a teoria, pois nos poupam de operações desnecessárias.
As propriedades vinculadas aos somatórios são, de certa maneira, mais “fáceis” de
serem operadas, ao passo de que as propriedades dos produtórios possuem uma carga
operatória maior, pois lidamos com potências, fatoriais, etc.
Tais propriedades são:
P1. – Produtório de uma constante (linearidade);
P2. – Produtório de uma constante por uma variável;
P3. – Produtório de um produto ou um quociente
𝑏
𝒃 𝑓(𝑘)
[ ∏𝒌=𝒂(𝒇(𝒌). 𝒈(𝒌)) ou ∏ (𝑔(𝑘))];
𝑘=𝑎
P4. – Produtório de uma variável linear
[∏𝒏𝒌=𝟏 𝒌];
P5. – Produtório de uma constante com expoente variável
𝒃

[∏ 𝜶𝒌 ] .
𝒌=𝒂
P6. – Logaritmo de um produto
𝑏

[log 𝑐 (∏ 𝑓(𝑘))] .
𝑎

Como na teoria acerca dos somatórios, também existem outras propriedades que
fogem do escopo da teoria, porém serão discutidas brevemente nos exercícios.
Agora, discutiremos cada uma destas propriedades.

P1. – Produtório de uma constante (linearidade):


“O produtório de uma constante é igual a constante elevada ao número de termos
do produtório.”
Em linguagem matemática, o caso geral, no qual o limite inferior 𝑎 é diferente de 1, é
tal que:
𝑏

∏ 𝛼 = 𝛼 𝑏−𝑎+1 .
𝑘=𝑎
Onde 𝛼 é uma constante não nula e diferente de um, 𝛼 ∈ ℝ∗ , 𝛼 ≠ 1.
Se o produtório se inicia no limite inferior igual a um, vale que:
𝑛

∏ 𝛼 = 𝛼𝑛 .
𝑘=1
Demonstração:
A iniciar pelo caso no qual o limite inferior é igual a 1:
𝑛

∏ 𝛼 = 𝛼. 𝛼. … . 𝛼.
𝑘=1
Temos que o número de termos deste produto é calculado a partir da fórmula do termo
geral de uma progressão aritmética de razão um, pois os índices 𝑘 (𝑘 = 1, 2, ..., 𝑛)
variam de acordo com esta.
Então, pelo termo geral da progressão aritmética, temos que o número de termos 𝑛𝑡 a
serem multiplicados é dado por:
𝑎𝑛 = 𝑎1 + (𝑛𝑡 − 1)𝑟 ⟹ 𝑛 = 1 + (𝑛𝑡 − 1) ∴ 𝒏𝒕 = 𝒏.
Assim,
𝑛

∏ 𝛼 = 𝛼 𝑛𝑡 = 𝛼 𝑛 .
𝑘=1
Para o caso geral, consideramos o limite inferior como sendo um número natural 𝑎
qualquer e o limite superior um número natural 𝑏 qualquer:
𝑏

∏ 𝛼 = 𝛼. 𝛼. … . 𝛼.
𝑘=𝑎
Temos, considerando os índices 𝑘 (𝑘 = 𝑎, (𝑎 + 1), … , (𝑏 − 1), 𝑏), novamente uma
P.A de razão um, porém com limite inferior (primeiro termo da P.A) igual a 𝑎 e limite
superior (último termo da P.A) igual a 𝑏.
Assim, o número de termos 𝑛𝑡 desta P.A é obtido da seguinte maneira:
𝑎𝑛 = 𝑎1 + (𝑛𝑡 − 1)𝑟 ⟹ 𝑏 = 𝑎 + (𝑛𝑡 − 1) ∴ 𝑛𝑡 = 𝑏 − 𝑎 + 1.
Portanto:
𝑏

∏ 𝛼 = 𝛼 𝑏−𝑎+1 .
𝑘=𝑎
Façamos alguns exemplos:
“1. Calcule o valor do produto:
∏13
𝑘=1 7.”
O número de termos 𝑛 deste produto é, novamente, obtido por meio do Termo Geral
de uma P.A de razão 1, primeiro termo igual a 1 e último termo igual a 13.
𝑎𝑛 = 𝑎1 + (𝑛 − 1)𝑟 ⟹ 13 = 1 + (𝑛 − 1) ∴ 𝑛 = 13.
Assim, temos que:
13

∏ 7 = 713 .
𝑘=1
“2. Obtenha o valor de ∏19
𝑘=6 3.”
O número de termos 𝑛 deste produto é dado por 19 = 6 + (𝑛 − 1), 𝑛 = 14.
Então:
19

∏ 3 = 314 .
𝑘=6
“3. Se:
16 11 4

(∏ 2) . (∏ 5) . (∏ 3) = 𝑘. 10𝑛 ,
𝑘=5 𝑘=3 𝑘=1
onde 𝑘 < 10 e 𝑛 é um natural, obtenha o valor numérico de 𝑛 + 𝑘, com uma
aproximação de duas casas decimais.”
Para facilitar as operações, definamos:
16

∏2 = 𝑎
𝑘=5
11

∏5 = 𝑏
𝑘=3
4

∏ 3 = 𝑐.
{ 𝑘=1
Assim, 𝑎𝑏𝑐 = 𝑘. 10𝑛 e, sejam:
𝒏𝒂 (número de termos do produto a); dado por 16 = 5 + (𝑛𝑎 − 1),
𝑛𝑎 = 12.
Assim, 𝑎 = 212 .
𝒏𝒃 (número de termos do produto b); dado por 11 = 3 + (𝑛𝑏 − 1),
𝑛𝑏 = 9.
Assim, 𝑏 = 59 .
𝒏𝒄 (número de termos do produto c); dado por 4 = 1 + (𝑛 − 1),
𝑛𝑐 = 4.
Assim, 𝑐 = 34 .
Então, temos que 𝑎𝑏𝑐 = (212 )(59 )(34 ) = 𝟐𝟗 . 23 . 𝟓𝟗 . 34 = (2.5)9 . 2³. 34 = 109 . 23 . 34
= 109 . 𝟐𝟑 . 𝟑𝟑 . 3 = (2.3)3 . 3.109 = 6³. 3.109 = 648.109 = 6,48.1011.
É importante que você, leitor, a esta altura, esteja familiarizado com as
propriedades da potenciação e radiciação.
Finalmente, temos que:
6,48.1011 = 𝑘. 10𝑛 .
Se 𝑘 < 10, temos que 𝑘 = 6,48 e 𝑛 = 11.
O valor de 𝑛 + 𝑘, com uma aproximação de duas casas decimais,
é 𝑛 + 𝑘 = 17,48 ≅ 17,5.
“4. Seja 𝛼, 𝑚, 𝑛 e 𝑝 constantes reais, com 𝛼 > 0, (𝑚, 𝑛, 𝑝) > 0 e 𝑛 + 𝑝 > 0.
Prove que:
𝑚+𝑛+𝑝 1
(𝑛+𝑝+1)
∏ √ln 𝑎 = (ln 𝑎)2 .”
𝑘=𝑚
Aplicação direta da propriedade pois, para calcularmos o número de termos deste
produto, devemos aplicar o Termo Geral da P.A.
Sendo 𝑥 o número de termos deste produto:
𝑎𝑛 = 𝑎1 + (𝑛 − 1)𝑟 ⟹ 𝑚 + 𝑛 + 𝑝 = 𝑚 + (𝑥 − 1). 1;
𝑥 (número de termos) = 𝑛 + 𝑝 + 1.
Como ln 𝑎 é uma constante, este produtório é igual a constante a ser multiplicada
elevada ao número de termos do produto.
𝑚+𝑛+𝑝 𝑚+𝑛+𝑝 1 1 𝑥 𝟏
(𝒏+𝒑+𝟏)
Assim: ∏ √ln 𝑎 = ∏ (ln 𝑎) = [(ln 𝑎) ] = (𝐥𝐧 𝒂)𝟐
2 2 .
𝑘=𝑚 𝑘=𝑚
P2. – Produtório de uma constante por uma variável:
“O produtório de uma constante por uma variável é igual ao produto entre a
constante elevada ao número de termos do produtório e o produtório da
variável.”
Matematicamente, o caso geral, no qual o limite inferior 𝑎 é diferente de 1:
𝑏 𝑏

∏[𝛼. 𝑓(𝑘)] = 𝛼 𝑏−𝑎+1 [∏ 𝑓(𝑘)] .


𝑘=𝑎 𝑘=𝑎
Sendo 𝛼 uma constante não nula e diferente de um, 𝛼 ∈ ℝ∗ , 𝛼 ≠ 1.
Se o produtório possui limite inferior igual a 1 e limite superior igual a um 𝑛 qualquer,
vale que:
𝑛 𝑛

∏[𝛼. 𝑓(𝑘)] = 𝛼 𝑛 [∏ 𝑓(𝑘)] .


𝑘=1 𝑘=1
Demonstração:
Iniciaremos as demonstrações com o caso particular no qual o limite inferior é 1 e o
limite superior é um natural 𝑛 qualquer:
𝑛

∏[𝛼. 𝑓(𝑘)] = [𝛼. 𝑓(1)]. [𝛼. 𝑓(2)]. … . [𝛼. 𝑓(𝑛 − 1)]. [𝛼. 𝑓(𝑛)]
𝑘=1
= 𝛼. 𝛼. … . 𝛼. 𝛼. 𝑓(1). 𝑓(2). … . 𝑓(𝑛 − 1). 𝑓(𝑛).
Perceba que temos 𝑛 fatores iguais a constante 𝛼 (verificando pelo termo geral das
Progressões Aritméticas; 𝑛 = 1 + (𝑛𝑡 − 1); 𝒏𝒕 = 𝒏, sendo 𝑛𝑡 o número de termos
desta progressão), assim, pela P1:
𝛼. 𝛼. … . 𝛼. 𝛼. 𝑓(1). 𝑓(2). … . 𝑓(𝑛 − 1). 𝑓(𝑛) = 𝑎𝑛 [𝑓(1). 𝑓(2). … . 𝑓(𝑛 − 1). 𝑓(𝑛)]
𝑛

= 𝑎𝑛 [∏ 𝑓(𝑘)] .
𝑘=1
Prosseguindo para o caso geral:
𝑏

∏[𝛼. 𝑓(𝑘)] = [𝛼. 𝑓(𝑎)]. [𝛼. 𝑓(𝑎 + 1)]. … . [𝛼. 𝑓(𝑏 − 1)]. [𝛼. 𝑓(𝑏)]
𝑘=𝑎
= 𝛼. 𝛼. … 𝛼. 𝛼. 𝑓(𝑎). 𝑓(𝑎 + 1). … . 𝑓(𝑏 − 1). 𝑓(𝑏).
Temos uma quantidade de 𝑏 − 𝑎 + 1 constantes 𝛼 sendo multiplicadas (já verificado
anteriormente, na demonstração da P1), assim:
𝛼. 𝛼. … 𝛼. 𝛼. 𝑓(𝑎). 𝑓(𝑎 + 1). … . 𝑓(𝑏 − 1). 𝑓(𝑏) = 𝛼 𝑏−𝑎+1 [𝑓(𝑎). 𝑓(2). … . 𝑓(𝑏 − 1). 𝑓(𝑏)]
𝑏

= 𝑎𝑏−𝑎+1 [∏ 𝑓(𝑘)] .
𝑘=𝑎
“1. Calcule:
5
∏𝑘=2(2𝑘 + 2).”
Temos que:
5 5 5
∏𝑘=2(2𝑘 + 2) = ∏𝑘=2[2(𝑘 + 1)] = 2𝑛 [∏𝑘=2(𝑘 + 1)], onde 𝑛 é obtido através do
termo geral da P.A:
𝑎𝑛 = 𝑎1 + (𝑛 − 1)𝑟 ⟹ 5 = 2 + (𝑛 − 1) ∴ 𝑛 = 4.
5 5

∏(2𝑘 + 2) = 24 [∏(𝑘 + 1)] = 16[3.4.5.6] = 5760.


𝑘=2 𝑘=2
“2. Obtenha o valor do produto:
∏121 4𝑘.”
12 12

∏ 4𝑘 = 4𝑛 (∏ 𝑘) .
1 1
O número 𝑛 é obtido, novamente, pelo termo geral da P.A:
12 = 1 + (𝑛 − 1), 𝑛 = 12.
12 12
12
∏ 4𝑘 = 2 (∏ 𝑘) = 412 (1.2.3. … .10.11.12) = 412 . 12! = 224 . 12!.
1 1
* 𝒏! = 𝒏(𝒏 − 𝟏)(𝒏 − 𝟐). … . 𝟑. 𝟐. 𝟏.
Observe que obtivemos um número extremamente grande se compararmos ao
somatório de mesmo argumento e mesmos limites. Isto é:
𝟏𝟐 𝟏𝟐

∏ 𝟒𝒌 ≫ ∑ 𝟒𝒌.
𝟏 𝟏
“3. Calcule:
6
𝑎) ∏ √7(𝑘 + 1);
𝑘=2
𝑏) ∏4𝑘=1 5(𝑘 + 5);
5
1
𝑐) ∏ 𝑘.”
𝑘=3 2
6 6 6
𝑛
𝑎) ∏ √7(𝑘 + 1) = ∏ √7√𝑘 + 1 = (√7) [∏ √𝑘 + 1] .
𝑘=2 𝑘=2 𝑘=2
𝑛 é tal que:
6 = 2 + (𝑛 − 1), 𝑛 = 5.
Assim:
6
5
∏ √7(𝑘 + 1) = (√7) [(√2 + 1)(√3 + 1)(√4 + 1)(√5 + 1)(√6 + 1)]
𝑘=2
5
= (√7) √3.4.5.6.7 = √76 . 6.5.4.3 = 7³. 2√6.5.3 = 7³. 2√90 = 7³. 2.3√10
= 2058√10.
4 4

𝑏) ∏ 5(𝑘 + 5) = 54 [∏(𝑘 + 5)] = 54 . 6.7.8.9 = 54 . 3024.


𝑘=1 1
5 5
1 1 𝑛
𝑐) ∏ 𝑘 = ( ) (∏ 𝑘) .
2 2
𝑘=3 𝑘=3
𝑛 é dado por:
5 = 3 + (𝑛 − 1), 𝑛 = 3.
Assim:
5
1 1 3 1 60 30 15
∏ 𝑘 = ( ) (∏5𝑘=3 𝑘) = . 3.4.5 = = = .
𝑘=3 2 2 8 8 4 2

P3. – Produtório de um produto ou um quociente (propriedade multiplicativa):


“O produtório de um produto ou de um quociente é o produto ou quociente dos
produtórios.”
Matematicamente:
𝑏 𝑏 𝑏

𝟏. ∏(𝑓(𝑘). 𝑔(𝑘)) = (∏ 𝑓(𝑘)) . (∏ 𝑔(𝑘))


𝑘=𝑎 𝑘=𝑎 𝑘=𝑎
𝑏 𝑏 𝑏
𝑓(𝑘) 1 ∏𝑏𝑘=𝑎 𝑓(𝑘)
𝟐. ∏ [ ] = (∏ 𝑓(𝑘)) . (∏ )= 𝑏
𝑔(𝑘) 𝑔(𝑘) ∏𝑘=𝑎 𝑔(𝑘)
𝑘=𝑎 𝑘=𝑎 𝑘=𝑎
Demonstração:
1:
𝑏

∏(𝑓(𝑘). 𝑔(𝑘))
𝑘=𝑎
= 𝑓(𝑎). 𝑔(𝑎). 𝑓(𝑎 + 1). 𝑔(𝑎 + 1). … . 𝑓(𝑏 − 1). 𝑔(𝑏 − 1). 𝑓(𝑏). 𝑔(𝑏)
= (𝑓(𝑎). 𝑓(𝑎 + 1). … . 𝑓(𝑏 − 1). 𝑓(𝑏)). (𝑔(𝑎). 𝑔(𝑎 + 1). … . 𝑔(𝑏 − 1). 𝑔(𝑏))
𝑏 𝑏

= (∏ 𝑓(𝑘)) . (∏ 𝑔(𝑘)) .
𝑘=𝑎 𝑘=𝑎
2:
𝑏
𝑓(𝑘) 𝑓(𝑎) 𝑓(𝑎 + 1) 𝑓(𝑏 − 1) 𝑓(𝑏) ∏𝑏𝑘=𝑎 𝑓(𝑘)
∏[ ]= . .…. . = .
𝑔(𝑘) 𝑔(𝑎) 𝑔(𝑎 + 1) 𝑔(𝑏 − 1) 𝑔(𝑏) ∏𝑏𝑘=𝑎 𝑔(𝑘)
𝑘=𝑎
Ou, de maneira análoga à primeira demonstração:
𝑏 𝑏 𝑏
𝑓(𝑘) 1
∏[ ] = (∏ 𝑓(𝑘)) . (∏ )
𝑔(𝑘) 𝑔(𝑘)
𝑘=𝑎 𝑘=𝑎 𝑘=𝑎
1 1 1 1
= 𝑓(𝑎). 𝑓(𝑎 + 1). … . 𝑓(𝑏 − 1). 𝑓(𝑏). . .…. .
𝑔(𝑎) 𝑔(𝑎 + 1) 𝑔(𝑏 − 1) 𝑔(𝑏)
𝑓(𝑎) 𝑓(𝑎 + 1) 𝑓(𝑏 − 1) 𝑓(𝑏)
= . .…. .
𝑔(𝑎) 𝑔(𝑎 + 1) 𝑔(𝑏 − 1) 𝑔(𝑏)
∏𝑏𝑘=𝑎 𝑓(𝑘)
= 𝑏 .
∏𝑘=𝑎 𝑔(𝑘)
“1. Calcule:
4
∏𝑘=2(𝑘³ − 1).”
Temos o seguinte produtório:
4

∏(𝑘 3 − 1) .
𝑘=2
Temos que o argumento deste produtório é uma diferença de cubos do tipo
𝑎³ − 𝑏 3 = (𝑎 − 𝑏)(𝑎2 + 𝑎𝑏 + 𝑏 2 ).
Assim:
4 4

∏(𝑘 3 − 1) = ∏[(𝑘 − 1)(𝑘 2 + 𝑘 + 1)].


𝑘=2 𝑘=2
Aplicando a P3:
4 4 4

∏[(𝑘 − 1)(𝑘 2 + 𝑘 + 1)] = [∏(𝑘 − 1)] . [∏(𝑘 2 + 𝑘 + 1)] .


𝑘=2 𝑘=2 𝑘=2
Portanto:
4 4 4

∏(𝑘 3 − 1) = [∏(𝑘 − 1)] . [∏(𝑘 2 + 𝑘 + 1)]


𝑘=2 𝑘=2 𝑘=2
= [(2 − 1)(3 − 1)(4 − 1)][(2 + 2 + 1)(3 + 3 + 1)(42 + 4 + 1)]
2 2

= 1.2.3.7.13.21 = 11466.
“2. Calcule:
13
𝑘
∏ .”
𝑘=1 𝑘+1
13
𝑘 ∏13𝑘=1 𝑘 1. 𝟐. 𝟑. 𝟒. 𝟓. 𝟔. 𝟕. 𝟖. 𝟗. 𝟏𝟎. 𝟏𝟏. 𝟏𝟐. 𝟏𝟑 1
∏ = 13 = = .
𝑘 + 1 ∏𝑘=1 𝑘 + 1 𝟐. 𝟑. 𝟒. 𝟓. 𝟔. 𝟕. 𝟖. 𝟗. 𝟏𝟎. 𝟏𝟏. 𝟏𝟐. 𝟏𝟑. 14 14
𝑘=1
O exemplo acima é o denominado Produto Telescópico, pois, de maneira análoga à
Soma Telescópica, os termos em negrito do produto irão se cancelar.
O Produto Telescópico, de maneira semelhante à Soma Telescópica, será visto e
discutido num item dedicado apenas a este.
“3. (SIMULADO IME) – Simplifique a expressão:
(6.12.18. … .300)
."
(2.6.10.14. … .98). (4.8.12.16. … .100)
Perceba que, no numerador desta fração temos o produto dos múltiplos de 6, os quais
variam de 1 até 50, pois 6.50 = 300.
Assim, no numerador temos o seguinte produto:
𝟓𝟎

(𝟔. 𝟏𝟐. 𝟏𝟖. … . 𝟑𝟎𝟎) = ∏ 𝟔𝒌.


𝒌=𝟏
No denominador, temos o produto de dois produtórios:
(2.6.10.14. … .98) e (4.8.12.16. … .100).
Como estamos lidando com um produto, podemos fazer a seguinte manipulação, sem
que alteremos o denominador:
(2.6.10.14. … .98). (4.8.12.16. … .100)
𝟓𝟎

= (𝟐. 𝟒. 𝟔. 𝟖. 𝟏𝟎. 𝟏𝟐. 𝟏𝟒. 𝟏𝟔. … . 𝟗𝟖. 𝟏𝟎𝟎) = ∏ 𝟐𝒌 .


𝒌=𝟏
* Pedimos para que você, leitor, verifique o exposto acima.
Assim, temos que:
(𝟔. 𝟏𝟐. 𝟏𝟖. … . 𝟑𝟎𝟎) ∏𝟓𝟎
𝒌=𝟏 𝟔𝒌
= 𝟓𝟎 .
(𝟐. 𝟔. 𝟏𝟎. 𝟏𝟒. … . 𝟗𝟖). (𝟒. 𝟖. 𝟏𝟐. 𝟏𝟔. … . 𝟏𝟎𝟎) ∏𝒌=𝟏 𝟐𝒌
Aplicando a P2 em ambos os produtórios, obtemos:
∏𝟓𝟎
𝒌=𝟏 𝟔𝒌 𝟔𝟓𝟎 ∏𝟓𝟎𝒌=𝟏 𝒌 𝟔𝟓𝟎
= 𝟓𝟎 𝟓𝟎 = 𝟓𝟎 = 𝟑𝟓𝟎 .
∏𝟓𝟎
𝒌=𝟏 𝟐𝒌 𝟐 ∏ 𝒌=𝟏 𝒌 𝟐
“4. Calcule:
1.3.5. … .99.101
."
11.13.15. … .89.91
Percebamos que 1.3.5. … .99.101 é o produto dos ímpares de 1 até 101.
Lembrando que um número ímpar qualquer é dado pela fórmula 𝑛í𝑚𝑝𝑎𝑟 = 2𝑘 − 1, o
número 𝑘 assumirá os valores de 1 até 51, pois:
2.1 − 1 = 1;
2.51 − 1 = 101.
Portanto:
51

1.3.5. … .99.101 = ∏(2𝑘 − 1) .


𝑘=1
O denominador da fração 11.13.15. … .89.91 é o produto dos ímpares de 11 até 91.
Assim, o número 𝑘 assumirá os valores de 6 até 46, pois:
2.6 − 1 = 11;
2.46 − 1 = 91.
Portanto:
46

11.13.15. … .89.91 = ∏(2𝑘 − 1) .


𝑘=6
Então:
1.3.5. … .99.101 ∏51
𝑘=1(2𝑘 − 1)
= 46 .
11.13.15. … .89.91 ∏𝑘=6(2𝑘 − 1)
Sabemos que:
51 5 46 51

∏(2𝑘 − 1) = [∏(2𝑘 − 1)] . [∏(2𝑘 − 1)] . [ ∏ (2𝑘 − 1)]


𝑘=1 𝑘=1 𝑘=6 𝑘=47
Esta é a propriedade recursiva de um produtório, a qual será demonstrada
posteriormente.
Porém, é intuitivo que:
𝑏 𝑝 𝑏

∏ 𝑓(𝑘) = (∏ 𝑓(𝑘)) . (∏ 𝑓(𝑘)) .


𝑘=𝑎 𝑘=𝑎 𝑝+1
Onde 𝑝 é tal que 𝑝 ∈ [𝑎, 𝑏[ (𝑎 ≤ 𝑝 < 𝑏).
Assim:
51 5 𝟒𝟔 51
∏𝑘=1(2𝑘 − 1) [∏𝑘=1(2𝑘 − 1)] . [∏𝒌=𝟔(𝟐𝒌 − 𝟏)] . [∏𝑘=47(2𝑘 − 1)]
46 = 𝟒𝟔
∏𝑘=6(2𝑘 − 1) ∏𝒌=𝟔(𝟐𝒌 − 𝟏)
5 51

= [∏(2𝑘 − 1)] . [ ∏ (2𝑘 − 1)] .


𝑘=1 𝑘=47
E, por fim:
5 51

[∏(2𝑘 − 1)] . [ ∏ (2𝑘 − 1)]


𝑘=1 𝑘=47
= (2.1 − 1)(2.2 − 1)(2.3 − 1)(2.4 − 1)(2.5 − 1)(2.47 − 1)(2.48 − 1)(2.49 − 1)(2.50 − 1). (2.51 − 1)
= 3.5.7.9.93.95.97.99.101.
* Será comum, ao final de um produtório cujo valor numérico é muito extenso, apenas
deixarmos indicado seu resultado, como no exemplo acima.
5
𝑥 6 −1
“5. Obtenha o valor de ∏ ( ).”
𝑥−1 3
Temos o seguinte produtório:
5
𝑥6 − 1
∏( ).
𝑥−1
𝑘=3
Podemos simplificar o argumento do produtório utilizando a diferença de dois cubos
da seguinte maneira:
𝒙𝟔 − 𝟏 (𝑥 2 − 1)(𝑥 4 + 𝑥 2 + 1) (𝑥 + 1)(𝑥 − 1)(𝑥 4 + 𝑥 2 + 1)
= =
𝒙−𝟏 (𝑥 − 1) (𝑥 − 1)
= (𝒙 + 𝟏)(𝒙𝟒 + 𝒙𝟐 + 𝟏).
Assim:
5 5
𝑥6 − 1
∏( ) = [∏((𝑥 + 1)(𝑥 4 + 𝑥 2 + 1))] .
𝑥−1
𝑘=3 𝑘=3

Aplicando a P3:
5 5 5

[∏((𝑥 + 1)(𝑥 4 + 𝑥 2 + 1))] = [∏(𝑥 + 1)] . [∏(𝑥 4 + 𝑥 2 + 1)]


𝑘=3 𝑘=3 𝑘=3
= (3 + 1)(4 + 1)(5 + 1)(3 + 3 + 1)(4 + 4 + 1)(54 + 52 + 1)
4 2 4 2

= 4.5.6. (34 + 32 + 1)(44 + 42 + 1)(54 + 52 + 1)


= 120. (91). (273). (651).
P4. – Produtório de uma variável linear:
“O produtório de uma variável linear é o fatorial do limite superior do
produtório.”
Em linguagem matemática:
𝑛

∏ 𝑘 = 𝑛!
𝑘=1
Para esta propriedade, é importante salientar que nos preocuparemos apenas com o
caso no qual o limite inferior é 1 e o limite superior é um número 𝑛, com 𝑛 > 1.
Pois, o caso geral ∏𝑏𝑘=𝑎 𝑘 utiliza o conceito de Fatorial de Pochhammer2, o qual foge
do escopo da teoria.
Assim, façamos a demonstração do caso indicado acima.
Demonstração:
𝑛

∏ 𝑘 = 1.2.3. … . (𝑛 − 3). (𝑛 − 2). (𝑛 − 1). 𝑛 = 𝒏!


𝑘=1
“1. Calcule os seguintes produtórios:
𝑎) ∏16𝑘=1 𝑘;
𝑏) ∏8𝑘=2 𝑘;
𝑐) ∏6𝑘=3 𝑘.”
16

𝑎) ∏ 𝑘 = 1.2.3. … .13.14.15.16 = 16!


𝑘=1
8

𝑏) ∏ 𝑘 = 2.3.4. … .6.7.8 = 8!
𝑘=2
6
2.3.4.5.6 6!
𝑐) ∏ 𝑘 = 3.4.5.6 = =
2 2
𝑘=3
“2. Calcule:
7
𝑘(𝑘 2 −5𝑘+6)
∏ (𝑘−3)
.”
𝑘=4
Temos que:
𝑘² − 5𝑘 + 6 = (𝑘 − 3)(𝑘 − 2).
Assim:
7 7 7
𝑘(𝑘 2 − 5𝑘 + 6) 𝑘(𝒌 − 𝟑)(𝑘 − 2)
∏ =∏ = ∏ 𝑘(𝑘 − 2) .
(𝑘 − 3) (𝒌 − 𝟑)
𝑘=4 𝑘=4 𝑘=4
Aplicando a P3:
7 7 7

∏ 𝑘(𝑘 − 2) = [∏ 𝑘] . [∏(𝑘 − 2)] .


𝑘=4 𝑘=4 𝑘=4

2
Fonte: Wolfram Alpha e Wolfram Math World.
7 7

[∏ 𝑘 ] . [∏(𝑘 − 2)] = [4.5.6.7]. [2.3.4.5] = [4.5.6.7]. [1.2.3.4.5]


𝑘=4 𝑘=4
1.2.3.4.5.6.7 7! 7! 5!
= [4.5.6.7]. 5! = [ ] . 5! = . 5! = .
2.3 6 6

P5. – Produtório de uma constante com expoente variável:


“O produtório de uma constante com expoente variável é a própria constante
elevada ao somatório dos expoentes.”
𝑏 1
𝑏 (𝑏2 −𝑎2 +𝑎+𝑏)
∏ 𝛼 𝑘 = 𝛼 ∑𝑘=𝑎 𝑘 = 𝛼 2 , onde α é uma constante real não nula e
𝑘=𝑎
estritamente positiva.
Se o produtório se inicia em 𝑘 = 1 (limite inferior) e termina em um 𝑛 qualquer,
vale que:
𝑛
𝑛 1
∏ 𝛼 𝑘 = 𝛼 ∑𝑘=1 𝑘 = 𝑎2𝑛(𝑛+1) .
𝑘=1
Demonstração:
Primeiramente o caso particular no qual o produtório se inicia em 𝑘 = 1 e se encerra
em um 𝑛 qualquer:
𝑛
𝟏
∏ 𝛼 𝑘 = 𝛼 1 . 𝛼². 𝛼³. … . 𝛼 𝑛−2 . 𝛼 𝑛−1 . 𝛼 𝑛 = 𝛼 1+2+3+⋯+(𝑛−1)+𝑛 = 𝜶𝟐𝒏(𝒏+𝟏) .
𝑘=1
* Utilizamos na demonstração acima a soma dos 𝑛 primeiros números naturais ou a
soma dos 𝑛 ternos de uma progressão aritmética de razão unitária.
Agora, a demonstração do caso geral, no qual o produtório se inicia em um 𝑎 inteiro
não nulo qualquer, e se encerra em um 𝑏, também inteiro e não nulo.
𝑏

∏ 𝛼 𝑘 = 𝛼 𝑎 . 𝛼 𝑎+1 . 𝛼 𝑎+2 . … . 𝛼 𝑏−2 . 𝛼 𝑏−1 . 𝛼 𝑏


𝑘=𝑎
= 𝛼 𝑎+(𝑎+1)+(𝑎+2)+⋯+(𝑏−2)+(𝑏−1)+𝑏 .
Veja que a soma dos termos equidistantes dos extremos é sempre a mesma:
𝑎 + 𝑏 = 𝑎 + 𝑏;
(𝑎 + 1) + (𝑏 − 1) = 𝑎 + 𝑏;
(𝑎 + 2) + (𝑏 − 2) = 𝑎 + 𝑏;
...
Temos, ainda, que o número de termos desta soma é dado por:
𝑏 = 𝑎 + (𝑛𝑡 − 1). 1 ⟹ 𝑛𝑡 = 𝑏 − 𝑎 + 1.
Então, seja S esta soma.
S = 𝑎 + (𝑎 + 1) + (𝑎 + 2) + ⋯ + (𝑏 − 2) + (𝑏 − 1) + 𝑏;
S = 𝑏 + (𝑏 − 1) + (𝑏 − 2) + ⋯ + (𝑎 + 2) + (𝑎 + 1) + 𝑎.
2S = (𝑎 + 𝑏) + ((𝑎 + 1) + (𝑏 − 1)) + ⋯ + ((𝑏 − 1) + (𝑎 + 1)) + (𝑏 + 𝑎).
Porém, temos que estas somas são todas iguais a 𝑎 + 𝑏.
Assim:
2S = (𝑎 + 𝑏) + (𝑎 + 𝑏) + ⋯ + (𝑎 + 𝑏) + (𝑎 + 𝑏);
2S = (𝑎 + 𝑏)(1 + 1 + ⋯ + 1 + 1) ⟸ (𝑏 − 𝑎 + 1) vezes.
1 1
S = (𝑎 + 𝑏)(𝑏 − 𝑎 + 1) = (𝑏 2 − 𝑎2 + 𝑎 + 𝑏).
2 2
Portanto:
1
𝑎+(𝑎+1)+(𝑎+2)+⋯+(𝑏−2)+(𝑏−1)+𝑏 (𝑏²−𝑎2 +𝑎+𝑏)
𝛼 = 𝛼2 .
* É de absoluta importância que você, caríssimo leitor, neste ponto da teoria, possua
uma certa afinidade com as propriedades aqui referenciadas, como a potenciação,
radiciação, sequências numéricas e etc, pois, como pôde ver, toda a nossa teoria está
alicerçada nestas.
“1. Se:
𝑝
∏3 𝜋 2𝑘 = 𝜋, obtenha o valor de 𝑝².”
Temos que:
𝑝 𝑝

∏ 𝜋 2𝑘 = ∏(𝜋 2 )𝑘 = 𝜋.
3 3
Expandindo o produtório, obtemos:
(𝜋 2 )3
. (𝜋 2 )4 . (𝜋 2 )5 . … . (𝜋 2 )𝑝−1 . (𝜋 2 )𝑝 = 𝜋;
𝜋 2(3+4+5+⋯+(𝑝−1)+𝑝) = 𝜋.
Tomando o logaritmo de base 𝜋 de ambos os lados, obtemos:
log 𝜋 𝜋 2(3+4+5+⋯+(𝑝−1)+𝑝) = log 𝜋 𝜋;
2(3 + 4 + 5 + ⋯ + (𝑝 − 1) + 𝑝) = 1;
Veja que:
3 + 𝑝 = 3 + 𝑝;
4 + (𝑝 − 1) = 3 + 𝑝;
5 + (𝑝 − 2) = 3 + 𝑝;
...
Ou seja, usaremos novamente a soma dos termos de uma P.A, pois os termos
equidistantes dos extremos possuem soma constante.
Assim, seja 𝑛 o número de termos desta soma:
𝑝 = 3 + (𝑛 − 1); 𝑛 = 𝑝 − 2.
1
Utilizando a soma da P.A S𝑛 = (𝑎1 + 𝑎𝑛 ) neste caso, obtemos:
2
1
3 + 4 + 5 + ⋯ + (𝑝 − 1) + 𝑝 = (3 + 𝑝)(𝑝 − 2).
2
Assim:
1
2. (3 + 𝑝)(𝑝 − 2) = 1;
2
(3 + 𝑝)(𝑝 − 2) = 1;
𝑝² + 𝑝 − 7 = 0.
√29 1
A única solução desta equação que satisfaz o problema é 𝑝 = − , pois o limite
2 2
superior deverá ser positivo.
Assim:
√29 1 29 1 √29 1 15 √29 15 − √29
𝑝= − ⟹ 𝑝2 = + − 2. . = − = .
2 2 4 4 2 2 2 2 2
“2. Sendo 𝑚 um natural não nulo, prove que:
𝑚 1
∏ 6𝑘 . 𝑘 = 𝑚! 62𝑚(𝑚+1) , ∀ 𝑚 ∈ ℕ∗ .”
𝑘=1
Temos o seguinte produtório:
𝑚

∏ 6𝑘 . 𝑘 .
𝑘=1
Apliquemos a P3 para separarmos este produtório em outros dois produtórios:
𝑚 𝑚 𝑚

∏ 6𝑘 . 𝑘 = (∏ 6𝑘 ) . (∏ 𝑘) .
𝑘=1 𝑘=1 𝑘=1
O primeiro produtório é tal que:
𝑚

∏ 6𝑘 = 6¹. 6². 6³. … . 6𝑚−1 . 6𝑚 = 61+2+3+⋯+(𝑚−1)+𝑚 .


𝑘=1
1
A soma 1 + 2 + 3 + ⋯ + (𝑚 − 1) + 𝑚 é equivalente a 𝑚(𝑚 + 1).
2
Assim:
𝑚
1
∏ 6𝑘 = 62𝑚(𝑚+1) .
𝑘=1
O segundo produtório é tal que:
𝑚

∏ 𝑘 = 1.2.3. … . (𝑚 − 1). 𝑚 = 𝑚!.


𝑘=1
Portanto:
𝑚 𝑚 1
𝑚(𝑚+1)
∏𝑘=1 6𝑘 . 𝑘 = (∏𝑘=1 6𝑘 ). (∏𝑚
𝑘=1 𝑘 ) = 6
2 . 𝑚!, como queríamos demonstrar.
“3. Obtenha o valor de
𝑚+𝑛
∏6 𝑒 𝑘−6 .”
Temos o seguinte produto:
𝑚+𝑛

∏ 𝑒 𝑘−6 = 𝑒 6−6 . 𝑒 7−6 . … . 𝑒 𝑚+𝑛−6 = 1. 𝑒 1 . 𝑒 2 . … . 𝑒 𝑚+𝑛−6


6
= 𝑒 1+2+⋯+(𝑚+𝑛−6) .
No expoente, temos a seguinte soma:
1 + 2 + ⋯ + (𝑚 + 𝑛 − 6), a qual é a soma de uma P.A de primeiro termo 1, razão 1,
último termo (𝑚 + 𝑛 − 6) e número de termos 𝑥 dado por
𝑚 + 𝑛 − 6 = 1 + (𝑥 − 1) ⟹ 𝑥 = 𝑚 + 𝑛 − 6.
Assim, esta soma é tal que:
1
1 + 2 + ⋯ + (𝑚 + 𝑛 − 6) = (𝑚 + 𝑛 − 6)(1 + (𝑚 + 𝑛 − 6)
2
1
= (𝑚 + 𝑛 − 6)(𝑚 + 𝑛 − 5).
2
Portanto:
𝑚+𝑛
𝟏
(𝒎+𝒏−𝟔)(𝒎+𝒏−𝟓)
∏ 𝑒 𝑘−6 = 𝑒 1+2+⋯+(𝑚+𝑛−6) = 𝒆𝟐 .
𝑘=6
P6. – O logaritmo de um produtório:
“O logaritmo de um produtório é o somatório dos logaritmos.”
Sejam 𝑟, 𝑚 e 𝑛 reais tais que 𝑟 > 0 ∧ 𝑟 ≠ 1 ∧ (𝑚, 𝑛) > 0,
em linguagem matemática, temos que:
𝑛 𝑛

log 𝑟 (∏ 𝑘) = ∑ (log 𝑟 𝑘) .
𝑘=𝑚 𝑘=𝑚
O estudo completo e aprofundado dos logaritmos será efetuado no item seguinte,
porém esta propriedade será apresentada agora, juntamente com as propriedades
gerais, justamente para que o próximo item flua com mais facilidade.
Demonstração:
Do estudo dos logaritmos, sabemos que log 𝑟 (𝑚. 𝑛) = log 𝑟 𝑚 + log 𝑟 𝑛, com os
números 𝑚, 𝑛 e 𝑟 satisfazendo as restrições acima apresentadas.
Então, o logaritmo de um produtório é tal que:
𝑛

log 𝑟 (∏ 𝑘) = log[𝑚. (𝑚 + 1). (𝑚 + 2). … . (𝑛 − 2). (𝑛 − 1). 𝑛].


𝑘=𝑚
Aplicando a supracitada propriedade logarítmica, obtemos:
log 𝑟 [𝑚. (𝑚 + 1). (𝑚 + 2). … . (𝑛 − 2). (𝑛 − 1). 𝑛]
= log 𝑟 𝑚 + log 𝑟 (𝑚 + 1) + log 𝑟 (𝑚 + 2) + ⋯ + log 𝑟 (𝑛 − 2) + log 𝑟 (𝑛 − 1) + log 𝑟 𝑛
𝒏

= ∑ (𝐥𝐨𝐠 𝒓 𝒌).
𝒌=𝒎
No apêndice sobre logaritmos do próximo capítulo, apresentaremos todas as
propriedades e suas respectivas demonstrações, além de uma brevíssima introdução ao
Logaritmo de um Número Complexo, a qual será feita no intuito de fortificar o
ferramental teórico para o capítulo 3, cujo objetivo principal é o aprofundamento dos
conceitos até lá apresentados.
“1. Calcule:
𝑝−𝑞
∑1 ln 𝑘 , onde 𝑝 − 𝑞 > 0, onde ln 𝑘 denota o logaritmo na base 𝑒 de 𝑘.”
Seja a soma:
𝑝−𝑞

∑ ln 𝑘 = ln 1 + ln 2 + ⋯ + ln(𝑝 − 𝑞).
𝑘=1
Temos, pela propriedade da soma de logaritmos, que:
𝑝−𝑞

ln 1 + ln 2 + ⋯ + ln(𝑝 − 𝑞) = ln(1.2. … . (𝑝 − 𝑞)) = ln [∏ 𝑘] .


1
Este produto é, por sua vez, tal que:
𝑝−𝑞

∏ 𝑘 = 1.2. … . (𝑝 − 𝑞) = (𝑝 − 𝑞)!.
1
Portanto:
𝑝−𝑞 𝑝−𝑞

∑ ln 𝑘 = ln 1 + ln 2 + ⋯ + ln(𝑝 − 𝑞) = ln [∏ 𝑘] = ln[(𝑝 − 𝑞)!].


𝑘=1 1
“2. Prove que:
𝑏+5
𝑘
log 2 (∏ ) = log 2 𝑎 − log 2 (𝑏 + 6), onde 𝑎 e 𝑏 são reais não nulos, não
𝑘=𝑎 𝑘+1
negativos e não unitários.”
Temos que:
𝑏+5
𝑘 𝑎 𝑎+1 𝑎+2 𝑏+5
log 2 (∏ ) = log 2 (( ).( ).( ).….( ))
𝑘+1 𝑎+1 𝑎+1+1 𝑎+2+1 𝑏−𝑎+1
𝑘=𝑎
𝑎 𝑎+1 𝑎+2 𝑎+3 𝑏+4 𝑏+5
= log 2 (( ).( ).( ).( ).….( ).( )) .
𝑎+1 𝑎+2 𝑎+3 𝑎+4 𝑏+5 𝑏+6
𝑎
Perceba que todos os termos irão se cancelar, com exceção de .
𝑏+6
𝑎
Assim, temos que este produto é igual a log 2 ( ), que por sua vez é equivalente a
𝑏+6
log 2 𝑎 − log 2 (𝑏 + 6), como queríamos demonstrar.
“3. Obtenha o valor de:
∑900
2 (ln(4𝑘 − 5)).”
Temos a seguinte soma:
900

∑(ln(4𝑘 − 5)) = ln 3 + ln 7 + ln 10 + ⋯ + ln 3595 .


𝑘=2
Observemos que os logaritmandos formam uma progressão aritmética de razão 4, pois
a sequência geradora destes é tal que:
𝑎𝑘 = 4𝑘 − 5 ⟹ 𝑎2 = 4.2 − 5 = 3;
𝑎3 = 4.3 − 5 = 7;
E assim por diante.
Então, segue que:
ln 3 + ln 7 + ln 10 + ⋯ + ln 900 = ln(3.7.10. … .3595)
900

= ln (∏(4𝑘 − 5)) .
𝑘=2
V. Logaritmos (revisão, propriedades e exercícios) e
o Produto Telescópico
1. – Definição de logaritmo e função logarítmica
Dados os números reais 𝑎 e 𝑏, com 𝑏 > 0, 𝑎 > 0 ∧ ≠ 1, verifica-se que:
𝑎 𝑥 = 𝑏 ⟺ 𝒙 = log 𝒂 𝒃.
O número 𝒂 é denominado base do logaritmo.
O número 𝒃 é denominado logaritmando.
O número 𝒙 é denominado logaritmo de 𝑏 na base 𝑎, o qual, garantida sua existência
pelas condições impostas acima, é o expoente que se deve elevar a base 𝒂 para que
obtenhamos 𝒃.
Do exposto acima, segue que a exponencial e o logaritmo são operações inversas.
Dada a função 𝑓: ℝ∗+ → ℝ, definida por 𝒇(𝒙) = 𝐥𝐨𝐠 𝒂 𝒙, com 𝑎 > 0 ∧ ≠ 1,
prova-se que:
1. – A função 𝑓(𝑥) = log 𝑎 𝑥 é injetora, pois existem 𝑥1 e 𝑥2 pertencentes ao domínio
da função, tais que:
𝑥1 ≠ 𝑥2 ⟹ 𝑓(𝑥1 ) ≠ 𝑓(𝑥2 ).
Ou, de maneira equivalente:
𝑥1 = 𝑥2 ⟹ 𝑓(𝑥1 ) = 𝑓(𝑥2 ).
2. – A função 𝑓(𝑥) = log 𝑎 𝑥 é sobrejetora, pois:
CD(𝑓) = Im(𝑓) = ℝ.
3. – Dos subitens 1 e 2, infere-se que a função 𝑓(𝑥) = log 𝑎 𝑥 é bijetora e, portanto,
admite uma função inversa 𝒇−𝟏 (𝒙) dada por 𝒇−𝟏 (𝒙) = 𝒂𝒙 .
4. – Se 𝑎 ∈ ]1, + ∞[ ou 𝑎 > 1, a função 𝑓(𝑥) = log 𝑎 𝑥 é estritamente crescente, pois:
∀ 𝒙𝟏 e 𝒙𝟐 ∈ ℝ∗+ , 𝒙𝟏 > 𝒙𝟐 ⟹ 𝒇(𝒙𝟏 ) > 𝒇(𝒙𝟐 ).
5. – Se 𝑎 ∈ ]0, 1[ ou 0 < 𝑎 < 1, a função 𝑓(𝑥) = log 𝑎 𝑥 é estritamente decrescente,
pois:
∀ 𝒙𝟏 e 𝒙𝟐 ∈ ℝ∗+ , 𝒙𝟏 > 𝒙𝟐 ⟹ 𝒇(𝒙𝟏 ) < 𝒇(𝒙𝟐 ).
Fazendo o uso de uma linguagem mais simples, a operação 𝒙 = 𝐥𝐨𝐠 𝒂 𝒃 nos fornece o
expoente que devemos elevar a base 𝒂 para obtermos 𝒃, ao passo que sua operação
inversa 𝒂𝒙 = 𝒃 nos fornece o “valor” da potência obtida ao fazermos 𝒂𝒙 .
6. – O gráfico da função logarítmica é exibido abaixo.

Fonte: www.educacao.globo.com.
7. – Dos gráficos acima, deduzimos que os interceptos da função (pontos nos quais o
gráfico intercepta os eixos coordenados) são:
Eixo ⃗⃗⃗⃗⃗
𝐎𝒚: 𝑓(𝑥) = log 𝑎 𝑥 ⟹ 𝑓(0) = log 𝑎 0.
Tomando 𝒑 = 𝒇(𝟎), segue que 𝒑 = 𝐥𝐨𝐠 𝒂 𝟎 ⟹ 𝒂𝒑 = 𝟎.
Concluímos, então, que a função logarítmica não intercepta o eixo das ordenadas,
pois, de 𝒂𝒑 = 𝟎, se concretiza um absurdo:
∄ 𝑝 tal que 𝑎𝒑 = 0, 𝑎 > 0 e 𝑎 ≠ 1.
Eixo ⃗⃗⃗⃗⃗
𝐎𝒙: 0 = log 𝑎 𝑥 ⟹ 𝒙 = 𝟏.
A função logarítmica 𝑓(𝑥) = log 𝑎 𝑥 sempre intercepta o eixo das abcissas no ponto
(1; 0).
8. – Os gráficos de 𝑓(𝑥) = log 𝑎 𝑥 e 𝑓 −1 (𝑥) = 𝑎 𝑥 são simétricos em relação à reta
bissetriz dos quadrantes ímpares (𝑏1,3 ), de equação 𝑦 = 𝑥.

Fonte: www.brainly.com.

As bases mais trabalhadas na matemática são as bases 𝟐, 𝟏𝟎 e 𝒆.


A base 2 é amplamente utilizada na computação, nos sistemas binários, nos sistemas
lógicos, etc.
A base 10 é a mais comum no estudo dos logaritmos, sua notação é, simplesmente:
𝐥𝐨𝐠 𝟏𝟎 𝒙 = 𝐥𝐨𝐠 𝒙.
O logaritmo de base 10 é utilizado na química, na meteorologia, na física-acústica, etc.
A base 𝒆, por apresentar propriedades interessantíssimas, é exaustivamente utilizada
no cômputo dos fenômenos naturais, como nas ciências biológicas, na Termologia,
incessantemente no Cálculo Diferencial e Integral, na economia, na modelagem de
𝑬𝒂
fenômenos químicos, como a Equação de Arrhenius (𝒌 = 𝐀𝒆−𝑹𝑻 ) e a equação de
Nernst (∆𝐆 = ∆𝐆𝟎 + 𝐑𝐓. 𝐥𝐧 𝐐), na modelagem de Equações Diferenciais, como
𝑑𝑥 𝑥 𝑑𝑦 𝑑2𝑦
= e = 𝑘. , na Mecânica Newtoniana, como na equação do
𝑑𝑡 𝑡 𝑑𝑥 𝑑𝑥 2
𝑑2𝑥 𝑘
Movimento Harmônico Simples, a qual é dada por + 𝑥 = 0 e em muitas outras
𝑑𝑡 2 𝑚
áreas, sendo a base logarítmica de mais relevância no espectro das ciências gerais.
As notações dos logaritmos e exponenciais de base 𝑒 são, respectivamente:
𝐥𝐨𝐠 𝒆 𝒙 = 𝐥𝐧 𝒙 e 𝒆𝒙 .
As mais corriqueiras denominações para o número 𝒆 incluem Constante de Euler,
Número de Euler, Número Neperiano, entre outros.
A notação 𝒆 é em homenagem ao matemático suíço Leonhard Euler, o qual, em
paralelo com John Napier, idealizador do conceito de logaritmo, nas totalidades de
seus trabalhos independentes, concluíram que:
𝑒 ≅ 2,718...
𝑛+1 𝑛
O número 𝒆 é definido, no Cálculo, como o limite para o qual a função 𝑓(𝑛) = ( )
𝑛
se aproxima quando 𝑛 tende a infinito:
𝒏+𝟏 𝒏
𝒆 = 𝐥𝐢𝐦 ( ) .
𝒏→∞ 𝒏
É também definido como:

𝟏
𝒆=∑ .
𝒌!
𝒌=𝟎
Ainda no campo do Cálculo Diferencial e Integral, o aparecimento deste número é
deveras usual.
1
Como, por exemplo, no cômputo da integral da função :
𝑥
1
∫ 𝑑𝑥 = 𝐥𝐧|𝒙| + 𝒄.
𝑥
Ou, ainda, no cômputo da integral da função exponencial 𝑒 𝑥 :
∫ 𝑒 𝑥 𝑑𝑥 = 𝒆𝒙 + 𝒄 .
Apresentaremos, no decorrer deste capítulo, a famosíssima Fórmula de Euler, a qual
relaciona os Números Complexos com as funções exponenciais.
O conhecimento desta fórmula é essencial para a apresentação dos
Logaritmos Complexos, os quais questionam e desafiam nosso senso comum a
respeito da definição de logaritmo sob os Números Reais.
A fórmula de Euler é dada por:
𝒆𝒊𝜽 = 𝐜𝐨𝐬 𝜽 + 𝒊. 𝐬𝐞𝐧 𝜽, onde 𝑖 = √− 1 é a unidade imaginária.
Salientamos, desde já, que a definição de Logaritmo Complexo será feita de maneira
breve, uma vez que esta, em sua totalidade, abandona o escopo desta teoria.

2. – Propriedades dos logaritmos


As principais propriedades operatórias dos logaritmos, as quais serão discutidas e
demonstradas neste apêndice, são:
P1. – Logaritmo de um produto;
P2. – Logaritmo de um quociente;
P3. – Logaritmo de uma potência;
P4. – Mudança de base;
P5. – Cologaritmo;
P6. – Antilogaritmo e Consequência da Definição.
P1. – Logaritmo de um produto:
“O logaritmo de um produto é a soma dos logaritmos.”
Matematicamente:
log 𝑟 (𝑚𝑛) = log 𝑟 𝑚 + log 𝑟 𝑛, onde 𝑟 > 0 ∧ 𝑏 ≠ 1, (𝑚, 𝑛) > 0.
Já fora demonstrado, nas propriedades dos produtórios, que:
𝑛 𝑛

log 𝑟 (∏ 𝑘) = ∑ (log 𝑟 𝑘).


𝑘=𝑚 𝑘=𝑚
Porém, demonstraremos esta propriedade sem o uso dos produtórios.
Demonstração:
Sejam:
𝑥 = log 𝑟 𝑚;
𝑦 = log 𝑟 𝑛;
𝑧 = log 𝑟 (𝑚𝑛).
Assim, da definição de logaritmo 𝐥𝐨𝐠 𝒂 𝒃 = 𝒄 ⟺ 𝒂𝒄 = 𝒃, temos:
I: 𝑟 𝑥 = 𝑚;
II: 𝑟 𝑦 = 𝑛;
III: 𝑟 𝑧 = 𝑚𝑛.
Fazendo I.II, obtemos:
I.II ⟹ 𝑟 𝑥 𝑟 𝑦 = 𝑚𝑛 ⟹ 𝑟 𝑥+𝑦 = 𝑚𝑛.
Mas 𝑚𝑛 = 𝑟 𝑧 :
𝑟 𝑥+𝑦 = 𝑚𝑛 ⟹ 𝑟 𝑥+𝑦 = 𝑟 𝑧 ⟹ 𝑥 + 𝑦 = 𝑧.
𝐥𝐨𝐠 𝒓 𝒎 + 𝐥𝐨𝐠 𝒓 𝒏 = 𝐥𝐨𝐠 𝒓 (𝒎𝒏), como queríamos demonstrar.

P2. – Logaritmo de um quociente:


“O logaritmo de um quociente é a diferença dos logaritmos.”
Matematicamente:
𝑚
log 𝑟 ( ) = log 𝑟 𝑚 − log 𝑟 𝑛.
𝑛
Esta propriedade é uma consequência direta da P1.
Demonstração:
Novamente, sejam:
𝑥 = log 𝑟 𝑚;
𝑦 = log 𝑟 𝑛;
𝑚
𝑧 = log 𝑟 ( ) .
𝑛
Da definição de logaritmo:
I: 𝑟 𝑥 = 𝑚;
II: 𝑟 𝑦 = 𝑛;
𝑚
III: 𝑟 𝑧 = .
𝑛
Fazendo I/II, ou seja, o quociente entre I e II, obtemos:
𝑟𝑥 𝑚 𝑚
I/II ⟹ = ⟹ 𝑟 𝑥−𝑦 = .
𝑟𝑦 𝑛 𝑛
𝑚 𝑧
Mas =𝑟 :
𝑛
𝑚
𝑟 𝑥−𝑦 = ⟹𝑟 𝑥−𝑦
= 𝑟 𝑧 ⟹ 𝑥 − 𝑦 = 𝑧.
𝑛
𝒎
𝐥𝐨𝐠 𝒓 𝒎 − 𝐥𝐨𝐠 𝒓 𝒏 = 𝐥𝐨𝐠 𝒓 ( ), demonstrando a propriedade.
𝒏

P3. – Logaritmo de uma potência:


“O logaritmo de uma potência é o produto do expoente desta pelo logaritmo.”
log 𝑟 𝑚𝑝 = 𝑝 . log 𝑟 𝑚, onde 𝑝 é um real não nulo.
Esta propriedade também será demonstrada com o uso dos produtórios.
Demonstração:
Sabemos que 𝑚𝑝 = 𝑚. 𝑚. … . 𝑚, onde 𝑚 se repete 𝑝 vezes.
Então, pela P1:
log 𝑟 𝑚𝑝 = log 𝑟 (𝑚. 𝑚. … . 𝑚) = log 𝑟 𝑚 + log 𝑟 𝑚 + ⋯ + log 𝑟 𝑚.
Esta soma possui 𝑝 termos iguais:
log 𝑟 𝑚 + log 𝑟 𝑚 + ⋯ + log 𝑟 𝑚 = log 𝑟 𝑚 (1 + 1 + ⋯ + 1)
= 𝒑. 𝐥𝐨𝐠 𝒓 𝒎.
Ou, de maneira análoga:
𝑝

log 𝑟 𝑚𝑝 = log 𝑟 (𝑚. 𝑚. … . 𝑚) = log 𝑟 (∏ 𝑚) .


𝑘=1
Vimos que o logaritmo de um produtório é o somatório dos logaritmos:
𝑝 𝑝

log 𝑟 (∏ 𝑚) = ∑ log 𝑟 𝑚 .
𝑘=1 𝑘=1
Como nosso produtório está numa variável 𝑘, e seu argumento é independente de 𝑘,
podemos trata-lo como uma constante:
𝑝 𝑝 𝑝

log 𝑟 (∏ 𝑚) = ∑ log 𝑟 𝑚 = (log 𝑟 𝑚) ∑ 1


𝑘=1 𝑘=1 𝑘=1
= 𝒑. 𝐥𝐨𝐠 𝒓 𝒎.
Esta propriedade pode ser estendida da seguinte maneira:
𝒑
𝐥𝐨𝐠 𝒓𝒒 𝒎𝒑 = 𝐥𝐨𝐠 𝒓 𝒎 .
𝒒
Demonstração:
Seja 𝑢 = log 𝑟𝑞 𝑚𝑝 , temos que:
(𝑟 𝑞 )𝑢 = 𝑚𝑝 ;
1 𝒑
𝑟 𝑞𝑢 = 𝑚𝑝 ⟹ 𝑞𝑢 = log 𝑟 𝑚𝑝 ⟹ 𝑢 = log 𝑟 𝑚𝑝 ⟹ 𝒖 = 𝐥𝐨𝐠 𝒓 𝒎.
𝑞 𝒒
Esta extensão da propriedade é útil nos casos em que ambos, logaritmo e sua base,
estão elevados a alguma potência.
P4. – Mudança de base:
“Todo logaritmo pode ser escrito como a razão entre os logaritmos do
logaritmando do primeiro e da base, numa outra base arbitrária.”
Em linguagem matemática:
Seja 𝑐 > 0 ∧ ≠ 1, vale que:
log𝑐 𝑏
log 𝑎 𝑏 = , com 𝑏 > 0 e 𝑎 > 0.
log𝑐 𝑎
Desta definição, decorre que:
𝐥𝐨𝐠 𝒃 𝒃 𝟏
𝐥𝐨𝐠 𝒂 𝒃 = = .
𝐥𝐨𝐠 𝒃 𝒂 𝐥𝐨𝐠 𝒃 𝒂
Demonstração:
Seja:
log𝑐 𝑏
= 𝑧;
log𝑐 𝑎
log 𝑐 𝑏 = 𝑧. log 𝑐 𝑎 ⟹ log 𝑐 𝑏 = log 𝑐 𝑎 𝑧 ;
𝑐 log𝑐 𝑏 = 𝑎 𝑧 ⟹ 𝒃 = 𝒂𝒛 .
Tomando o logaritmo na base 𝑎 de ambos os lados:
𝑏 = 𝑎 𝑧 ⟹ log 𝑎 𝑏 = log 𝑎 𝑎 𝑧 ;
𝑧 = log 𝑎 𝑏.
log𝑐 𝑏
Mas 𝑧 = :
log𝑐 𝑎
𝐥𝐨𝐠 𝒄 𝒃
𝑧 = log 𝑎 𝑏 ⟹ 𝐥𝐨𝐠 𝒂 𝒃 = , como queríamos demonstrar.
𝐥𝐨𝐠 𝒄 𝒂
Alguns exemplos:
log𝑒 7 ln 7
1. – log 5 7 = = ;
log𝑒 5 ln 5
log4 13
2. – ln 13 = ;
log4 𝑒
log100 2
3. – log 3 2 = .
log100 3

P5. – Cologaritmo:
“O cologaritmo é definido como o oposto de um dado logaritmo.”
Assim:
colog 𝑎 𝑏 = log 𝑎 (1/𝑏) = log 𝑎 (𝑏−1 ) = − log 𝑎 𝑏.
Ou:
colog𝑎 𝑏
= − 1.
log𝑎 𝑏

P6. – Antilogaritmo e Consequência da Definição:


“O Antilogaritmo é definido como a operação inversa da logaritmação.”
Temos, então, que:
anti log 𝑎 𝑏 = 𝑎𝑏 .
Percebamos que a antilogaritmação é um sinônimo da exponenciação, pois:
Definindo antilog 𝑎 𝑏 ≡ 𝑎𝑏 = 𝑥, então 𝑏 = log 𝑎 𝑥.
Consequência da Definição de logaritmo:
Sabemos que 𝑎 𝑥 = 𝑏 ⟹ 𝑥 = log 𝑎 𝑏.
Porém, 𝑎 𝑥 = 𝑏 ⟹ 𝒂𝐥𝐨𝐠𝒂 𝒃 = 𝒃.
Esta é consequência da definição dos logaritmos.
Exemplos:
“1. Determine 𝑥 que satisfaça a igualdade log 5 (𝑥 2 − 5𝑥) = log 5 (6𝑥).”
Temos a equação log 5 (𝑥 2 − 5𝑥) = log 5 (6𝑥).
Primeiramente, definamos as condições de existências logaritmos para que possamos
prosseguir com a resolução:
I: 𝑥² − 5𝑥 > 0 ⟹ 𝑥 ∈ ] − ∞, 0[ ∪ ]5, + ∞[ ou, simplesmente, 𝑥 < 0 ∨ 𝑥 > 5.
II: 6𝑥 > 0 ⟹ 𝑥 > 0.
Tomando o Antilogaritmo de base 5 de ambos os lados:
antilog 5 (log 5 (𝑥 2 − 5𝑥)) = antilog 5 (log 5 (6𝑥)).
2
5log5 (𝑥 −5𝑥) = 5log5 (6𝑥) ⟹ 𝑥 2 − 5𝑥 = 6𝑥.
𝑥 2 − 11𝑥 = 0 ⟹ 𝑥 = 0 ∨ 𝑥 = 11.
Porém, pela condição II, 𝑥 > 0.
Assim, temos apenas uma solução, 𝑥 = 11.
S = {11}.
Insistimos no fato de que tomar o Antilogaritmo dos dois lados de uma igualdade é,
nada mais, nada menos, que tomar a exponencial para que cancelemos os logaritmos.
Observe:
log 7 100 = log 7 𝑥.
Tomando a exponencial na base 7 de ambos os lados (antilogaritmo de base 7),
obtemos:
7log7 100 = 7log7 𝑥 ⟹ 𝑥 = 100.
A operação acima é idêntica a:
log 7 100 = log 7 𝑥 ⟹ antilog 7 ( log 7 100) = antilog 7 (log 7 𝑥);
7log7 100 = 7log7 𝑥 .
Veja que obtivemos a mesma expressão.
“2. Resolva, no âmbito dos números reais, a inequação:
ln(3𝑥 3 − 5𝑥 2 + 𝑥) < ln(3𝑥 2 − 𝑥).”
Primeiramente, as condições de existência dos logaritmos são:
1 1
I: 3𝑥 3 − 5𝑥 2 + 𝑥 > 0 ⟹ 𝑥 > (√5 + 13) ∨ 0 < 𝑥 < (5 − √13);
6 6
1
II: 3𝑥 2 − 𝑥 > 0 ⟹ 𝑥 < 0 ∨ 𝑥 > .
3
1
Fazendo I ∩ II, obtemos 𝑥 > (5 + √13).
6
Caso pairem dúvidas a respeito das condições acima, solicitamos a consulta de um
livro que trate das inequações e suas formas de resolução.
Como estamos trabalhando com o logaritmo de base 𝑒, não devemos nos preocupar
com as restrições desta, pois 𝑒 > 1.
* Se 𝑎 ∈ ]1, + ∞[ ou 𝑎 > 1, a função 𝑓(𝑥) = log 𝑎 𝑥 é estritamente crescente, pois:
∀ 𝒙𝟏 e 𝒙𝟐 ∈ ℝ∗+ , 𝒙𝟏 > 𝒙𝟐 ⟹ 𝒇(𝒙𝟏 ) > 𝒇(𝒙𝟐 ).
Do lembrete acima, é imediato que:
ln(3𝑥 3 − 5𝑥 2 + 𝑥) < ln(3𝑥 2 − 𝑥) ⟹ 𝟑𝒙𝟑 − 𝟓𝒙𝟐 + 𝒙 < 𝟑𝒙𝟐 − 𝒙.
3𝑥 3 − 3𝑥 2 − 5𝑥 2 + 2𝑥 < 0;
3𝑥 3 − 8𝑥 2 + 2𝑥 < 0 ⟹ 𝑥(𝑥 2 − 8𝑥 + 2) < 0.
Resolvendo esta inequação pelos métodos convencionais, obtemos:
1 1
3
(4 − √10) < 𝑥 < 3 (4 + √10) ∨ 𝑥 < 0.
1
Da condição inicial 𝑥 > (5 + √13), temos que:
6
1 1
S = {𝑥 ∈ ℝ: (5 + √13) < 𝑥 < (4 + √10)}.
6 3
A seguir, traremos uma pequena lista de exercícios sobre os Logaritmos, cuja
resolução destes é altamente recomendada para que possamos prosseguir, com
segurança, nossa teoria.

V – 1. Exercícios propostos – Logaritmos


⊕⊕ − 1. (FME – IEZZI) Se 𝑎, 𝑏 e 𝑐 são reais positivos, com 𝑎 ≠ 1 e 𝑎𝑐 ≠ 1,
prove que:
log 𝑎 𝑏 = log 𝑎𝑐 [𝑏(1 + log 𝑎 𝑐)].

⊕⊕ − 2. (FME – IEZZI) Se 𝑎, 𝑏 e 𝑐 são reais positivos, não unitários e 𝑎 = 𝑏𝑐,


mostre que:
1 1
=1+ .
log𝑎 𝑐 log𝑏 𝑐
* Um número 𝑝 é não unitário quando 𝒑 ≠ 𝟏.

⊕⊕⊕ − 3. (FME – IEZZI) Se 𝑥 = log 𝑐 (𝑎𝑏), 𝑦 = log 𝑏 (𝑎𝑐) e 𝑧 = log 𝑎 (𝑏𝑐),


prove que:
1 1 1
+ + = 1.
1+𝑥 1+𝑦 1+𝑧

⊕⊕ − 4. (FME – IEZZI) Se 𝑎 e 𝑏 são reais positivos, prove que:


𝑎log 𝑏 = 𝑏 log 𝑎 .

⊕⊕⊕ − 5. (FME – IEZZI) Se 𝑎 e 𝑏 são raízes da equação 𝑥 2 − 𝑝𝑥 + 𝑞 = 0, com


𝑝 > 0 ∧ 0 < 𝑞 ≠ 1, demonstre que:
log 𝑞 𝑎𝑎 + log 𝑞 𝑏 𝑏 + log 𝑞 𝑎𝑏 + log 𝑞 𝑏 𝑎 = 𝑝.

⊕⊕⊕ − 6. (FME – IEZZI) Se 𝑎, 𝑏 e 𝑐 são reais positivos, prove que:


𝑎 log 𝑐 𝑏 log 𝑎 𝑐 log 𝑏
(𝑏 ) .( ) .( ) = 1.
𝑐 𝑎
⊕⊕⊕ − 7. (FME – IEZZI) Se 0 < 𝑥 ≠ 1, demonstre que:
1 1 1 1 1
+ +⋯+ = (1 − ) . .
log 𝑥 2 . log 𝑥 4 log 𝑥 4 . log 𝑥 8 log 𝑥 2𝑛−1 . log 𝑥 2𝑛 𝑛 (log 𝑥 2)2
Sugestão: Verifique, caro leitor, que a soma acima é uma soma telescópica.

2. – Introdução ao Logaritmo Complexo – Números Complexos


Antes de iniciarmos, recordemos as principais propriedades dos Números Complexos.
No auge das mais diversas contribuições ao estudo da Matemática, mostrou-se
absolutamente necessário um rigoroso parecer às situações nas quais eram obtidas, por
meio das resoluções de equações polinomiais, raízes de índice par com radicando
negativo.
3
Por exemplo, a equação 𝑥 − 1 = 0, no âmbito dos números reais, é tal que:
𝑥 3 − 1 = 0 ⟹ (𝑥 − 1)(𝑥 2 + 𝑥 + 1) = 0;
𝑥 = 1 ∨ 𝑥 2 + 𝑥 + 1 = 0.
O número 1 é a dita solução trivial da equação, pois 13 − 1 = 0.
Porém, na equação 𝑥 2 + 𝑥 + 1 = 0, se obtivermos o discriminante:
𝑥 2 + 𝑥 + 1 = 0 ⟹ ∆ = 12 − 4.1.1 = − 3.
𝟏 √− 𝟑
Do exposto acima, as raízes desta equação são tais que 𝒙 = − ± .
𝟐 𝟐
O fato acima, quando analisado sob a ótica dos Reais, concretiza um absurdo,
pois, numa equação quadrática qualquer 𝑎𝑥 2 + 𝑏𝑥 + 𝑐 = 0, verifica-se que:
1. ∆ = 𝑏 2 − 4𝑎𝑐 > 0 ⟹ A equação admite duas raízes reais distintas.
2. ∆ = 𝑏 2 − 4𝑎𝑐 = 0 ⟹ A equação admite uma única raiz real (duas raízes reais
iguais).
2
3. ∆ = 𝑏 − 4𝑎𝑐 < 0 ⟹ A equação não admite raiz real.
Das definições acima, percebeu-se necessária a criação de um novo conjunto
matemático, o qual auxiliaria na solução do absurdo acima e alteraria
permanentemente o rumo da Matemática, talvez como nenhuma outra teoria jamais
ousara antes.
Assim nasce o conjunto dos Números Complexos.
Este conjunto está alicerçado no conceito de Número Imaginário, os quais deram
início às rigorosas construções axiomáticas deste conjunto.
Fora, então, convencionado que:
ℕ ⊂ ℤ ⊂ ℚ ⊂ ℝ ⊂ ℂ, onde ℂ é o conjunto dos Números Complexos.
Para dar continuidade à limitação natural da raiz quadrada de um número negativo
exibida pelo conjunto dos Reais, fora definido que:
Dada a equação 𝑥² + 1 = 0, antes sem solução, esta agora apresentaria duas soluções
imaginárias, dadas por − √− 1 e √− 1, e √− 1 receberia o nome de
Unidade Imaginária, ou Constante Imaginária, e seria a base de toda a nova teoria
do recém-criado conjunto dos Números Complexos, adotando 𝑖 = √− 1, com a
propriedade 𝑖² = − 1 e corrigindo a deficiência natural dos números Reais.
Relembrando que, para que seja necessária a criação de um novo conjunto numérico, é
necessário a descoberta de uma falha ou uma limitação natural grave de um certo
conjunto, falha esta que impossibilite a resolução de problemas cujo Universo seja o
conjunto em questão e, de acordo com a falha descoberta, dar início a rigorosa
formulação de um novo conjunto que satisfaça todas as condições de seu antecessor,
buscando não entrar em discordância com este e, de maneira adicional, oferecer uma
solução às deficiências do mesmo.
1. O primeiro conjunto, o conjunto dos naturais (ℕ), não comporta situações como
𝑥 + 1 = 0.
Sendo assim, não existe solução natural para este problema:
∄ 𝑥 ∈ ℕ: 𝑥 = − 1.
2. O segundo conjunto, o conjunto dos inteiros (ℤ), introduz a noção de número
negativo e propõe a solução para 𝑥 + 1 = 0 ⟹ 𝑥 = − 1.
Porém falha em problemas como 2𝑥 = 3.
A equação 2𝑥 = 3 não possui solução nos números inteiros.
∄ 𝑥 ∈ ℤ: 2𝑥 = 3.
3. Então, fora introduzido o conjunto dos números racionais (ℚ), os quais são
𝑝 3
representados por uma razão , onde 𝑞 ≠ 0, solucionando a equação 2𝑥 = 3 ⟹ 𝑥 = .
𝑞 2
Porém, este falha na resolução de problemas como 𝑥² = 2.
A equação 𝑥² = 2 não apresenta solução no conjunto dos números racionais.
∄ 𝑥 ∈ ℚ: 𝑥² = 2.
4. Assim, fora introduzido o conjunto dos números irracionais 𝕀 ou (ℝ ∖ ℚ),
englobando todos os números não obtidos pela razão de dois inteiros, e solucionando
𝑥² = 2 ⟹ 𝑥 2 = ± √2.
Porém, os irracionais também falham na resolução de problemas como 5𝑥 = 4.
A equação 5𝑥 = 4 não apresenta solução irracional, e sim racional.
∄ 𝑥 ∈ 𝕀: 5𝑥 = 4.
Mas, como dito anteriormente, este problema possui solução na ótica dos números
racionais e, sendo assim, a união destes dois conjuntos (racionais e irracionais) recebe
o nome de Conjunto dos Números Reais, ℚ ⋃ 𝕀 = ℝ, e a criação deste conjunto
supriria todas as limitações apresentadas pelos racionais e irracionais até então.
Mas, os reais falham na resolução de problemas como 𝑥² + 1 = 0, pois não existe 𝑥
real tal que seu quadrado resulte em um número negativo.
∄ 𝑥 ∈ ℝ: 𝑥² + 1 = 0.
5. Enfim, com o advento dos Números Complexos, este problema fora solucionado:
Adotando 𝑖 = √− 1 ⟹ 𝑥 2 = − 1 ⟹ 𝑥 = ± 𝑖.
Percebamos que o conjunto dos Números Complexos cumpre todas as condições
anteriores, pois engloba todos os conjuntos antecessores e apresenta solução para os
problemas nos quais nos deparamos com uma raiz de índice par cujo radicando é um
número negativo.
Dito isto, temos, finalmente, que:
Um número complexo 𝑧 é tal que 𝑧 = 𝑥 + 𝑖. 𝑦, onde 𝑥 e 𝑦 ∈ ℝ e 𝑖 é a supracitada
unidade imaginária, com 𝑖² = − 1.
O número 𝑥 é denominado Parte Real de 𝑧, 𝑥 = Re(𝑧) e 𝑦 é denominado
Parte Imaginária de 𝑧 (pois acompanha 𝑖), 𝑦 = Im(𝑧).
Disto, segue que todo complexo 𝑧 é dado por 𝑧 = Re(𝑧) + 𝑖. Im(𝑧).
Da definição trivial de um número complexo, seguem duas consequências:
1. Se um complexo 𝑧 apresentar parte real nula, Re(𝑧) = 0, este complexo é dito
Imaginário Puro e pertence ao conjunto
ℂ ∖ ℝ = 𝕀𝕞
(Conjunto dos números Complexos – Conjunto dos números reais), ou seja, pertence
ao conjunto dos Números Imaginários Puros.
𝒛 = 𝟎 + 𝒊. 𝐈𝐦(𝒛) ⟹ 𝒛 ∈ ℂ ∖ ℝ ⟹ 𝒛 ∈ 𝕀𝕞.
2. Se um complexo 𝑧 apresentar parte imaginária nula, Im(𝑧) = 0, este complexo é
dito Número Real e pertence a ℝ.
𝒛 = 𝐑𝐞(𝒛) + 𝟎. 𝒊 ⟹ 𝒛 ∈ ℝ.
O número complexo 𝑧̅ = 𝑥 − 𝑖. 𝑦 é denominado Conjugado de 𝑧, pois há a inversão
do sinal de sua parte imaginária.
Da definição de Conjugado, decorre que:
1. 𝑧 + 𝑧̅ = (𝑥 + 𝑖. 𝑦) + (𝑥 − 𝑖. 𝑦) = 2𝑥 = 2. Re(𝑧).
2. 𝑧 − 𝑧̅ = (𝑥 + 𝑖. 𝑦) − (𝑥 − 𝑖. 𝑦) = 2. 𝑖. 𝑦 = 2. 𝑖. Im(𝑧).
3. 𝑧. 𝑧̅ = (𝑥 + 𝑖𝑦)(𝑥 − 𝑖𝑦) = 𝑥² + 𝑦².
𝑧 𝑥+𝑖.𝑦 (𝑥+𝑖.𝑦)(𝑥+𝑖.𝑦) (𝑥+𝑖.𝑦)² 𝑥²−𝑦 2 +2𝑖𝑥𝑦
4. = = (𝑥−𝑖.𝑦)(𝑥+𝑖.𝑦) = = .
𝑧̅ 𝑥−𝑖.𝑦 𝑥²+𝑦² 𝑥²+𝑦²
5. Dados 𝑧 = 𝑎 + 𝑖. 𝑏 e 𝑤 = 𝑐 + 𝑖. 𝑑 complexos, segue que:
5.1.
𝑧̅̅̅̅̅̅̅̅
+ 𝑤 = (𝑎 ̅̅̅̅̅̅̅̅̅̅̅̅̅̅̅̅̅̅̅̅̅̅̅̅̅̅̅
+ 𝑖. 𝑏) + (𝑐 + 𝑖. 𝑑) = ̅̅̅̅̅̅̅̅̅̅̅̅̅̅̅̅̅̅̅̅̅̅̅̅
(𝑎 + 𝑐) + 𝑖(𝑏 + 𝑑) = (𝑎 + 𝑐) − 𝑖(𝑏 + 𝑑)
= (𝑎 − 𝑖. 𝑏) + (𝑐 − 𝑖. 𝑑) = 𝑧̅ + 𝑤 ̅, ou seja, o conjugado da soma é a soma dos
conjugados.
5.2.
𝑧. 𝑤 = (𝑎
̅̅̅̅̅ ̅̅̅̅̅̅̅̅̅̅̅̅̅̅̅̅̅̅̅̅̅̅̅
+ 𝑖. 𝑏)(𝑐 + 𝑖. 𝑑) = (𝑎𝑐 ̅̅̅̅̅̅̅̅̅̅̅̅̅̅̅̅̅̅̅̅̅̅̅̅̅̅̅̅̅̅̅̅̅
+ 𝑖. 𝑎. 𝑑 + 𝑖. 𝑏. 𝑐 − 𝑏𝑑) = (𝑎𝑐 ̅̅̅̅̅̅̅̅̅̅̅̅̅̅̅̅̅̅̅̅̅̅̅̅̅̅̅̅̅̅
− 𝑏𝑑) + 𝑖(𝑎𝑑 + 𝑏𝑐)
= (𝑎𝑐 − 𝑏𝑑) − 𝑖(𝑎𝑑 + 𝑏𝑐) = (𝑎 − 𝑖. 𝑏)(𝑐 − 𝑖. 𝑑) = 𝑧̅. 𝑤 ̅, ou seja, o conjugado do
produto é o produto dos conjugados.
5.3.
𝑧̅̅ = ̅̅̅̅̅̅̅̅̅̅̅̅
(𝑎 − 𝑖. 𝑏) = 𝑎 + 𝑖. 𝑏, ou seja, o conjugado do conjugado de um número
complexo é ele próprio.
A notação 𝑧 = 𝑥 + 𝑖. 𝑦 é denominada Forma Retangular de 𝑧, pois esta pode ser posta
na forma de um par ordenado (𝑥; 𝑦) no plano Complexo (Plano de Argand – Gauss).
O Plano Complexo possui a mesma forma do Plano Cartesiano, eixos perpendiculares
e uma origem comum, porém, o eixo O𝑥 ⃗⃗⃗⃗⃗ ganha a alcunha de Eixo Real, e o eixo O𝑦 ⃗⃗⃗⃗⃗
passa a ser chamado Eixo Imaginário:
O Plano Complexo.

Todo número complexo 𝑧 = 𝑥 + 𝑖. 𝑦 = Re(𝑧) + 𝑖. Im(𝑧) = (𝑥; 𝑦), com o advento


deste plano, passa a ser interpretado como um vetor de origem
no ponto O = (0; 0) e extremidade no ponto Z = (𝑥; 𝑦).
Assim, todas as propriedades geométricas dos vetores são válidas na representação
vetorial de um complexo.
Ainda na representação geométrica ou vetorial, percebe-se que o vetor ⃗⃗⃗⃗⃗
OZ
(vetor de origem no ponto O = (0; 0) e extremidade no ponto Z = (𝑥; 𝑦)) faz um
ângulo 𝜃 (argumento de 𝒛) com o eixo real, de tal maneira que:

Percebamos que as representações geométricas de um complexo 𝑧 = 𝑥 + 𝑖. 𝑦 e seu


conjugado 𝑧̅ = 𝑧 ∗ = 𝑥 − 𝑖. 𝑦 são simétricas em relação ao eixo real.
O número 𝑟 é denominado Módulo de 𝒛 e representa a distância do afixo
(ponto Z = (𝑥; 𝑦)) de 𝑧 até a origem O = (0; 0), e pode ser representado tanto por 𝑟
quanto por |𝑧|.
Seu cálculo é feito aplicando a distância entre dois pontos (Teorema de Pitágoras),
considerando os pontos Z = (𝑥; 𝑦) e O = (0; 0):
𝑟² = 𝑥² + 𝑦² ⟹ 𝑟 = |𝑧| = √𝑥² + 𝑦².
Vejamos, também, que:
𝑥
𝐈. cos 𝜃 = ⟹ 𝒙 = 𝒓. 𝐜𝐨𝐬(𝜽)
𝑟
𝑦
𝐈𝐈. sen 𝜃 = ⟹ 𝒚 = 𝒓. 𝐬𝐞𝐧(𝜽)
𝑟
sen 𝜃 𝑦 𝑦 𝒚
𝐈𝐈⁄𝐈 ⟹ = ⟹ 𝑡𝑔 𝜃 = ⟹ 𝜽 = 𝒂𝒓𝒄 𝒕𝒈 ( )
{ cos 𝜃 𝑥 𝑥 𝒙

Da forma retangular, 𝑧 = 𝑥 + 𝑖. 𝑦 ⟹ 𝑧 = 𝑟. cos 𝜃 + 𝑖. 𝑟. sen 𝜃


= 𝒓(𝐜𝐨𝐬(𝜽) + 𝒊. 𝐬𝐞𝐧(𝜽)).
A forma acima é denominada Forma Trigonométrica de 𝑧, a qual segue as mesmas
propriedades da forma retangular.
A forma trigonométrica 𝑧 = 𝑟(cos(𝜃) + 𝑖. 𝑠𝑒𝑛(𝜃)) é frequentemente abreviada para
𝒛 = 𝒓. 𝐜𝐢𝐬(𝜽).
Percebamos que um número complexo 𝑧 qualquer, quando posto em sua forma
trigonométrica, independe de 𝑥 e 𝑦, passando a depender somente de 𝑟 e 𝜃.
Assim, um complexo 𝑧 = (𝑥; 𝑦) é escrito na forma 𝑧 = (𝑟; 𝜃),
𝑦
onde 𝑟 = √𝑥² + 𝑦² e 𝜃 = 𝑎𝑟𝑐 𝑡𝑔 ( ).
𝑥
Do exposto acima:
𝑦
𝒛 = (𝒙; 𝒚) ⟹ (𝒓; 𝜽) ⟹ (√𝑥² + 𝑦²; 𝑎𝑟𝑐 𝑡𝑔 ( )).
𝑥
A forma acima é denominada Forma Polar de 𝑧:
𝑦
Forma polar ⟹ 𝑧 = (𝑟; 𝜃) = (√𝑥² + 𝑦²; 𝑎𝑟𝑐 𝑡𝑔 ( )).
𝑥
Ainda, da definição de módulo, decorre que
(deixaremos que você, senhor leitor, realize as demonstrações):
𝑧 |𝑧|
1. | | = |𝑤|, com 𝑤 ≠ 0, ou seja, o módulo da razão é a razão dos módulos.
𝑤
2. |𝑧|. |𝑤| = |𝑧. 𝑤|, ou seja, o módulo do produto é o produto dos módulos.
3. |𝑧|𝑛 = |𝑧 𝑛 |, ou seja, a n-ésima potência do módulo é o módulo da n-ésima
potência.
3. |𝑧| = |𝑧̅|, o módulo de um complexo e seu conjugado são iguais.
4. |𝑧 + 𝑤| ≤ |𝑧| + |𝑤| (desigualdade triangular), o módulo da soma é menor ou
igual que a soma dos módulos.
* Sugestão: Tente demonstrar a quarta propriedade geometricamente.
5. Re(𝑧) ≤ |Re(𝑧)| ≤ |𝑧|.
6. Im(𝑧) ≤ |Im(𝑧)| ≤ |𝑧|.
O argumento do número complexo 𝑧 é o ângulo 𝜃 cujo vetor complexo faz com o eixo real, e
sua notação é 𝜃 = arg 𝑧.
Percebamos que existem infinitos argumentos para um complexo:
arg 𝑧 = 𝜃 + 2𝑘𝜋, 𝑘 ∈ ℤ.
Denomina-se argumento principal de 𝒛 o menor argumento possível de 𝑧, para 𝑘 = 0.
“1. Passe para a forma trigonométrica/polar:
𝑎) 𝑧 = 1 + 𝑖;
1 √3
𝑏) 𝑧 = − 𝑖 ;
2 2
𝑐) 𝑧 = 7 − 2𝑖.”
𝑎) 𝑧 = 1 + 𝑖;
|𝑧| = |1 + 𝑖| = √12 + 12 = √2;
1 𝜋
arg 𝑧 = arg(1 + 𝑖) = arc tg ( ) = arc tg(1) = ;
1 4
𝜋 𝜋 𝜋 𝜋
Assim, 𝑧 = 𝑟. cis(𝜃) ⟹ 𝑧 = √2. cis ( ) = √2 (cos ( ) + 𝑖. 𝑠𝑒𝑛 ( )) = (√2; ).
4 4 4 4
1 √3
𝑏) − 𝑖 ;
2 2
2
1 √3 1 2 √3 1 3 4
|𝑧| = | − 𝑖 | = √( ) + ( ) = √ + = √ = 1.
2 2 2 2 4 4 4
√3
1 √3 2 𝜋
arg 𝑧 = arg ( − 𝑖 ) = arc tg (− 1 ) = arc tg(− √3) = − 3 ;
2 2
2
𝜋 𝜋 5𝜋 5𝜋
Assim, 𝑧 = 𝑟. cis(𝜃) ⟹ 1. cis (− ) = cis (− + 2𝜋) = cis ( ) = (1; ).
3 3 3 3
𝑐) 𝑧 = 7 − 2𝑖;
|𝑧| = |7 − 2𝑖| = √72 + 22 = √49 + 4 = √53;
2 2
arg 𝑧 = arg(7 − 2𝑖) = arc tg (− ) = − arc tg ( ).
7 7
Perceba que o argumento deste complexo não é um arco notável, logo, é necessário o
conhecimento das funções trigonométricas inversas.
Caso você, leitor, não esteja familiarizado com as funções trigonométricas inversas, consulte
um livro sobre trigonometria.
2
Assim, 𝑧 = 𝑟. cis(𝜃) ⟹ 𝑧 = √53. cis (− arc tg ( )).
7

3. – Introdução ao Logaritmo Complexo – A Fórmula de Euler


Em meados do século XVIII, os matemáticos da época dedicaram sua atenção aos
misteriosos Números Imaginários, os quais carregavam propriedades até então
inimagináveis, pois desafiavam toda a lógica dos Números Reais e se contrapunham
a natureza destes.
Eis então que Abraham de Moivre, célebre matemático francês, amigo pessoal de Isaac
Newton, publica as denominadas Fórmulas de Moivre, as quais estabelecem que, dado um
complexo 𝑧 = 𝑟. cis(𝜃) = 𝑟(cos(𝜃) + 𝑖. sen(𝜃)), vale que:
1. 𝑧 𝑛 = 𝑟 𝑛 . cis(𝑛𝜃) = 𝑟 𝑛 (cos(𝑛𝜃) + 𝑖. sen(𝑛𝜃)), onde 𝑛 ∈ ℕ∗ .
Esta é a Primeira Fórmula de Moivre (exponenciação de um complexo).
𝒏 𝑛 𝜃 2𝑘𝜋 𝑛 𝜃 2𝑘𝜋 𝜃 2𝑘𝜋
2. √𝑧 = √𝑟. cis ( +
𝑛 𝑛
) = √𝑟. (cos (𝑛 + 𝑛
) + 𝑖. sen (𝑛 + 𝑛
)), onde 𝑛 ∈ ℕ∗ e 𝑘 ∈ ℤ.
Esta é a Segunda Fórmula de Moivre (radiciação de um complexo).
Estas fórmulas traçaram o paralelo entre a trigonometria e os Números Imaginários.
As descobertas de Moivre abalaram a comunidade científica na época, pois trouxeram uma
elegante conexão entre a trigonometria e os números complexos.
Assim, no ano de 1748, Leonhard Euler, um dos maiores matemáticos de todos os tempos,
presenteia a comunidade matemática com a denominada Fórmula de Euler, criando novos
horizontes para o avanço desta ciência.
Euler verificou que:
𝒊.𝜽
𝒆 = 𝐜𝐨𝐬 𝜽 + 𝒊. 𝐬𝐞𝐧 𝜽.
Onde 𝑒 é a base dos Logaritmos Naturais (ou Neperianos), 𝑖 é a unidade imaginária, e 𝜃 é o
argumento deste complexo.
A fórmula acima garantiu extrema notoriedade a Euler, pois, num curto período de tempo,
tornou-se uma das fórmulas matemáticas mais famosas de todos os tempos, com inúmeras
aplicações nas ciências abstratas.
Assim, todo número complexo 𝑧, de módulo 𝑟 e argumento 𝜃 pode ser escrito da seguinte
forma:
𝒛 = 𝒓(𝐜𝐨𝐬(𝜽) + 𝒊. 𝐬𝐞𝐧(𝛉)) = 𝒓. 𝒆𝒊.𝜽 ; 𝒛̅ = 𝒓(𝐜𝐨𝐬(𝜽) − 𝒊. 𝐬𝐞𝐧(𝛉)) = 𝒓. 𝒆−𝒊.𝜽 .
Um interessante fato ocorre se tomarmos, na fórmula de Euler, 𝜽 = 𝝅:
𝑒 𝑖.𝜃 = cos 𝜃 + 𝑖. sen 𝜃 ⟹ 𝑒 𝑖.𝜋 = cos 𝜋 + 𝑖. sen 𝜋 ⟹ 𝑒 𝑖.𝜋 = − 1:
𝒆𝒊.𝝅 + 𝟏 = 𝟎.
A identidade acima é denominada Identidade de Euler e é considerada a mais bela fórmula
matemática da história, pois congrega, de maneira simples, as cinco constantes mais
importantes da matemática em uma só igualdade:
𝒆: A base dos logaritmos naturais;
𝝅: A constante universal obtida pela razão entre o perímetro de qualquer circunferência e seu
diâmetro;
𝒊: A unidade imaginária, dotada da propriedade 𝑖 2 = − 1;
𝟏: O elemento neutro da multiplicação, o primeiro número natural;
O: O elemento neutro da adição, o “início” de toda a matemática.
A Fórmula de Euler, além de possibilitar a interpretação exponencial de um Número
Complexo, facilita, de maneira extraordinária, o trabalho algébrico típico dos complexos.
Sendo assim, vale que:
1. 𝑟 𝑛 . 𝑒 𝑖.𝑛.𝜃 = 𝑟 𝑛 (cos(𝑛. 𝜃) + 𝑖. sen(𝑛. 𝜃)) (Primeira Fórmula de Moivre);
1 𝜃 2𝑘𝜋
𝑖( + ) 𝑛 𝜃 2𝑘𝜋 𝜃 2𝑘𝜋
2. 𝑟 𝑛 𝑒 𝑛 𝑛 = √𝑟 (cos ( + ) + 𝑖. sen (𝑛 + )) (Segunda Fórmula de Moivre);
𝑛 𝑛 𝑛
3. 𝑟1 . 𝑒 𝑖.𝜃1 . 𝑟2 . 𝑒 𝑖.𝜃2 = 𝑟1 . 𝑟2 (cos(𝜃1 + 𝜃2 ) + 𝑖. sen(𝜃1 + 𝜃2 )), onde 𝑟1 , 𝑟2 , 𝜃1 e 𝜃2 são,
respectivamente, os módulos de dois complexos distintos e os argumentos destes complexos,
também distintos.
𝑟1 .𝑒 𝑖.𝜃1 𝑟
4. = ( 1) (cos(𝜃1 − 𝜃2 ) + 𝑖. sen(𝜃1 − 𝜃2 )).
𝑟2 .𝑒 𝑖.𝜃2 𝑟2
As definições acima podem ser facilmente demonstradas pelas propriedades da potenciação e
da radiciação, partindo de 𝑧 = 𝑟. 𝑒 𝑖.𝜃 = 𝑟. cis(𝜃), por serem a “adequação”, para a forma
exponencial, das definições da forma trigonométrica (ou forma polar) de um Complexo.
* Caro leitor, não nos preocuparemos com a demonstração da Fórmula de Euler, uma vez
que esta exige o conhecimento de conceitos avançados do Cálculo Numérico, como as
Séries de Taylor – MacLaurin para as funções sen 𝑥, cos 𝑥 e 𝑒 𝑥 , e conceitos não triviais de
Limites, os quais, obviamente, abandonam o nível da teoria.

4. – O Logaritmo Complexo
Vimos, no item anterior, que a Fórmula de Euler estabelece a conexão entre os números
complexos e as exponenciais, de tal forma que, dado um número complexo 𝑧, vale que:
𝑧 = 𝑟. 𝑒 𝑖.𝜃 , onde 𝑒 𝑖.𝜃 = cos(𝜃) + 𝑖. sen(𝜃), onde 𝑟 = |𝑧| é o módulo de 𝑧 e 𝜃 = arg(𝑧).
Se tomarmos o logaritmo natural (ln) de ambos os lados, segue que:
ln 𝑧 = ln(𝑟. 𝑒 𝑖.𝜃 ) ⟹ ln 𝑧 = ln(𝑟) + ln(𝑒 𝑖.𝜃 ) ⟹ 𝐥𝐧 𝒛 = 𝐥𝐧(𝒓) + 𝒊. 𝜃.
Mas, sendo 𝜃 = arg(𝑧) o argumento deste complexo 𝑧, sabemos que este possui um número
infinito de arcos côngruos, de tal maneira que 𝜃 = arg(𝑧) + 2𝑘𝜋.
Assim:
ln 𝑧 = ln(𝑟) + 𝑖. 𝜃 ⟹ 𝐥𝐧 𝒛 = 𝐥𝐧(𝒓) + 𝒊. (𝐚𝐫𝐠(𝒛) + 𝟐𝒌𝝅).
Esta é a definição de Logaritmo Complexo.
Como o argumento de 𝑧 pode assumir infinitos valores côngruos, recebe o nome de
Logaritmo Principal de 𝒛 aquele cujo argumento 𝜃 é o menor possível.
Utilizando a propriedade da mudança de base para uma base 𝑎 qualquer,
com 𝑎 > 0 ∧ 𝑎 ≠ 1, obtemos:
𝑧 = 𝑟. 𝑒 ⟹ log 𝑎 𝑧 = log 𝑎 (𝑟. 𝑒 𝑖.𝜃 ) ⟹ 𝐥𝐨𝐠 𝒂 𝒛 = 𝐥𝐨𝐠 𝒂 𝒓 + 𝐥𝐨𝐠 𝒂 𝒆𝒊.𝜽 .
𝑖.𝜃

Deste modo, podemos obter os logaritmos de números negativos, imaginários e complexos,


não nos limitando às condições do logaritmando > 0, característica dos reais.
Ainda assim, a base deverá ser não nula e não unitária, porém esta também pode assumir
valores complexos.
Por exemplo, obtenhamos o valor do logaritmo principal de − 1:
Temos que – 1 = 𝑒 𝑖.𝜋 , assim:
ln(− 1) = ln(𝑒 𝑖.𝜋 ) = 𝒊. 𝝅.
De fato: ln(− 1) = 𝑢 ⟹ 𝑒 𝑢 = − 1 ⟹ 𝒖 = 𝒊. 𝝅.
“1. Obtenha o logaritmo principal de:
𝑎) log 3 (− 1);
𝑏) log 2 (1 − 𝑖);
𝑐) ln(𝑖);
𝑑) ln(− 𝑒).”
𝑎) Da Identidade de Euler, temos que – 1 = 𝑒 𝑖.𝜋 = cos(𝜋) + 𝑖. sen(𝜋).
Assim:
log 3 (− 1) = log 3 (𝑒 𝑖.𝜋 ).
Efetuando a mudança de base para a base 𝑒:
ln(𝑒 𝑖.𝜋) 𝒊.𝝅
log 3 (𝑒 𝑖.𝜋 ) = = .
ln 3 𝐥𝐧 𝟑
Verificando que o resultado obtido é, de fato, verdadeiro:
𝑖.𝜋 1 1 𝑖.𝜋
𝑢 𝑖.𝜋.
3 =−1⟹3 𝑙𝑛 3 =3 ln 3 = (3 ln 3 ) .
𝟏 𝐥𝐧 𝒆
* 𝟑𝐥𝐧 𝟑 = 𝟑𝐥𝐧 𝟑 = 𝟑𝐥𝐨𝐠𝟑 𝒆 = 𝒆.
Enfim:
1 𝑖.𝜋
(3ln 3 ) = 𝒆𝒊.𝝅 = − 𝟏.
ln(1−𝑖)
𝑏) log 2 (1 − 𝑖) = .
ln 2
O número complexo 1 − 𝑖, quando em sua forma polar, é tal que:
𝜋 𝜋 𝜋 𝜋
1 − 𝑖 = √2 (cos (− ) + 𝑖. sen (− )) = √2 (cos ( ) − 𝑖. sen ( ))
4 4 4 4
𝜋
𝜋
= √2. cis (− ) = √2. 𝑒 −𝑖.4 .
4
Assim:
𝜋
−𝑖. 𝜋
ln(1−𝑖) ln(√2.𝑒 4) ln √2−𝑖( +2𝑘𝜋)
4
log 2 (1 − 𝑖) = = = .
ln 2 ln 2 ln 2
Como buscamos o valor principal, seja 𝑘 = 0:
𝜋 𝝅
ln √2−𝑖( +2.0.𝜋) 𝐥𝐧 √𝟐−𝒊.
4 𝟒
log 2 (1 − 𝑖) = = .
ln 2 𝐥𝐧 𝟐
𝑐) O número 𝑖, quando em sua forma polar, é tal que:
𝜋
𝜋 𝜋 𝜋
𝑖 = cos ( ) + 𝑖. sen ( ) = cis ( ) = 𝑒 𝑖.2 .
2 2 2
Assim:
𝜋
𝑖. 𝜋 𝜋
ln(𝑖) = ln (𝑒 ) = 𝑖 ( + 2𝑘𝜋) = 𝑖. .
2
2 2
𝑑) ln(− 𝑒) = ln(− 1. 𝑒) = ln(− 1) + ln 𝑒 = ln(− 1) + 1 = 𝑖. 𝜋 + 1.

V – 2. Exercícios propostos – Logaritmos Complexos


⊕⊕ − 1. Prove, pela Fórmula de Euler, que:
𝑎) cos(𝛼 + 𝛽) = cos(𝛼) . cos(𝛽) − sen(𝛼). sen(𝛽);
𝑏) sen(𝛼 + 𝛽) = sen(𝛼). cos(𝛽) + sen(𝛽). cos(𝛼).

R:
Sejam:
𝑖.𝛼
𝟏: 𝑒 = cos(𝛼) + 𝑖. sen(𝛼)
{ ;
𝟐: 𝑒 𝑖.𝛽 = cos(𝛽) + 𝑖. sen(𝛽)
Fazendo 1.2:
𝑒 𝑖.𝛼 . 𝑒 𝑖.𝛽 = (cos(𝛼) + 𝑖. sen(𝛼)). (cos(𝛽) + 𝑖. sen(𝛽));
𝑒 𝑖(𝛼+𝛽) = cos(𝛼) . cos(𝛽) + 𝑖. sen(𝛽). cos(𝛼) + 𝑖. sen(𝛼). cos(𝛽) + 𝑖². sen(𝛼). sen(𝛽)
= cos(𝛼) . cos(𝛽) + 𝑖(sen(𝛽). cos(𝛼) + 𝑠𝑒𝑛(𝛼). cos(𝛽)) − sen(𝛼). sen(𝛽)
= (cos(𝛼) . cos(𝛽) − sen(𝛼). sen(𝛽)) + 𝑖(sen(𝛽). cos(𝛼) + sen(𝛼). cos(𝛽)).
Assim: 𝑒 𝑖(𝛼+𝛽) = cos(𝛼 + 𝛽) + 𝑖. sen(𝛼 + 𝛽)
= (cos(𝛼) . cos(𝛽) − sen(𝛼). sen(𝛽)) + 𝑖(sen(𝛽). cos(𝛼) + sen(𝛼). cos(𝛽)).
Perceba que obtivemos um número complexo cujas partes reais e imaginárias são,
respectivamente: Re(𝑧) = cos(𝛼) . cos(𝛽) − sen(𝛼). sen(𝛽);
Im(𝑧) = sen(𝛽). cos(𝛼) + sen(𝛼). cos(𝛽).
Assim:
cos(𝛼 + 𝛽) = Re(𝑧) = cos(𝛼) . cos(𝛽) − sen(𝛼). sen(𝛽)
{ ;
sen(𝛼 + 𝛽) = Im(𝑧) = sen(𝛽). cos(𝛼) + sen(𝛼). cos(𝛽)
Como queríamos demonstrar.

⊕⊕ − 2. Obtenha:
𝑎) log 5 (5 + 𝑖);
𝑏) ln(1 − 𝑖. √2);
𝑐) log 𝑖 (− 1);
R:
ln(− 1) ln(𝑒 𝑖.𝜋 ) 𝑖. 𝜋
log 𝑖 (− 1) = = 𝜋 = 𝜋 = 𝟐.
ln 𝑖 𝑖.
ln (𝑒 2 ) 𝑖. 2
𝑑) log − 5 4;
R:
ln 4 ln 4 𝐥𝐧 𝟒
log − 5 4 = = = .
ln(− 5) ln(5.𝑒 𝑖.𝜋 ) 𝐥𝐧 𝟓+𝒊.𝝅
𝑒) log (1−𝑖) (1 + 𝑖).
R:
𝜋
𝑖. 𝝅
ln(1+𝑖) ln(√2.𝑒 4 ) 𝐥𝐧 √𝟐+𝒊.
𝟒
log (1−𝑖) (1 + 𝑖) = = 𝜋 = 𝝅 .
ln(1−𝑖) ln(√2.𝑒
−𝑖.
4) 𝐥𝐧 √𝟐−𝒊.
𝟒

⊕⊕ − 3. Prove que os números 𝑖 𝑖 e 𝑖 −𝑖 são reais.

⊕⊕ − 4. Demonstre, utilizando a Fórmula de Euler, as duas Fórmulas de Moivre:


1: 𝑧 𝑛 = 𝑟 𝑛 (cos(𝑛. 𝜃) + 𝑖. sen(𝑛. 𝜃));
𝑛 𝑛 𝜃 2𝑘𝜋 𝜃 2𝑘𝜋
2: √𝑧 = √𝑟 (cos ( + ) + 𝑖. sen (𝑛 + )).
𝑛 𝑛 𝑛
Sabendo-se que 𝑟 = |𝑧| e 𝑧 = 𝑟. 𝑒 𝑖.𝜃 = 𝑟(cos(𝜃) + 𝑖. sen(𝜃)).

⊕⊕ − 5. Dada a Fórmula de Euler:


𝒆𝒊.𝜽 = 𝐜𝐨𝐬(𝜽) + 𝒊. 𝐬𝐞𝐧(𝜽)
{ ;
𝒆−𝒊.𝜽 = 𝐜𝐨𝐬(𝜽) − 𝒊. 𝐬𝐞𝐧(𝜽)
Verifique que:
1
𝑎) cos(𝜃) = (𝑒 𝑖.𝜃 + 𝑒 −𝑖.𝜃 );
2
1
𝑏) sen(𝜃) =
2𝑖
(𝑒 𝑖.𝜃 − 𝑒 −𝑖.𝜃 ).
A seguir, daremos continuidade ao estudo dos produtórios, apresentando e demonstrando o
Produto Telescópico, propriedade análoga à Soma Telescópica.
Toda esta revisão acerca dos Logaritmos e Números Complexos servirá como um diferencial
para a construção de um sólido raciocínio lógico-abstrato, pré-requisito para as matérias
vistas nas universidades.

5. – O Produto Telescópico
De maneira análoga a Soma Telescópica, a qual fora vista no capítulo de somatórios, o
Produto Telescópico é uma técnica de absoluta importância para o cômputo de produtórios
não tão convencionais, como, por exemplo:
1000
𝑘 2 +𝑘+1
P𝑛 = ∏ (𝑘 2 +3𝑘+3).
𝑘=1
O produto acima, como podemos observar, é absolutamente complicado, pois, em seu
argumento, temos a razão entre dois polinômios quadráticos.
Assim, como a Soma Telescópica, o Produto Telescópico é de extrema utilidade para a
simplificação de problemas como este.

Definição intuitiva da Propriedade Telescópica – Produtórios:


Seja uma sequência 𝑎𝑛 , cujos termos são dados por:
𝑎
𝑎𝑛 = 𝑘+1, onde 𝑘 varia de 1 até 𝑛.
𝑎𝑘
O denominado Produto Telescópico é tal que:
𝒂𝟐 𝒂𝟑 𝒂𝟒 𝒂𝒏 𝑎𝑛+1
Ptelescópico = ( ) . ( ) . ( ) . … . ( ).( ).
𝑎1 𝒂𝟐 𝒂𝟑 𝒂𝒏−𝟏 𝒂𝒏
Perceba que os termos destacados irão todos se cancelar, resultando em:
𝒂𝒏+𝟏
𝐏𝐭𝐞𝐥𝐞𝐬𝐜ó𝐩𝐢𝐜𝐨 = ( ).
𝒂𝟏
𝑎
Caso a sequência seja tal que 𝑎𝑛 = ( 𝑘 ), vale que:
𝑎𝑘+1
𝑎1 𝒂𝟐 𝒂𝟑 𝒂𝒏−𝟏 𝒂𝒏 𝒂𝟏
Ptelescópico = ( ) . ( ) . ( ) . … . ( ).( )=( ).
𝒂𝟐 𝒂𝟑 𝒂𝟒 𝒂𝒏 𝑎𝑛+1 𝒂𝒏+𝟏

Definição formal da Propriedade Telescópica – Produtórios:


Consideremos a sequência 𝑎𝑛 :
𝑎
𝑎𝑛 : ℤ∗ → ℝ∗ , 𝑎𝑛 = 𝑘+1, com 𝑘 variando de 1 até 𝑛.
𝑎𝑘
Seja P𝑛 = 𝑎1 . 𝑎2 . … . 𝑎𝑛 .
Definamos o operador algébrico ∇ tal que:
𝒂
𝛁 𝒂𝒏 = 𝒏+𝟏.
𝒂𝒏
Este operador, quando aplicado no produto P𝑛 , resulta em:
𝒂𝟐 𝒂𝟑 𝒂𝒏 𝒂𝒏+𝟏 𝒂𝒏+𝟏
𝛁 𝐏𝑛 = ( ) . ( ) . … . ( ).( ) ⟹ 𝛁 𝐏𝑛 = ( ).
𝒂𝟏 𝒂𝟐 𝒂𝒏−𝟏 𝒂𝒏 𝒂𝟏
Utilizando a notação de produtório, temos que:
𝑛

P𝑛 = ∏ 𝑎𝑘 .
𝑘=1
𝒂𝒏+𝟏
𝛁 𝐏𝑛 = ( ).
𝒂𝟏
Ou, de maneira equivalente, considerando o argumento do produto uma função 𝑓(𝑘):
𝒃 𝒃
𝒇(𝒌 + 𝟏) 𝒇(𝒃 + 𝟏)
𝛁 𝐏𝑛 = ∏ 𝛁 𝒇(𝒌) = ∏ ( )= .
𝒇(𝒌) 𝒇(𝒂)
𝒌=𝒂 𝒌=𝒂
Demonstração:
Temos que:
𝑏
𝑓(𝑘 + 1) 𝑓(𝑎 + 1) 𝑓(𝑎 + 2) 𝑓(𝑎 + 3) 𝑓(𝑏 + 1)
∇ P𝑛 = ∏ ( )=( ).( ).( ).….( ).
𝑓(𝑘) 𝑓(𝑎) 𝑓(𝑎 + 1) 𝑓(𝑎 + 2) 𝑓(𝑏)
𝑘=𝑎
Percebamos que, com exceção de 𝑓(𝑎) e 𝑓(𝑏 + 1), todos os outros termos se cancelarão.
Então, o produto acima se torna:
𝒃
𝒇(𝒌+𝟏) 𝒇(𝒃+𝟏)
𝛁 𝐏𝒏 = ∏ ( )= .
𝒌=𝒂 𝒇(𝒌) 𝒇(𝒂)
Assim como a Soma Telescópica, o resultado acima é importantíssimo para o estudo dos
produtórios, porém não é tão difundido quanto a primeira.
A propriedade acima é utilizada, por exemplo, na computação, para simplificar as operações
dos algoritmos, na estatística, na probabilidade, etc.
Para que seja possível a aplicação deste conceito, deve-se manipular a estrutura do
𝑓(𝑘+1) 𝑓(𝑘)
argumento do produtório a fim de chegar em ou .
𝑓(𝑘) 𝑓(𝑘+1)

“1. Obtenha o valor de:


100
1
∏ (1 − 𝑘).”
𝑘=2
Seja o seguinte produto:
100
1
∏ (1 − ) .
𝑘
𝑘=2
1 𝑘−1 𝑓(𝑘−1)
O argumento deste produtório é 1 − = , o qual já se encontra no padrão , onde
𝑘 𝑘 𝑓(𝑘)
𝑓(𝑘) = 𝑘.
Então, podemos aplicar a propriedade, de tal modo que:
𝑏
𝑓(𝑘 − 1) 𝑓(𝑎 − 1) 𝑓(𝑎) 𝑓(𝑏 − 2) 𝑓(𝑏 − 1) 𝒇(𝒂 − 𝟏)
∏( )=( ).( ).….( ).( )=( ).
𝑓(𝑘) 𝑓(𝑎) 𝑓(𝑎 + 1) 𝑓(𝑏 − 1) 𝑓(𝑏) 𝒇(𝒃)
𝑘=𝑎
Assim:
100
1 𝑓(𝑎−1) 2−1 𝟏
∏ (1 − 𝑘) = 𝑓(𝑏)
=
100
=
𝟏𝟎𝟎
.
𝑘=2
“2. Calcule:
500
2𝑘+1
∏ (1 + 𝑘2
).”
𝑘=1
O argumento deste produto é tal que:
2𝑘+1 𝑘²+2𝑘+1 (𝑘+1)2 𝑓(𝑘+1)
1+ = = = , onde 𝑓(𝑘) = 𝑘².
𝑘2 𝑘² 𝑘² 𝑓(𝑘)
Assim, basta aplicar a propriedade:
500 500
2𝑘+1 (𝑘+1)² 𝑓(500+1) 5012
∏ (1 + 𝑘2
)=∏ 𝑘²
=
𝑓(1)
=
12
= 𝟓𝟎𝟏².
𝑘=1 𝑘=1
“3. Calcule:
1000
𝑘²+2𝑘
∏ ( 𝑘²−1 ).”
𝑘=2
O argumento do produto é tal que:
𝑘 2 +2𝑘 𝑘 2 +2𝑘+1−1 (𝑘+1)2 −1 𝑓(𝑘+1)
= = = , onde 𝑓(𝑘) = 𝑘² − 1.
𝑘 2 −1 𝑘 2 −1 𝑘²−1 𝑓(𝑘)
Deste modo:
1000 1000
𝑘² + 2𝑘 (𝑘 + 1)2 − 1 𝑓(1000 + 1) 𝟏
∏( )= ∏( )= = (𝟏𝟎𝟎𝟏𝟐 − 𝟏).
𝑘² − 1 𝑘² − 1 𝑓(2) 𝟑
𝑘=2 𝑘=2
“4. Se:
25
𝑘3 − 1 𝑚
∏( 3 ) = .
𝑘 + 3𝑘 2 + 3𝑘 17000 + 𝑛
𝑘=2
Obtenha o valor de 𝑚 + 𝑛.”
O argumento do produto em questão é tal que:
𝑘 3 −1 𝑘 3 −1 𝒌𝟑 −𝟏 𝑓(𝑘)
= = = , onde 𝑓(𝑘) = 𝑘 3 − 1.
𝑘 3 +3𝑘 2 +3𝑘 𝑘³+3𝑘 2 +3𝑘+1−1 (𝒌+𝟏)𝟑 −𝟏 𝑓(𝑘+1)
Deste modo:
25 25 3
𝑘3 − 1 𝑘 −1 𝑓(2) 23 − 1 7
∏( 3 ) = ∏ ( ) = = = .
𝑘 + 3𝑘 2 + 3𝑘 ( 𝑘 + 1) 3 − 1 𝑓(25 + 1) 263 − 1 17575
𝑘=2 𝑘=2
Assim, 𝑚 = 7 e 𝑛 = 575.
𝒎 + 𝒏 = 𝟓𝟖𝟐.
100
1+𝑎𝑘−1
“5. Calcule ∏ 𝑘 , onde 𝑎 é uma constante real não nula e diferente de um.”
𝑘=1 1+𝑎
100
1+𝑎𝑘−1
É dado o produto ∏ 𝑘 .
𝑘=1 1+𝑎
Tomando 𝑓(𝑘) = 1 + 𝑎𝑘−1 , temos que 𝑓(𝑘 + 1) = 1 + 𝑎𝑘+1−1 = 1 + 𝑎𝑘 .
Assim, vale a propriedade Telescópica:
𝒃
𝒇(𝒌) 𝒇(𝒂)
∏ = .
𝒌=𝒂 𝒇(𝒌+𝟏) 𝒇(𝒃+𝟏)

Enfim, segue que:


100
1+𝑎𝑘−1 𝒇 (𝟏 )
∏ 𝑘 = , onde 𝑓(𝑘) = 1 + 𝑎𝑘−1 .
𝒇(𝟏𝟎𝟏)
𝑘=1 1+𝑎
𝒇(𝟏) 1 + 𝑎1−1 𝟐
= = .
𝒇(𝟏𝟎𝟏) 1 + 𝑎101−1 𝟏 + 𝒂𝟏𝟎𝟎
100
𝑟²−1
“6. Obtenha o valor de ∏ .”
𝑟=3 𝑟²−4
𝑟 2 −1
O argumento do produtório fornecido é , o qual pode ser simplificado da seguinte
𝑟 2 −4
maneira:
2
𝑟 − 1 (𝑟 + 1)(𝑟 − 1)
= .
𝑟 2 − 4 (𝑟 + 2)(𝑟 − 2)
100 100 100 100
𝑟²−1 (𝑟+1)(𝑟−1) 𝑟+1 𝑟−1
Assim, ∏ =∏ = (∏ ) . (∏ ).
𝑟=3 𝑟²−4 𝑟=3 (𝑟+2)(𝑟−2 ) 𝑟=3 𝑟+2 𝑟=3 𝑟−2
Assim, definamos:
100 100 100
𝑟²−1 𝑟+1 𝑟−1
∏ = 𝒙. 𝒚, onde 𝒙 = ∏ e𝒚=∏ .
𝑟=3 𝑟²−4 𝑟=3 𝑟+2 𝑟=3 𝑟−2
Perceba que ambos 𝑥 e 𝑦 estão no padrão para a aplicação da propriedade telescópica:
𝑏
𝑓(𝑘) 𝑓(𝑎)
𝟏. ∏ =
𝑘=𝑎 𝑓(𝑘+1) 𝑓(𝑏+1)
𝑏 ;
𝑓(𝑘) 𝑓(𝑏)
𝟐. ∏ =
{ 𝑘=𝑎 𝑓(𝑘−1) 𝑓(𝑎−1)
Dito isto e aplicando 1 para 𝑥 e 𝟐 para 𝑦:
100 100
𝑟+1 𝑓1 (𝑟) 𝑓1 (3)
𝑥=∏ ⟹𝑥=∏ = , onde 𝑓1 (𝑟) = 𝑟 + 1.
𝑟=3 𝑟+2 𝑟=3 𝑓1 (𝑟+1) 𝑓1 (100+1)
100
𝑓1 (𝑟) 𝑓1 (3) 4 2
𝑥=∏ = = = .
𝑟=3 𝑓1 (𝑟+1) 𝑓1 (101) 102 51
100
𝑟−1 𝑓2 (𝑟) 𝑓2 (100)
𝑦=∏ ⟹ 𝑦 = ∏100
𝑟=3 = , onde 𝑓2 (𝑟) = 𝑟 − 1.
𝑟=3 𝑟−2 𝑓2 (𝑟−1) 𝑓2 (3−1)
100
𝑟−1 𝑓2 (100) 99
𝑦=∏ = = = 99.
𝑟=3 𝑟−2 𝑓2 (2) 1
Enfim:
100
𝑟²−1 2 198 𝟔𝟔
∏ = 𝑥. 𝑦 = ( ) . 99 = = .
𝑟=3 𝑟²−4 51 51 𝟏𝟕
“7. (HARVARD – MIT) – Calcule:
2016
𝑘²
∏ .”
𝑘=2 𝑘²−1
2016
𝑘²
Dado o produto ∏ , seu argumento pode ser simplificado da seguinte maneira:
𝑘=2 𝑘²−1
𝑘2 𝑘.𝑘 𝒌 𝒌
= =( ).( ).
𝑘 2 −1 (𝑘+1)(𝑘−1) 𝒌+𝟏 𝒌−𝟏
Então:
2016 2016 2016 2016
𝑘² 𝑘 𝑘 𝑘 𝑘
∏ = ∏ [( ).( )] = (∏ ) . (∏ ).
𝑘² − 1 𝑘+1 𝑘−1 𝑘+1 𝑘−1
𝑘=2 𝑘=2 𝑘=2 𝑘=2
2016 2016
𝑘 𝑓(𝑘) 𝑓(2)
1: ∏ =∏ = , onde 𝑓(𝑘) = 𝑘.
𝑘=2 𝑘+1 𝑘=2 𝑓(𝑘+1) 𝑓(2016+1)
2016 2016
𝑘 𝑓(𝑘) 𝑓(2) 𝟐
∏ =∏ = = .
𝑘+1 𝑓(𝑘 + 1) 𝑓(2017) 𝟐𝟎𝟏𝟕
𝑘=2 𝑘=2
2016 2016
𝑘 𝑓(𝑘) 𝑓(2016)
2: ∏ =∏ = , onde 𝑓(𝑘) = 𝑘.
𝑘=2 𝑘−1 𝑘=2 𝑓(𝑘−1) 𝑓(2−1)
2016 2016
𝑘 𝑓(𝑘) 𝑓(2016)
∏ =∏ = = 𝟐𝟎𝟏𝟔.
𝑘−1 𝑓(𝑘 − 1) 𝑓(1)
𝑘=2 𝑘=2
Deste modo:
2016 2016
𝑘 𝑘 2 𝟒𝟎𝟑𝟐
(∏ ) . (∏ )=( ) . 2016 = .
𝑘+1 𝑘−1 2017 𝟐𝟎𝟏𝟕
𝑘=2 𝑘=2
“8.
cos 91° 7 cos 92° 7 cos 94° 7 cos 359° 7 cos7 (𝑝°)
( ) .( ) .( ) .….( ) = .
cos 92° cos 93° cos 95° cos 360° cos7 (𝑞°)
𝑝 + 𝑞 = ?.”
cos 91° 7 cos 92° 7 cos 94° 7 cos 359° 7
( ) .( ) .( ) .….( )
cos 92° cos 93° cos 95° cos 360°
cos 91° 𝐜𝐨𝐬 𝟗𝟐° 𝐜𝐨𝐬 𝟗𝟑° 𝐜𝐨𝐬 𝟑𝟓𝟗° 7 cos 7 (91°)
=( . . .…. ) = .
𝐜𝐨𝐬 𝟗𝟐° 𝐜𝐨𝐬 𝟗𝟑° 𝐜𝐨𝐬 𝟗𝟒° cos 360° cos 7 (360°)
𝑝 + 𝑞 = 91 + 360 = 𝟒𝟓𝟏.”
29
𝑘²+5𝑘+6 1
“8. Dado que ∏ = , onde 𝑝 > 0, mostre que
𝑘=1 𝑘²+7𝑘+12 𝑝²+7
1
√𝑝 − 4 < 3.”
√𝑝
O argumento do produtório em questão é tal que:
𝑘² + 5𝑘 + 6 𝑘 2 + 5𝑘 + 6 𝑘 2 + 5𝑘 + 6
= =
𝑘² + 7𝑘 + 12 (𝑘 2 + 2𝑘 + 1) + 5𝑘 + 11 (𝑘 + 1)2 + 5𝑘 + 5 + 6
𝑘 2 + 5𝑘 + 6 𝑓(𝑘)
= = .
(𝑘 + 1)2 + 5(𝑘 + 1) + 6 𝑓(𝑘 + 1)
Assim:
29 29
𝑘²+5𝑘+6 𝑓(𝑘)
∏ =∏ , onde 𝑓(𝑘) = 𝑘 2 + 5𝑘 + 6.
𝑘=1 𝑘²+7𝑘+12 𝑘=1 𝑓(𝑘+1)
29 29
𝑘²+5𝑘+6 𝑓(𝑘) 𝑓(1) 𝑓(1) 12
∏ =∏ = = =
𝑘=1 𝑘²+7𝑘+12 𝑘=1 𝑓(𝑘+1) 𝑓(29+1) 𝑓(30) 30²+5.30+6
12 𝟏
= = .
1056 𝟖𝟖
1 1
= ⟹ 𝑝2 = 81 ⟹ 𝒑 = 𝟗, pois 𝑝 > 0.
𝑝2 +7 88
1 1 1 1 35
√𝑝 − 4 < 3 ⟹ √9 − =3− =3− = .
√𝑝 4√9 4.3 12 12
35 𝟑𝟔 35
Temos, então, que
12
< 𝟏𝟐 = 𝟑, pois 12 ≅ 2,9 < 3, como queríamos demonstrar.
“9. Sejam 𝑎 e 𝑏 reais positivos e diferentes de zero, com 𝑎 > 𝑏, tais que:
5
= 𝑎𝑏.
1 1 1 1 1
(1 − 2) (1 − 3) (1 − 4) . … . (1 − ) (1 − 2016)
2015
Obtenha 𝑎 − 𝑏.”
Temos que:
5 5
𝑎𝑏 = = .
1 1 1 1 1 2016
1
(1 − 2) (1 − 3) (1 − 4) . … . (1 − ) (1 − 2016) ∏ (1 − )
2015 𝑘 𝑘=2
2016
1
No denominador temos o produtório ∏ (1 − 𝑘), o qual pode ser reescrito da seguinte
𝑘=2
maneira:
2016 2016
1 𝑘−1
∏ (1 − 𝑘) = ∏ ( 𝑘
).
𝑘=2 𝑘=2
𝑓(𝑘−1)
Percebamos que este já se encontra na forma , onde 𝑓(𝑘) = 𝑘.
𝑓(𝑘)
Assim:
2016
𝑘−1 𝑓(𝑎 − 1) 𝑓(2 − 1) 𝑓(1) 1
∏( )=( )= = = .
𝑘 𝑓(𝑏) 𝑓(2016) 𝑓(2016) 2016
𝑘=2
5
Temos que 𝑎𝑏 = 1 = 𝟓. 𝟐𝟎𝟏𝟔.
2016
Assim, 𝑎 = 2016 e 𝑏 = 5, pois 𝑎 > 𝑏 e 𝒂 − 𝒃 = 𝟐𝟎𝟏𝟏.

E assim se encerra nossa teoria acerca dos produtórios.


A seguir, apresentaremos, no item VI, os exercícios resolvidos, cobrindo toda a teoria
apresentada.
Após o término deste capítulo, apresentaremos, no capítulo 3, um breve apêndice com a
teoria das Séries Telescópicas, a qual será abordada de maneira rápida, para que possamos
avançar para os exercícios finais de aprofundamento, os quais não apresentarão uma
mudança significativa na dificuldade, porém exigirão uma dose adicional de raciocínio
abstrato, manipulações algébricas favoráveis, desdobramentos, desenvolturas matemáticas
levemente mais apuradas e, até mesmo, “certos truques”, pois contemplarão toda a teoria até
lá apresentada.
VI. Exercícios resolvidos
1. Notação Pi
⊕ − 1. Passe para a notação Pi os seguintes produtos:
𝛾 𝛾 𝛾 𝛾
𝑎) ( ) . ( ) . ( ) . … . ( 2016);
3 9 27 3
𝑏) sec(𝛼) . sec 3 . (𝛼). sec 5 (𝛼). … . sec 2𝑛−1 (𝛼);
𝑐) (𝑓(𝑥)). (𝑓(𝑥 + 1)). (𝑓(𝑥 + 2)). … . (𝑓(𝑥 + (𝑛 − 1)));
𝑘
𝑑) (𝑥 + 𝑎). (𝑥 + 𝑎)10 . (𝑥 + 𝑎)100 . (𝑥 + 𝑎)1000 . … . (𝑥 + 𝑎)10 ;
𝑒) 𝜋. 𝜋. 𝜋. … . 𝜋, onde 𝜋 se repete 375 vezes.

R:
2016
𝛾 𝛾 𝛾 𝛾 𝛾
𝑎) ( ) . ( ) . ( ) . … . ( 2016) = ∏ (3𝑘 );
3 9 27 3 𝑘=1
3 5 (𝛼). 2𝑛−1 (𝛼) 𝑛
𝑏) sec(𝛼) . sec . (𝛼). sec … . sec = ∏𝑘=1 sec 2𝑘−1 (𝛼);
𝑐) (𝑓(𝑥)). (𝑓(𝑥 + 1)). (𝑓(𝑥 + 2)). … . (𝑓(𝑥 + (𝑛 − 1))) = ∏𝑛−1
𝑘=0 𝑓(𝑥 + 𝑘);
𝑛
10 100 1000 10𝑛 𝑘
(𝑥 + 𝑎)10 ;
𝑑)(𝑥 + 𝑎). (𝑥 + 𝑎) . (𝑥 + 𝑎) . (𝑥 + 𝑎) . … . (𝑥 + 𝑎) =∏
𝑘=0
375

𝑒) 𝜋. 𝜋. 𝜋. … . 𝜋 = 𝜋 375 = ∏ 𝜋.
𝑘=1

⊕⊕ − 2. Calcule:
5
𝑎) ∏ √𝑘;
𝑘=2
19
𝑏) ∏𝑘=3 𝛼 𝑘 , onde 𝛼 ∈ ℝ∗+ , 𝛼 > 1;
20
1 √5
𝑐) ∏ (2 + ).
𝑘=1 2

R:
5
𝑎) ∏ √𝑘 = (√2). (√3). (√4). (√5) = √2.3.4.5 = √120.
𝑘=2
𝑏) Temos o produto de uma constante com expoente variável, de tal modo que:
19

∏ 𝛼 𝑘 = 𝛼³. 𝛼 4 . 𝛼 5 . … . 𝛼 19 = 𝛼 3+4+5+⋯+19 = 𝛼 187 .


𝑘=3
𝑐) Temos o produto de uma constante, o qual é dado por:
20 20
1 √5 1 √5
∏( + ) = ( + ) .
2 2 2 2
𝑘=1
⊕⊕ − 3. Calcule o valor de:
𝑎) ∏30
𝑘=7 9𝑘;
𝑏) ∏17
𝑘=3 7.

R:
𝑎) Temos o produto de uma constante por uma variável linear:
30 30

∏ 9𝑘 = 9𝑛 (∏ 𝑘) .
𝑘=7 𝑘=7
Temos que 𝑛 é o número de termos deste produto:
𝑎𝑛 = 𝑎1 + (𝑛 − 1)𝑟 ⟹ 30 = 7 + (𝑛 − 1) ⟹ 𝒏 = 𝟐𝟒.
30 30

∏ 9𝑘 = 924 (∏ 𝑘 ) = 924 (7.8.9.10. … .28.29.30)


𝑘=7 𝑘=7
1.2.3. … .7.8.9.10. … .28.29.30 𝟑𝟎!
= 924 ( ) = 𝟗𝟐𝟒 ( ) .
1.2.3.4.5.6 𝟔!
* Neste tipo de problema, é essencial o conhecimento do fatorial e das manipulações
algébricas fundamentais.
𝑏) Temos o produto de uma constante, o qual é dado por:
17

∏ 7 = 7𝑛 .
𝑘=3
𝑛 é o número de termos, obtido através de:
17 = 3 + (𝑛 − 1) ⟹ 𝒏 = 𝟏𝟓.
Assim:
17

∏ 7 = 715 .
𝑘=3

⊕⊕⊕ − 4. Prove que:


𝑛 𝑛−𝑚+1
1 𝑘−1 𝑚 1 1
𝑎) ∏ ( )( ) = ( − )( ) , onde 𝑎, 𝑏 e 𝑐 são constantes reais
𝑘=𝑚 𝑎−𝑏𝑐 𝑘 𝑛 𝑛 𝑎−𝑏𝑐
não nulas e 𝑎 ≠ 𝑏𝑐.
𝑛 1 𝑛
(𝑝−𝑞+1)𝑒 𝑘 1
𝑏) ∏ = 𝑒 2𝑛(𝑛+1) (𝑝 − 𝑞 + 1)𝑛 ( ) , com 𝑝 e 𝑞 constantes não nulas
𝑘=1 𝑝+𝑞 𝑝+𝑞
e 𝑝 ≠ − 𝑞 e 𝑒 é a constante de Euler.

R:
𝑎) Por 𝑎, 𝑏 e 𝑐 serem constantes, vale a P2 (produtório de uma constante por uma variável):
𝑛 𝑥 𝑛
1 𝑘−1 1 𝑘−1
∏( )( )=( ) ∏( ).
𝑎 − 𝑏𝑐 𝑘 𝑎 − 𝑏𝑐 𝑘
𝑘=𝑚 𝑘=𝑚
1
O expoente 𝑥 da constante ( ) é o número de termos deste produto, obtido pelo Termo
𝑎−𝑏𝑐
Geral de uma Progressão Aritmética:
𝑎𝑛 = 𝑎1 + (𝑛 − 1)𝑟 ⟹ 𝑛 = 𝑚 + (𝑥 − 1) ⟹ 𝒙 = 𝒏 − 𝒎 + 𝟏.
Assim:
𝑥 𝑛 𝑛−𝑚+1 𝑛
1 𝑘−1 1 𝑘−1
( ) ∏( )=( ) ∏( ).
𝑎 − 𝑏𝑐 𝑘 𝑎 − 𝑏𝑐 𝑘
𝑘=𝑚 𝑘=𝑚
𝑛
𝑘−1 𝑓(𝑘−1)
O produto ∏ ( ) é um produto telescópico, do tipo , onde 𝑓(𝑘) = 𝑘, cuja
𝑘=𝑚 𝑘 𝑓(𝑘)

fórmula é:
𝑛
𝑘−1 𝑓(𝑚 − 1) 𝒎 − 𝟏 𝒎 𝟏
∏( )= = = − .
𝑘 𝑓(𝑛) 𝒏 𝒏 𝒏
𝑘=𝑚
Enfim:
𝑛 𝑛−𝑚+1 𝑛 𝑛−𝑚+1 𝑚
1 𝑘−1 1 𝑘−1 1 1
∏ ( )( )=( ) ∏ ( )=( ) ( − ),
𝑘=𝑚 𝑎−𝑏𝑐 𝑘 𝑎−𝑏𝑐 𝑘=𝑚 𝑘 𝑎−𝑏𝑐 𝑛 𝑛
como queríamos demonstrar.

𝑏) Sendo 𝑝 e 𝑞 constantes, vale, novamente, a P2:


𝑛 𝑛
(𝑝 − 𝑞 + 1)𝑒 𝑘 𝑝−𝑞+1 𝑥
∏ =( ) ∏ 𝑒𝑘 .
𝑝+𝑞 𝑝+𝑞
𝑘=1 𝑘=1
Novamente, seja 𝑥 o número de termos de termos deste produto:
𝑛 = 1 + (𝑥 − 1) ⟹ 𝒙 = 𝒏.
Assim:
𝑛 𝑛
𝑝−𝑞+1 𝑥 𝑘
𝑝−𝑞+1 𝑛
( ) ∏𝑒 = ( ) ∏ 𝑒𝑘.
𝑝+𝑞 𝑝+𝑞
𝑘=1 𝑘=1
𝑛
O produto ∏𝑘=1 𝑒 𝑘 é o produto de uma constante com expoente variável, de tal modo que
vale a P5:
𝑛

∏ 𝑒 𝑘 = 𝑒¹. 𝑒². 𝑒 3 . … . 𝑒 𝑛−1 . 𝑒 𝑛 = 𝑒 1+2+3+⋯+(𝑛−1)+𝑛 .


𝑘=1
Temos a soma dos 𝑛 primeiros naturais (ou a soma dos 𝑛 primeiros termos de uma P.A de
razão 1) no expoente, a qual fora demonstrada anteriormente no capítulo 0:
1
1+2+3+⋯+(𝑛−1)+𝑛 𝑛(𝑛+1)
𝑒 =𝑒 2 .
Assim:
𝑛 𝑛
(𝑝 − 𝑞 + 1)𝑒 𝑘 𝑝−𝑞+1 𝑛 𝑘
𝑝 − 𝑞 + 1 𝑛 1𝑛(𝑛+1)
∏ =( ) ∏𝑒 = ( ) 𝑒2
𝑝+𝑞 𝑝+𝑞 𝑝+𝑞
𝑘=1 𝑘=1
1
1 𝑛
𝑛(𝑛+1)
=𝑒 2 (𝑝 − 𝑞 + 1 )𝑛 ( ) , C.Q.D.
𝑝+𝑞
2. Propriedades e Produto Telescópico

⊕⊕ − 5. Calcule:
𝑎) ∏𝑛+8
𝑘=1 𝑘 ;
500
𝑘
𝑏) ∏ ( );
𝑘=2 √𝑘²−1
2 3 4 2000
𝑐) ( ) . ( ) . ( ) . … . ( ).
3 4 5 2001

R:
𝑎) Temos o produto de uma variável linear, a qual varia de 1 até 𝑛 + 8, de tal modo que:
𝑛+8

∏ 𝑘 = 1.2.3. … . (𝑛 + 6)(𝑛 + 7)(𝑛 + 8) = (𝒏 + 𝟖)!


𝑘=1
𝑏) Façamos a manipulação do argumento deste produtório, da seguinte maneira:
𝑘 √𝑘. √𝑘 √𝑘 √𝑘 𝑓(𝑘) 𝑓(𝑘)
= =( ).( )=( ).( ).
√𝑘² − 1 √(𝑘 + 1)(𝑘 − 1) √𝑘 + 1 √𝑘 − 1 𝑓(𝑘 + 1) 𝑓(𝑘 − 1)
* 𝒇(𝒌) = √𝒌.
Assim:
500 500 500
𝑘 √𝑘. √𝑘 √𝑘 √𝑘
∏ =∏ = ∏ [( ).( )]
𝑘=2
√𝑘² − 1 𝑘=2
√(𝑘 + 1)(𝑘 − 1) 𝑘=2
√ 𝑘 + 1 √𝑘 − 1
500 500
√𝑘 √𝑘
= (∏ ) . (∏ ).
𝑘=2
√𝑘 + 1 𝑘=2
√𝑘 − 1
Veja que ambos os produtos se encontram nos padrões do Produto Telescópico, assim:
500
√𝑘 𝑓(2) √𝟐
𝟏: ∏ = = .
√𝑘 + 1 𝑓(500 + 1) √𝟓𝟎𝟏
𝑘=2
500
√𝑘 𝑓(500) √500
𝟐: ∏ = = = √500 = 𝟏𝟎√𝟓.
√𝑘 − 1 𝑓(2 − 1) √2 − 1
𝑘=2
Enfim, o produto é:
500 500 500
𝑘 √𝑘 √𝑘 √2 𝟏𝟎√𝟏𝟎
∏ = (∏ ) . (∏ )=( ) . (10√5) = .
𝑘=2
√𝑘² − 1 𝑘=2
√𝑘 + 1 𝑘=2
√𝑘 − 1 √501 √𝟓𝟎𝟏
2 3 4 2000 𝟐
𝑐) ( ) . ( ) . ( ) . … . ( )= .
3 4 5 2001 𝟐𝟎𝟎𝟏

⊕⊕⊕ − 6. Sendo 𝑎 e 𝑏 constantes reais, com (𝑎, 𝑏) > 1, prove que:


𝑛
√𝑘 + 1(√𝑎 + 𝑏 − √𝑎 − 𝑏) 𝑛
∏ = √𝑛 + 1(√𝑎 + 𝑏 − √𝑎 − 𝑏) .
𝑘=1
√𝑘
R:
Faça você.

VII. Exercícios propostos


⊕ − 1. Transponha para a notação de produtório:
𝑎) 𝑥. 𝑥². 𝑥³. … . 𝑥 359 ;
1 1 1 1 1
𝑏) (1 − ) (1 − ) (1 − ) . … . (1 − ) (1 − 2016);
2 3 4 2015
𝑑 𝑑 𝑑 𝑑
𝑐) ( 𝑓(𝑥)) . ( 𝑓(𝑥 + 1)) . ( 𝑓(𝑥 + 2)) . … . ( 𝑓(𝑥 + (𝑛 − 1))) .
𝑑𝑥 𝑑𝑥 𝑑𝑥 𝑑𝑥
𝑑
* 𝑓(𝑥) denota a derivada primeira da função 𝑓 com relação a 𝑥.
𝑑𝑥

𝑑) (∫ cos(𝑥) 𝑑𝑥) . (∫ cos(𝑥 + 1) 𝑑𝑥) . (∫ cos(𝑥 + 2) 𝑑𝑥) . … . (∫ cos(𝑥 + 2016) 𝑑𝑥) .


∗ ∫ 𝑓(𝑥) 𝑑𝑥 denota a primitiva (integral) da função 𝑓.

⊕⊕ − 2. Calcule:
5
𝑎) ∏ (√𝑘 + √𝑘 − 1);
𝑘=1
475
𝑏) ∏𝑘=0 𝑖 𝑘 ;
* 𝑖 = √− 1.
1
𝑐) ∏𝑛 −𝑘 ;
𝑘=0 5

⊕⊕⊕ − 3. Prove que:


2016
1 2!
𝑎) ∏ ( ) = (2016!)−2 ;
𝑘
𝑘=1
1 1 1 𝑛 + 2017
𝑏) (1 + ) (1 + ) . … . (1 + )= .
𝑛 𝑛+1 𝑛 + 2016 𝑛

⊕⊕⊕ − 4. Verifique que:


𝑚+𝑛 𝑛 1
− − 1
1 𝑘−5 2 2 (𝑛+1)(2𝑚+𝑛−10)
∏ ( )𝛼 = (4 + √3) 𝛼2 , onde 𝛼 é uma constante real não
√4+√3
𝑘=𝑚
nula e positiva.

⊕⊕⊕ − 5. Prove que:


4+𝜋
2016
𝜋 1
tan( )
4
∏ tan (𝑘 + ) cot ( (4𝑘 + 𝜋 + 4)) = 𝜋 .
𝑘=1 4 4 tan(2017+ )
4
⊕⊕⊕ − 6. (IME) Obtenha as raízes da equação 𝑧² + 2𝑖𝑧 + 2 − 4𝑖 = 0 e localize-as no
plano complexo.
* 𝑖 é a unidade imaginária tal que 𝑖² = − 1.

⊕⊕ − 7. Prove que:
155
√𝑘
log 3 (∏ ) < 3.
𝑘=3
√𝑘 − 1
Dados: log 3 5 + log 3 √10 ≅ 2,5.

2020
√𝑘(𝑘−1)
⊕⊕⊕ − 8. Prove que ∏ ( 𝑘√𝑘+1 ) → 0, isto é, prove que o produto em questão é
𝑘=2
muito próximo de zero.

⨁⨁⨁ − 9. Calcule os seguintes produtos telescópicos:


3² − 1 4² − 1 501² − 1
𝑎) ( ).( ).….( )
2² − 1 3² − 1 500² − 1
7 8 9 160
𝑏) ( ) . ( ) . ( ) . … . ( );
8 9 10 161
𝑛
𝛼 𝑘+1 −𝛼 𝑘
𝑐) ∏ , onde 𝛼 ∈ ℝ∗+ ;
𝑘=1 𝛼 𝑘 −𝛼 𝑘−1
300
1+cos(𝜃)
𝑑) ∏ ;
𝜃=0 1+cos(𝜃+1)

⊕⊕ − 10. Resolva a seguinte inequação:


log 𝑒 (𝑥 2 − 4𝑥 + 1) < log 𝑒 (6𝑥 2 − 3𝑥).

⊕⊕⊕ − 11. Calcule:


9 10 11 6000
√( ).( ).( ).….( ).
10 11 12 6001

⊕⊕⊕ − 12. Calcule os produtos:


200
𝜋
𝑎) ∏ ;
𝑘=0 𝑘+1
9
𝑏) ∏ (√𝑘 + 1 − √𝑘 − 1);
𝑘=5
12
𝑐) ∏ (𝑘√𝑘);
𝑘=1
5000

𝑑) ln (∏ 𝑒 𝑘 ) .
𝑘=1
⊕⊕⊕ − 13. Demonstre que:
𝑛
𝑟 4 +2 𝑛4 +2
∏ ((𝑟−1)4 ) = , onde 𝑛 > 𝑚 > 0.
𝑟=𝑚 +2 𝑚4 −4𝑚3 +6𝑚2 −4𝑚+3
* Sugestão: Binômio de Newton.

⊕⊕⊕ − 14. Calcule o produto dos 242 termos da sequência


𝑎𝑛 = (− 260 , 3−60 , 259 , − 359 , − 258 , 3−58 , ... ).

⊕⊕⊕ − 15. Prove:


𝑛
1
𝑘 𝑛(𝑛+1)
∏[(𝑟. cis(𝜃)] = (𝑟. cis(𝜃))2 .
0
* 𝑟. cis(𝜃) = 𝑟(cos(𝜃) + 𝑖. sen(𝜃)).

⊕⊕⊕ − 16. Sendo 𝑎 e 𝑏 constantes reais, com (𝑎, 𝑏) > 1, prove que:
𝑛
√𝑘 + 1(√𝑎 + 𝑏 − √𝑎 − 𝑏) 𝑛
∏ = √𝑛 + 1(√𝑎 + 𝑏 − √𝑎 − 𝑏) .
𝑘=1
√𝑘

⊕⊕ − 17. Demonstre que:


2.4.6. … .2𝑛 2𝑛
= .
(1.2.3. … . 𝑛)2 𝑛!

⊕⊕⊕ − 18. Calcule:


100
2𝑘 + 1
∏ ( 𝑘+1 ).
2 +1
𝑘=0

⊕⊕ − 19. Prove que:


𝑛 𝑛−𝑚+1
1 𝜋 2𝑘−1
1 1 1
∏ [( − ) . ( 2𝑘 )] = ( − ) . 𝜋 𝑚−𝑛−1 .
√ 𝑝 √ 𝑞 𝜋 √ 𝑝 √ 𝑞
𝑘=𝑚

⊕⊕ − 20. Prove que o produto de 2 números da forma 𝑥𝑘 , com 𝑘 = [1, 2] é máximo


𝑐2
quando a soma 𝑥1 + 𝑥2 é constante e igual a 𝑐, e é numericamente igual a P𝑚á𝑥 = .
4

† − 21. Prove que:


sen(2α) sen(4α) sen(8α) sen(2𝑛 𝛼) ∏𝑛𝑘=1 sen(2𝑘 . α). (sec 𝑛 (𝛼))
( ).( ).( ).….( 𝑛 )= 1 .
2 cos(𝛼) 4cos(𝛼) 8cos(α) 2 cos(𝛼) 𝑛(𝑛+1)
22
* Sugestão: Utilize o Princípio da Indução Finita.
Capítulo 3 – Exercícios de aprofundamento, gabarito e
resoluções
1. – Apêndice geral
A1. – Trigonometria
1. – Arco da soma:
sen(𝛼 + 𝛽) = sen(𝛼). cos(𝛽) + sen(𝛽). cos(𝛼)
cos(𝛼 + 𝛽) = cos(𝛼) . cos(𝛽) − sen(𝛼). sen(𝛽)
tan(𝛼) + tan(𝛽)
tan(𝛼 + 𝛽) =
{ 1 − tan(𝛼) . tan(𝛽)

2. – Arco da diferença:
sen(𝛼 − 𝛽) = sen(𝛼). cos(𝛽) − sen(𝛽). cos(𝛼)
cos(𝛼 − 𝛽) = cos(𝛼) . cos(𝛽) + sen(𝛼). sen(𝛽)
tan(𝛼) − tan(𝛽)
tan(𝛼 − 𝛽) =
{ 1 + tan(𝛼) . tan(𝛽)

3. – Redução ao primeiro quadrante:


𝜋
sen ( − 𝜃) = cos(𝜃)
2
{ 𝜋
;
cos ( − 𝜃) = sen(θ)
2

4. – Fórmulas de Prostaférese (transformação em produto):


1 1
𝐈: sen(𝛼) + sen(𝛽) = 2sen ( (𝛼 + 𝛽)) cos ( (𝛼 − 𝛽))
2 2
1. Senos: { ;
1 1
𝐈𝐈: sen(𝛼) − sen(𝛽) = 2sen ( (𝛼 − 𝛽)) cos ( (𝛼 + 𝑏))
2 2

1 1
𝐈: cos(𝛼) + cos(𝛽) = 2 cos ( (𝛼 + 𝛽)) cos ( (𝛼 − 𝛽))
2 2
2. Cossenos: { ;
1 1
𝐈𝐈: cos(α) − cos(𝛽) = − 2sen ( (𝛼 + 𝛽)) sem ( (𝛼 − 𝛽))
2 2

sen(𝛼+𝛽)
tan(𝛼) + tan(𝛽) =
cos(𝛼) cos(𝛽)
3. Tangentes: { sen(𝛼−𝛽)
;
tan(𝛼) − tan(𝛽) =
cos(α) cos(β)

5. – Fórmulas do arco duplo:


sen(2𝜃) = 2sen(𝜃) cos(𝜃)
2 (𝜃)
{cos(2𝜃) = 2cos − 1 ou cos(2𝜃) = 1 − 2sen2 (𝜃) ou cos(2𝜃) = cos 2 (𝜃) − sen2 (𝜃)
2 tan(𝜃)
tan(2𝜃) =
1−tan2 (𝜃)
6. – Fórmulas do arco triplo:
sen(3𝜃) = 3sen(𝜃) − 4sen3 (𝜃)
cos(3𝜃) = 4cos 3 (𝜃) − 3 cos(𝜃)
3 tan(𝜃) − tan3 (𝜃)
tan(3𝜃) =
{ 1 − 3tan2 (𝜃)

7. – Fórmulas do arco metade (bissecção de arcos):


𝜃 1 − cos(𝜃)
sen ( ) = ±√
2 2

𝜃 1 + cos(𝜃)
cos ( ) = ±√
2 2

𝜃 1 − cos(𝜃)
tan ( ) = ±√
{ 2 1 + cos(𝜃)

8. – Identidades harmônicas:
𝑏
𝐈: 𝑎. sen(𝜃) + 𝑏. cos(𝜃) = A. sen (𝜃 + arc tan ( ))
𝑎
1. Senos: { 𝑏
;
𝐈𝐈: 𝑎. sen(𝜃) − 𝑏. cos(𝜃) = A. sen (𝜃 − arc tan ( ))
𝑎

𝑏
𝐈: 𝑎. cos(𝜃) + 𝑏. sen(𝜃) = A. cos (𝜃 − arc tan ( ))
𝑎
2. Cossenos: { 𝑏
;
𝐈𝐈: 𝑎. cos(𝜃) − 𝑏. sen(𝜃) = A. cos (𝜃 + 𝑎𝑟𝑐 𝑡𝑎𝑛 ( ))
𝑎
𝒃 𝒃
Onde A = √𝒂𝟐 + 𝒃𝟐 e 𝐚𝐫𝐜 𝐭𝐚𝐧 ( ) = 𝜶 ⟹ 𝐭𝐚𝐧(𝜶) = .
𝒂 𝒂

A2. – Sequências numéricas (progressões aritméticas e geométricas)


1. Termo geral:
𝐏. 𝐀: 𝑎𝑛 = 𝑎1 + (𝑛 − 1)𝑟 ou 𝑎𝑛 = 𝑎𝑚 + (𝑛 − 𝑚)𝑟
{ ;
𝐏. 𝐆: 𝑎𝑛 = 𝑎1 . 𝑞 𝑛−1 ou 𝑎𝑛 = 𝑎𝑚 . 𝑞 𝑛−𝑚

2. Soma:
1 1
𝐏. 𝐀: S𝑛 = 𝑛(𝑎1 + 𝑎𝑛 ) ou S𝑛 = 𝑛(2𝑎1 + (𝑛 − 1)𝑟)
2 2
{ 𝑎1 (𝑞 𝑛 −1) ;
𝐏. 𝐆: S𝑛 =
𝑞−1

3. Produto:
P.G: |P𝑛 | = √(𝑎1 . 𝑎𝑛 )𝑛 .
4. Limite da soma dos infinitos termos de uma P.G:
𝑎
lim𝑛→∞ S𝑛 = 1 , se e somente se |𝑞| < 1.
1−𝑞

A3. – Potenciação
1
1. 𝑎. 𝑎. 𝑎 … . 𝑎. 𝑎 = 𝑎𝑛 e 𝑎−𝑛 = ;
𝑎𝑛
𝑎𝑛
2. 𝑎𝑛 . 𝑎𝑚 = 𝑎𝑛+𝑚 e = 𝑎𝑛−𝑚 ;
𝑎𝑚
𝑎 𝑛 𝑎𝑛
3. (𝑎. 𝑏)𝑛 = 𝑎𝑛 . 𝑏 𝑛 e ( ) = ;
𝑏 𝑏𝑛
0
4. 𝑎 = 1;
5. 𝑎𝑛 = 𝑒 𝑛.ln 𝑎 .

A4. – Radiciação
1
𝑛
1. √𝑎 = 𝑎 ; 𝑛

√𝑎 𝑎
2. √𝑎. √𝑏 = √𝑎𝑏 e =√ ;
√𝑏 𝑏
𝑛 𝑚 𝑚𝑛
3. √ √𝑎 = √𝑎;
1
4. √𝑎 = 𝑎.

A5. – Fatoração básica e produtos notáveis


1. 𝑎2 − 𝑏 2 = (𝑎 + 𝑏)(𝑎 − 𝑏);
2. 𝑎3 + 𝑏 3 = (𝑎 + 𝑏)(𝑎2 − 𝑎𝑏 + 𝑏 2 );
3. 𝑎3 − 𝑏 3 = (𝑎 − 𝑏)(𝑎2 + 𝑎𝑏 + 𝑏 2 );
4. 𝑎4 − 𝑏 4 = (𝑎 + 𝑏)(𝑎 − 𝑏)(𝑎2 + 𝑏 2 );
5. 𝑎𝑛 − 𝑏 𝑛 = (𝑎 − 𝑏)(𝑎𝑛−1 + 𝑎𝑛−2 𝑏 + ⋯ + 𝑏 𝑛−2 𝑎 + 𝑏 𝑛−1 );
6. (𝑎 + 𝑏)2 = 𝑎2 + 2𝑎𝑏 + 𝑏 2 ;
7. (𝑎 + 𝑏)3 = 𝑎3 + 𝑏 3 + 3𝑎𝑏(𝑎 + 𝑏);
8. (𝑎 − 𝑏)3 = 𝑎3 − 𝑏 3 − 3𝑎𝑏(𝑎 − 𝑏);
𝑛
𝑛
9. (𝑎 + 𝑏) = ∑ (𝑛𝑘)𝑎𝑛−𝑘 𝑏 𝑘 (Teorema do Binômio de Newton);
𝑘=0
10. (𝑎 − 𝑏)𝑛 = ∑𝑛𝑘=0(𝑛𝑘)𝑎𝑛−𝑘 (− 𝑏)𝑘 ;
11. (𝑎 + 𝑏 + 𝑐)2 = 𝑎2 + 𝑏 2 + 𝑐 2 + 2(𝑎𝑏 + 𝑎𝑐 + 𝑏𝑐);
12. Se 𝑎 + 𝑏 + 𝑐 = 0, então 𝑎3 + 𝑏 3 + 𝑐 3 = 3𝑎𝑏𝑐;
𝟏3. (𝑎 + 𝑏)2 − (𝑎 − 𝑏)2 = 4𝑎𝑏;
14. (𝑎2 + 𝑏 2 )2 = (𝑎2 − 𝑏 2 )2 + 4𝑎2 𝑏 2 (Identidade de Platão);
15. 𝑎4 + 4𝑏 4 = (𝑎2 + 2𝑏 2 + 2𝑎𝑏)(𝑎2 + 2𝑏 2 − 2𝑎𝑏)
(Identidade de Sophie – Germain);
16. (𝑎2 + 𝑏 2 )(𝑐² + 𝑑 2 ) − (𝑎. 𝑐 + 𝑏. 𝑑)2 = (𝑎. 𝑑 − 𝑏. 𝑐)2 (Identidade de Lagrange);
(𝑥−𝑏)(𝑥−𝑐) (𝑥−𝑐)(𝑥−𝑎) (𝑥−𝑎)(𝑥−𝑏)
17.
(𝑎−𝑏)(𝑎−𝑐)
+ (𝑏−𝑐)(𝑏−𝑎) + (𝑐−𝑎)(𝑐−𝑏) = 𝟏 (Identidade da interpolação).
A6. – Desigualdades elementares
1. 𝑎 + 𝑏 ≥ 2√𝑎𝑏 (Desigualdade das médias de Cauchy);
1
2. 𝑎 + ≥ 2 (Desigualdade da soma dos inversos).
𝑎

2. – A Série Telescópica
A definição mais simples de uma Série Telescópica é o limite da Soma Telescópica
associada (Soma Parcial) quando seu número de termos tende a infinito:
Considerando a série ∑∞𝑘=1(𝑎𝑘+1 − 𝑎𝑘 ), temos que:
∞ 𝒏

∑(𝑎𝑘+1 − 𝑎𝑘 ) = 𝐥𝐢𝐦 [∑(𝒂𝒌+𝟏 − 𝒂𝒌 )] .


𝒏→∞
𝑘=1 𝒌=𝟏
A soma ∑𝑛𝑘=1(𝑎𝑘+1 − 𝑎𝑘 ) é denominada Soma Parcial ou Soma Análoga.
Por ser uma série, esta pode não convergir e, em função disto, nos limitaremos aos
casos nos quais a série converge absolutamente, por serem enormemente mais
simples.
O critério de convergência da série telescópica é tal que:
𝐥𝐢𝐦 (𝒂𝒌+𝟏 − 𝒂𝒌 ) = 𝐋 ⟺ ∃ 𝐥𝐢𝐦 𝒂𝒌.
𝒌→∞ 𝒌→∞
Exemplo:
“1. Obtenha o valor de:

1
∑ .”
𝑘=1 𝑘(𝑘+1)
Temos que:
∞ 𝑛
1 1
∑ = lim (∑ ).
𝑘(𝑘 + 1) 𝑛→∞ 𝑘(𝑘 + 1)
𝑘=1 𝑘=1
O argumento desta soma (soma parcial), quando em frações parciais, é tal que:
1 1 1
= − .
𝑘(𝑘 + 1) 𝑘 𝑘 + 1
𝑛 𝑛
1 1 1 1
⟹∑ = ∑( − )=1− .
𝑘(𝑘 + 1) 𝑘 𝑘+1 𝑛+1
𝑘=1 𝑘=1
𝑛 𝑛
1 1 1 1
⟹ lim (∑ ) = lim [∑ ( − )] = lim (1 − ).
𝑛→∞ 𝑘(𝑘 + 1) 𝑛→∞ 𝑘 𝑘+1 𝑛→∞ 𝑛+1
𝑘=1 𝑘=1
Temos, das propriedades operatórias dos limites, que o limite da soma é equivalente
a soma dos limites: 𝐥𝐢𝐦𝒙→𝒂 (𝒇(𝒙) ± 𝒈(𝒙)) = 𝐥𝐢𝐦𝒙→𝒂 𝒇(𝒙) ± 𝐥𝐢𝐦𝒙→𝒂 𝒈(𝒙).
1 1
⟹ lim (1 − ) = lim 1 − lim ( ).
𝑛→∞ 𝑛+1 𝑛→∞ 𝑛→∞ 𝑛 + 1
1
Verifique você, leitor, que o denominador da função , quando 𝑛 → ∞, se aproxima
𝑛+1
cada vez mais de zero.
1
Ou seja, o limite da função 𝑓(𝑛) = , quando 𝑛 tende a infinito, é zero.
𝑛+1
Caso queira você, leitor, uma prova deste fato, consulte um livro básico de
1
Pré-Cálculo que contenha o gráfico da função e você constatará que, a medida que
𝑛+1
𝑛 assume valores cada vez maiores, se aproximando do infinito (pela direita do eixo
das abcissas), o valor de 𝑓(𝑛) tende a zero, sendo o eixo das abcissas uma assíntota
1
horizontal da função 𝑓(𝑛) = .
𝑛+1
Deste modo:
1 1
lim (1 − ) = lim 1 − lim ( ) = 1 + 0 = 𝟏.
𝑛→∞ 𝑛+1 𝑛→∞ 𝑛→∞ 𝑛 + 1
∞ 𝑛 𝑛
1 1 1 1 1
⟹∑ = lim (∑ ) = lim [∑ ( − )] = lim (1 − ) = 𝟏.
𝑘 (𝑘 + 1) 𝑛→∞ 𝑘 (𝑘 + 1) 𝑛→∞ 𝑘 𝑘+1 𝑛→∞ 𝑛+1
𝑘=1 𝑘=1 𝑘=1

Vale ressaltar que não iremos nos aprofundar neste conceito, pois o conceito de limite e suas
definições não constam em nossa pauta teórica.
Portanto, a Série Telescópica não será explorada de maneira abrangente, como feito nas
universidades, será apresentada apenas uma visão geral desta.

3. – Exercícios finais de aprofundamento e desafios


† − 1. (ROMÊNIA – TITU ANDREESCU) Calcule a soma:
1 1 1
+ + ⋯+ .
√1 + √2 √2 + √3 √𝑛 − 1 + √𝑛

† − 2. (ROMÊNIA – TITU ANDREESCU) Prove que:


1 1 1
+ +⋯+ ≥ 24.
√1 + √3 √5 + √7 √9997 + √9999

† − 3. (HARVARD TOURNAMENT 2006) Calcule:


22 3² 4² 5² 20062
( ).( ).( ).( ).….( ).
2² − 1 3² − 1 4² − 1 5² − 1 20062 − 1

⊕⊕⊕ − 4. Prove que são números naturais:

𝑎)√2√2√2√2 … ;

𝑏) √7 + 4√3 + √7 − 4√3.

⊕⊕⊕ − 5. Demonstre a fórmula do produto dos 𝑛 primeiros termos de uma progressão


geométrica finita:
|P𝑛 | = √(𝑎1 . 𝑎𝑛 )𝑛 , onde 𝑎1 e 𝑎𝑛 são, respectivamente, o primeiro termo e o termo de índice
𝑛 desta progressão geométrica.
† − 6. (RÚSSIA – LIDSKI) Demonstre que, se os números positivos 𝑎1 , 𝑎2 , ..., 𝑎𝑛 formam
uma progressão aritmética, nesta ordem, vale que:
1 1 1 𝑛−1
+ + ⋯+ = .
√𝑎1 + √𝑎2 √𝑎2 + √𝑎3 𝑎 + 𝑎
√ 𝑛−1 √ 𝑛 √ 1 √ 𝑛 𝑎 + 𝑎

† − 7. (RÚSSIA – LIDSKI) Demonstre que, se os números 𝑎1 , 𝑎2 , ..., 𝑎𝑛 são diferentes de


zero e formam uma progressão aritmética, nesta ordem, vale que:
1 1 1 1 𝑛−1
+ + +⋯+ = .
𝑎1 . 𝑎2 𝑎2 . 𝑎3 𝑎3 . 𝑎4 𝑎𝑛−1 . 𝑎𝑛 𝑎1 . 𝑎𝑛

⊕⊕ − 8. Calcule a soma dos 𝑛 primeiros termos de uma progressão aritmética de razão


𝑟 > 0 sem o uso do Princípio da Indução Finita.

⊕⊕⊕ − 9. (ROMÊNIA – TITU ANDREESCU) Calcule:


𝑛
1
∑ .
𝑘=1
(𝑘 + 1)√𝑘 + 𝑘√𝑘 + 1

⊕⊕⊕ − 10. (ROMÊNIA – TITU ANDREESCU – ADAPTADA) Calcule:


2016
1
∏ (1 − ).
𝑘2
𝑘=2

⊕⊕⊕ − 11. (PUTNAM AND BEYOND – TITU ANDREESCU)


Calcule, sabendo-se que o processo de radiciação é infinito:

√1 + √1 + √1 + √1 + ⋯ .

⊕⊕⊕ − 12. Prove que √2 + √2 + √2 + ⋯ = √2√2√2 … .

⊕⊕⊕ − 13. Calcule:



3 3
∑( − ).
𝑘 2 (𝑘 + 1)2
𝑘=1

⊕⊕⊕ − 14. Prove que:


𝑛

∏[sen(𝜃 2 − 𝑘 2 ). cossec((𝜃 − 𝑘 − 1)(𝜃 + 𝑘 + 1))] = sen(𝜃 2 ) cossec(𝜃 2 − (𝑛 + 1)²) .


𝑘=0
⊕⊕⊕ − 15. Demonstre que a soma dos cubos dos 𝑛 primeiros números naturais é igual ao
quadrado da soma dos 𝑛 primeiros números naturais:
1
1³ + 2³ + 3³ + ⋯ + (𝑛 − 1)3 + 𝑛3 = 𝑛2 (𝑛 + 1)2 .
4

⊕⊕⊕ − 16. (OBM 2004 – 1ª FASE – NÍVEL 3)


Sejam:
1001 1001
𝑘² 𝑘2
𝑎=∑ ( )e𝑏 = ∑ ( ).
𝑘=1 2𝑘−1 𝑘=1 2𝑘+1
Determine o inteiro 𝑛 mais próximo de 𝑎 − 𝑏.

1+𝑖
⊕⊕⊕ − 17. (ITA 2004) Dado o complexo 𝑧 = , calcule:
√2
60

|∑ 𝑧 𝑛 | .
𝑛=1

S𝑚 𝑚2
⊕⊕⊕ − 18. Numa progressão aritmética, verifica-se que = , onde S𝑚 e S𝑛 são as
S𝑛 𝑛2
somas dos 𝑚 primeiros termos e dos 𝑛 primeiros termos desta progressão, respectivamente,
com 𝑚 ≠ 𝑛.
Prove que a razão desta progressão é o dobro do primeiro termo.

⊕⊕⊕ − 19. Calcule:


1 1 1
+ + ⋯+ .
1.2.3 2.3.4 2015.2016.2017

24
1
⊕⊕⊕ − 20. O valor numérico de ∑ é:
𝑛=1 √2𝑛+√2𝑛+2
√2
A( ) .
2
B ( ) √2.
√2
C( ) .
3
D ( ) 3√2.
E ( ) 2√2.

⊕⊕⊕ − 21. Calcule:


𝑛+𝑚+1
1 1 𝑘
∏ − .( ) .
𝑘+1
𝑘=𝑚 √ 𝑝 + √𝑞 − 1 √ 𝑞 − √𝑝 + 1
[( ) ]
⊕⊕⊕ − 22. Demonstre o teorema do Determinante de Vandermonde para uma matriz de
ordem 3:
1 1 1
Se A 3 x 3 = [𝑎1 𝑎2 𝑎3 ], onde 𝑎1 , 𝑎2 e 𝑎3 ∈ ℝ ∖ {0}, então
𝑎12 𝑎22 𝑎32
|A| = det A = (𝑎3 − 𝑎2 )(𝑎3 − 𝑎1 )(𝑎2 − 𝑎1 ).

† − 23. Calcule:
13 𝑚

∑ (∑ (2𝑘 + 5)) .
𝑚=1 𝑘=1
* Ver resolução.

† − 24. Dado que:


𝑛 𝑚
(𝑘 + 1)² 1
∑ (∏ ) = 𝑛(2𝑛2 + 9𝑛 + 13).
𝑘² 𝑎
𝑚=1 𝑘=1
Obtenha o valor da constante 𝑎.
* Ver resolução.

† − 25. Calcule:
5 𝑚 5 𝑚

∏ (∑ 𝑘) + ∑ (∏ 𝑘) .
𝑚=1 𝑘=1 𝑚=1 𝑘=1

† − 26. (IME 2014) Determine o(s) valor(es) de 𝑥, inteiros e positivos, que satisfaz(em) a
equação:
𝑥 𝑦−1

𝑥² = ∑ [∏(𝑦 − 𝑧)] .
𝑦=1 𝑧=0

† − 27. Tendo em mente as seguintes propriedades da derivação:


𝑑
I: (𝑥 𝑛 ) = 𝑛. 𝑥 𝑛−1 , onde 𝑛 é um inteiro qualquer.
𝑑𝑥
𝑑 𝑑 𝑑
II: (𝑓(𝑥 ) + 𝑔(𝑥 )) = (𝑓 (𝑥 )) + (𝑔(𝑥 ))
𝑑𝑥 𝑑𝑥 𝑑𝑥
(a derivada da soma é a soma das derivadas).
𝑑
III: (𝑘) = 0, onde 𝑘 é uma constante real qualquer.
𝑑𝑥
Calcule a derivada de:
𝑥2 𝑥3 𝑥𝑛
S𝑛 = (𝑥 + 𝑘) + ( + 𝑘) + ( + 𝑘) + ⋯ + ( + 𝑘), levando em consideração que
2 3 𝑛
|𝑥| < 1.
𝑘 1
⊕⊕⊕ − 28. Dado que ∑𝑛𝑘=0 (𝑘+1)! = 1 − (𝑛+1)!, calcule a série abaixo:
1 2 3 4
+ + + +⋯
2! 3! 4! 5!

† − 29. (PUTNAM 1984)


𝑎) Verifique que:
𝑘
6 3𝑘 3𝑘+1
= − .
(3𝑘+1 − 2𝑘+1 )(3𝑘 − 2𝑘 ) 3𝑘 − 2𝑘 3𝑘+1 − 2𝑘+1
* Expanda em frações parciais.
𝑏) Calcule o valor de:

6𝑘
∑ .
(3𝑘+1 − 2𝑘+1 )(3𝑘 − 2𝑘 )
𝑘=1
* Ver resolução.

⊕⊕⊕ − 30. Calcule:


2016 𝑚−7

∏ ( ∑ (𝑘 2 + 5𝑘)) .
𝑚=7 𝑘=1

⊕⊕⊕ − 31. (IME 2016) O valor do somatório abaixo é:


15
𝜋
∑ Im (cis 2𝑘−1 ( )) .
36
𝑘=1
2+√3
A( ) 𝜋 .
4sen(36)
2−√3
B( ) 𝜋 .
4sen( )
36

1
C( ) 𝜋 .
4sen(36)
π
D ( ) sen ( ).
36
1
E( ) .
4
* Observação: cis(𝛼) = cos(𝛼) + 𝑖. sen(𝛼), Im(𝑧) é a parte imaginária do
𝑛
número complexo 𝑧 e cis 𝑛 (𝛼) = (cis(𝛼)) = cis(𝑛. 𝛼).

⊕⊕⊕ − 32. Calcule:


4 2𝑛 6 𝑚+9
𝑘
∏ (∏ ) − ∑ ( ∑ (𝑝 − 5)) .
𝑘+1
𝑛=1 𝑘=3 𝑚=1 𝑝=1

1
† − 33. Sabendo-se que P𝑛 = ∏ (1 + 𝑘 )
𝑘=0 22

pode ser escrito na forma √1 + √1 + 𝑎, com 1 + √1 + 𝑎 > 0, obtenha o valor do


1
logaritmo, na base (1.000.000³ − 1 ), da diferença entre o número 𝛽 que
1+
√ 7+7
excede 𝑎 em 32 unidades e o número 𝛾 que é numericamente igual a 𝛼 2 − 25.
* Dica: P𝑛 converge para um número natural.

⊕⊕⊕ − 34. Se:


2 3
7 7 2√𝑚 + √𝑛 7
1+( )+( ) +( ) +⋯= .
√100 + √140 √100 + √140 √100 + √140 2√𝑚 + √𝑛 − 𝑝√𝑞
Calcule 𝑚 + 𝑛 + 𝑝 + 𝑞.

⊕⊕⊕ − 35. (IME 2007) Considere a sequência de triângulos retângulos cuja lei de
formação é dada por:
2
𝑎𝑘+1 = 𝑎𝑘
3
{ 4 ;
𝑏𝑘+1 = 𝑏𝑘
5
onde 𝑎𝑘 e 𝑏𝑘 , para 𝑘 ≥ 1, são os comprimentos dos catetos do 𝑘-ésimo triângulo retângulo.
Dado que 𝑎1 = 30 𝑢. 𝑐 (unidades de comprimento) e 𝑏1 = 42 𝑢. 𝑐, determine o valor da
soma das áreas de todos os triângulos, em (𝑢. 𝑐)² (unidades de área), quando 𝑘 → ∞.

† − 36. (IIT – JEE – ÍNDIA) Dado que:


48 47 46 2 1 1 1 1
+ + +⋯+ + = 51⁄2 + 𝑘. (1 + + + ⋯ + ), o valor da
2.3 3.4 4.5 48.49 49.50 2 3 50
constante 𝑘 é:
A ( ) − 1.
1
B( )− .
2
C ( ) 1.
D ( ) 2.

5000
⊕⊕⊕ − 37. Dado o número complexo 𝑧𝑟 , tal que 𝑧𝑟 = ∑𝑘=0 𝑖 𝑘 , calcule o valor de
(𝑧𝑟 )2016 + |𝑧𝑟 |2016 .
∗ 𝑖 = √− 1.

⊕⊕⊕ − 38. Resolva a seguinte equação, considerando 𝜃 ∈ [0, 𝜋):



∑𝑛=0(cos(𝜃))2𝑛 = 5.
⊕⊕⊕ − 39. (OPM – ADAPTADA) Seja 𝑓 uma função tal que 𝑓(1) = 17 e
𝑛
𝑓(𝑛) = , ∀ 𝑛 ∈ ℕ > 1.
𝑓(𝑛−1)
Calcule:
𝑓(1). 𝑓(2). 𝑓(3). … . 𝑓(2015). 𝑓(2016).

⊕⊕⊕ − 40. Mostre que numa progressão aritmética de razão 𝑟 > 0, na qual S𝑘 é a soma
dos 𝑘 primeiros termos desta progressão, verifica-se que:
1
(S𝑛 − S𝑛−1 ) + (S𝑛−1 − S𝑛−2 ) + (S𝑛−2 − S𝑛−3 ) + ⋯ + (S3 − S2 ) + (S2 − S1 ) = (𝑎 (𝑛 − 2) + 𝑛. 𝑎𝑛 ).
2 1

⊕⊕⊕ − 41. (CAIO GUIMARÃES)


𝜃 𝜃
Prove que 1 + cis(𝜃) = 2 cos ( ) cis ( ),
2 2
sabendo-se que cis(𝜃) = cos(𝜃) + 𝑖. sen(𝜃).

⊕⊕ − 42. (FUVEST – 2016)


Utilize as propriedades logarítmicas para simplificar a expressão:
1 1 1
S= + + .
2. log 2 2016 5. log 3 2016 10. log 7 2016
Assim, o valor de S é:
1
A( ) .
2
1
B( ) .
3
1
C( ) .
5
1
D( ) .
7
1
E( ) .
10

⊕⊕⊕ − 43. Calcule:


1 1 1 1
S𝑛 = + + +⋯+ 𝑛 .
2. log 2 2016 4. log 2 2016 8. log 2 2016 2 . log 2 2016
* Sugestão: Mudança de base.

⊕⊕⊕ − 44 (IIT – JEE) Dado 𝑛 = 2010! , o valor de


1 1 1
+ +⋯+ é igual a:
log2 𝑛 log3 𝑛 log2010 𝑛
A ( ) − 1.
B ( ) 0.
C ( ) 1.
D ( ) 2.
* Sugestão: Novamente, mudança de base.
⊕⊕⊕ − 45. Dada a equação 𝑎𝑥 2 + 𝑏𝑥 + 𝑐 = 0, na qual 𝑎, 𝑏 e 𝑐 são números e 𝑎 ≠ 0,
cujas raízes são os números 𝑥1 e 𝑥2 , distintos entre si, prove que:
𝑏 √∆
𝑎) 𝑥 = − ± , ∆ = 𝑏 2 − 4𝑎𝑐.
2𝑎 2𝑎
𝑏
𝑏) 𝑥1 + 𝑥2 = − .
𝑎
𝑐
𝑐) 𝑥1 . 𝑥2 = .
𝑎
√∆
𝑑) 𝑥1 − 𝑥2 = , se 𝑥1 > 𝑥2 .
𝑎
√∆
𝑒) 𝑥2 − 𝑥1 = − , se 𝑥1 > 𝑥2 .
𝑎

⊕⊕⊕ − 46. (OPCIONAL 1 – TEOREMA DAS SOMAS DE NEWTON)


𝑎) Prove que toda equação de segundo grau pode ser escrita na forma 𝑥 2 − S. 𝑥 + P = 0,
onde S e P são, respectivamente, a soma e o produto de suas raízes.
𝑏) Prove o Teorema das Somas de Newton para uma equação polinomial de grau 2:
Dada a equação 𝑎𝑥 2 + 𝑏𝑥 + 𝑐 = 0, cujas raízes são 𝑥1 e 𝑥2 , define-se S𝑘 = 𝑥1𝑘 + 𝑥2𝑘 ,
onde 𝑘 é um inteiro qualquer.
Assim:
𝒂. 𝐒𝒌+𝟐 + 𝒃. 𝐒𝒌+𝟏 + 𝒄. 𝐒𝒌 = 𝟎.
𝑐) Calcule, aplicando o Teorema das Somas de Newton, a expressão:
10 10
1 √5 1 √5
( + ) +( − ) .
2 2 2 2
𝑑) Calcule:
6 6
(√5 + 2√3) + (√5 − 2√3) .

† − 47. (OPCIONAL 2 – TEOREMA DOS POLINÔMIOS SIMÉTRICOS)


Sejam os números reais 𝛼, 𝛽 e 𝛾, definamos a soma de suas 𝑘-ésimas potências da seguinte
forma S𝑘 = 𝛼 𝑘 + 𝛽 𝑘 + 𝛾 𝑘 , onde 𝑘 ∈ ℕ ∖ {0}.
Sejam, também, 𝜎1 = 𝛼 + 𝛽 + 𝛾, 𝜎2 = 𝛼. 𝛽 + 𝛼. 𝛾 + 𝛽. 𝛾 e 𝜎3 = 𝛼. 𝛽. 𝛾.
Verifique, pelo processo indutivo, que para todo 𝑘 natural não nulo, vale que:
S𝑘 = (𝛼 + 𝛽 + 𝛾)(𝛼 𝑘−1 + 𝛽 𝑘−1 + 𝛾 𝑘−1 ) − (𝛼. 𝛽 + 𝛼. 𝛾 + 𝛽. 𝛾)(𝛼 𝑘−2 + 𝛽 𝑘−2 + 𝛾 𝑘−2 ) +
𝛼. 𝛽. 𝛾(𝛼 𝑘−3 + 𝛽 𝑘−3 + 𝛾 𝑘−3 ) ⟹ 𝐒𝒌 = 𝝈𝟏 . 𝐒𝒌−𝟏 − 𝝈𝟐 . 𝐒𝒌−𝟐 + 𝝈𝟑 . 𝐒𝒌−𝟑 .

† − 48. (OPCIONAL 3)
3
Considere a equação 𝑧 − 6. 𝑧 + 5 = 0, na variável 𝑧 complexa, cujas raízes são os números
𝑧1 , 𝑧2 e 𝑧3 .
Obtenha, sem resolver a equação, o valor da expressão:
3 3 3

𝜓(𝑧) = ∑ 𝑧𝑘 + ∑ 𝑧𝑘2 + ∑ 𝑧𝑘3 .


𝑘=1 𝑘=1 𝑘=1
⊕⊕⊕ − 49.
𝑎) Prove a fórmula da diferença de quadrados:
𝑎2 − 𝑏 2 = (𝑎 + 𝑏)(𝑎 − 𝑏).
𝑏) Dado que:
995
1000² − (𝑘 + 1)² 𝑎
∏ = .
1000² − 𝑘² 𝑏
𝑘=1
Obtenha 𝑎 + 𝑏.

⊕⊕⊕ − 50.
Prove a fórmula da diferença de cubos:
𝑎3 − 𝑏 3 = (𝑎 − 𝑏)(𝑎2 + 𝑏 2 + 𝑎𝑏).
𝑏) Sendo 𝑎 ∈ ℝ, com 𝑎 > 0 ≠ 1, prove que:
𝑛
(𝑎 − 𝑘)(𝑎2 + 𝑎𝑘 + 1) 𝑎3
∏ = .
(𝑎 − 𝑘 − 1)(𝑎2 + 𝑎𝑘 + 𝑎 + 𝑘 2 + 2𝑘 + 1) (𝑎 − 𝑛 − 1)(𝑎2 + 𝑎𝑛 + 𝑎 + 𝑛2 + 2𝑛 + 1)
𝑘=0

⊕⊕⊕ − 51. Obtenha o domínio D𝑓 da função 𝑓 abaixo:


∑3𝑘=0 𝑥 𝑘
𝑓(𝑥) = √ 2 .
∑𝑘=1 𝑥 𝑘

† − 52. A solução da desigualdade abaixo é um intervalo da forma (𝑎, 𝑏).


7 7

𝑥 4 + 𝑥 3 + ∑ 𝑥 𝑘 − ∑ 𝑥 𝑘 < 0.
𝑘=1 3
Calcule o valor de 𝑏 − 𝑎.
* (𝑎, 𝑏) = {𝑥 ∈ ℝ: 𝑎 < 𝑥 < 𝑏}.

† − 53. (INSTITUTO DE TECNOLOGIA DE ILLINOIS)


Se 𝑧 = cos(𝜃) + 𝑖. sen(𝜃), calcule, para 𝜃 = 2°, o valor de
15

∑ Im( 𝑧 2𝑟−1 ).
𝑟=1

1 3 5 7
† − 54. O valor de 𝑥 em 1 + 2𝑥 + 3𝑥² + 4𝑥³ + ⋯ = + + + + ⋯ é:
2 4 8 16
1
A( ) .
√2
√3
B( ) .
2
√3
C( )1− .
3
√2
D( )1 − .
2
† − 55. (SPIEGEL) Se 𝑛 ∈ ℕ > 1, determine o valor da soma:
2𝜋 4𝜋 6𝜋 2(𝑛 − 1)𝜋
sen ( ) + sen ( ) + sen ( ) + ⋯ + sen ( ).
𝑛 𝑛 𝑛 𝑛

† − 56. (SBM) Determine o valor da soma S abaixo, para 𝑛 ∈ ℕ:


𝜋 2𝜋 3𝜋 𝑛𝜋
S = sen ( ) + sen ( ) + sen ( ) + ⋯ + sen ( ) .
𝑛 𝑛 𝑛 𝑛

† − 57. (CAIO GUIMARÃES) Determine uma expressão para as seguintes somas:


𝑎) cos(𝑥) + cos(2𝑥) + cos(3𝑥) + ⋯ + cos(𝑛𝑥);
𝑏) sen(𝑥) + sen(2𝑥) + sen(3𝑥) + ⋯ + sen(𝑛𝑥).

† − 58. Calcule, sendo 𝑟 > 0 ≠ 1 a razão da progressão aritmética


(𝛼, 𝛼 + 𝑟, 𝛼 + 2𝑟, ..., 𝛼 + 𝑛𝑟), as somas:
𝑎) cos(𝛼) + cos(𝛼 + 𝑟) + cos(𝛼 + 2𝑟) + ⋯ + cos(𝛼 + 𝑛𝑟);
𝑏) sen(𝛼) + sen(𝛼 + 𝑟) + sen(𝛼 + 2𝑟) + ⋯ + sen(𝛼 + 𝑛𝑟).

2016
𝜋 𝜋
† − 59. Se ∑ cos ( + 𝑘 ) = 𝑎 + 𝑏√𝑐, com {𝑎, 𝑏, 𝑐} ⊂ ℚ ∖ {0},
𝑘=0 5 7

determine o valor de 𝑎 + 𝑏 + 𝑐.

2016
𝜋 𝜋 𝜋
† − 60. Prove que ∑ sen ( + 𝑘 ) = sen ( ).
𝑘=0 3 9 3
4. – Gabarito e resoluções
Capítulo 0
1. 2𝑛 + 1 + 2𝑛 + 3 = 4𝑛 + 4 = 4(𝑛 + 1) = 2.2(𝑛 + 1) ⟹ par.
2. Sejam os ímpares 2𝑘 + 1, 2𝑘 + 3, 2𝑘 + 5 e 2𝑘 + 7.
(2𝑘 + 1) + (2𝑘 + 3) + (2𝑘 + 5) + (2𝑘 + 7) = 8𝑘 + 16 = 2(𝑘 + 8) ⟹ par.
3. Sejam os ímpares do tipo 2𝑝 + 1.
Tomando 𝑘 = 1 ⟹ (2𝑝 + 1) + (2𝑝 + 3) = 2.2(𝑝 + 1) ⟹ par.
Tomando 𝑘 = 𝑛 ⟹ (2𝑝 + 1) + (2𝑝 + 3) + (2𝑝 + 5) + ⋯ + (2𝑝 + 𝑛)
1
= 2. 𝑛. 𝑝 + (1 + 2 + ⋯ + 𝑛) = 2. 𝑛. 𝑝 + 𝑛(𝑛 + 1)
2
4.𝑛.𝑝+𝑛²+𝑛 𝑛(4𝑝+𝑛+1) (4𝑝+𝑛+1)
= = = 2𝑛 ⟹ par.
2 2 4
4. Aplicando a fórmula da soma da P.G (ver apêndice), obtemos:
𝑛
1. (𝑟 𝑛 − 1) 𝑟 𝑛 − 1 𝟏 − 𝒓𝒏
1 + 𝑟 + 𝑟² + ⋯ = 𝑟 = = =− .
𝑟−1 𝑟−1 𝟏−𝒓
5. Seja S = 2 + 4 + 6 + ⋯ + 2𝑛.
(1+𝑛)𝑛
S = 2(1 + 2 + 3 + ⋯ + 𝑛) = 2 ( ) = 𝒏(𝒏 + 𝟏).
2
6. Dado que 1 + 3 + 5 + ⋯ + (2𝑛 − 1) = 𝑛², seja 𝑛 = 1:
2.1 − 1 = 1².
Seja 𝑛 = 𝑘:
1 + 3 + 5 + ⋯ + (2𝑘 − 1) = 𝑘².
Agora, para 𝑘 + 1:
𝟏 + 𝟑 + 𝟓 + ⋯ + (𝟐𝒌 − 𝟏) + 2(𝑘 + 1) − 1) = (𝑘 + 1)²;
𝑘² + 2𝑘 + 2 − 1 = 𝑘 2 + 2𝑘 + 1;
𝒌² + 𝟐𝒌 + 𝟏 = 𝒌² + 𝟐𝒌 = 𝟏, como queríamos demonstrar.
7. 9𝑛 − 1 é divisível por 8..
91 −1
Seja 𝑛 = 1: = 𝟏.
8
Hipótese: 9𝑘 − 1 é, também, divisível por 8.
Para 𝑘 + 1:
9
9𝑘+1 −1 9𝑘 .9−1 9𝑘 .9−9 9(𝟗𝒌 −𝟏)
= = = ⟹ É divisível por 8.
8 8 8 8
8. 𝑎) Sejam 𝑎 e 𝑏 reais, com 𝑎 > 𝑏 > 0.
Se 𝑎 > 𝑏, então √𝑎 > √𝑏:
2
√𝑎 > √𝑏 ⟹ √𝑎 − √𝑏 > 0 ⟹ (√𝑎 − √𝑏) ≥ 0 ⟹ 𝑎 − 2√𝑎𝑏 + 𝑏 ≥ 0
⟹ 𝒂 + 𝒃 ≥ 𝟐√𝒂𝒃.
1
𝑏) Tomando 𝑏 = na desigualdade de Cauchy, obtemos:
𝑎
1 1 𝟏
𝑎 + ≥ 2√𝑎. ⟹ 𝒂 + ≥ 𝟐.
𝑎 𝑎 𝒂
2
1
9. 1³ + 2³ + ⋯ + 𝑛3 = ( 𝑛(𝑛 + 1)) .
2
Para 𝑛 = 1:
2
1
1³ = ( . 1. (1 + 1)) ⟹ 1 = 1.
2
Suponhamos a validade para um natural 𝑘:
2
1 3
1³ + 2³ + ⋯ + 𝑘 = ( 𝑘(𝑘 + 1)) .
2
Verificando a validade para 𝑘 + 1:
2
𝟑 3 1
𝟏³ + 𝟐³ + ⋯ + 𝒌 + (𝑘 + 1) = ( (𝑘 + 1)(𝑘 + 1 + 1)) ;
2
2 2
1 3 1
(2 𝑘(𝑘 + 1)) + (𝑘 + 1) = (2 (𝑘 + 1)(𝑘 + 2)) ;
1 1
𝑘 2 (𝑘 + 1)2 + (𝑘 + 1)3 = (𝑘 + 1)2 (𝑘 + 2)2 ;
4 4
1 1
(𝑘 + 1)2 ( 𝑘 2 + (𝑘 + 1)) = (𝑘 + 1)²(𝑘 + 2)²;
4 4
𝑘 2 +4𝑘+4 1
(𝑘 + 1)2 ( ) = (𝑘 + 1)²(𝑘 + 2)²;
4 4
𝟏 𝟏
(𝒌 + 𝟏)𝟐 (𝒌 + 𝟐)𝟐 = (𝒌 + 𝟏)𝟐 (𝒌 + 𝟐)𝟐 .
𝟒 𝟒
Q.E.D.

Capítulo 1
Frações parciais
1
1. .
2
2. Demonstração a cargo do leitor.
1 1
3. 𝑎) − .
2(𝑥−3) 2(𝑥+3)
𝑥−𝑎 A B
𝑏) (𝑥−𝑏)(𝑥−𝑐) = + ;
𝑥−𝑏 𝑥−𝑐
𝑥 − 𝑎 = A(𝑥 − 𝑐) + B(𝑥 − 𝑏).
Tomando 𝑥 = 𝑐:
𝑐−𝑎
𝑐 − 𝑎 = B(𝑐 − 𝑏) ⟹ B = .
𝑐−𝑏
Tomando 𝑥 = 𝑏:
𝑏−𝑎
𝑏 − 𝑎 = A(𝑏 − 𝑐) ⟹ A = .
𝑏−𝑐
Assim:
𝑏−𝑎 𝑐−𝑎
𝑥−𝑎 𝑏−𝑐 𝑐−𝑏 𝑏−𝑎 𝑐−𝑎
(𝑥−𝑏)(𝑥−𝑐)
= + = + .
𝑥−𝑏 𝑥−𝑐 (𝑏−𝑐)(𝑥−𝑏) (𝑐−𝑏)(𝑥−𝑐)
12 2
𝑐) − + 1.
5(𝑥−6) 5(𝑥−1)
106 6
𝑑) 𝑥 + − + 5.
5(𝑥−5) 5𝑥
1
𝑒) .
𝑥+1
𝑏−𝑎 𝑐−𝑎 𝑑−𝑎
𝑓) (𝑏−𝑐)(𝑏−𝑑)(𝑥−𝑏) − (𝑏−𝑐)(𝑐−𝑑)(𝑥−𝑐) − (𝑏−𝑑)(𝑑−𝑐)(𝑥−𝑑).
4.
𝑥³ − 4𝑥 = 𝑥(𝑥 2 − 4) = 𝑥(𝑥 + 2)(𝑥 − 2).
8 A B C
= + + ⟹ 8 = A(𝑥 − 2)(𝑥 + 2) + B. 𝑥(𝑥 + 2) + C. 𝑥(𝑥 − 2).
𝑥 3 −4𝑥 𝑥 𝑥−2 𝑥+2
𝑥 = 2 ⟹ 8 = 8B ⟹ B = 1.
𝑥 = 0 ⟹ 8 = − 4A ⟹ A = − 2.
𝑥 = − 2 ⟹ 8 = 8C ⟹ C = 1.
8 2 1 1
=− + + .
𝑥 3 −4𝑥 𝑥 𝑥−2 𝑥+2
1 1 1 1 1 1 1 1
+ + = + + = − 1 + + = 𝟎.
1+A 1+B 1+C 1−2 1+1 1+1 2 2

Capítulo 1 – Exercícios propostos – Somatórios


1.
𝑛−1
𝑎) 𝑥 + 𝑥² + 𝑥³ + ⋯ + 𝑥 𝑛−1 = ∑𝑘=1 𝑥 𝑘 ;
𝑏) arc sen(𝑥) + arc sen(𝑥 + 1) + arc sen(𝑥 + 2) + ⋯ + arc sen(𝑥 + 𝑛)
𝑛

= ∑ arc sen(𝑥 + 𝑘).


𝑘=0
𝑛−1
1 1 1 1 1
𝑐) + + +⋯+ = ∑( ).
1+1 1+2 1+3 1 + (𝑛 − 1) 1+𝑘
𝑘=1
1 1 1
𝑑) 1. log 𝑎 1 + log 𝑎 2 + log 𝑎 3 + ⋯ + log 𝑎 𝑛
2 3 𝑛
𝑛 𝑛
1 𝑘
=∑ log 𝑎 𝑘 ou ∑ log 𝑎 √𝑘.
𝑘=1 𝑘 𝑘=1
𝑛+𝑚

𝑒) 1 + 𝑒 + 𝑒² + ⋯ + 𝑒 𝑛+𝑚 = ∑ 𝑒 𝑘 .
𝑘=0
𝑛
1 1 1 1 1
𝑓) + + +⋯+ =∑
5 10 15 5𝑛 5𝑘
𝑘=1
2.
4 4 4

𝑎) ∑(𝑘 3 − 5𝑘) = ∑ 𝑘³ − 5 ∑ 𝑘 = 13 + 23 + 33 + 43 − 5(1 + 2 + 3 + 4)


𝑘=1 𝑘=1 𝑘=1
= 𝟓𝟎.
𝑏) Temos o somatório de uma constante.
Assim:
5

∑ √6 + √3 = 𝑛. √6 + √3.
𝑘=1
5 = 1 + (𝑛 − 1) ⟹ 𝑛 = 5.
5

∑ √6 + √3 = 𝟓. √𝟔 + √𝟑.
𝑘=1
5

𝑐) ∑ √𝑘 − √𝑘 − 1 = (√3 − √2) + (√4 − √3) + (√5 − √4)


𝑘=3

= (√𝟑 − √𝟐) + (√𝟒 − √𝟑) + √𝟑.


3

𝑑) ∑ 𝑘𝑘 = 11 + 22 + 33 = 1 + 4 + 27 = 𝟑𝟔.
𝑘=1
4 4 4 4
1 1 1
𝑒) ∑ ( (𝑘 2 + 4𝑘)) = ∑ (𝑘 2 + 4𝑘) = (∑ 𝑘 2 + 4 ∑ 𝑘)
𝑘=1
√5 √5 𝑘=1 √5 𝑘=1 𝑘=1
1 2 1 70
= (1 + 22 + 32 + 42 + 4(1 + 2 + 3 + 4)) = (30 + 40) = = 𝟏𝟒√𝟓.
√5 √5 √5
6
1 1 1 1 1
𝑓) ∑ ( ) = ( ) + ( ) + ( ) + ( )
𝑥2 − 3 32 − 3 42 − 3 52 − 3 62 − 3
𝑥=3
1 1 1 1 𝟏𝟑𝟕
= + + + = .
6 13 22 33 𝟒𝟐𝟗
3. Tendo em mente as seguintes propriedades operatórias dos logaritmos
𝐥𝐨𝐠 𝒓 𝒎 𝒑
= 𝐥𝐨𝐠 𝒏 𝒎 e 𝐥𝐨𝐠 𝒓𝒒 𝒎𝒑 = 𝐥𝐨𝐠 𝒓 𝒎, temos:
𝐥𝐨𝐠 𝒓 𝒏 𝒒
4 𝑛 4 4 4
log 1⁄2 √32 𝑛
1 1
∑ = ∑(log 8𝑛+2 √32 ) = ∑ log 8𝑛+2 32 𝑛 = ∑ log 𝑛+2 32
8
log 1⁄2 8𝑛+2 𝑛
𝑛=1 𝑛=1 𝑛=1 𝑛=1
1 4 4 4
1 5 5 1
= ∑ 𝑛 log 8 32 = ∑ . ( ) = (∑ )
𝑛+2 𝑛(𝑛 + 2) 3 3 𝑛(𝑛 + 2)
𝑘=1 𝑘=1 𝑘=1
5 1 1 1 1 5 68 𝟏𝟕
= ( + + + )= ( )= .
3 3 10 15 24 3 120 𝟏𝟖
Gab. D.
4. Ver apêndice sobre as Fórmulas de Prostaférese.
6° 6°
1 1
𝑎) ∑ (sen 𝜃 + cos 𝜃) = ( ∑ sen 𝜃 + cos 𝜃)
𝑚 𝑚
𝜃=3° 𝜃=3°
1
= (sen 3° + cos 3° + sen 4° + cos 4° + sen 5° + cos 5° + sen 6° + cos 6°)
𝑚
1
= ((sen 3° + sen 4°) + (sen 5° + sen 6°) + (cos 3° + cos 4°) + (cos 5 ° + cos 6°))
𝑚
1 3° + 4° 4° − 3° 5° + 6° 6° − 5°
= ((2sen ( ) cos ( )) + (2sen ( ) cos ( ))
𝑚 2 2 2 2
3° + 4° 4° − 3° 5° + 6° 6° − 5°
+ (2 cos ( ) cos ( )) + (2 cos ( ) cos ( )))
2 2 2 2

𝟏 𝟗 𝛑
𝟒√𝟐 𝐜𝐨𝐬 (( ) °) 𝐜𝐨𝐬(𝟏°) 𝐬𝐞𝐧 (( ) ° + )
𝟐 𝟐 𝟒
= .
𝒎
𝑛 𝑛
𝑘+5 1
𝑏) ∑ = [∑(𝑘 + 5)]
𝑎 + 𝑏 + 𝑐 (𝑎 + 𝑏 + 𝑐)
𝑘=1 𝑘=1
𝑛 𝑛
1
= [∑ 𝑘 + ∑ 5]
(𝑎 + 𝑏 + 𝑐)
𝑘=1 𝑘=1
1 𝑛(𝑛 + 1) 1 𝑛² + 𝑛 + 10𝑛
= [ + 5𝑛] = [ ]
𝑎+𝑏+𝑐 2 𝑎+𝑏+𝑐 2
1 𝑛²+11𝑛 𝒏²+𝟏𝟏𝒏
= [ ]= .
𝑎+𝑏+𝑐 2 𝟐(𝒂+𝒃+𝒄)
5.
1 1 1 1
S= + + + ⋯+ .
√2 + √1 √3 + √2 √4 + √3 √81 + √80
80 80
1
S=∑ = ∑(√𝑘 + 1 − √𝑘).
𝑘=1
√ 𝑘 + 1 + √𝑘 𝑘=1
Aplicando a propriedade telescópica:
80
∑ (√𝑘 + 1 − √𝑘) = 𝑓(80 + 1) − 𝑓(1), onde 𝑓(𝑘) = √𝑘:
𝑘=1
80

∑(√𝑘 + 1 − √𝑘) = 𝑓(𝑘 + 1) − 𝑓(𝑘) = √81 − √1 = 𝟖.


𝑘=1
1 1 1
6. S = + + ⋯+ .
√𝟒+√𝟕 √𝟕+√𝟏𝟎 √𝟑𝟗𝟕+√𝟒𝟎𝟎
A sequência 𝒂𝒌𝟏 é dada por 𝑎𝑘1 = 4 + (𝑛 − 1). 3 ⟹ 𝒂𝒌𝟏 = 𝟑𝒌 + 𝟏.
A sequência 𝒂𝒌𝟐 é dada por 𝑎𝑘2 = 7 + (𝑛 − 1). 3 ⟹ 𝒂𝒌𝟐 = 𝟑𝒌 + 𝟒.
Para obtermos o limite superior (número de termos) do somatório, façamos:
3𝑘 + 1 = 397 ⟹ 𝒌 = 𝟏𝟑𝟐.
3𝑘 + 4 = 400 ⟹ 𝒌 = 𝟏𝟑𝟐.
O número de termos é 132.
Assim:
132
1 1 1 1
+ + ⋯+ =∑ .
√𝟒 + √𝟕 √𝟕 + √𝟏𝟎 √𝟑𝟗𝟕 + √𝟒𝟎𝟎 𝑘=1
√𝟑𝒌 + 𝟏 + √𝟑𝒌 + 𝟒
132 132
1 1
∑ = ∑ (√3𝑘 + 4 − √3𝑘 + 1).
√3𝑘 + 1 + √3𝑘 + 4 3
𝑘=1 𝑘=1
Observe que:
𝒇(𝒌) = √𝟑𝒌 + 𝟏 ⟹ 𝑓(𝑘 + 1) = √3(𝑘 + 1) + 1 ⟹ 𝒇(𝒌 + 𝟏) = √𝟑𝒌 + 𝟒.
Assim, obtivemos o padrão para o uso da propriedade telescópica:
132
1 1 132
∑ (√3𝑘 + 4 − √3𝑘 + 1) = (∑𝑘=1 𝑓 (𝑘 + 1) − 𝑓(𝑘)),
𝑘=1 3 3
onde 𝑓(𝑘) = √3𝑘 + 1.
132 132
1 1 1
∑ (√3𝑘 + 4 − √3𝑘 + 1) = (∑ 𝑓(𝑘 + 1) − 𝑓(𝑘)) = (𝑓(132 + 1) − 𝑓(1))
3 3 3
𝑘=1 𝑘=1
1 1
= (𝑓(133) − 𝑓(1)) = (√3.133 + 1 − √3.1 + 1) = 𝟔.
3 3
7. É dada a função 𝑓(𝑥) = 4𝑥² − 1 ⟹ 𝑓(𝑥) = (2𝑥 + 1)(2𝑥 − 1).
1 1 1
Assim, seja S = + +⋯+ .
𝑓(1) 𝑓(2) 𝑓(100)
100 100
1 1 1 1 1
S= + + ⋯+ =∑ =∑ .
𝑓(1) 𝑓(2) 𝑓(100) 𝑓(𝑥) (2𝑥 + 1)(2𝑥 − 1)
𝑥=0 𝑥=1
1
O argumento (2𝑥+1)(2𝑥−1), em frações parciais, é equivalente a
1 1 1 1
− = − .
2(2𝑥−1) 2(2𝑥+1) 4𝑥−2 4𝑥+2
100 100 100
1 1 1 1 1
∑ = ∑( − ) = ∑( − ).
(2𝑥 + 1)(2𝑥 − 1) 2(2𝑥 − 1) 2(2𝑥 + 1) 4𝑥 − 2 4𝑥 + 2
𝑥=1 𝑥=1 𝑥=1
1 1 1
Se tomarmos 𝑔(𝑥) = , é imediato que 𝑔(𝑥 + 1) = = .
4𝑥−2 4(𝑥+1)−2 4𝑥+2
1
Assim, o argumento está na forma telescópica 𝑔(𝑥) − 𝑔(𝑥 + 1), onde 𝑔(𝑥) = .
4𝑥−2
Enfim:
100 100
1 1
∑( − ) = ∑(𝑔(𝑘) − 𝑔(𝑘 + 1)) = 𝑔(1) − 𝑔(100 + 1)
4𝑥 − 2 4𝑥 + 2
𝑥=1 𝑥=1
1 1 1 1 𝟏𝟎𝟎
= − = − = .
4.1 − 2 4.101 − 2 2 402 𝟐𝟎𝟏
3
8. ∑𝑘=1(𝑎 − 𝑘)𝑥 𝑘 = (𝑎 − 1)𝑥 + (𝑎 − 2)𝑥 2 + (𝑎 − 3)𝑥 3 .
𝛼² + 𝛽² + 𝛾² = (𝛼 + 𝛽 + 𝛾)2 − 2(𝛼𝛽 + 𝛼𝛾 + 𝛽𝛾).
Pelas relações de Girard dos polinômios de terceiro grau:
𝑎−2 𝑎−1
𝛼+𝛽+𝛾 =− e 𝛼𝛽 + 𝛼𝛾 + 𝛽𝛾 = .
𝑎−3 𝑎−3
Assim:
2
𝑎−2 2 𝑎−1
𝛼² + 𝛽² + 𝛾² = (𝛼 + 𝛽 + 𝛾) − 2(𝛼𝛽 + 𝛼𝛾 + 𝛽𝛾) = (− ) − 2( )
𝑎−3 𝑎−3
(𝑎 − 2)2 (𝑎 − 1) 𝒂𝟐 − 𝟒𝒂 + 𝟐
= −2 =− 𝟐 .
(𝑎 − 3)(𝑎 − 3) 𝑎−3 𝒂 − 𝟔𝒂 + 𝟗
9.
60 60
3 1 1 60
𝑎) ∑ =∑ (3𝑘−1 − 3𝑘+2) = ∑𝑘=1(𝑓(𝑘) − 𝑓(𝑘 + 1)),
𝑘=1 (3𝑘−1)(3𝑘+2) 𝑘=1
1
onde 𝑓(𝑘) = .
3𝑘−1
Assim:
60
1 1 1 1 𝟒𝟓
∑( − ) = 𝑓(1) − 𝑓(60 + 1) = − = .
3𝑘 − 1 3𝑘 + 2 3.1 − 1 3.61 − 1 𝟗𝟏
𝑘=1
40 40 40
5 5 5 5 5 5 5
𝑏) ∑ = ∑( − ) = ∑( − + − )
𝑘(𝑘 − 2) 2(𝑘 − 2) 2𝑘 2(𝑘 − 2) 2𝑘 2(𝑘 − 1) 2(𝑘 − 1)
𝑘=3 𝑘=3 𝑘=3
40 40
5 5 5 5
= ∑( − )+∑( − ).
2(𝑘 − 1) 2𝑘 2(𝑘 − 2) 2(𝑘 − 1)
𝑘=3 𝑘=3
Perceba o padrão telescópico nas duas somas acima.
Assim:
40 40
5 5 5 5
∑( − )+∑( − )
2(𝑘 − 1) 2𝑘 2(𝑘 − 2) 2(𝑘 − 1)
𝑘=3 𝑘=3
40 40 5
= ∑𝑘=3(𝑓(𝑘 − 1) − 𝑓(𝑘)) + ∑𝑘=3(𝑔(𝑘 − 1) − 𝑔(𝑘)), onde 𝑓(𝑘) =
2𝑘
5 5
e 𝑔(𝑘 ) = = .
2(𝑘−1) 2𝑘−2
40 40
5 5 5 5
∑( − )+∑( − )
2(𝑘 − 1) 2𝑘 2(𝑘 − 2) 2(𝑘 − 1)
𝑘=3 𝑘=3
= (𝑓(3 − 1) − 𝑓(40)) + (𝑔(3 − 1) − 𝑔(40)) = (𝑓(2) − 𝑓(40)) + (𝑔(2) − 𝑔(40))
5 5 5 5 𝟐𝟐𝟔𝟏
=( − )+( − )= .
4 80 2 78 𝟔𝟐𝟒
25 25
1 1 1 25 1
𝑐) ∑ =∑ (16𝑘 − 16(𝑘+1)) = ∑𝑘=1(𝑓(𝑘) − 𝑓(𝑘 + 1)), onde 𝑓(𝑘) = 16𝑘.
𝑘=1 4𝑘(4𝑘+4) 𝑘=1
25
1 1 1 1 𝟐𝟓
∑( − ) = 𝑓(1) − 𝑓(25 + 1) = − = .
16𝑘 16(𝑘 + 1) 16 16.26 𝟒𝟏𝟔
𝑘=1
500 500

𝑑) ∑ (ln(𝑘 + 2) − ln(𝑘)) = ∑ (ln(𝑘 + 2) − ln(𝑘) + ln(𝑘 + 1) − ln(𝑘 + 1))


𝑘=2 𝑘=2
500 500

= ∑(ln( 𝑘 + 2) − ln(𝑘 + 1)) + ∑ (ln(𝑘 + 1) − ln(𝑘))


𝑘=2 𝑘=2
500
500
=∑ (𝑓(𝑘 + 1) − 𝑓(𝑘)) + ∑𝑘=2(𝑔(𝑘 + 1) − 𝑔(𝑘)),
𝑘=2
onde 𝑓(𝑘) = ln(𝑘 + 1) e 𝑔(𝑘) = ln(𝑘).
Enfim:
500 500

∑(ln( 𝑘 + 2) − ln(𝑘 + 1)) + ∑ (ln(𝑘 + 1) − ln(𝑘))


𝑘=2 𝑘=2
500 500

= ∑(𝑓(𝑘 + 1) − 𝑓(𝑘)) + ∑(𝑔(𝑘 + 1) − 𝑔(𝑘))


𝑘=2 𝑘=2
= (𝑓(500 + 1) − 𝑓(2)) + (𝑔(500 + 1) − 𝑔(2))
= (ln(501 + 1) − (ln(2 + 1)) + (ln(501) − ln(2))
𝟓𝟎𝟐 𝟓𝟎𝟏 𝟓𝟎𝟐.𝟓𝟎𝟏
= 𝐥𝐧 ( ) + 𝐥𝐧 ( ) ou 𝐥𝐧 ( ).
𝟑 𝟐 𝟔
49 49

𝑒) ∑(√𝑘 − √𝑘 − 2) = ∑(√𝑘 − √𝑘 − 2 + √𝑘 − 1 − √𝑘 − 1)
𝑘=2 𝑘=2
49 49

= ∑(√𝑘 − √𝑘 − 1) + ∑(√𝑘 − 1 − √𝑘 − 2)
𝑘=2 𝑘=2
49
49
=∑ (𝑓(𝑘) − 𝑓(𝑘 − 1)) + ∑𝑘=2(𝑔(𝑘) − 𝑔(𝑘 − 1)), 𝑓(𝑘) = √𝑘 e 𝑔(𝑘) = √𝑘 − 1.
𝑘=2
49 49

∑(√𝑘 − √𝑘 − 1) + ∑(√𝑘 − 1 − √𝑘 − 2)
𝑘=2 𝑘=2
49 49

= ∑(𝑓(𝑘) − 𝑓(𝑘 − 1)) + ∑(𝑔(𝑘) − 𝑔(𝑘 − 1))


𝑘=2 𝑘=2
= (𝑓(49) − 𝑓(2 − 1)) + (𝑔(49) − 𝑔(2 − 1)) = (√49 − √1) + (√48 − 0)
= 𝟔 + 𝟒√𝟑.
𝟖𝟓
𝑓) − .
𝟑𝟔
𝑔) 𝐜𝐨𝐬(𝟗𝟏°) − 𝟏.
ℎ) 𝟏 − 𝒆𝟒𝟑𝟕 .
𝑖) √𝒂𝒏−𝟏 − 𝟏.
10. ∑3𝑘=0(𝑘 + 𝑖(𝑘 + 1)) = 𝑖 + (1 + 2𝑖) + (2 + 3𝑖) + (3 + 4𝑖)
= (1 + 2 + 3) + 𝑖(1 + 2 + 3 + 4) = 6 + 10𝑖.
𝑧𝑟 = 𝑚 + (𝑚 + 4)𝑖 ⟹ 𝑚 + (𝑚 + 4)𝑖 = 6 + 10𝑖 ⟹ 𝑚 = 6.
(𝑚, 𝑚 + 2, 𝑚 + 4) ⟹ (6, 8, 10) ⟹ P.A de razão 2.
𝑝 𝑝 𝑝 𝑝
11. ∑𝑘=1(4𝑘 + 5) = 225 ⟹ ∑𝑘=1 4𝑘 + ∑𝑘=1 5 = 225 ⟹ 4 ∑𝑘=1 𝑘 + 5𝑝 = 225.
(1 + 𝑝)𝑝
4(1 + 2 + ⋯ + 𝑝) + 5𝑝 = 225 ⟹ 4 ( ) + 5𝑝 = 225 ⟹ 2𝑝(1 + 𝑝) + 5𝑝 = 225.
2
2𝑝 + 7𝑝² − 225 = 0 ⟹ 𝒑 = 𝟗.
1 1
12. 1 + (𝑥 − 2) + (𝑥 − 2)2 + ⋯ + (𝑥 − 2)𝑛 + ⋯ = 7 ⟹ =7⟹ = 7.
1−(𝑥−2) 3−𝑥
20 𝟐𝟎
𝑥= ⟹ S = { }.
7 𝟕
1
13. − .
3√5
14.
∞ ∞ ∞
1 1 1 1 𝑛
∑ = ∑ = ∑ 2− 2
𝑛=0
(√2 − 1)√2𝑛 (√2 − 1) 𝑛=0 √2𝑛 (√2 − 1) 𝑛=0
1
1 1 −1 √𝟐
= ( ) = √2 = .
(√2 − 1) 1 − 1 1−
1 𝟑 − 𝟐√𝟐
√2 √2
15.
𝑎) 𝟏𝟓. 𝟓𝟓𝟒.
𝑏) 𝟑𝟓.
𝟏𝟏
𝑑) .
𝟔𝟕
1
16. 𝑎) = √𝑘 + 1 − √𝑘
√𝑘+√𝑘+1
(multiplicando numerador e denominador pelo conjugado do denominador).
𝑛

∑(√𝑘 + 1 − √𝑘) = (√𝟐 − 1) + (√𝟑 − √𝟐) + ⋯ + (√𝑛 − 1 − √𝒏) = √𝒏 + 𝟏 − 𝟏.


𝑘=1
143
1 1 1 1
𝑏) + + ⋯+ =∑
1 + √2 √2 + √3 √143 + √144 𝑘=1
√𝑘 + √𝑘 + 1
143

= ∑(√𝑘 + 1 − √𝑘) = √143 + 1 − 1 = 𝟏𝟏.


𝑘=1
1 1 1
𝑐) + +⋯+ = √225 − 1 = 𝟏𝟒.
1 + √2 √2 + √3 √224 + √225
17. 𝑎) 𝟐.
𝑏) 4.
2 3
1 1 1 1 √𝟏𝟓 𝟏𝟓
𝑐)1 + +( ) +( ) +⋯= = + .
√3√5 √3√5 √3√5 1 𝟏𝟒 𝟏𝟒
1−
√3√5
1
2 3
1 1 1 1 − 5 √7 √𝟕
𝑑) +( ) +( ) +⋯= =− .
1 − 5 √7 1 − 5 √7 1 − 5 √7 1 𝟑𝟓
1−
1 − 5 √7
𝟕 𝟒
𝑒) + .
𝟑 √𝟕
18.
𝑎) Definamos as sequências 𝑎𝑛 = (2, 9, 16, ..., 1395) e 𝑏𝑛 = (9, 16, 23, ..., 1402), de tal
modo que:
𝑎𝑛 = 𝑎1 + (𝑛𝑎 − 1)𝑟 ⟹ 𝑎𝑛 = 2 + (𝑛𝑎 − 1)7 ⟹ 𝑎𝑛 = 7𝑛𝑎 − 5.
𝑎𝑛 +5 1395+5
O número de termos 𝑛𝑎 desta sequência é 𝑛𝑎 = ⟹ 𝑛𝑎 = ⟹ 𝑛𝑎 = 200.
7 7
𝑏𝑛 = 𝑏1 + (𝑛𝑏 − 1)𝑟 ⟹ 𝑏𝑛 = 9 + (𝑛𝑏 − 1)7 ⟹ 𝑏𝑛 = 7𝑛𝑏 + 2.
𝑏𝑛 −2 1402−2
𝑛𝑏 = ⟹ 𝑛𝑏 = ⟹ 𝑛𝑏 = 200.
7 7
𝑛𝑎 = 𝑛𝑏 = 𝑛.
Assim:
200
1 1 1 1 1
+ + +⋯+ =∑ .
2.9 9.16 16.23 1395.1402 (7𝑛 − 5)(7𝑛 + 2)
𝑛=1
200 200
1 1 1 𝟓𝟎
∑ = ∑( − )= .
(7𝑛 − 5)(7𝑛 + 2) 7(7𝑛 − 5) 7(7𝑛 + 2) 𝟕𝟎𝟏
𝑛=1 𝑘=1
435
1 1 1 1 1 𝟏𝟒𝟓
𝑏) + + +⋯+ =∑ = .
3.7 7.11 11.15 1739.1743 (4𝑛 − 1)(4𝑛 + 3) 𝟏𝟕𝟒𝟑
𝑛=1
𝟏𝟎𝟎𝟎
𝑐) .
𝟔𝟎𝟎𝟗
19. 𝑏) 𝑛 − 7 = 1 + (𝑛𝑠 − 1) ⟹ 𝒏𝒔 = 𝒏 − 𝟕.
𝑐) 3.19 − 2 = 1 + (𝑛 − 1)3 ⟹ 𝒏 = 𝟏𝟗.
𝑑) 6.79 − 3 = 21 + (𝑛 − 1)6 ⟹ 𝒏 = 𝟕𝟔.
𝑛−1
𝑓) 𝑎𝑛 = 𝑎1 𝑞 ⟹ 5𝑛−9 = 53 . 5𝑛𝑠−1 ⟹ 5𝑛−9 = 5𝑛𝑠+2 ⟹ 𝒏𝒔 = 𝒏 − 𝟏𝟏.

Capítulo 2 – V – 1. Exercícios propostos – Logaritmos


1. log 𝑎 𝑏 = log 𝑎𝑐 [𝑏(1 + log 𝑎 𝑐)].
Efetuando, no segundo membro (lado direito da igualdade), uma mudança de base para a
base 𝑎, obtemos:
log 𝑎 [𝑏(1 + log 𝑎 𝑐)]
log 𝑎 𝑏 = ⟹ log 𝑎 𝑏. log 𝑎 𝑎𝑐 = log 𝑎 [𝑏(1 + log 𝑎 𝑐)]
log 𝑎 𝑎𝑐
⟹ log 𝑎 𝑏(log 𝑎 𝑎 + log 𝑎 𝑐) = log 𝑎 [𝑏(1 + log 𝑎 𝑐)]
⟹ log 𝑎 𝑏(1 + log 𝑎 𝑐) = log 𝑎 [𝑏(1 + log 𝑎 𝑐)]
⟹ 𝐥𝐨𝐠 𝒂 [𝒃(𝟏 + 𝐥𝐨𝐠 𝒂 𝒄)] = 𝐥𝐨𝐠 𝒂 [𝒃(𝟏 + 𝐥𝐨𝐠 𝒂 𝒄)].
2. Se 𝑎 = 𝑏𝑐, então:
log 𝑐 𝑎 = log 𝑐 (𝑏𝑐) ⟹ 𝑙𝑜𝑔𝑐 𝑎 = log 𝑐 𝑏 + log 𝑐 𝑐 ⟹ log 𝑐 𝑎 = 1 + log 𝑐 𝑏
log 𝑎 𝑎 log 𝑏 𝑏 𝟏 𝟏
⟹ =1+ ⟹ =𝟏+ .
log 𝑎 𝑐 log 𝑏 𝑐 𝐥𝐨𝐠 𝒂 𝒄 𝐥𝐨𝐠 𝒃 𝒄
3. 1 + 𝑥 = 1 + log 𝑐 (𝑎𝑏) = log 𝑐 𝑐 + log 𝑐 (𝑎𝑏) = log 𝑐 (𝑎𝑏𝑐).
1 1 log 𝑐 𝑐
= = = log 𝑎𝑏𝑐 𝑐 .
1 + 𝑥 log 𝑐 (𝑎𝑏𝑐) log 𝑐 (𝑎𝑏𝑐)
1 + 𝑦 = 1 + log 𝑏 (𝑎𝑐) = log 𝑏 𝑏 + log 𝑏 (𝑎𝑐) = log 𝑏 (𝑎𝑏𝑐).
1 1 log 𝑏 𝑏
= = = log 𝑎𝑏𝑐 𝑏 .
1 + 𝑦 log 𝑏 (𝑎𝑏𝑐) log 𝑏 (𝑎𝑏𝑐)
1 + 𝑧 = 1 + log 𝑎 (𝑏𝑐) = log 𝑎 𝑎 + log 𝑎 (𝑏𝑐) = log 𝑎 (𝑎𝑏𝑐).
1 1 log 𝑎 𝑎
= = = log 𝑎𝑏𝑐 𝑎 .
1 + 𝑧 log 𝑎 (𝑎𝑏𝑐) log 𝑎 (𝑎𝑏𝑐)
1 1 1
+ + = log 𝑎𝑏𝑐 𝑐 + log 𝑎𝑏𝑐 𝑏 + log 𝑎𝑏𝑐 𝑎 = log 𝑎𝑏𝑐 (𝑎𝑏𝑐) = 𝟏.
1+𝑥 1+𝑦 1+𝑧
4. 𝑎log 𝑏 = 𝑏 log 𝑎 ⟹ log(𝑎log 𝑏 ) = log(𝑏 log 𝑎 ) ⟹ log 𝑏. log 𝑎 = log 𝑎. log 𝑏 ⟹ 𝒂 = 𝒃.
5. 𝑥² − 𝑝𝑥 + 𝑞 = 0;
log 𝑞 𝑎𝑎 + log 𝑞 𝑏 𝑏 + log 𝑞 𝑎𝑏 + log 𝑞 𝑏 𝑎 = 𝑎. log 𝑞 𝑎 + 𝑏. log 𝑞 𝑏 + 𝑏. log 𝑞 𝑎 + 𝑎. log 𝑞 𝑏
= 𝑎. log 𝑞 𝑎 + 𝑏. log 𝑞 𝑎 + 𝑏. log 𝑞 𝑏 + 𝑎. log 𝑞 𝑏 = (𝑎 + 𝑏). log 𝑞 𝑎 + (𝑎 + 𝑏). log 𝑞 𝑏
= (𝑎 + 𝑏)(log 𝑞 𝑎 + log 𝑞 𝑏) = (𝑎 + 𝑏)(log 𝑞 (𝑎𝑏)).
Das Relações de Girard (soma e produto), temos que 𝑎 + 𝑏 = 𝑝 e 𝑎𝑏 = 𝑞.
Assim:
(𝑎 + 𝑏)(log 𝑞 (𝑎𝑏)) = 𝑝. log 𝑞 𝑞 = 𝒑.
Assim:
log 𝑞 𝑎 + log 𝑞 𝑏 + log 𝑞 𝑎𝑏 + log 𝑞 𝑏 𝑎 = 𝒑.
𝑎 𝑏

6. Demonstração análoga ao exercício 4.


7.
𝑛
1 1 1 1
+ +⋯+ = ∑ .
log 𝑥 2 . log 𝑥 4 log 𝑥 4 . log 𝑥 8 log 𝑥 2𝑛−1 . log 𝑥 2𝑛 log 𝑥 2𝑘−1 . log 𝑥 2𝑘
𝑘=2
1 1 1
= = .
log 𝑥 2𝑘−1 . log 𝑥 2𝑘 (𝑘 − 1). log 𝑥 2. 𝑘. log 𝑥 2 𝑘(𝑘 − 1). (log 𝑥 2)2
𝑛 𝑛 𝑛
1 1 1 1
∑ = ∑ = ∑ . .
log 𝑥 2𝑘−1 . log 𝑥 2𝑘 𝑘(𝑘 − 1). (log 𝑥 2)2 𝑘(𝑘 − 1) (log 𝑥 2)²
𝑘=2 𝑘=2 𝑘=2
1
O fator permanece constante, pois a soma está na variável 𝑘.
(log𝑥 2)²
Assim:
𝑛 𝑛
1 1 1 1
∑ . =( )∑ .
𝑘(𝑘 − 1) (log 𝑥 2)² (log 𝑥 2)² 𝑘(𝑘 − 1)
𝑘=2 𝑘=2
𝑛
1
A soma ∑ é uma conhecida soma telescópica, e, portanto, terá sua
𝑘=2 𝑘(𝑘−1)
resolução omitida.
𝑛
1 1
∑ =1− .
𝑘(𝑘 − 1) 𝑛
𝑘=2
Enfim:
𝑛
1 1 𝟏 𝟏
( )∑ =( ) . (𝟏 − ) .
(log 𝑥 2)² 𝑘(𝑘 − 1) (𝐥𝐨𝐠 𝒙 𝟐)² 𝒏
𝑘=2
Capítulo 2 – V – 2. Exercícios propostos – Logaritmos Complexos
2.
ln(5 + 𝑖)
𝑎) log 5 (5 + 𝑖) = .
ln 5
1
1 1 𝑖.arc tan( )
* 5 + 𝑖 = √26 (cos (arc tan ( )) + 𝑖. sen (arc tan ( ))) = √26. 𝑒 5 .
5 5
1
ln (√26. 𝑒
𝑖.arc tan( )
5 ) 𝟏
ln(5 + 𝑖) 𝐥𝐧(√𝟐𝟔) + 𝒊. 𝐚𝐫𝐜 𝐭𝐚𝐧 ( )
log 5 (5 + 𝑖) = = = 𝟓 .
ln 5 ln 5 𝐥𝐧 𝟓
𝑏) ln(1 − 𝑖. √2) = ln(√3. 𝑒 𝑖.arc tan(− √2) ) = ln(√3) + 𝑖. arc tan(− √2)
= 𝐥𝐧(√𝟑) − 𝒊. 𝐚𝐫𝐜 𝐭𝐚𝐧(√𝟐).
3.
𝜋 𝜋 𝑖 𝜋 𝝅
I. 𝑒 𝑖.2 = 𝑖 ⟹ (𝑒 𝑖.2 ) = 𝑖 𝑖 ⟹ 𝑒 𝑖.𝑖.2 = 𝑖 𝑖 ⟹ 𝒊𝒊 = 𝒆−𝟐 ∈ ℝ.
𝜋 𝜋 −𝑖 𝜋 𝝅
𝑖. 𝑖. −𝑖 −𝑖.𝑖. −𝑖 −𝑖
II. 𝑒 2 = 𝑖 ⟹ (𝑒 ) 2 =𝑖 ⟹𝑒 2 =𝑖 ⟹𝑖 = 𝒆 ∈ ℝ.
𝟐

4.
𝑛
1: 𝑧 = 𝑟. 𝑒 𝑖.𝜃 ⟹ 𝑧 𝑛 = (𝑟. 𝑒 𝑖.𝜃 ) ⟹ 𝑧 𝑛 = 𝑟 𝑛 . 𝑒 𝑖.𝑛.𝜃 .
𝒛𝒏 = 𝒓𝒏 (𝐜𝐨𝐬(𝒏. 𝜽) + 𝒊. 𝐬𝐞𝐧(𝒏. 𝜽)).
1 1 1 1 𝜃
2: 𝑧 = 𝑟. 𝑒 𝑖.𝜃 ⟹ 𝑧 𝑛 = (𝑟. 𝑒 𝑖.𝜃 )𝑛 ⟹ 𝑧 𝑛 = 𝑟 𝑛 . 𝑒 𝑖.𝑛 .
𝟏 𝟏 𝜽 𝜽
𝒛𝒏 = 𝒓𝒏 (𝐜𝐨𝐬 ( ) + 𝒊. 𝐬𝐞𝐧 ( ))
𝒏 𝒏
𝒏 𝒏 𝜽 + 𝟐𝒌𝝅 𝜽 + 𝟐𝒌𝝅
⟹ √𝒛 = √𝒓 (𝐜𝐨𝐬 ( ) + 𝒊. 𝐬𝐞𝐧 ( )) .
𝒏 𝒏
5.
𝑎) Somando ambas as equações:
𝟏
𝑒 𝑖.𝜃 + 𝑒 −𝑖.𝜃 = 2 cos(𝜃) ⟹ 𝐜𝐨𝐬(𝜽) = (𝒆𝒊.𝜽 + 𝒆−𝒊.𝜽 ).
𝟐
𝑏) Subtraindo ambas as equações:
𝑖.𝜃 −𝑖.𝜃
𝑒 −𝑒 = cos(𝜃) + 𝑖. sen(𝜃) − (cos(𝜃) − 𝑖. sen(𝜃))
𝟏
⟹ 𝑒 𝑖.𝜃 − 𝑒 −𝑖.𝜃 = 2. 𝑖. sen(𝜃) ⟹ 𝐬𝐞𝐧(𝜽) = (𝒆𝒊.𝜽 − 𝒆−𝒊.𝜽 ).
𝟐𝒊

Capítulo 2 – Exercícios propostos – Produtórios


1.
359
𝑎) 𝑥. 𝑥². 𝑥³. … . 𝑥 359 = ∏𝑘=1 𝑥 𝑘 .
2016
1
𝑏) ∏ (1 − 𝑘).
𝑘=2
𝑛−1
𝑑
𝑐) ∏ ( 𝑓(𝑥 + 𝑘)).
𝑘=0 𝑑𝑥
2016

𝑑) ∏ (∫(cos 𝑥 + 𝑘) 𝑑𝑥) .
𝑘=0
2.
5

𝑎) ∏ (√𝑘 + √𝑘 − 1)
𝑘=1

= √1. √2 + √2 − 1. √3 + √3 − 1. √4 + √4 − 1. √5 + √5 − 1

= √3. √3 + √2. √4 + √3. √7 = √7.3. √(3 + √2)(4 + √3)

= √𝟐𝟏. √𝟏𝟐 + 𝟒√𝟐 + 𝟑√𝟑 + √𝟔.


475

𝑏) ∏ 𝑖 𝑘 = 𝑖 1 . 𝑖². 𝑖³. … . 𝑖 475 = 𝑖 1+2+3+⋯+475 = 𝑖 113.050 = 𝑖 113.054 mod 4 = 𝑖 2 = − 𝟏.


𝑘=0
Lembre-se, caro leitor, das potências cíclicas da unidade imaginária:
𝑖 0 = 1;
𝑖 1 = 𝑖;
𝑖² = − 1;
𝑖 3 = − 𝑖;
𝑖 4 = 1;
𝑖 5 = 𝑖;
...
𝑖 𝑛 = 𝑖 𝒏 𝐦𝐨𝐝 𝟒 .
𝒂 𝐦𝐨𝐝 𝒃 = resto da divisão de 𝒂 por 𝒃.
1 1 1 1 𝟏
𝑐) 𝑛 −𝑘
= −1 −2 −𝑛
= 1−2−3−⋯−𝑛 = 1 = 𝟓𝟐𝒏(𝒏+𝟏) .
∏𝑘=0 5 5 .5 .….5 5
5−2𝑛(𝑛+1)
3.
2016 2016 2016
1 2! 1 2 1 1 1 1 1
𝑎) ∏ ( ) = ∏ ( ) = ∏ 2 = ( ) . ( 2 ) ( 2 ) . … . ( )
𝑘 𝑘 𝑘 1 2 3 20162
𝑘=1 𝑘=1 𝑘=1
1 1
= = = (𝟐𝟎𝟏𝟔!)−𝟐 .
(1.2.3. … .2016)² (2016!)2
1 1 1
𝑏) (1 + ) . (1 + ) . … . (1 + )
𝑛 𝑛+1 𝑛 + 2016
𝒏+𝟏 𝒏+𝟐 𝒏+𝟑 𝒏 + 𝟐𝟎𝟏𝟔 𝑛 + 2017
=( ).( ).( ).….( ).( )
𝑛 𝒏+𝟏 𝒏+𝟐 𝒏 + 𝟐𝟎𝟏𝟓 𝒏 + 𝟐𝟎𝟏𝟔
Os termos em negrito irão se cancelar, restando apenas:
1 𝒏 + 𝟐𝟎𝟏𝟕
( ) . (𝑛 + 2017) = .
𝑛 𝒏
4.
𝑚+𝑛 𝑥 𝑚+𝑛
1 1
∏( ) 𝛼 𝑘−5 = ( ) ∏ 𝛼 𝑘−5 .
𝑘=𝑚
√4 + √ 3 √4 + √3 𝑘=𝑚
O expoente 𝑥 é obtido através da fórmula do termo geral de uma progressão
aritmética, da seguinte maneira:
𝑚 + 𝑛 = 𝑚 + (𝑥 − 1) ⟹ 𝑥 = 𝑛 + 1.
Assim:
𝑥 𝑚+𝑛 𝑛+1 𝑚+𝑛
1 1
( ) ∏ 𝛼 𝑘−5 = ( ) ∏ 𝛼 𝑘−5 .
√4 + √3 𝑘=𝑚 √4 + √ 3 𝑘=𝑚
Temos o produtório de uma constante com expoente variável:
𝑚+𝑛

∏ 𝛼 𝑘−5 = 𝛼 𝑚−5 . 𝛼 𝑚−4 . 𝛼 𝑚−3 . … . 𝛼 𝑚+𝑛−5 = 𝛼 (𝑚−5)+(𝑚−4)+(𝑚−3)+⋯+(𝑚+𝑛−5)


𝑘=𝑚
1 1
(𝑛+1)(2𝑚+𝑛−10)
= 𝛼 2((𝑚−5)+(𝑚+𝑛−5))(𝑛+1) = 𝛼2 .
Assim:
𝑚+𝑛 𝑛+1
1 𝑘−5
1 1
(𝑛+1)(2𝑚+𝑛−10)
∏( )𝛼 =( ) 𝛼2
𝑘=𝑚
√4 + √ 3 √4 + √ 3
1 1
(𝑛+1)(2𝑚+𝑛−10) 1 1
(𝑛+1)(2𝑚+𝑛−10)
= 𝑛+1 𝛼 2 = 1 𝛼 2
(𝑛+1)
(√4 + √3) (4 + √3)2
1 𝒏 𝟏
−( (𝑛+1)) 1(𝑛+1)(2𝑚+𝑛−10) − − 𝟏
(𝒏+𝟏)(𝟐𝒎+𝒏−𝟏𝟎)
= (4 + √3) 2 𝛼2 = (𝟒 + √𝟑) 𝟐 𝟐
𝜶𝟐 .
5.
𝜋 1 𝜋
Se 𝑓(𝑘) = tan (𝑘 + ), veja que cot ( (4𝑘 + 𝜋 + 4)) = cot (𝑘 + + 1)
4 4 4
1
= 𝜋 .
tan (𝑘 + 4 + 1)
Assim, o produto em questão já se encontra em sua forma telescópica e é dado por:
2016 2016
𝜋 1 𝜋 1
∏ tan (𝑘 + ) cot ( (4𝑘 + 𝜋 + 4)) = ∏ tan (𝑘 + ) .
4 4 4 tan (𝑘 + 𝜋 + 1)
𝑘=1 𝑘=1 4
2016 𝜋 𝜋 𝟒+𝝅
tan (𝑘 + ) 𝑓(1) tan (1 + ) 𝐭𝐚𝐧 ( )
=∏ 4 4 𝟒
𝜋 = 𝑓(2016 + 1) = 𝜋 = 𝝅 .
𝑘=1 tan (𝑘 + 1 + 4 ) tan (2017 + 4 ) 𝐭𝐚𝐧 (𝟐𝟎𝟏𝟕 + 𝟒)
6. 𝑧² + 2𝑖𝑧 + 2 − 4𝑖 = 0.
2𝑖 √(2𝑖 2 ) − 4(2 − 4𝑖) √− 4 − 8 + 16𝑖
𝑧=− ± ⟹𝑧 =−𝑖±
2 2 2
√− 12+16𝑖 1 − 12+16𝑖
⟹𝑧=−𝑖± ⟹ 𝑧 = − 𝑖 ± √− 12 + 16𝑖 ⟹ 𝑧 = − 𝑖 ± √
2 2 4
⟹ 𝑧 = − 𝑖 ± √− 3 + 4𝑖.
Veja que − 3 + 4𝑖 = 1 − 4 + 4𝑖 = 1 + 2.2. 𝑖 − 4 = (1 + 2𝑖)².
Assim:
𝑧 = − 𝑖 ± √− 3 + 4𝑖 ⟹ 𝑧 = − 𝑖 ± √(1 + 2𝑖)2 ⟹ 𝑧 = − 𝑖 ± (1 + 2𝑖)
⟹ 𝑧 = − 𝑖 + (1 + 2𝑖) ⟹ 𝑧 = 1 + 𝑖 ∨ 𝑧 = − 𝑖 − (1 + 2𝑖) ⟹ 𝑧 = − 1 − 3𝑖.
Assim, S = {1 + 𝑖, − 1 − 3𝑖}.
7.
155
√𝑘
O produto ∏ já se encontra em sua forma telescópica:
𝑘=3 √𝑘−1
155 155
√𝑘 𝑓(𝑘)
∏ =∏ , onde 𝑓(𝑘) = √𝑘.
𝑘=3 √𝑘−1 𝑘=3 𝑓(𝑘−1)
Assim:
155 155
√𝑘 𝑓(𝑘) 𝑓(155) 𝑓(155) √155 155
∏ =∏ = = = =√ .
√𝑘 − 1 𝑓(𝑘 − 1) 𝑓(3 − 1) 𝑓(2) √2 2
𝑘=3 𝑘=3
155
√𝑘 155
log 3 (∏ ) = log 3 √ = log 3 (√155) − log 3 (√2) .
√𝑘 − 1 2
𝑘=3
log 3 (√155) − log 3 (√2)
1 1 1 𝟏 𝟏𝟓𝟓
= log 3 155 − log 3 2 = (log 3 155 − log 3 2) = (𝐥𝐨𝐠 𝟑 ( )) .
2 2 2 𝟐 𝟐
155 155 155
Como = 77, 5 < 81, temos que log 3 ( ) < log 3 81 ⟹ log 3 ( ) < 4
2 2 2
𝟏 𝟏𝟓𝟓
𝐥𝐨𝐠 𝟑 ( ⟹ )<𝟐
𝟐 𝟐
𝟏 𝟏𝟓𝟓
E, consequentemente, 𝐥𝐨𝐠 𝟑 ( ) < 𝟑, demonstrando o fato apresentado.
𝟐 𝟐
8.
2020 2020 2020 2020
√𝑘(𝑘 − 1) √𝑘 𝑘−1 √𝑘 𝑘−1
∏( ) = ∏ [( ).( )] = [∏ ( )] . [∏ ( )] .
𝑘 √ 𝑘 + 1 √ 𝑘 + 1 𝑘 √ 𝑘 + 1 𝑘
𝑘=2 𝑘=2 𝑘=2 𝑘=2
Obtivemos o produto de dois produtos telescópicos:
2020 2020
√𝑘 𝑓(𝑘) 𝑓(2) √2
∏ ( )=∏ (𝑓(𝑘+1)) = 𝑓(2020+1) = , onde 𝑓(𝑘) = √𝑘.
𝑘=2 √𝑘+1 𝑘=2 √2021
2020 2020
𝑘−1 𝑓(𝑘−1) 𝑓(2−1) 1
∏ ( 𝑘
)=∏ ( 𝑓(𝑘)
) = 𝑓(2020) = 2020, onde 𝑓(𝑘) = 𝑘.
𝑘=2 𝑘=2
Assim:
2020 2020
√𝑘 𝑘−1 √2 1 √2
[∏ ( )] . [∏ ( )] = . = .
√ 𝑘 + 1 𝑘 √ 2021 2020 2020√2021
𝑘=2 𝑘=2

Utilizando uma calculadora científica qualquer, verifique que este número é


aproximadamente igual a 0,00001557..., sendo, portanto, muito próximo de zero.
9.
500
3² − 1 4² − 1 5012 − 1 (𝑘 + 1)2 − 1
𝑎) ( ).( ).….( ) = ∏( ).
2² − 1 3² − 1 5002 − 1 𝑘2 − 1
𝑘=2
500
(𝑘+1)2 −1 𝑓(500+1) (5012 −1) 𝟓𝟎𝟏²−𝟏
∏ ( 𝑘 2 −1
)= 𝑓(2)
=
22 −1
=
𝟑
, onde 𝑓(𝑘) = 𝑘² − 1.
𝑘=2
𝟕
𝑏) .
𝟏𝟔𝟏
𝑐) 𝜶𝒏 .
𝟐
𝑑) .
𝟏 + 𝐜𝐨𝐬(𝟑𝟎𝟏)
10.
Condições de existência:
1. 𝑥² − 4𝑥 + 1 > 0 ⟹ 𝑥 > 2 + √3 ∨ 𝑥 < 2 − √3.
1
2. 6𝑥² − 3𝑥 > 0 ⟹ 𝑥 > ∨ 𝑥 < 0.
2
De 1 ∩ 2 ⟹ 𝒙 > 𝟐 + √𝟑 ∨ 𝒙 < 𝟎.
Assim:
ln(𝑥 − 4𝑥 + 1) < ln(6𝑥 − 3𝑥) ⟺ 𝑥 2 − 4𝑥 + 1 < 6𝑥 2 − 3𝑥 ⟹ 𝟓𝒙𝟐 + 𝒙 − 𝟏 > 𝟎.
2 2
1 1
Desta última, tiramos que 𝑥 > (√21 − 1) ∨ 𝑥 < 10 (− 1 − √21). 3.
10
𝟏
De 1 ∩ 2 ∩ 3 ⟹ 𝒙 > 𝟐 + √𝟑 ∨ 𝒙 < (− 𝟏 − √𝟐𝟏).
𝟏𝟎
𝟏
S = {𝒙 ∈ ℝ: 𝒙 > 𝟐 + √𝟑 ∨ 𝒙 < (− 𝟏 − √𝟐𝟏)}.
𝟏𝟎
11.
9 𝟑√𝟔𝟎𝟎𝟏
√ = .
6001 𝟔𝟎𝟎𝟏
12.
𝝅𝟐𝟎𝟏
𝑎) .
𝟐𝟎𝟏!
𝑑) 𝐥𝐧(𝒆𝟏+𝟐+⋯+𝟓𝟎𝟎𝟎 ) = 𝟏 + 𝟐 + ⋯ + 𝟓𝟎𝟎𝟎.
13.
Já em sua forma telescópica, o produtório em questão é tal que:
𝑛
𝑟 4 +2 𝑓(𝑟) 𝑓(𝑛) 𝒏𝟒 −𝟐
∏ ((𝑟−1)4+2) = 𝑓(𝑟−1) = 𝑓(𝑚−1) = (𝒎−𝟏)𝟒−𝟐, onde 𝑓(𝑟) = 𝑟 4 + 2.
𝑟=𝑚
Pelo Teorema do Binômio de Newton, temos que:
(𝒂 − 𝒃)𝟒 = 𝒂𝟒 − 𝟒𝒂𝟑 𝒃 + 𝟔𝒂𝟐 𝒃𝟐 − 𝟒𝒂𝒃𝟑 + 𝒃𝟒 .
Consequentemente:
(𝑚 − 1)4 = 𝑚4 − 4𝑚3 + 6𝑚2 − 4𝑚 + 1
⟹ (𝑚 − 1)4 + 2 = 𝑚4 − 4𝑚3 + 6𝑚2 − 4𝑚 + 3.
𝑛
𝑟4 + 2 𝑛4 − 2
⟹ ∏( )= 4 .
(𝑟 − 1)4 + 2 𝑚 − 4𝑚3 + 6𝑚2 − 4𝑚 + 3
𝑟=𝑚
14. P242 = − 1.
15.
𝑛

∏[𝑟. cis(𝜃)]𝑘 = 1. [𝑟. cis(𝜃)]. [𝑟. cis(𝜃)]2 . [𝑟. cis(𝜃)]³. … . [𝑟. cis(𝜃)]𝑛
𝑘=0
𝟏
1+2+⋯+𝑛 𝒏(𝒏+𝟏)
= (𝑟. cis(𝜃)) = (𝒓. 𝐜𝐢𝐬(𝜽))𝟐 .
16.
𝑛 𝑛
√𝑘 + 1(√𝑎 + 𝑏 − √𝑎 − 𝑏) 𝑥 √𝑘 + 1
∏ = (√𝑎 + 𝑏 − √𝑎 − 𝑏) ∏ .
𝑘=1
√𝑘 𝑘=1
√𝑘
O expoente 𝑥 é o número de termos deste produto, e é dado por:
𝑛 = 1 + (𝑥 − 1) ⟹ 𝑥 = 𝑛.
𝑛 𝑛
𝑥 √𝑘 + 1 𝑛 √𝑘 + 1
(√𝑎 + 𝑏 − √𝑎 − 𝑏) ∏ = (√𝑎 + 𝑏 − √𝑎 − 𝑏) ∏ .
𝑘=1
√𝑘 𝑘=1
√𝑘
𝑛 𝑛
√𝑘 + 1 𝑓(𝑘 + 1) √𝑛 + 1
∏ =∏ = = √𝑛 + 1.
√𝑘 𝑓(𝑘) √1
𝑘=1 𝑘=1
Assim:
𝑛
√𝑘 + 1(√𝑎 + 𝑏 − √𝑎 − 𝑏) 𝑛
∏ = √𝑛 + 1 (√𝑎 + 𝑏 − √𝑎 − 𝑏) .
𝑘=1
√𝑘
17.
2.4.6. … .2𝑛 2.1.2.2.2.3. … .2. 𝑛 2𝑛 . 𝑛! 𝟐𝒏
= = = .
(1.2.3. … . 𝑛)² (𝑛!)² 𝑛!. 𝑛! 𝒏!
18.
𝟐
.
𝟐𝟏𝟎𝟏 + 𝟏
19.
𝑛
𝜋 2𝑘−1
1 1
∏ [( − ) . ( 2𝑘 )]
√𝑝 √𝑞 𝜋
𝑘=𝑚
𝑥 𝑛 𝑥 𝑛
1 𝜋 2𝑘 . 𝜋 −1
1 1 1
=( − ) ∏( 2𝑘
) = ( − ) ∏ 𝜋 −1 .
√𝑝 √𝑞 𝑘=𝑚 𝜋 √𝑝 √𝑞 𝑘=𝑚
𝑛 = 𝑚 + (𝑥 − 1) ⟹ 𝑥 = 𝑛 − 𝑚 + 1.
𝑛−𝑚+1 𝑛 𝑛−𝑚+1
1 1 −1
1 1
( − ) ∏𝜋 =( − ) (𝜋 −1 )𝑛−𝑚+1
√𝑝 √𝑞 𝑘=𝑚
√𝑝 √𝑞
𝑛−𝑚+1 𝒏−𝒎+𝟏
1 1 𝟏 𝟏
=( − ) (𝜋 −1 )𝑛−𝑚+1 =( − ) . 𝝅𝒎−𝒏−𝟏 .
√ 𝑝 √𝑞 √ 𝒑 √𝒒
20.
Sejam 𝑥1 e 𝑥2 tais que 𝑥1 + 𝑥2 = 𝑐.
𝑥1 + 𝑥2 = 𝑐 ⟹ 𝑥1 = 𝑐 − 𝑥2 .
𝑥1 𝑥2 = 𝑓(𝑥2 ) = (𝑐 − 𝑥2 )(𝑥2 ) = 𝑐𝑥2 − 𝑥22 .
A função 𝑓(𝑥2 ) = 𝑥1 𝑥2 = 𝑐𝑥2 − 𝑥22 possui valor máximo, e este é obtido através das
fórmulas do vértice de uma função quadrática da seguinte maneira:
𝑏 𝑐 𝑐
𝑥𝑣 ⟹ Coordenada 𝑥𝑣 do vértice V = (𝑥𝑣 , 𝑦𝑣 ) ⟹ 𝑥𝑣 = − ⟹− = .
2𝑎 2(− 1) 2
𝑐
𝑦𝑣 ⟹ Coordenada 𝑦𝑣 do vértice, valor máximo que ocorre para 𝑥 = 𝑥𝑣 = :
2
2 2 2 2
𝑐 𝑐 𝑐 𝑐 𝑐 1 𝑐
𝑦𝑣 = 𝑓(𝑥𝑣 ) = P𝑚á𝑥 = 𝑓 ( ) = 𝑐 ( ) − ( ) = − = (𝑐 2 − )
2 2 2 2 4 2 2
𝒄𝟐
= .
𝟒
* Esta questão também pode ser resolvida através dos métodos do Cálculo Diferencial,
derivando a função 𝑓(𝑥2 ) com respeito à variável 𝑥2 , igualando esta derivada a zero e
substituindo o valor obtido em 𝑓(𝑥2 ) = 𝑐𝑥2 − 𝑥22 , obtendo, da mesma maneira, o
𝑐2
valor .
4
21.
Veja que:
sen(2𝛼) 1
sen(2𝛼) = 2sen(𝛼) cos(𝛼) ⟹ = sen(2𝛼) sec(𝛼) .
2 cos(𝛼) 2
sen(4𝛼) 1
= sen(4𝛼) sec(𝛼) .
4 cos(𝛼) 4
...
𝑛
sen(2 𝛼) 1
= sen(2𝑛 𝛼) sec(𝛼) .
2𝑛 cos(𝛼) 2𝑛
Assim:
sen(2α) sen(4α) sen(8α) sen(2𝑛 𝛼)
( ).( ).( ).….( 𝑛 )
2 cos(𝛼) 4cos(𝛼) 8cos(α) 2 cos(𝛼)
1 1 1
= ( sen(2𝛼) sec(𝛼)) . ( sen(4𝛼) sec(𝛼)) . … . ( 𝑛 sen(2𝑛 𝛼) sec(𝛼))
2 4 2
𝑛
sen(2𝛼). sen(4𝛼). … . sen(2 𝛼). sec 𝛼 ∏𝑘=1 sen(2𝑘 𝛼). (sec(𝛼))𝑛
𝑛 𝑛
= =
2.4. … . 2𝑛 21+2+⋯+𝑛
∏𝒏𝒌=𝟏 𝐬𝐞𝐧(𝟐𝒌 𝜶). (𝐬𝐞𝐜(𝜶))𝒏
= 𝟏
.
𝒏(𝒏+𝟏)
𝟐𝟐

A ideia central desta exercício é a reunião das propriedades dos produtórios,


envolvendo, também, a noção do arco duplo e seus múltiplos.
É importante frisar que esta não é uma questão de nível olímpico ou pré-universitário,
porém exige um conhecimento prévio dos conceitos mencionados acima.
Capítulo 3 – Exercícios finais de aprofundamento e desafios
1.
𝑛−1
1 1 1 1
+ + ⋯+ =∑ .
√1 + √2 √2 + √3 √𝑛 − 1 + √𝑛 𝑘=1
√𝑘 + √𝑘 + 1
𝑛−1 𝑛−1 𝑛−1
1 1 1 √𝑘 + 1 − √𝑘
∑ =∑ = ∑( ).( )
𝑘=1
√𝑘 + √𝑘 + 1 𝑘=1
√ 𝑘 + 1 + √𝑘 𝑘=1
√ 𝑘 + 1 + √𝑘 √ 𝑘 + 1 − √𝑘
𝑛−1 𝑛−1
√𝑘 + 1 − √𝑘
=∑ = ∑(√𝑘 + 1 − √𝑘).
𝑘+1−𝑘
𝑘=1 𝑘=1
A soma obtida é uma soma telescópica clássica:
𝑛−1 𝑛−1

∑(√𝑘 + 1 − √𝑘) = ∑(𝑓(𝑘 + 1) − 𝑓(𝑘)) = 𝑓(𝑛 − 1 + 1) − 𝑓(1) = √𝒏 − 𝟏.


𝑘=1 𝑘=1
2.
1 1 1
+ +⋯+ = S.
√𝟏 + √𝟑 √𝟗𝟗𝟗𝟕 + √𝟗𝟗𝟗𝟗
√𝟓 + √𝟕
Veja que:
𝒂𝒏 = (1, 5, ..., 9997).
𝒃𝒏 = (𝟑, 7, ..., 9999).
𝒂𝒏 = 𝟏 + (𝒏 − 𝟏)𝟒 ⟹ 𝒂𝒏 = 𝟒𝒏 − 𝟑.
𝒃𝒏 = 𝟑 + (𝒏 − 𝟏)𝟒 ⟹ 𝒃𝒏 = 𝟒𝒏 − 𝟏.
𝑎𝑛 = 𝑎1 + (𝑛𝑎 − 1)𝑟 ⟹ 9997 = 1 + (𝑛𝑎 − 1)4 ⟹ 𝑛𝑎 = 2500.
𝑏𝑛 = 𝑏1 + (𝑛𝑏 − 1)𝑟 ⟹ 9999 = 3 + (𝑛𝑏 − 1)4 ⟹ 𝑛𝑏 = 2500.
𝒏𝒂 = 𝒏𝒃 = 𝒏.
2500 2500
1 1 √4𝑛 − 3 − √4𝑛 − 1
S= ∑ = ∑( ).( )
𝑛=1
√4𝑛 − 3 + √4𝑛 − 1 𝑛=1
√4𝑛 − 3 + √4𝑛 − 1 √4𝑛 − 3 − √4𝑛 − 1
2500 2500
√4𝑛 − 3 − √4𝑛 − 1 √4𝑛 − 3 − √4𝑛 − 1
= ∑ = ∑
4𝑛 − 3 − (4𝑛 − 1) −2
𝑛=1 𝑛=1
2500 2500
√4𝑛 − 1 − √4𝑛 − 3 1
= ∑ = ( ∑ √4𝑛 − 1 − √4𝑛 − 3) .
2 2
𝑛=1 𝑛=1
Percebamos que, considerando 𝑓(𝑛) = 4𝑛 − 3, temos que 𝑓(𝑛 + 1) = 4(𝑛 + 1) − 3
⟹ 𝑓(𝑛 + 1) = 4𝑛 + 4 − 3 = 4𝑛 + 1.
Assim:
2500 2500
1 1
(∑ √4𝑛 − 1 − √4𝑛 − 3) = 2 (∑ √4𝑛 − 1 − √4𝑛 − 3 + √4𝑛 + 1 − √4𝑛 + 1)
2 𝑛=1 𝑛=1
2500 2500
1
= (( ∑ √4𝑛 + 1 − √4𝑛 − 3) + ( ∑ √4𝑛 − 1 − √4𝑛 + 1)) .
2
𝑛=1 𝑛=1
Veja que:
2500
1 1
( ∑ √4𝑛 + 1 − √4𝑛 − 3) = (√10.001 − 1) > 𝟐𝟒.
2 2
𝑛=1
2500
1
A parcela (∑
2
√4𝑛 − 1 − √4𝑛 + 1) não é uma soma telescópica comum, pois
𝑛=1
não se encontra nas formas 𝑓(𝑘 + 1) − 𝑓(𝑘) ou 𝑓(𝑘) − 𝑓(𝑘 + 1), porém é fácil
verificar que esta soma nos fornece um resultado negativo.
Assim, fica demonstrado que a soma em questão é tal que:
2500 2500
1
(( ∑ √4𝑛 + 1 − √4𝑛 − 3) + ( ∑ √4𝑛 − 1 − √4𝑛 + 1)) ≥ 24.
2
𝑛=1 𝑛=1
3.
2006
22 32 42 52 20062 𝑘²
( 2 ).( 2 ).( 2 ).( 2 ).….( ) = ∏ ( ).
2 −1 3 −1 4 −1 5 −1 20062 − 1 𝑘² − 1
𝑘=2
Manipulando este produtório, obtemos:
2006 2006 2006 2006
𝑘² 𝑘. 𝑘 𝑘 𝑘
∏( )= ∏( ) = (∏ ) . (∏ ).
𝑘² − 1 (𝑘 + 1)(𝑘 − 1) 𝑘+1 𝑘−1
𝑘=2 𝑘=2 𝑘=2 𝑘=2
Perceba que obtivemos dois produtos telescópicos:
2006 2006
𝑘 𝑓(𝑘) 𝑓(2) 2 2
(∏ 𝑘+1
) = (∏ 𝑓(𝑘+1)
) = 𝑓(2006+1) = 2006+1 = 2007, onde 𝑓(𝑘) = 𝑘.
𝑘=2 𝑘=2
2006 2006
𝑘 𝑓(𝑘) 𝑓(2006) 2006
(∏ 𝑘−1
) = (∏ 𝑓(𝑘−1)
)= 𝑓(2−1)
=
2−1
= 2006, onde 𝑓(𝑘) = 𝑘.
𝑘=2 𝑘=2
Assim:
2006 2006
𝑘 𝑘 2 𝟒𝟎𝟏𝟐
(∏ ) . (∏ )=( ) . 2006 = .
𝑘+1 𝑘−1 2007 𝟐𝟎𝟎𝟕
𝑘=2 𝑘=2
4.
𝑎) Façamos:

𝑥 = √2√2√2√2 … ⟹ 𝑥 2 = 2√𝟐√𝟐√𝟐 … ⟹ 𝑥 2 = 2𝑥 ⟹ 𝒙 = 𝟐.

* Não foi especificado, mas o processo de radiciação é infinito.


2

𝑏) 𝑥 = √7 + 4√3 + √7 − 4√3 ⟹ 𝑥 2 = (√7 + 4√3 + √7 − 4√3)

⟹ 𝑥 2 = 7 + 4√3 + 2. √7 + 4√3. √7 − 4√3 + 7 − 4√3

⟹ 𝑥 2 = 14 + 2. √7 + 4√3. √7 − 4√3 ⟹ 𝑥 2 = 14 + 2√(7 + 4√3)(7 − 4√3)

⟹ 𝑥 2 = 14 + 2√49 − 48 ⟹ 𝑥 2 = 14 + 2 ⟹ 𝑥 2 = 16 ⟹ 𝒙 = 𝟒.
5.
Dada a P.G (𝑎1 , 𝑎2 , ..., 𝑎𝑛 ), definamos P𝑛 = 𝑎1 . 𝑎2 . … . 𝑎𝑛 :
Deste modo, vale que:
P = 𝑎1 . 𝑎2 . … . 𝑎𝑛
{ 𝑛 ;
P𝑛 = 𝑎𝑛 . 𝑎𝑛−1 . … . 𝑎1
Multiplicando ambas as equações:
2
P𝑛 = (𝑎1 . 𝑎𝑛 ). (𝑎2 . 𝑎𝑛−1 ). … . (𝑎𝑛 . 𝑎1 ).
Sendo uma progressão geométrica, vale a seguinte propriedade:
𝑎1 . 𝑎𝑛 = 𝑎2 . 𝑎𝑛−1 = 𝑎3 . 𝑎𝑛−2 = ⋯ = 𝑎𝑛 𝑎1 .
Assim:
P𝑛 = (𝑎1 . 𝑎𝑛 ). (𝑎2 . 𝑎𝑛−1 ). … . (𝑎𝑛 . 𝑎1 ) ⟹ P𝑛2 = (𝑎1 . 𝑎𝑛 ). (𝑎1 . 𝑎𝑛 ). … . (𝑎1 . 𝑎𝑛 )
2

⟹ P𝑛2 = 𝑎1𝑛 . 𝑎𝑛𝑛 ⟹ P𝑛2 = (𝑎1 . 𝑎𝑛 )𝑛 ⟹ |𝐏𝒏 | = √(𝒂𝟏 . 𝒂𝒏 )𝒏 .


Devemos tomar o módulo do produto para não introduzirmos raízes no campo
complexo.
6.
Solução 1.
Na notação sigma, temos que:
𝑛
1 1 1 1
+ +⋯+ =∑ .
√𝑎1 + √𝑎2 √𝑎2 + √𝑎3 √𝑎𝑛−1 + √𝑎𝑛 𝑘=2 √𝑎𝑘−1 + √𝑎𝑘
Manipulando o argumento deste somatório, obtemos:
1 √𝑎𝑘−1 − √𝑎𝑘 √𝑎𝑘 − √𝑎𝑘−1
= = .
√𝑎𝑘−1 + √𝑎𝑘 𝑎𝑘−1 − 𝑎𝑘 𝑎𝑘 − 𝑎𝑘−1
Estando os termos 𝑎1 , 𝑎2 , ..., 𝑎𝑛 em progressão aritmética, temos que todas as
diferenças entre dois termos consecutivos da forma 𝑎𝑘 − 𝑎𝑘−1 é constante e igual
para todas estas diferenças.
Esta diferença é a razão da P.A:
𝒂𝟐 − 𝒂𝟏 = 𝒂𝟑 − 𝒂𝟐 = 𝒂𝟒 − 𝒂𝟑 = ⋯ = 𝒂𝒏−𝟏 − 𝒂𝒏−𝟐 = 𝒂𝒏 − 𝒂𝒏−𝟏 = 𝒓.
Deste modo:
√𝒂𝒌 − √𝒂𝒌−𝟏 √𝒂𝒌 − √𝒂𝒌−𝟏
= .
𝒂𝒌 − 𝒂𝒌−𝟏 𝒓
𝑛 𝑛 𝑛
1 √𝑎𝑘 − √𝑎𝑘−1 1
∑ = ∑( ) = ∑(√𝑎𝑘 − √𝑎𝑘−1 ) .
√𝑎𝑘−1 + √𝑎𝑘 𝑘=2 𝑟 𝑟
𝑘=2 𝑘=2
Percebamos que esta soma já se encontra em sua forma telescópica:
𝑛 𝑛
1 1 1
∑(√𝑎𝑘 − √𝑎𝑘−1 ) = ∑(𝑓(𝑘) − 𝑓(𝑘 − 1)) = (𝑓(𝑛) − 𝑓(2 − 1))
𝑟 𝑟 𝑟
𝑘=2 𝑘=2
1 √𝒂 𝒏 − √ 𝒂 𝟏
= (√𝑎𝑛 − √𝑎2−1 ) = .
𝑟 𝒓
* 𝒇(𝒌) = √𝒂𝒌 .
Enfim:
√𝑎𝑛 − √𝑎1 √𝑎𝑛 − √𝑎1 √𝑎𝑛 + √𝑎1 𝑎𝑛 − 𝑎1 𝑎1 + (𝑛 − 1)𝑟 − 𝑎1
=( ).( )= =
𝑟 𝑟 √𝑎𝑛 + √𝑎1 𝑟(√𝑎𝑛 + √𝑎1 ) 𝑟(√𝑎𝑛 + √𝑎1 )
(𝑛 − 1)𝒓 𝒏−𝟏
= = .
𝒓(√𝑎𝑛 + √𝑎1 ) √𝒂𝒏 + √𝒂𝟏

Solução 2.
Estando os números 𝑎1 , 𝑎2 , ... , 𝑎𝑛 em P.A, vale que:
𝑎2 = 𝑎1 + 𝑟;
𝑎3 = 𝑎1 + 2𝑟;
...
𝑎𝑛−1 = 𝑎1 + (𝑛 − 2)𝑟;
𝑎𝑛 = 𝑎1 + (𝑛 − 1)𝑟.
Assim:
1 1 1
+ + ⋯+
√𝑎1 + √𝑎2 √𝑎2 + √𝑎3 √𝑎𝑛−1 + √𝑎𝑛
1 1 1
= + + ⋯+ .
√𝑎1 + √𝑎1 + 𝑟 √𝑎1 + 𝑟 + √𝑎1 + 2𝑟 √𝑎1 + (𝑛 − 2)𝑟 + √𝑎1 + (𝑛 − 1)𝑟
Multiplicando pelo conjugado do denominador, obtemos:
1 √𝑎1 − √𝑎1 + 𝑟 √𝑎1 + 𝑟 − √𝑎1
= = .
√𝑎1 + √𝑎1 + 𝑟 𝑎1 − (𝑎1 + 𝑟) 𝑟
1 √𝑎1 + 𝑟 − √𝑎1 + 2𝑟 √𝑎1 + 2𝑟 − √𝑎1 + 𝑟
= = .
√𝑎1 + 𝑟 + √𝑎1 + 2𝑟 𝑎1 + 𝑟 − (𝑎1 + 2𝑟) 𝑟
...
1 √𝑎1 +(𝑛−2)𝑟−√𝑎1 +(𝑛−1)𝑟 √𝑎1 +(𝑛−1)𝑟−√𝑎1 +(𝑛−2)𝑟
= = .
√𝑎1 +(𝑛−2)𝑟+√𝑎1 +(𝑛−1)𝑟 𝑎1 +(𝑛−2)𝑟−(𝑎1 +(𝑛−1)𝑟) 𝑟
Então:
1 1 1
+ + ⋯+
√𝑎1 + √𝑎1 + 𝑟 √𝑎1 + 𝑟 + √𝑎1 + 2𝑟 √𝑎1 + (𝑛 − 2)𝑟 + √𝑎1 + (𝑛 − 1)𝑟
√𝑎1 + 𝑟 − √𝑎1 √𝑎1 + 2𝑟 − √𝑎1 + 𝑟 √𝑎1 + (𝑛 − 1)𝑟 − √𝑎1 + (𝑛 − 2)𝑟
= + +⋯+
𝑟 𝑟 𝑟
1
= ((√𝒂𝟏 + 𝒓 − √𝑎1 ) + (√𝒂𝟏 + 𝟐𝒓 − √𝒂𝟏 + 𝒓) + ⋯ + (√𝑎1 + (𝑛 − 1)𝑟 − √𝒂𝟏 + (𝒏 − 𝟐)𝒓)) .
𝑟
Perceba que todos os termos se cancelarão, com exceção de √𝑎1 e √𝑎1 + (𝑛 − 1)𝑟.
Assim:
√𝑎1 + 𝑟 − √𝑎1 √𝑎1 + 2𝑟 − √𝑎1 + 𝑟 √𝑎1 + (𝑛 − 1)𝑟 − √𝑎1 + (𝑛 − 2)𝑟
+ + ⋯+
𝑟 𝑟 𝑟
1 √𝑎𝑛 − √𝑎1 √𝑎𝑛 − √𝑎1 √𝑎𝑛 + √𝑎1
= (√𝑎1 + (𝑛 − 1)𝑟 − √𝑎1 ) = =( ).( )
𝑟 𝑟 𝑟 𝑎 +
√ 𝑛 √ 1 𝑎
𝑎𝑛 − 𝑎1 𝑎1 + (𝑛 − 1)𝑟 − 𝑎1 (𝑛 − 1)𝑟 𝒏−𝟏
= = = = .
𝑟(√𝑎𝑛 + √𝑎1 ) 𝑟(√𝑎𝑛 + √𝑎1 ) 𝑟(√𝑎𝑛 + √𝑎1 ) √𝒂 𝒏 + √ 𝒂 𝟏
7.
Transpondo esta soma para a notação sigma:
𝑛
1 1 1 1 1
+ + + ⋯+ =∑ .
𝑎1 . 𝑎2 𝑎2 . 𝑎3 𝑎3 . 𝑎4 𝑎𝑛−1 . 𝑎𝑛 𝑎𝑘−1 . 𝑎𝑘
𝑘=2
Tendo em mente que
𝑎2 − 𝑎1 = 𝑎3 − 𝑎2 = 𝑎4 − 𝑎3 = ⋯ = 𝑎𝑛−1 − 𝑎𝑛−2 = 𝑎𝑛 − 𝑎𝑛−1 = 𝑟, temos que:
1 1 𝑟 𝑟 𝑎𝑘 − 𝑎𝑘−1 𝑎𝑘 𝑎𝑘−1
= . = = = −
𝑎𝑘−1 . 𝑎𝑘 𝑎𝑘−1 . 𝑎𝑘 𝑟 𝑎𝑘−1 . 𝑎𝑘 . 𝑟 𝑎𝑘−1 . 𝑎𝑘 . 𝑟 𝑎𝑘−1 . 𝑎𝑘 . 𝑟 𝑎𝑘−1 . 𝑎𝑘 . 𝑟
1 1 1 1 1
= − = ( − ).
𝑎𝑘−1 . 𝑟 𝑎𝑘 . 𝑟 𝑟 𝑎𝑘−1 𝑎𝑘
Enfim:
𝑛 𝑛 𝑛
1 1 1 1 1 1 1
∑ =∑ ( − ) = ∑( − ).
𝑎𝑘−1 . 𝑎𝑘 𝑟 𝑎𝑘−1 𝑎𝑘 𝑟 𝑎𝑘−1 𝑎𝑘
𝑘=2 𝑘=2 𝑘=2
𝑛
1 1 1 1 1 1 1 1 1 1 𝑎𝑛 − 𝑎1
∑( − )= ( − )= ( − )= ( )
𝑟 𝑎𝑘−1 𝑎𝑘 𝑟 𝑎2−1 𝑎𝑛 𝑟 𝑎1 𝑎𝑛 𝑟 𝑎1 . 𝑎𝑛
𝑘=2
1 𝑎1 + (𝑛 − 1)𝑟 − 𝑎1 (𝑛 − 1)𝑟 𝒏 − 𝟏
= ( )= = .
𝑟 𝑎1 . 𝑎𝑛 𝑎1 . 𝑎𝑛 . 𝑟 𝒂𝟏 . 𝒂𝒏
8.
Seja S𝑛 = 𝑎1 + 𝑎2 + ⋯ + 𝑎𝑛 .
Sabemos, da propriedade dos termos equidistantes dos extremos de uma P.A, que:
(𝑎1 + 𝑎𝑛 ) = (𝑎2 + 𝑎𝑛−1 ) = (𝑎3 + 𝑎𝑛−2 ) = ⋯ = (𝑎𝑛−1 + 𝑎2 ) = (𝑎𝑛 + 𝑎1 ).
Deste modo:
S = 𝑎1 + 𝑎2 + ⋯ + 𝑎𝑛
{ 𝑛 +
S𝑛 = 𝑎𝑛 + 𝑎𝑛−1 + ⋯ + 𝑎1
Somando ambas as equações:
2S𝑛 = (𝑎1 + 𝑎𝑛 ) + (𝑎2 + 𝑎𝑛−1 ) + ⋯ + (𝑎𝑛 + 𝑎1 ).
2S𝑛 = (𝑎1 + 𝑎𝑛 ) + (𝑎1 + 𝑎𝑛 ) + ⋯ + (𝑎1 + 𝑎𝑛 )
⟹ 2S𝑛 = (𝑎1 + 𝑎𝑛 )(1 + 1 + ⋯ + 1) ⟹ 2S𝑛 = (𝑎1 + 𝑎𝑛 )𝑛
𝟏
⟹ 𝐒𝒏 = (𝒂𝟏 + 𝒂𝒏 )𝒏.
𝟐
9. Reveja o exercício resolvido de número 12 da teoria das Somas Telescópicas.
𝑛 𝑛 𝑛
1 √𝑘 + 1 − √𝑘 √𝑘 + 1 √𝑘
∑ =∑ = ∑( − )
𝑘=1
(𝑘 + 1)√𝑘 + 𝑘√𝑘 + 1 𝑘=1
√𝑘√𝑘 + 1 𝑘=1
√𝑘√𝑘 + 1 √𝑘√𝑘 + 1
𝑛
1 1 1 1 1 √𝒏 + 𝟏 − 𝟏
= ∑( − )= − =1− = .
𝑘=1
√𝑘 √𝑘 + 1 √1 √𝑛 + 1 √𝑛 + 1 √𝒏 + 𝟏
* Resolva este exercício para 𝑛 = 2016.
10.
2016 2016 2016
1 𝑘² − 1 (𝑘 + 1)(𝑘 − 1)
∏ (1 − 2 ) = ∏ ( )= ∏[ ]
𝑘 𝑘² 𝑘. 𝑘
𝑘=2 𝑘=2 𝑘=2
2016 2016
𝑘+1 𝑘−1 2016 + 1 2−1 𝟐𝟎𝟏𝟕
= (∏ ) . (∏ )=( ).( )= .
𝑘 𝑘 2 2016 𝟒𝟎𝟏𝟐
𝑘=2 𝑘=2
11.

𝑥 = √1 + √1 + √1 + √1 + ⋯;

𝑥² = 1 + √1 + √1 + √1 + ⋯ ⟹ 𝑥 2 = 1 + 𝑥 ⟹ 𝒙𝟐 − 𝒙 − 𝟏 = 𝟎.
As raízes desta equação são tais que:
1 √1 − 4(−1) 1 √5
𝑥= ± ⟹𝑥= ± .
2 2 2 2
Obtivemos as seguintes raízes:
1 √5 1 √5
𝑥= + ∨𝑥 = − .
2 2 2 2
Como estes números são provenientes de uma raiz quadrada, devemos ter que 𝑥 > 0.
1 √5
Assim, descartamos o número 𝑥 = − , pois tomando √5 = 2 + 𝑘,
2 2
com 0 ≤ 𝑘 < 1 (pois √5 ≠ 3) temos:
1 √5 1 2+𝑘 1−2+𝑘 −1+𝑘
𝑥= − ⟹𝑥= − ⟹𝑥= ⟹𝑥= .
2 2 2 2 2 2
Para qualquer valor de 𝑘 restrito às condições 0 ≤ 𝑘 < 1, teremos 𝑥 < 0.
Enfim:
𝟏 √𝟓
𝒙= + .
𝟐 𝟐
Este número é conhecido como Número de Ouro ou Razão Áurea,
e recebe o símbolo Φ.
* Esperamos que você, leitor, compreenda o formalismo acima, pois escolhemos não
tomar um valor arredondado para √5, justamente para deixar claro que, para qualquer
valor de 𝑘 ∈ [0, 1[, o arredondamento da raiz de 5 será tal que 𝑥 < 0.
Caso você, atento leitor, não tenha compreendido a ideia acima:
√5 = 2, 23606797749979 ou, simplesmente √5 = 2, 2.
1 √5 1 2, 2 𝟏, 𝟐
𝑥= − ⟹𝑥= − ⟹𝒙=− < 𝟎.
2 2 2 2 𝟐
12.

𝑥 = √2 + √2 + √2 + ⋯ e 𝑦 = √2√2√2 …
𝑥² = 2 + √2 + √2 + ⋯ ⟹ 𝑥 = 2.
𝑦² = 2√2√2 … ⟹ 𝑦 = 2.
Assim, 𝒙 = 𝒚.
13.
Temos uma série telescópica:
∞ 𝑛
3 3 3 3
∑( 2 − ) = lim [∑ ( − )] .
𝑘 (𝑘 + 1)2 𝑛→∞ 𝑘 2 (𝑘 + 1)2
𝑘=1 𝑘=1
Primeiramente devemos obter o resultado da soma telescópica acima:
𝑛
3 3 3 3 3
∑( 2
− 2
) = 𝑓(1) − 𝑓(𝑛 + 1) = 2 − 2
=3− .
𝑘 (𝑘 + 1) 1 (𝑛 + 1) (𝑛 + 1)²
𝑘=1
3
* 𝑓(𝑘) = .
𝑘2
Assim:
𝑛
3 3 3
lim [∑ ( − ) ] = lim [3 − ].
𝑛→∞ 𝑘 2 (𝑘 + 1)2 𝑛→∞ (𝑛 + 1)2
𝑘=1
Utilizando novamente a propriedade do limite de uma soma:
lim (𝑓(𝑥) ± 𝑔(𝑥)) = lim 𝑓(𝑥) ± lim 𝑔(𝑥).
𝑥→𝑎 𝑛→∞ 𝑛→∞
3 3
⟹ lim [3 − ] = lim 3 − lim ( ).
𝑛→∞ (𝑛 + 1)2 𝑛→∞ 𝑛→∞ (𝑛 + 1)2
O limite da função 𝑦 = 3, quando seu argumento tende a infinito é, de maneira óbvia,
igual a 3, uma vez que a função 𝑦 = 3 representa uma função constante, uma reta
paralela ao eixo das abcissas que intercepta o eixo das ordenadas no ponto (0; 3) e,
para todo 𝑥 pertencente ao domínio desta função (D𝑓 = ℝ) , o valor de 𝑓(𝑥) é
constante e igual a 3.
3
O limite da função 𝑦 = (𝑛+1)2 , quando 𝑛 → ∞, é igual a zero, uma vez que, para
3
valores muito grandes de 𝑛, cada vez mais próximos do infinito, o quociente (𝑛+1)2
se
aproxima cada vez mais de zero.
Assim:
∞ 𝑛
3 3 3 3 3
∑( 2− ) = lim [∑ ( − ) ] = lim [3 − ]
𝑘 (𝑘 + 1)2 𝑛→∞ 𝑘 2 (𝑘 + 1)2 𝑛→∞ (𝑛 + 1)2
𝑘=1 𝑘=1
3
= lim 3 − lim ( ) = 3 − 0 = 𝟑.
𝑛→∞ 𝑛→∞ (𝑛 + 1)2
14.
𝑛

∏[sen(𝜃 2 − 𝑘 2 ). cossec((𝜃 − 𝑘 − 1)(𝜃 + 𝑘 + 1))]


𝑘=0
𝑛 𝑛
2 2)
sen(𝜃 2 − 𝑘 2 )
= ∏[sen(𝜃 − 𝑘 . cossec(𝜃² − (𝑘 + 1)²)] = ∏ [ ].
sen(𝜃² − (𝑘 + 1)²)
𝑘=0 𝑘=0
𝑛 𝑛
sen(𝜃 2 − 𝑘 2 ) 𝑓(𝑘) 𝑓(0)
∏[ ]=∏ = .
sen(𝜃² − (𝑘 + 1)²) 𝑓(𝑘 + 1) 𝑓(𝑛 + 1)
𝑘=0 𝑘=0
* 𝑓(𝑘) = sen(𝜃 − 𝑘 2 ). 2

𝑓(0) sen(𝜃 2 − 0²)


= 2
= 𝐬𝐞𝐧(𝜽𝟐 ). 𝐜𝐨𝐬𝐬𝐞𝐜(𝜽𝟐 − (𝒏 + 𝟏)𝟐 ).
𝑓(𝑛 + 1) sen(𝜃 − (𝑛 + 1)²)
15.
Demonstre tal fato através do Princípio da Indução Finita.
16.
1001
𝑘2
𝑎= ∑( )
2𝑘 − 1
𝑘=1
1001
𝑘²
𝑏= ∑( )
{ 2𝑘 + 1
𝑘=1
1001 1001 1001
𝑘2 𝑘2 𝑘2 𝑘²
𝑎−𝑏 = ∑ ( )− ∑ ( )= ∑( − ).
2𝑘 − 1 2𝑘 + 1 2𝑘 − 1 2𝑘 + 1
𝑘=1 𝑘=1 𝑘=1
1001 1001
𝑘 2 (2𝑘 + 1) − 𝑘 2 (2𝑘 − 1) 𝑘 2 (2𝑘 + 1 − (2𝑘 − 1))
= ∑( )= ∑( )
(2𝑘 − 1)(2𝑘 + 1) (2𝑘 − 1)(2𝑘 + 1)
𝑘=1 𝑘=1
1001 1001
2𝑘² 𝑘²
= ∑( )=2∑( ).
(2𝑘 − 1)(2𝑘 + 1) (2𝑘 − 1)(2𝑘 + 1)
𝑘=1 𝑘=1
O argumento desta soma, quando expandido em frações parciais, é tal que:
𝑘² 1 1 1
= − + .
(2𝑘 − 1)(2𝑘 + 1) 8(2𝑘 − 1) 8(2𝑘 + 1) 4
1001 1001
𝑘² 1 1 1
2∑( )=2∑( − + )
(2𝑘 − 1)(2𝑘 + 1) 8(2𝑘 − 1) 8(2𝑘 + 1) 4
𝑘=1 𝑘=1
1001 1001 1001
1 1 1 1 1 1
= ∑( − + )= ∑( − )+ ∑ .
4(2𝑘 − 1) 4(2𝑘 + 1) 2 4(2𝑘 − 1) 4(2𝑘 + 1) 2
𝑘=1 𝑘=1 𝑘=1
1001
1 1 1 1
⟹ ∑( − ) = 𝑓(1) − 𝑓(1001 + 1) = − .
4(2𝑘 − 1) 4(2𝑘 + 1) 8.1 − 4 8.1002 − 4
𝑘=1
1001
1 1
⟹ ∑ = 1001. .
2 2
𝑘=1
Enfim, obtemos:
1 1 1 1
− + 1001. = 1001. .
8.1 − 4 8.1002 − 4 2 2
1 1
O número − é muito próximo de zero, assim, consideremos apenas a
8.1−4 8.1002−4
1
parcela 1001. ⟹ 500,5 ⟹ O inteiro mais próximo de 500,5 é 𝟓𝟎𝟏.
2
17.
Veja que:
𝜋
1 + 𝑖 √2. cis (4 ) 𝝅
𝑧= = = 𝐜𝐢𝐬 ( ) .
√2 √2 𝟒
Esta soma nos fornecerá um número complexo resultante, e o problema nos pede o
módulo deste complexo:
60 1 2 60
𝜋 𝜋 𝜋
∑ 𝑧 𝑛 = (cis ( )) + (cis ( )) + ⋯ + (cis ( )) .
4 4 4
𝑛=1
𝜋
Obtivemos a soma de uma progressão geométrica de razão cis ( ), e, aplicando a
4
𝑎1 (𝑞 𝑛 −1)
fórmula da soma da P.G ⟹ S𝑛 = :
𝑞−1

𝜋 𝜋 60
cis ( ) ((cis ( − 1)
𝜋
1
𝜋
2
𝜋
60
4 4 ))
(cis ( )) + (cis ( )) + ⋯ + (cis ( )) = 𝜋 .
4 4 4 cis (4 ) − 1
𝑛
Aplicando a Primeira Fórmula de Moivre ⟹ (cis(𝜃)) = cis(𝑛. 𝜃):
𝜋 𝜋 60
cis ( ) ((cis ( )) − 1) cis (𝜋) (cis (60 𝜋) − 1) cis (𝜋) (cis(15𝜋) − 1)
4 4 4 4 4
𝜋 = 𝜋 = 𝜋
cis (4 ) − 1 cis ( 4 ) − 1 cis (4 ) − 1
1+𝑖
(− 2)
√2 − √2(1 + 𝑖) 2(1 + 𝑖) 2(1 + 𝑖)
= = =− = .
1+𝑖 1 + 𝑖 − √2 1 + 𝑖 − √2 √2 − 1 − 𝑖
−1
√2 √2
Temos, das propriedades do módulo de um número complexo, que o módulo de um
quociente é o quociente entre os módulos:
𝑧 |𝑧|
| | = |𝑤|, onde 𝑤 ≠ 0.
𝑤
Assim:
60 60 𝑛 60 60 𝑛
𝜋 2(1 + 𝑖) 𝜋
∑ 𝑧 𝑛 = ∑ (cis ( )) = ⟹ |∑ 𝑧 𝑛 | = |∑ (cis ( )) |
4 √2 − 1 − 𝑖 4
𝑛=1 𝑛=1 𝑛=1 𝑛=1

2(1 + 𝑖) |2 + 2𝑖| √2² + 2² √8 2√2


=| |= = = =
√2 − 1 − 𝑖 |(√2 − 1) − 𝑖| √3 − 2√2 + 1 √4 − 2√2
√(√2 − 1)² + 1²

2 1 2 + √2
= 2√ = 2√ = 2√ = √𝟒 + 𝟐√𝟐.
2(2 − √2) 2 − √2 2
60 60 𝑛
𝜋
⟹ |∑ 𝑧 | = |∑ (cis ( )) | = √𝟒 + 𝟐√𝟐.
𝑛
4
𝑛=1 𝑛=1
18.
O termo geral 𝑎𝑛 , em função de 𝑚, pode ser escrito como:
𝑎𝑛 = 𝑎𝑚 + (𝑛 − 𝑚)𝑟.
A fórmula da soma de uma Progressão Aritmética é:
1
S𝑛 = 𝑛(𝑎1 + 𝑎𝑛 ).
2
Assim:
1
S𝑚 2 𝑚(𝑎1 + 𝑎𝑚 ) 𝑚2 𝑚(𝑎1 + 𝑎𝑚 ) 𝑚2 𝑎1 + 𝑎𝑚 𝑚
= = 2 ⟹ = 2 ⟹ =
S𝑛 1 𝑛 𝑛(𝑎 1 + 𝑎𝑛 ) 𝑛 𝑎1 + 𝑎𝑛 𝑛
𝑛(𝑎1 + 𝑎𝑛 )
2
⟹ (𝑎1 + 𝑎𝑚 )𝑛 = 𝑚(𝑎1 + 𝑎𝑛 ) ⟹ (𝑎1 + 𝑎𝑚 )𝑛 = 𝑚(𝑎1 + 𝑎𝑚 + (𝑛 − 𝑚)𝑟)
⟹ (𝑎1 + 𝑎𝑚 )𝑛 = 𝑚. 𝑎1 + 𝑚. 𝑎𝑚 + 𝑚(𝑛 − 𝑚)𝑟
⟹ (𝑎1 + 𝑎𝑚 )𝑛 − 𝑚(𝑎1 + 𝑎𝑚 ) = 𝑚(𝑛 − 𝑚)𝑟
⟹ (𝑎1 + 𝑎𝑚 )(𝑛 − 𝑚) = 𝑚(𝑛 − 𝑚)𝑟 ⟹ (𝑎1 + 𝑎𝑚 )(𝑛 − 𝑚) − 𝑚(𝑛 − 𝑚)𝑟 = 0
⟹ (𝑛 − 𝑚)(𝑎1 + 𝑎𝑚 − 𝑚𝑟) = 0.
Conforme a informação de que que 𝑚 − 𝑛 ≠ 0, então a única parcela nula possível é
𝑎1 + 𝑎𝑚 − 𝑚𝑟.
𝑎1 + 𝑎𝑚 − 𝑚𝑟 = 0 ⟹ 𝑎1 + 𝑎1 + (𝑚 − 1)𝑟 − 𝑚𝑟 = 0
⟹ 2𝑎1 + 𝑚𝑟 − 𝑟 − 𝑚𝑟 = 0 ⟹ 𝒓 = 𝟐𝒂𝟏 .
19.
2016
1 1 1 1
+ + ⋯+ = ∑ .
1.2.3 2.3.4 2015.2016.2017 (𝑘 − 1)𝑘(𝑘 + 1)
𝑘=2
2016 2016
1 1 1 1
⟹ ∑ = ∑( ( − ))
(𝑘 − 1)𝑘(𝑘 + 1) 2 (𝑘 − 1)𝑘 𝑘(𝑘 + 1)
𝑘=2 𝑘=2
2016 2016 2016
1 1 1 1 1 1
= [∑ ( − )] = [ ∑ −∑ ]
2 (𝑘 − 1)𝑘 𝑘(𝑘 + 1) 2 (𝑘 − 1)𝑘 𝑘(𝑘 + 1)
𝑘=2 𝑘=2 𝑘=2
2016 2016
1 1 1 1 1
= [∑ ( − )− ∑ ( − )]
2 𝑘−1 𝑘 𝑘 𝑘+1
𝑘=2 𝑘=2
1 1 1 1 1 1 1 1
= (1 − −( − )) = (1 − − + )
2 2016 2 2017 2 2016 2 2017
1 1 1 1 𝟏 𝟏 𝟏
= − − + = + − .
2 4032 4 4034 𝟒 𝟒𝟎𝟑𝟒 𝟒𝟎𝟑𝟐
20.
24 24
1 1 √2𝑛 − √2𝑛 + 2
∑ = ∑( . )
𝑛=1
√ 2𝑛 + √ 2𝑛 + 2 𝑛=1
√ 2𝑛 + √ 2𝑛 + 2 √ 2𝑛 − √ 2𝑛 + 2
24 24 24
√2𝑛 − √2𝑛 + 2 √2𝑛 + 2 − √2𝑛 1
=∑ =∑ = (∑ √2𝑛 + 2 − √2𝑛) .
−2 2 2
𝑛=1 𝑛=1 𝑛=1
Veja que:
𝑓(𝑛) = √2𝑛 ⟹ 𝑓(𝑛 + 1) = √2(𝑛 + 1) = √2𝑛 + 2.
24 24
1 1 1
⟹ (∑ √2𝑛 + 2 − √2𝑛) = (∑ 𝑓(𝑛 + 1) − 𝑓(𝑛)) = (𝑓(24 + 1) − 𝑓(1)).
2 2 2
𝑛=1 𝑛=1
* 𝑓(𝑛) = √2𝑛.
1 1 1 1
(𝑓(24 + 1) − 𝑓(1)) = (√2.25 − √2.1) = (√50 − √2) = (5√2 − √2)
2 2 2 2
1
= (4√2) = 𝟐√𝟐.
2
Gab. E.
21.
𝑛+𝑚+1
1 1 𝑘
∏ − .( )
𝑘+1
𝑘=𝑚 √𝑝 + √𝑞 − 1 √ 𝑞 − √𝑝 + 1
[( ) ]
𝑥
𝑛+𝑚+1
1 1 𝑘
= − ∏ ( ).
𝑘+1
√ 𝑝 + √𝑞 − 1 √ 𝑞 − √𝑝 + 1 𝑘=𝑚
( )
𝑛 + 𝑚 + 1 = 𝑚 + (𝑥 − 1) ⟹ 𝑥 = 𝑛 + 2.
𝑥
𝑛+𝑚+1
1 1 𝑘
⟹ − ∏ ( )
𝑘+1
√ 𝑝 + √𝑞 − 1 √ 𝑞 − √𝑝 + 1 𝑘=𝑚
( )
𝑛+2
𝑛+𝑚+1
1 1 𝑘
= − ∏ ( )
𝑘+1
√ 𝑝 + √𝑞 − 1 √ 𝑞 − √𝑝 + 1 𝑘=𝑚
( )
𝒏+𝟐

𝟏 𝟏 𝒎
= − .
𝒎+𝒏+𝟐
√ 𝒑 + √𝒒 − 𝟏 √ 𝒒 − √𝒑 + 𝟏
( )
22.
Por problemas de fatoração, é mais cômodo desenvolvermos este determinante pelo
Teorema de Laplace.
Tomando a primeira linha para facilitar os cálculos, obtemos:
1 1 1
|𝑎1 𝑎2 𝑎3 | = 𝑎11 . A11 + 𝑎12 . A12 + 𝑎13 . A13 .
𝑎12 𝑎22 𝑎32
Mas 𝑎11 = 𝑎12 = 𝑎13 = 1:
1 1 1
⟹ |𝑎1 𝑎2 𝑎3 | = 𝑎11 . A11 + 𝑎12 . A12 + 𝑎13 . A13 = A11 + A12 + A13 .
𝑎12 𝑎22 𝑎32
Lembrando que A𝑖𝑗 = (− 1)𝑖+𝑗 . D𝑖𝑗 , onde D𝑖𝑗 é o determinante resultante do “corte”
da linha 𝑖 e coluna 𝑗 do determinante da matriz antiga.
Assim, temos que:
𝑎2 𝑎3
A11 = (− 1)1+1 . | 2 | = 𝑎2 . 𝑎32 − 𝑎3 . 𝑎22 = 𝑎2 . 𝑎3 (𝑎3 − 𝑎2 ).
𝑎2 𝑎32
𝑎1 𝑎3
A12 = (− 1)1+2 . | 2 | = − (𝑎1 . 𝑎32 − 𝑎3 . 𝑎12 ) = − (𝑎1 . 𝑎3 (𝑎3 − 𝑎1 )).
𝑎1 𝑎32
𝑎1 𝑎2
A13 = (− 1)1+3 . | 2 | = 𝑎1 . 𝑎22 − 𝑎2 . 𝑎12 = 𝑎1 . 𝑎2 (𝑎2 − 𝑎1 ).
𝑎1 𝑎22
A11 + A12 + A13 = 𝑎2 . 𝑎3 (𝑎3 − 𝑎2 ) − 𝑎1 . 𝑎3 (𝑎3 − 𝑎1 ) + 𝑎1 . 𝑎2 (𝑎2 − 𝑎1 )
= (𝒂𝟑 − 𝒂𝟐 )(𝒂𝟑 − 𝒂𝟏 )(𝒂𝟐 − 𝒂𝟏 ).
23.
Não devemos nos deixar levar pela estranheza ao visualizarmos um somatório duplo
(somatório cujo argumento é um outro somatório).
O conceito de um somatório duplo é exatamente o mesmo, porém, tratamos como
variável (argumento) todo um outro somatório, o denominado Somatório Interno ou
Somatório Argumento.
Não incorporamos a teoria acerca dos somatórios e produtórios ênuplos
(duplos, triplos, etc) pois seu procedimento de cálculo é semelhante ao de somatórios
tradicionais, porém, é estritamente necessária a interpretação correta do problema,
atentando-se aos limites (inferiores e superiores), realizando as manipulações
algébricas corretas e etc, buscando evitar enganos relativamente “comuns” no cômputo
de somatórios ênuplos.
13 𝒎

⟹ ∑ (∑(𝟐𝒌 + 𝟓)) .
𝑚=1 𝒌=𝟏
Primeiramente, é feito o cômputo do somatório interno (argumento):
𝑚 𝑚 𝑚 𝑚 𝑚

∑(2𝑘 + 5) = ∑ 2𝑘 + ∑ 5 = 2 ∑ 𝑘 + ∑ 5
𝑘=1 𝑘=1 𝑘=1 𝑘=1 𝑘=1
1
= 2(1 + 2 + ⋯ + (𝑚 − 1) + 𝑚) + 5𝑚 = 2 ( 𝑚(𝑚 + 1)) + 5𝑚
2
= 𝑚(𝑚 + 1) + 5𝑚 = 𝑚(𝑚 + 1 + 5) = 𝒎(𝒎 + 𝟔).
Ou seja:
13 𝑚 𝟏𝟑

∑ (∑ (2𝑘 + 5)) = ∑ 𝒎(𝒎 + 𝟔) = 𝟏𝟑𝟔𝟓.


𝑚=1 𝑘=1 𝒎=𝟏
24.
Primeiramente, devemos calcular o produtório interno:
𝑚
(𝑘 + 1)2
⟹∏ 2
= (𝑚 + 1)2 .
𝑘
𝑘=1
Assim:
𝑛 𝑚 𝒏
(𝑘 + 1)²
∑ (∏ ) = ∑ (𝒎 + 𝟏)𝟐 .
𝑘²
𝑚=1 𝑘=1 𝒎=𝟏
𝑛

⟹ ∑ (𝑚 + 1)2 = 22 + 32 + 42 + ⋯ + (𝑛 + 1)2 .
𝑚=1
Já demonstramos, na teoria acerca do Princípio da Indução Finita, que:
𝑛
1
∑ 𝑚² = 1² + 2² + 3² + ⋯ + 𝑛2 = 𝑛(𝑛 + 1)(2𝑛 + 1).
6
𝑚=1
Assim:
𝑛

∑ (𝑚 + 1)2 = 22 + 32 + 42 + ⋯ + (𝑛 + 1)2
𝑚=1
= 𝟏² + 𝟐² + 𝟑² + ⋯ + 𝒏𝟐 + (𝑛 + 1)2 − 1²
1 1
= 𝑛(𝑛 + 1)(2𝑛 + 1) + (𝑛 + 1)² − 1 = 𝑛(𝑛 + 1)(2𝑛 + 1) + 𝑛(𝑛 + 2)
6 6
1 (𝑛 + 1)(2𝑛 + 1) + 6𝑛 + 12
= 𝑛 ( (𝑛 + 1)(2𝑛 + 1) + (𝑛 + 2)) = 𝑛 ( )
6 6
1 𝟏
= 𝑛(2𝑛2 + 𝑛 + 2𝑛 + 1 + 6𝑛 + 12) = 𝒏(𝟐𝒏𝟐 + 𝟗𝒏 + 𝟏𝟑).
6 𝟔
Assim, 𝒂 = 𝟔.
25.
𝑚
1
⟹ ∑ 𝑘 = 1 + 2 + ⋯ + 𝑚 = 𝑚(𝑚 + 1).
2
𝑘=1
𝑚

⟹ ∏ 𝑘 = 1.2. … . 𝑚 = 𝑚!
𝑘=1
Assim:
5 𝑚 5 𝑚 𝟓 𝟓
𝟏
∏ (∑ 𝑘) + ∑ (∏ 𝑘) = ∏ 𝒎(𝒎 + 𝟏) + ∑ 𝒎!
𝟐
𝑚=1 𝑘=1 𝑚=1 𝑘=1 𝒎=𝟏 𝒎=𝟏
5
1
= ( ) 2.6.12.20.30 + 1 + 2 + 6 + 24 + 120
2
144.600
= + 153 = 𝟐𝟖𝟓𝟑.
32
26. Seja 𝑥 tal que:
𝑥 𝑦−1

𝑥² = ∑ [∏(𝑦 − 𝑧)] .
𝑦=1 𝑧=0
Desenvolvendo o produtório argumento, obtemos:
𝑦−1

∏(𝑦 − 𝑧) = (𝑦 − 0)(𝑦 − 1)(𝑦 − 2). … . (𝑦 − (𝑦 − 2))(𝑦 − (𝑦 − 1))


𝑧=0
= 𝑦(𝑦 − 1)(𝑦 − 2). … .2.1 = 𝒚!
Assim:
𝑥 𝑦−1 𝑥

𝑥² = ∑ [∏(𝑦 − 𝑧)] ⟹ 𝑥 2 = ∑ 𝑦!
𝑦=1 𝑧=0 𝑦=1
Desenvolvendo este somatório:
𝑥

∑ 𝑦! = 1! + 2! + 3! + ⋯ + 𝑥!
𝑦=1
𝑥

⟹ 𝑥 2 = ∑ 𝑦! ⟹ 𝒙𝟐 = 𝟏! + 𝟐! + 𝟑! + ⋯ + 𝒙!
𝑦=1
Podemos tomar valores de 𝑥 que satisfaçam esta igualdade da seguinte maneira:
Tomando 𝑥 = 1 ⟹ 12 = ∑1𝑦=1 𝑦! ⟹ 12 = 1! ✓.
Tomando 𝑥 = 2 ⟹ 2² = ∑2𝑦=1 𝑦! ⟹ 𝟐𝟐 ≠ 𝟏! + 𝟐! ⟹ 2 não é solução.
Tomando 𝑥 = 3 ⟹ 32 = ∑3𝑦=1 𝑦! ⟹ 32 = 1! + 2! + 3! ⟹ 32 = 9 ✓.
Podemos verificar, para 𝑦 > 3, que a igualdade nunca ocorrerá.
Assim, as únicas soluções são 𝒙 = 𝟏 e 𝒙 = 𝟑.
S = {1, 3}.
27.
𝑥2 𝑥3 𝑥𝑛
S𝑛 = (𝑥 + 𝑘) + ( + 𝑘) + ( + 𝑘) + ⋯ + ( + 𝑘).
2 3 𝑛
Derivando ambos os lados com relação a 𝑥, obtemos:
𝑑 𝑑 𝑥2 𝑥3 𝑥𝑛
S = ((𝑥 + 𝑘) + ( + 𝑘) + ( + 𝑘) + ⋯ + ( + 𝑘))
𝑑𝑥 𝑛 𝑑𝑥 2 3 𝑛
𝑑 𝑑 𝑑 𝑥2 𝑑 𝑥3 𝑑 𝑥𝑛
⟹ S = (𝑥 + 𝑘) + ( + 𝑘) + ( + 𝑘) + ⋯ + ( + 𝑘)
𝑑𝑥 𝑛 𝑑𝑥 𝑑𝑥 2 𝑑𝑥 3 𝑑𝑥 𝑛
𝑑 𝑑 𝑑 𝑑 𝑥2 𝑑 𝑑 𝑥3 𝑑 𝑑 𝑥𝑛 𝑑
⟹ S𝑛 = 𝑥+ 𝑘+ ( )+ 𝑘+ ( )+ 𝑘 + ⋯+ ( )+ 𝑘.
𝑑𝑥 𝑑𝑥 𝑑𝑥 𝑑𝑥 2 𝑑𝑥 𝑑𝑥 3 𝑑𝑥 𝑑𝑥 𝑛 𝑑𝑥
𝑑
Mas 𝑘 = 0:
𝑑𝑥
𝑑
S𝑛 = 1 + 𝑥 + 𝑥² + ⋯ + 𝑥 𝑛−1 .
𝑑𝑥
Obtivemos a soma de uma progressão geométrica de primeiro termo 1 e razão 𝑥:
𝑑 𝑛−1
𝒅 𝒙𝒏 − 𝟏
S = 1 + 𝑥 + 𝑥² + ⋯ + 𝑥 ⟹ 𝐒 = .
𝑑𝑥 𝑛 𝒅𝒙 𝒏 𝒙−𝟏
28.
1 2 3 4 𝑘
+ + + + ⋯ = ∑∞
𝑘=1 (𝑘+1)!.
2! 3! 4! 5!
Assim, temos que:
∞ 𝑛
𝑘 𝑘
∑ = lim (∑ ).
(𝑘 + 1)! 𝑛→∞ (𝑘 + 1)!
𝑘=1 𝑘=1
Mas:
𝑛
𝑘 1
∑ =1− .
(𝑘 + 1)! (𝑛 + 1)!
𝑘=1
Assim:
∞ 𝑛
𝑘 𝑘 1
∑ = lim (∑ ) = lim (1 − )
(𝑘 + 1)! 𝑛→∞ (𝑘 + 1)! 𝑛→∞ (𝑛 + 1)!
𝑘=1 𝑘=1
1
= lim 1 − lim ( ).
𝑛→∞ 𝑛→∞ (𝑛 + 1)!
Uma vez que função fatorial possui uma taxa de crescimento altíssima
1
lim (1 − )
𝑛→∞ (𝑛 + 1)!
1
= lim𝑛→∞ 1 − lim𝑛→∞ ((𝑛+1)!), a função (𝑛 + 1)! , quando 𝑛 → ∞, se aproxima do
1
infinito e, como consequência, o quociente (𝑛+1)!, quando 𝑛 → ∞, tende a zero, pois o
denominador é infinitamente maior que o numerador.
Assim:
1 1
lim (1 − ) = lim 1 − lim ( ) = 1 − 0 = 𝟏.
𝑛→∞ (𝑛 + 1)! 𝑛→∞ 𝑛→∞ (𝑛 + 1)!
29.
𝑎)
𝑘
6 A B
= +
(3𝑘+1 − 2𝑘+1 )(3𝑘 − 2𝑘 ) (3𝑘+1 − 2𝑘+1 ) 3𝑘 − 2𝑘
6𝑘 A(3𝑘 − 2𝑘 ) + B(3𝑘+1 − 2𝑘+1 )
⟹ 𝑘+1 =
(3 − 2𝑘+1 )(3𝑘 − 2𝑘 ) (3𝑘+1 − 2𝑘+1 )(3𝑘 − 2𝑘 )
⟹ 6𝑘 = A(3𝑘 − 2𝑘 ) + B(3𝑘+1 − 2𝑘+1 ) ⟹ 6𝑘 = A. 3𝑘 − A. 2𝑘 + B. 3𝑘+1 − B. 2𝑘+1
⟹ 6𝑘 = 3𝑘 (A + 3B) − 2𝑘 (A + 2B).
A
Tomando A + 3B = 0, B = − :
3
𝑘 𝑘 2A 𝑘+1
6 = − 2 (A − )⟹A=−3 , B = 3𝑘 .
3
Assim:
6𝑘 3𝑘 3𝑘+1
= − .
(3𝑘+1 − 2𝑘+1 )(3𝑘 − 2𝑘 ) 3𝑘 − 2𝑘 3𝑘+1 − 2𝑘+1
∞ ∞
6𝑘 3𝑘 3𝑘+1
𝑏) ∑ 𝑘+1 = ∑( 𝑘 − )
(3 − 2𝑘+1 )(3𝑘 − 2𝑘 ) 3 − 2𝑘 3𝑘+1 − 2𝑘+1
𝑘=1 𝑘=1
𝑛
3𝑘 3𝑘+1
= lim [∑ 𝑘 − ].
𝑛→∞ 3 − 2𝑘 3𝑘+1 − 2𝑘+1
𝑘=1
Calculemos esta soma telescópica:
𝑛 𝑛
3𝑘 3𝑘+1
∑( 𝑘 − ) = ∑ (𝑓(𝑘) − 𝑓(𝑘 + 1)) = 𝑓(1) − 𝑓(𝑛 + 1)
3 − 2𝑘 3𝑘+1 − 2𝑘+1
𝑘=1 𝑘=1
3𝑛+1
= 3 − 𝑛+1 .
3 − 2𝑛+1
Assim:
𝑛
3𝑘+1 3𝑘 3𝑛+1
lim [∑ ( 𝑘+1 − )] = lim [3 − 𝑛+1 ]
𝑛→∞ 3 − 2𝑘+1 3𝑘 − 2𝑘 𝑛→∞ 3 − 2𝑛+1
𝑘=1
3𝑛+1 3𝑛+1
= lim 3 − lim ( 𝑛+1 ) = 3 − lim ( 𝑛+1 ) = 3 − 1 = 𝟐.
𝑛→∞ 𝑛→∞ 3 − 2𝑛+1 𝑛→∞ 3 − 2𝑛+1
30.
𝑚−7 𝑚−7 𝑚−7

∑ (𝑘² + 5𝑘) = ∑ 𝑘² + ∑ 5𝑘
𝑘=1 𝑘=1 𝑘=1
1 1
= (𝑚 − 7)((𝑚 − 7) + 1)(2(𝑚 − 7) + 1)) + 5 ( (𝑚 − 7)((𝑚 − 7) + 1))
6 2
1 5
= (𝑚 − 7)(𝑚 − 6)(2𝑚 − 13) + (𝑚 − 7)(𝑚 − 6)
6 2
1 5 4𝑚 − 26 + 30
= (𝑚 − 7)(𝑚 − 6) ( (2𝑚 − 13) + ) = (𝑚 − 7)(𝑚 − 6) ( )
6 2 12
4𝑚 + 4 1
= (𝑚 − 7)(𝑚 − 6) ( ) = (𝑚 − 7)(𝑚 − 6)(𝑚 + 1).
12 3
Assim:
2016 𝑚−7 2016
1
∏ ( ∑ (𝑘² + 5𝑘)) = ∏ [ (𝑚 − 7)(𝑚 − 6)(𝑚 + 1)] .
3
𝑚=7 𝑘=1 𝑚=7
1
Perceba que, se tomarmos 𝑚 = 7, obteremos (7 − 7)(7 − 6)(7 + 1) = 0.
3
Assim, como se trata de um produtório, todos os termos seguintes serão multiplicados
por 0.
Assim:
2016 𝑚−7 2016
1
∏ ( ∑ (𝑘² + 5𝑘)) = ∏ [ (𝑚 − 7)(𝑚 − 6)(𝑚 + 1)] = 𝟎.
3
𝑚=7 𝑘=1 𝑚=7
31.
15 15
2𝑘−1
𝜋 𝜋 2𝑘−1
∑ Im (cis ( )) = ∑ Im (cis ( ) ).
36 36
𝑘=1 𝑘=1
* Lembrando que cis(𝛼) = cos(𝛼) + 𝑖. sen(𝛼).
15
𝜋 2𝑘−1
⟹ ∑ Im (cis ( ) )
36
𝑘=1
𝜋 𝜋 3 𝜋 5 𝜋 27 𝜋 29
= Im (cis ( )) + Im (cis ( ) ) + Im (cis ( ) ) + ⋯ + Im (cis ( ) ) + Im (cis ( ) )
36 36 36 36 36
𝜋 𝜋 3 𝜋 5 𝜋 27 𝜋 29
= Im (cis ( ) + (cis ( ) ) + (cis ( ) ) + ⋯ + (cis ( ) ) + (cis ( ) )) .
36 36 36 36 36
𝜋
Seja 𝑧 = cis ( ):
36
𝜋 𝜋 3 𝜋 5 𝜋 27 𝜋 29
⟹ Im (cis ( ) + (cis ( ) ) + (cis ( ) ) + ⋯ + (cis ( ) ) + (cis ( ) ))
36 36 36 36 36
= Im(𝑧 + 𝑧 3 + 𝑧 5 + ⋯ + 𝑧 27 + 𝑧 29 ).
Obtivemos uma progressão geométrica de razão 𝑧²:
3 5 27 29 )
𝑧(𝑧 30 − 1)
⟹ Im(𝑧 + 𝑧 + 𝑧 + ⋯ + 𝑧 + 𝑧 = Im ( 2 ).
𝑧 −1
𝑧(𝑧 30 − 1) 𝑧 30 − 1
⟹ = 𝑧( 2 ).
𝑧2 − 1 𝑧 −1
𝜋
Uma vez que = 5°, vale que:
36
𝑧² − 1 = cis(10°) − 1 = cos(10°) + 𝑖. sen(10°)
2 (5°)
= 1 − 2sen + 𝑖. sen(10°) − 1 = 2𝑖²sen2 (5°) + 𝑖. 2sen(5°) cos(5°)
= 2𝑖sen(5°)(𝑖. sen(5°) + cos(5°)) = 2𝑖sen(5°)cis(5°).
𝑛
Lembrando que (cis(𝛼)) = cis(𝑛. 𝛼), perceba que:
𝑧 30 − 1 cis(30.5°) − 1 cis(150°) − 1
𝑧( 2 ) = cis(5°) ( )=
𝑧 −1 2𝑖sen(5°)cis(5°) 2𝑖sen(5°)

√3 1 √3 1 √3 − 2 1
− + 𝑖 − 1 (− 2 − 1) + 𝑖 2 (− 2 ) + 𝑖 2
= 2 2 = = .
𝜋 𝜋 𝜋
2𝑖sen (36) 2𝑖sen (36) 2𝑖sen (36)
Multiplicando numerador e denominador por − 𝑖:
√3 − 2 1 √3 − 2 1
(− ) +𝑖 − 𝑖 (− ) +
2 2 𝑖 2 2
⟹ 𝜋 . = 𝜋 .
2𝑖sen (36) 𝑖 − 2sen (36)
Como devemos obter a parte imaginária deste complexo:
√3 − 2 1
− 𝑖 (− ) +
2 2 1 2 + √3
𝜔= 𝜋 ⟹ ω = − 𝜋 + 𝑖 𝜋
− 2sen (36) 4sen (36) 4sen (36)
𝟐 + √𝟑
⟹ 𝐈𝐦(𝝎) = 𝝅 .
𝟒𝐬𝐞𝐧 (𝟑𝟔)
Gab. A.
32.
Primeiramente, calculemos o produtório:
2𝑛
𝑘 2𝑛 𝑓(𝑘) 𝑓(3) 3
⟹∏ =∏ = = .
𝑘+1 𝑘=3 𝑓(𝑘 + 1) 𝑓(2𝑛 + 1) 2𝑛 + 1
𝑘=3
∗ 𝑓(𝑘) = 𝑘.
Deste modo:
4 2𝑛 4 4
𝑘 3 4
1 4
1 1 1 1 34
∏ (∏ ) = ∏( ) = 3 ∏( )=3 ( . . . )= .
𝑘+1 2𝑛 + 1 2𝑛 + 1 3 5 7 9 945
𝑛=1 𝑘=3 𝑛=1 𝑛=1
A seguir, calculemos o somatório:
𝑚+9 𝑚+9 𝑚+9 𝑚+9

⟹ ∑ (𝑝 − 5) = ∑ 𝑝 − ∑ 5 = ∑ 𝑝 − 5(𝑚 + 9)
𝑝=1 𝑝=1 𝑝=1 𝑝=1
= (1 + 2 + 3 + ⋯ + (𝑚 − 8) + (𝑚 − 9)) − 5(𝑚 + 9)
1 1
= (𝑚 + 9)(1 + (𝑚 + 9)) − 5(𝑚 + 9) = (𝑚 + 9)(𝑚 + 10) − 5(𝑚 + 9)
2 2
1 𝑚 + 10 − 10 1
= (𝑚 + 9) ( (𝑚 + 10) − 5) = (𝑚 + 9) ( ) = 𝑚(𝑚 + 9).
2 2 2
Deste modo:
6 𝑚+9 6 6
1 1
∑ ( ∑ (𝑝 − 5)) = ∑ ( 𝑚(𝑚 + 9)) = ∑ 𝑚(𝑚 + 9)
2 2
𝑚=1 𝑝=1 𝑚=1 𝑚=1
1
= (10 + 22 + 36 + 52 + 70 + 90) = 140.
2
Enfim:
4 2𝑛 6 𝑚+9
𝑘 𝟑𝟒
∏ (∏ ) − ∑ ( ∑ (𝑝 − 5)) = − 𝟏𝟒𝟎.
𝑘+1 𝟗𝟒𝟓
𝑛=1 𝑘=3 𝑚=1 𝑝=1
33.

1
Dado que P𝑛 = ∏ (1 + 𝑘 ), têm-se que:
𝑘=0 22

1
∏ (1 + 𝑘 ) = 2.
𝑘=0 22
(Utilizando métodos computacionais para o cômputo deste limite).
Deste modo:
√1 + √1 + 𝛼 = 2 ⟹ 𝛼 = 8.
De acordo com o enunciado, temos que:
𝛽 = 𝛼 + 32 = 8 + 32 = 40.
𝛾 = 𝛼² − 25 = 82 − 25 = 64 − 25 = 39.
Sendo 𝑏 > 0 ≠ 1 uma base qualquer, veja que:
log 𝑏 (𝛽 − 𝛾) = log 𝑏 (40 − 39) = log 𝑏 1 = 𝟎.
34.
Temos o limite da soma dos infinitos termos de uma progressão geométrica de razão
0 < 𝑞 < 1.
Deste modo:
2 3
7 7 7 1
1+( )+( ) +( ) +⋯=
√100 + √140 √100 + √140 √100 + √140 7
1−
√100 + √140
1 1 √100 + 2√35 √50.2 + 2√35
=
7
= = =
1− √100 + 2√35 − 7 √100 + 2√35 − 7 √50.2 + 2√35 − 7
√100 + 2√35
√100 + 2√35

√2(50 + √35)
√2. √50 + √35
= = .
√2(50 + √35) − 7 √2. √50 + √35 − 7

Multiplicando numerador e denominador desta última fração por √2:


√2. √50 + √35 √2. √50 + √35 √2 𝟐√𝟓𝟎 + √𝟑𝟓
= . = .
√2. √50 + √35 − 7 √2. √50 + √35 − 7 √2 𝟐√𝟓𝟎 + √𝟑𝟓 − 𝟕√𝟐
Deste modo, 𝑚 = 50, 𝑛 = 35, 𝑝 = 7 e 𝑞 = 2.
⟹ 𝑚 + 𝑛 + 𝑝 + 𝑞 = 50 + 35 + 7 + 2 = 𝟗𝟒.
35.
2 4
Dadas as recorrências 𝑎𝑘+1 = 𝑎𝑘 , 𝑏𝑘+1 = 𝑏𝑘 e sabendo-se que 𝑎1 = 30 𝑢. 𝑐
3 5
e 𝑏1 = 42 𝑢. 𝑐, temos que:
2 2 2
𝑎1 = 30 ⟹ 𝑎1+1 = 𝑎1 ⟹ 𝑎2 = 𝑎1 ⟹ 𝑎2 = . 30 = 20.
3 3 3
4 4 4 168
𝑏1 = 42 ⟹ 𝑏1+1 = 𝑏1 ⟹ 𝑏2 = 𝑏1 ⟹ 𝑏2 = . 42 = .
5 5 5 5
2 2 40 2 2
𝑎3 = 𝑎2 = . 20 = = ( ) 𝑎1 .
3 3 3 3
4 4 168 672 4 2
𝑏3 = 𝑏2 = . = = ( ) 𝑏1 .
5 5 5 25 5
Perceba que:
40
𝑎3 ( 3 ) 40 2 𝑎2
𝐈. = = = = .
𝑎2 20 60 3 𝑎1
672 4
𝑏3 4 . 42 4 𝑏2
𝐈𝐈. = 25 = = 5 = = .
𝑏2 168 5 42 5 𝑏1
5
De I e II, podemos afirmar que, tanto os termos 𝑎𝑘 quanto os termos 𝑏𝑘 constituem
progressões geométricas.
1
Lembrando que a área de um triângulo é dada por A𝑡𝑟 = 𝑏ℎ, temos:
2
A área A1 do primeiro triângulo (𝑘 = 1), de catetos 𝑎1 e 𝑏1 é:
1 1 1
A1 = 𝑏ℎ ⟹ A1 = 𝑎1 𝑏1 ⟹ A1 = . 30.42 = 630 (𝑢. 𝑐)2 .
2 2 2
A área A2 , de catetos 𝑎2 e 𝑏2 é:
1 𝟏 2 4 8
A2 = 𝑎2 𝑏2 ⟹ A2 = . . 𝒂𝟏 . . 𝒃𝟏 = .A .
2 𝟐 3 5 15 1
A área A3 , de catetos 𝑎3 e 𝑏3 é:
1 𝟏 2 2 4 2 8 2
A3 = 𝑎3 𝑏3 ⟹ . ( ) . 𝒂𝟏 . ( ) . 𝒃𝟏 = ( ) . A1 .
2 𝟐 3 5 15
Mantendo este processo até o infinito, verificamos que estas áreas compõem uma progressão
8
aritmética de razão e primeiro termo A1 = 630.
15
Deste modo, o limite da soma destas áreas é:
A1 630 630 630.15
lim A𝑛 = = = = = 𝟏𝟑𝟓𝟎 (𝒖. 𝒄)².
𝑛→∞ 1−𝑞 1− 8 7 7
( )
15 15
36.
Solução 1.
Veja que:
48 47 46 2 1 1 2 46 47 48
+ + + ⋯+ + = + + ⋯+ + + .
2.3 3.4 4.5 48.49 49.50 𝟒𝟗. 𝟓𝟎 𝟒𝟖. 𝟒𝟗 𝟒. 𝟓 𝟑. 𝟒 𝟐. 𝟑
Deste modo, se fizermos uso do termo geral das progressões aritméticas para determinarmos
a recorrência da sequência dos denominadores, temos que:
𝒂𝒏 = (49, 48, ..., 4, 3, 2) ⟹ 𝒂𝒏 = 49 + (𝑛𝑎 − 1)(− 1) ⟹ 𝒂𝒏 = 50 − 𝑛.
𝒃𝒏 = (50, 49, ..., 5, 4, 3) ⟹ 𝒃𝒏 = 50 + (𝑛𝑏 − 1)(− 1) ⟹ 𝒃𝒏 = 51 − 𝑛.
Temos que o número de termos de cada uma destas somas é 48.
Enfim:
48
1 2 46 47 48 𝑛
+ +⋯+ + + =∑ .
49.50 48.49 4.5 3.4 2.3 (50 − 𝑛)(51 − 𝑛)
𝑛=1
Em frações parciais:
48 48
𝑛 51 50
∑ = ∑( − ).
(50 − 𝑛)(51 − 𝑛) 𝑛 − 51 𝑛 − 50
𝑛=1 𝑛=1
Expandindo esta soma:
48
51 50
∑( − )
𝑛 − 51 𝑛 − 50
𝑛=1
51 50 51 50 51 50 51 50
=( − )+( − )+( − ) + ⋯( − )=
1 − 51 1 − 50 2 − 51 2 − 50 3 − 51 3 − 50 48 − 51 48 − 50
51 50 51 50 51 50 51 50
= (− − (− )) + (− − (− )) + (− − (− )) + ⋯ + (− − (− ))
50 49 49 48 48 47 3 2
51 50 51 50 51 50 51 50
= (− + ) + (− + ) + (− + ) + ⋯ + (− + )
50 49 49 48 48 47 3 2
51 51 51 51 50 50 50 50
= (− − − − ⋯− ) + ( + + + ⋯+ ).
50 49 48 3 49 48 47 2
51 51 51 51 50 50 50 50
⟹ (− − − − ⋯− ) + ( + + +⋯+ )
50 49 48 3 49 48 47 2
1 1 1 1 1 1 1 1
= − 51 ( + + + ⋯ + ) + 50 ( + + + ⋯+ ).
50 49 48 3 49 48 47 2
1 1 1 1
Percebamos que o fator + + + ⋯ + é comum às parcelas acima.
49 48 47 3
1 1 1 1
Assim, seja 𝑝 = + + + ⋯+ .
49 48 47 3
Deste modo:
1 1 1 1 1 1 1 1
⟹ − 51 ( + + + ⋯ + ) + 50 ( + + +⋯+ )
50 49 48 3 49 48 47 2
1 1 51 50
= − 51 ( + 𝑝) + 50 (𝑝 + ) = − − 51𝑝 + 50𝑝 +
50 2 50 2
51 50 1 1
= − − 𝑝 + 25 = − − − 𝑝 + 25 = − 1 − − 𝑝 + 25.
50 50 50 50
1 1 1 1
Mas 𝑝 = + + + ⋯ + :
49 48 47 3
1 1 1 1 1
⟹ 1− − 𝑝 + 25 = − 1 − + 25 − ( + + ⋯+ )
50 50 49 48 3
1 1 1 1 1 1
= − 1 + 25 − ( + + ⋯ + ) = 25 − ( + + ⋯ + + 1)
50 49 3 50 49 3
1 1 1 1 1 1 1 1 1 1
= 25 + − − ( + + ⋯ + + 1) = 25 + − ( + + ⋯ + + + 1)
2 2 50 49 3 2 50 49 3 2
51 1 1 1 1
= −( + + ⋯ + + + 1) .
2 50 49 3 2
Enfim, se compararmos com o lado direito da igualdade do enunciado, inferimos que
𝒌 = − 𝟏.
Gab. A.

Solução 2.
1 1 1
Sabendo-se que = − , temos:
𝑛(𝑛+1) 𝑛 𝑛+1
48 47 46 2 1
+ + + ⋯+ +
2.3 3.4 4.5 48.49 49.50
48 48 47 47 2 2 1 1
= ( − )+( − )+⋯+( − )+( − )
2 3 3 4 48 49 49 50
48 1 1 1 1 48 1 1 1 1 1 1
= − − −⋯− − = + 1 + − 1 − − − − ⋯− −
2 3 4 49 50 2 2 2 3 4 49 50
51 1 1 1 1
= − (1 + + + ⋯ + + ).
2 2 3 49 50
51 1 1 1 1 51 1 1 1 1
⟹ − (1 + + + ⋯ + + )= + 𝑘. (1 + + + ⋯ + + )
2 2 3 49 50 2 2 3 49 50
⟹ 𝒌 = − 𝟏.
Gab. A.
37. Veja que:
5000

𝑧𝑟 = S5001 = ∑ 𝑖 𝑘 = 1 + 𝑖 + 𝑖² + 𝑖³ + ⋯ + 𝑖 4999 + 𝑖 5000 .


𝑘=0
As potências de 𝑖 acima constituem uma progressão geométrica de razão 𝑞 = 𝑖, primeiro
termo 𝑎1 = 1 e 5001 termos.
Assim:
𝑎1 (𝑞𝑛 − 1) 1(𝑖 5001 − 1)
S𝑛 = ⟹ 𝑧𝑟 = S5001 = .
𝑞−1 𝑖−1
Das potências de 𝑖, temos que:
𝑛 𝑛 mod 4
𝑖 =𝑖 , onde 𝑛 mod 4 denota o resto da divisão de 𝑛 por 4.
⟹ 𝑖 5001 = 𝑖 5001 mod 4 .
O resto da divisão de 5001 por 4 é 1, assim:
𝑖 5001 = 𝑖 5001 mod 4 = 𝑖 1 = 𝑖.
(𝑖 5001 − 1) 𝑖−1
⟹ 𝑧𝑟 = S5001 = ⟹ 𝑧𝑟 = S5001 = = 1.
𝑖−1 𝑖−1
Deste modo, 𝑧𝑟 = 1, (𝑧𝑟 )2016 = 1 e |𝑧𝑟 |2016 = |1|2016 = 1.
A expressão pedida é (𝑧𝑟 )2016 + |𝑧𝑟 |2016 = 1 + 1 = 𝟐.
38.
Temos que:

∑ (cos(𝜃))2𝑛 = 5 ⟹ 1 + (cos(𝜃))2 + (cos(𝜃))4 + ⋯ = 5.


𝑛=0
Obtivemos a soma dos infinitos termos de uma progressão geométrica de primeiro termo
igual a 1 e razão igual a (cos(𝜃))2 .
Deste modo, aplicando a fórmula do limite da soma dos infinitos termos de uma progressão
geométrica, temos:
1
⟹ 1 + (cos(𝜃))2 + (cos(𝜃))4 + ⋯ = 5 ⟹ =5
1 − (cos(𝜃))2
4
⟹ 5 − 5(cos(𝜃))2 = 1 ⟹ − 5(cos(𝜃))2 = − 4 ⟹ (cos(𝜃))2 =
5
2 √5 2 √5
cos(𝜃) = ou cos(𝜃) = − .
5 5
Assim:
2√5 2 √5 2√5
𝜃 = arc cos ( ) ou 𝜃 = arc cos (− ) = − arc cos ( ).
5 5 5
Esboçando ambas as soluções no ciclo trigonométrico, e lembrando-se que o eixo dos
cossenos é o eixo das abcissas, verifica-se que ambos os arcos satisfazem a equação
Assim:
𝟐√𝟓 𝟐√𝟓
𝐒 = {𝜽 = 𝐚𝐫𝐜 𝐜𝐨𝐬 ( ) 𝐨𝐮 𝜽 = − 𝐚𝐫𝐜 𝐜𝐨𝐬 ( )} .
𝟓 𝟓
39.
𝑛
Dado que 𝑓(𝑛) = , temos que:
𝑓(𝑛−1)
2
𝑓(2) = ;
𝑓(1)
3
𝑓(3) = ;
𝑓(2)
4
𝑓(4) = ;
𝑓(3)
...
2015
𝑓(2015) = ;
𝑓(2014)
2016
𝑓(2016) = .
𝑓(2015)
O produto em questão é tal que:
2
𝑓(1). 𝑓(2) = 𝑓(1). = 2;
𝑓(1)
4
𝑓(3). 𝑓(4) = 𝑓(3). = 4;
𝑓(3)
6
𝑓(5). 𝑓(6) = 𝑓(5). = 6;
𝑓(5)
8
𝑓(7). 𝑓(8) = 𝑓(7). = 8;
𝑓(7)
...
2016
𝑓(2015). 𝑓(2016) = 𝑓(2015). = 2016.
𝑓(2015)
Perceba que, agrupando os termos dois a dois, teremos um total de 1008 produtos.
Deste modo, o produto resultante é tal que:
𝑓(1). 𝑓(2). 𝑓(3). … . 𝑓(2015). 𝑓(2016)
= (𝑓(1). 𝑓(2)). (𝑓(3). 𝑓(4)). (𝑓(5). 𝑓(6)). (𝑓(7). 𝑓(8)). … . (𝑓(2015). 𝑓(2016))
= 2.4.6.8. … .2016 = (2.1). (2.2). (2.3). (2.4). … . (2.1008)
= 21008 (1.2.3.4. … .1008) = 𝟏𝟎𝟎𝟖! 𝟐𝟏𝟎𝟎𝟖 .
40.
Dado que S𝑘 é a soma dos 𝑘 primeiros termos de uma progressão aritmética, temos:
S1 = 𝑎1 ;
S2 = 𝑎1 + 𝑎2 ;
S3 = 𝑎1 + 𝑎2 + 𝑎3 ;
...
S𝑛−1 = 𝑎1 + 𝑎2 + 𝑎3 + ⋯ + 𝑎𝑛−1 ;
S𝑛 = 𝑎1 + 𝑎2 + 𝑎3 + ⋯ + 𝑎𝑛 .
Perceba o padrão:
S2 − S1 = (𝑎1 + 𝑎2 ) − 𝑎1 = 𝑎2 .
S3 − S2 = (𝑎1 + 𝑎2 + 𝑎3 ) − (𝑎1 + 𝑎2 ) = 𝑎3 .
S4 − S3 = (𝑎1 + 𝑎2 + 𝑎3 + 𝑎4 ) − (𝑎1 + 𝑎2 + 𝑎3 ) = 𝑎4 .
...
S𝑛 − S𝑛−1 = (𝑎1 + 𝑎2 + 𝑎3 + ⋯ + 𝑎𝑛−1 + 𝑎𝑛 ) − (𝑎1 + 𝑎2 + 𝑎3 + ⋯ + 𝑎𝑛−1 ) = 𝑎𝑛 .
Deste modo, temos que:
(S𝑛 − S𝑛−1 ) + (S𝑛−1 − S𝑛−2 ) + (S𝑛−2 − S𝑛−3 ) + ⋯ + (S3 − S2 ) + (S2 − S1 )
= 𝑎𝑛 + 𝑎𝑛−1 + ⋯ + 𝑎3 + 𝑎2 = 𝒂𝟏 + 𝑎2 + 𝑎3 + ⋯ + 𝑎𝑛 − 𝒂𝟏
1 1 1
= 𝑛(𝑎1 + 𝑎𝑛 ) − 𝑎1 = (𝑛(𝑎1 + 𝑎𝑛 ) − 2𝑎1 ) = (𝑛. 𝑎1 + 𝑛. 𝑎𝑛 − 2𝑎1 )
2 2 2
𝟏
= (𝒂𝟏 (𝒏 − 𝟐) + 𝒏. 𝒂𝒏 ), como queríamos demonstrar.
𝟐
41.
Dado que cis(𝜃) = cos(𝜃) + 𝑖. sen(𝜃), temos que:
1 + cis(𝜃) = (1 + cos(𝜃)) + 𝑖. sen(𝜃).
𝜃
Das fórmulas do arco duplo, temos que cos(𝜃) = 2cos 2 ( ) − 1
2
𝜃
⟹ 1 + cos(𝜃) = 2cos 2 ( ).
2
𝜃
⟹ 1 + cis(𝜃) = (1 + cos(𝜃)) + 𝑖. sen(𝜃) = 2cos 2 ( ) + 𝑖. sen(𝜃).
2
𝜃 𝜃
Novamente aplicando as fórmulas do arco duplo, temos que sen(𝜃) = 2sen ( ) cos ( ).
2 2
𝜃
⟹ 1 + cis(𝜃) = (1 + cos(𝜃)) + 𝑖. sen(𝜃) = 2cos 2 ( ) + 𝑖. sen(𝜃)
2
𝜃 𝜃 𝜃 𝜃 𝜃 𝜃
= 2cos 2 ( ) + 𝑖. 2sen ( ) cos ( ) = 2cos ( ) (cos ( ) + 𝑖. sen ( ))
2 2 2 2 2 2
𝜽 𝜽
= 𝟐 𝐜𝐨𝐬 ( ) 𝐜𝐢𝐬 ( ) .
𝟐 𝟐
42.
1
Uma vez que log 𝑎 𝑏 = (propriedade da mudança de base), temos:
log𝑏 𝑎
1 1 1 1
= . = . log 2016 2.
2. log 2 2016 2 log 2 2016 2
1 1 1 1
= . = . log 2016 3.
5. log 3 2016 5 log 3 2016 5
1 1 1 1
= . = . log 2016 7 .
10. log 7 2016 10 log 7 2016 10
1 1 1
⟹S= + +
2. log 2 2016 5. log 3 2016 10. log 7 2016
1 1 1
= . log 2016 2 + . log 2016 3 + . log 2016 7
2 5 10
5. log 2016 2 + 2. log 2016 3 + log 2016 7 log 2016 32 + log 2016 9 + log 2016 7
= =
10 10
1 1 𝟏
= log 2016 (32.9.7) = 𝐥𝐨𝐠 𝟐𝟎𝟏𝟔 𝟐𝟎𝟏𝟔 = .
10 10 𝟏𝟎
Gab. E.
43.
Veja que:
1 1 1 1
+ + + ⋯+ 𝑛
2. log 2 2016 4. log 2 2016 8. log 2 2016 2 . log 2 2016
1 1 1 1
= . log 2016 2 + . log 2016 2 + . log 2016 2 + ⋯ + 𝑛 . log 2016 2
2 4 8 2
1 1 1 1
= log 2016 2 . ( + + + ⋯ + 𝑛 )
2 4 8 2
−1 −2 −3 −𝑛
2−1 (2−𝑛 − 1)
= log 2016 2. (2 + 2 + 2 + ⋯ + 2 ) = log 2016 2. ( )
2−1 − 1
2−1−𝑛 − 2−1
= log 2016 2. ( −1
) = 𝐥𝐨𝐠 𝟐𝟎𝟏𝟔 𝟐. (𝟏 − 𝟐−𝒏 ) .
2 −1
44.
1
Uma vez que = log 𝑛 𝑎, decorre que:
log𝑎 𝑛
1 1 1
⟹ + + ⋯+ = log 𝑛 2 + log 𝑛 3 + ⋯ + log 𝑛 2010
log 2 𝑛 log 3 𝑛 log 2010 𝑛
= log 𝑛 (2.3. … .2010) = log 𝑛 (1.2.3. … .2010) = log 𝑛 2010! .
Mas 𝑛 = 2010! :
⟹ log 𝑛 2010! = log 2010! 2010! = 𝟏.
Gab. C.
45.
𝑎) Dada a quadrática 𝑎𝑥² + 𝑏𝑥 + 𝑐 = 0, dividamos ambos os lados por 𝑎:
𝑎𝑥 2 + 𝑏𝑥 + 𝑐 𝑏 𝑐
𝑎𝑥² + 𝑏𝑥 + 𝑐 = 0 ⟹ = 0 ⟹ 𝑥 2 + 𝑥 + = 0.
𝑎 𝑎 𝑎
Devemos transformar o lado esquerdo da igualdade num quadrado perfeito, e, para tal,
façamos a seguinte manipulação algébrica:
𝑏 𝑐 2𝑏 𝑐 2𝑏 𝑐
𝑥2 + 𝑥 + = 0 ⟹ 𝑥2 + 𝑥 + = 0 ⟹ 𝑥2 + 𝑥=−
𝑎 𝑎 2𝑎 𝑎 2𝑎 𝑎
𝒃 𝑐
⟹ 𝑥2 + 2 𝑥=− .
𝟐𝒂 𝑎
𝑏2
Perceba que, por intermédio desta manipulação, podemos adicionar o fator em ambos os
4𝑎2
lados da igualdade:
2
𝑏 𝑐 2
𝑏 𝑏2 𝑏2 𝑐 𝒃 𝟐 𝒃𝟐 𝒄
𝑥 + 2 𝑥 = − ⟹ 𝑥 + 2 𝑥 + 2 = 2 − ⟹ (𝒙 + ) = −
2𝑎 𝑎 2𝑎 4𝑎 4𝑎 𝑎 𝟐𝒂 𝟒𝒂𝟐 𝒂
𝑏 𝑏2 𝑐 𝑏 𝑏 2 − 4𝑎𝑐
⟹𝑥+ =±√ 2− ⟹𝑥+ =±√
2𝑎 4𝑎 𝑎 2𝑎 4𝑎2
𝑏 √𝑏 2 − 4𝑎𝑐 𝒃 √𝒃𝟐 − 𝟒𝒂𝒄 𝒃 √∆
⟹𝑥+ =± ⟹𝒙=− ± ⟹𝒙=− ± .
2𝑎 2𝑎 𝟐𝒂 𝟐𝒂 𝟐𝒂 𝟐𝒂
* Caro leitor, caso pairem dúvidas acerca do raciocínio acima, consulte uma teoria que cubra com eficiência
os tópicos de álgebra elementar, incluindo as equações quadráticas e suas definições.
𝑏) As raízes 𝑥1 e 𝑥2 da equação 𝑎𝑥² + 𝑏𝑥 + 𝑐 = 0, tal que ∆ = 𝑏² − 4𝑎𝑐, são:
𝑏 √∆ 𝑏 √∆
𝑥1 = − + e 𝑥2 = − − .
2𝑎 2𝑎 2𝑎 2𝑎
Efetuando a soma 𝑥1 + 𝑥2 :
𝑏 √∆ 𝑏 √∆ 𝑏 𝑏 2𝑏 𝒃
𝑥1 + 𝑥2 = − + + (− − )=− − =− =− .
2𝑎 2𝑎 2𝑎 2𝑎 2𝑎 2𝑎 2𝑎 𝒂
𝑏 √∆ 𝑏 √∆
𝑐) Efetuando o produto entre as raízes 𝑥1 = − + e 𝑥2 = − − , temos:
2𝑎 2𝑎 2𝑎 2𝑎
2 2
𝑏 √∆ 𝑏 √∆ 𝑏 ∆ 𝑏 −∆
𝑥1 . 𝑥2 = (− + ) . (− − )= 2− 2= .
2𝑎 2𝑎 2𝑎 2𝑎 4𝑎 4𝑎 4𝑎2
Mas ∆ = 𝑏² − 4𝑎𝑐:
𝑏 2 − ∆ 𝑏 2 − (𝑏 2 − 4𝑎𝑐) 4𝑎𝑐 𝒄
⟹ = = 2= .
4𝑎2 4𝑎2 4𝑎 𝒂
𝑏 √∆ 𝑏 √∆ 𝑏 𝑏 √∆ √∆
𝑑) 𝑥1 − 𝑥2 = − + − (− − )=− + + +
2𝑎 2𝑎 2𝑎 2𝑎 2𝑎 2𝑎 2𝑎 2𝑎
√∆
. =
𝒂
𝑏 √∆ 𝑏 √∆ 𝑏 𝑏 √∆ √∆
𝑒) 𝑥2 − 𝑥1 = − − − (− + )=− + − −
2𝑎 2𝑎 2𝑎 2𝑎 2𝑎 2𝑎 2𝑎 2𝑎
√∆
=−
.
𝒂
46.
𝑎) Seja a equação 𝑎𝑥² + 𝑏𝑥 + 𝑐 = 0, onde 𝑎 ≠ 0.
Dividindo ambos os lados por 𝑎:
𝑏 𝑐
⟹ 𝑎𝑥 2 + 𝑏𝑥 + 𝑐 = 0 ⟹ 𝑥 2 + 𝑥 + = 0.
𝑎 𝑎
Mas, fora demonstrado que a soma S e o produto P das raízes desta equação são
𝑏 𝑐
respectivamente iguais a S = − e P = :
𝑎 𝑎
𝑏 𝑐 𝒃 𝒄
⟹ 𝑥 2 + 𝑥 + = 0 ⟹ 𝑥 2 − (− ) 𝑥 + = 0 ⟹ 𝒙𝟐 − 𝐒. 𝒙 + 𝐏 = 𝟎.
𝑎 𝑎 𝒂 𝒂
𝑘 𝑘
𝑏) Dado que S𝑘 = 𝑥1 + 𝑥2 e, uma vez que os números 𝑥1 e 𝑥2 são raízes da equação
𝑎𝑥² + 𝑏𝑥 + 𝑐 + 0, temos que:
𝐈: P(𝑥1 ) = 0 ⟹ 𝑎. 𝑥1 ² + 𝑏. 𝑥1 + 𝑐 = 0
{ ;
𝐈𝐈: P(𝑥2 ) = 0 ⟹ 𝑎. 𝑥2 ² + 𝑏. 𝑥2 + 𝑐 = 0
Multiplicando I por 𝑥1𝑘 e II por 𝑥2𝑘 :
𝐈: 𝑎. 𝑥1 ². 𝑥1𝑘 + 𝑏. 𝑥1 . 𝑥1𝑘 + 𝑐. 𝑥1𝑘 = 0 ⟹ 𝑎. 𝑥1𝑘+2 + 𝑏. 𝑥1𝑘+1 + 𝑐. 𝑥1 = 0
{ ;
𝐈𝐈: 𝑎. 𝑥2 ². 𝑥2𝑘 + 𝑏. 𝑥2 . 𝑥2𝑘 + 𝑐. 𝑥2𝑘 = 0 ⟹ 𝑎. 𝑥2𝑘+2 + 𝑏. 𝑥2𝑘+1 + 𝑐. 𝑥2 = 0
Fazendo I + II:
𝐈 + 𝐈𝐈 ⟹ 𝑎. 𝑥1𝑘+2 + 𝑏. 𝑥1𝑘+1 + 𝑐. 𝑥1 + 𝑎. 𝑥2𝑘+2 + 𝑏. 𝑥2𝑘+1 + 𝑐. 𝑥2 = 0
⟹ 𝑎. (𝑥1𝑘+2 + 𝑥2𝑘+2 ) + 𝑏. (𝑥1𝑘+1 + 𝑥2𝑘+1 ) + 𝑐. (𝑥1 + 𝑥2 ) = 0 ⟹ 𝒂. 𝐒𝒌+𝟐 + 𝒃. 𝐒𝒌+𝟏 + 𝒄. 𝐒𝒌 = 𝟎.
𝑐)
1 √5
Passo 1. Obter um polinômio de grau mínimo que possua os números 𝑥1 = + e
2 2
1 √5
𝑥2 = − como raízes.
2 2
Passo 2. Utilizar o Teorema das Somas de Newton para obter qualquer soma do tipo
S𝑘 = 𝑥1𝑘 + 𝑥2𝑘 .
Dito isto:
Passo 1. O polinômio de grau mínimo, neste caso, é um polinômio quadrático, uma vez que
desejamos encontrar a soma das potências de dois números reais.
Lembrando que toda equação quadrática pode ser escrita na forma 𝑥² − S. 𝑥 + P = 0, onde S
denota a soma de suas raízes e P o produto das mesmas, temos:
1 √5 1 √5 1 √5 1 √5 1 √5 1 √5
𝑥1 = + , 𝑥2 = − ⟹ 𝑥2 − ( + +( − )) . 𝑥 + (( + ) ( − )) =0
2 2 2 2 2 2 2 2 2 2 2 2

⟹ 𝒙𝟐 − 𝒙 − 𝟏 = 𝟎.
Passo 2. Lembrando que o Teorema das Somas de Newton é tal que:
𝒂. 𝒙² + 𝒃. 𝒙 + 𝒄 = 𝟎 ⟹ 𝒂. 𝐒𝒌+𝟐 + 𝒃. 𝐒𝒌+𝟏 + 𝒄. 𝐒𝒌 = 𝟎, onde 𝐒𝒌 = 𝒙𝒌𝟏 + 𝒙𝒌𝟐 .
⟹ 𝑥 2 − 𝑥 − 1 = 0 ⟹ S𝑘+2 − S𝑘+1 − S𝑘 = 0 ⟹ 𝐒𝒌+𝟐 = 𝐒𝒌+𝟏 + 𝐒𝒌 .
Deste modo, façamos o cálculo destas somas até 𝑘 = 8, uma vez que podemos obter
qualquer valor destas somas em função das anteriores.
Antes, porém, tenhamos em mente que:
0 0
1 √5 1 √5
S0 = 𝑥10 + 𝑥20 = ( + ) + ( − ) = 1 + 1 = 2.
2 2 2 2
1 √5 1 √5 1 1
S1 = 𝑥11 + 𝑥21 = ( + ) + ( − ) = + = 1.
2 2 2 2 2 2
⟹ 𝐒𝒌+𝟐 = 𝐒𝒌+𝟏 + 𝐒𝒌 :
2 2
1 √5 1 √5
𝑘 = 0 ⟹ S2 = S1 + S0 ⟹ S2 = 1 + 2 = 3 ⟹ S2 = ( + ) + ( − ) = 3.
2 2 2 2
3 3
1 √5 1 √5
𝑘 = 1 ⟹ S3 = S2 + S1 ⟹ S3 = 3 + 1 = 4 ⟹ S3 = ( + ) + ( − ) = 4.
2 2 2 2
4 4
1 √5 1 √5
𝑘 = 2 ⟹ S4 = S3 + S2 ⟹ S4 = 4 + 3 = 7 ⟹ S4 = ( + ) + ( − ) = 7.
2 2 2 2
5 5
1 √5 1 √5
𝑘 = 3 ⟹ S5 = S4 + S3 ⟹ S5 = 7 + 4 = 11 ⟹ S5 = ( + ) + ( − ) = 11.
2 2 2 2
6 6
1 √5 1 √5
𝑘 = 4 ⟹ S6 = S5 + S4 ⟹ S6 = 11 + 7 = 18 ⟹ S6 = ( + ) + ( − ) = 18.
2 2 2 2
7 7
1 √5 1 √5
𝑘 = 5 ⟹ S7 = S6 + S5 ⟹ S7 = 18 + 11 = 29 ⟹ S7 = ( + ) + ( − ) = 29.
2 2 2 2
8 8
1 √5 1 √5
𝑘 = 6 ⟹ S8 = S7 + S6 ⟹ S8 = 29 + 18 = 47 ⟹ S8 = ( + ) + ( − ) = 47.
2 2 2 2
9 9
1 √5 1 √5
𝑘 = 7 ⟹ S9 = S8 + S7 ⟹ S9 = 47 + 29 = 76 ⟹ S9 = ( + ) + ( − ) = 76.
2 2 2 2
𝟏𝟎 𝟏𝟎
𝟏 √𝟓 𝟏 √𝟓
𝒌 = 𝟖 ⟹ 𝐒𝟏𝟎 = 𝐒𝟗 + 𝐒𝟖 ⟹ 𝐒𝟏𝟎 = 𝟕𝟔 + 𝟒𝟕 = 𝟏𝟐𝟑 ⟹ 𝐒𝟏𝟎 =( + ) +( − )
𝟐 𝟐 𝟐 𝟐
= 𝟏𝟐𝟑.
𝑑) O polinômio de grau mínimo que contém as raízes (5 + 2√3) e (5 − 2√3) é tal que:
𝑥² − (5 + 2√3 + (5 − 2√3)) . 𝑥 + ((5 + 2√3)(5 − 2√3)) = 0
⟹ 𝒙𝟐 − 𝟏𝟎. 𝒙 + 𝟏𝟑 = 𝟎.
Pelo Teorema das Somas de Newton:
𝒙𝟐 − 𝟏𝟎. 𝒙 + 𝟏𝟑 = 𝟎 ⟹ 𝐒𝒌+𝟐 − 𝟏𝟎. 𝐒𝒌+𝟏 + 𝟏𝟑. 𝐒𝒌 = 𝟎
⟹ 𝐒𝒌+𝟐 = 𝟏𝟎. 𝐒𝒌+𝟏 − 𝟏𝟑. 𝐒𝒌 .
Primeiramente:
S0 = 2.
S1 = 5 + 2√3 + (5 − 2√3) = 10.
Enfim:
S2 = 10. S1 − 13. S0 ⟹ S2 = 10.10 − 13.2 = 100 − 26 = 74.
S3 = 10. S2 − 13. S1 ⟹ S3 = 10.74 − 13.10 = 610.
S4 = 10. S3 − 13. S2 ⟹ S3 = 10.610 − 13.74 = 5138.
S5 = 10. S4 − 13. S3 ⟹ S5 = 10.5138 − 13.610 = 43450.
𝐒𝟔 = 𝟏𝟎. 𝐒𝟓 − 𝟏𝟑. 𝐒𝟒 ⟹ 𝐒𝟔 = 𝟏𝟎. 𝟒𝟑𝟒𝟓𝟎 − 𝟏𝟑. 𝟓𝟏𝟑𝟖 = 𝟑𝟔𝟕𝟕𝟎𝟔.
47. Demonstração.
(Verifique o exposto para 𝑘 = 1, 𝑘 = 2 e 𝑘 = 3.)
48.
Uma vez que S𝑘 = 𝑧1𝑘 + 𝑧2𝑘 + 𝑧3𝑘 e dado que
3 3 3

𝜓(𝑧) = ∑ 𝑧𝑘 + ∑ 𝑧𝑘2 + ∑ 𝑧𝑘3


𝑘=1 𝑘=1 𝑘=1
= (𝑧1 + 𝑧2 + 𝑧3 ) + (𝑧1 + 𝑧2 + 𝑧3 ) + (𝑧13 + 𝑧23
2 2 2
+ 𝑧33 ),
temos, aplicando o Teorema das Somas de Newton:
𝑧³ − 6. 𝑧 + 5 = 0 ⟹ S𝑘+3 + 0. S𝑘+2 − 6. S𝑘+1 + 5. S𝑘 = 0
⟹ 𝐒𝒌+𝟑 = 𝟔. 𝐒𝒌+𝟏 − 𝟓. 𝐒𝒌 .
Lembrando que:
S0 = 𝑧10 + 𝑧20 + 𝑧30 = 3.
S1 = 𝑧1 + 𝑧2 + 𝑧3 = 0 (Relações de Girard).
S2 = 𝑧1 + 𝑧2 + 𝑧32 = (𝑧1 + 𝑧2 + 𝑧3 )2 − 2(𝑧1 𝑧2 + 𝑧2 𝑧3 + 𝑧1 𝑧3 ) = 02 − 2(− 6)
2 2

= 0 + 12 = 12.
𝑘 = 0 ⟹ S0+3 = 6. S0+1 − 5. S0 ⟹ S3 = 6. S1 − 5. S0 ⟹ S3 = 6.0 − 5.3 = − 15.
Desta forma, a expressão 𝜓(𝑧) é tal que:
3 3 3

𝜓(𝑧) = ∑ 𝑧𝑘 + ∑ 𝑧𝑘2 + ∑ 𝑧𝑘3 = (𝑧1 + 𝑧2 + 𝑧3 ) + (𝑧12 + 𝑧22 + 𝑧32 ) + (𝑧13 + 𝑧23 + 𝑧33 )
𝑘=1 𝑘=1 𝑘=1
= 0 + 12 + (− 15) = − 𝟑.
49. 𝑎) Partindo do quadrado da soma de 𝑎 e 𝑏:
(𝑎 + 𝑏)2 = 𝑎² + 2𝑎𝑏 + 𝑏².
Para obtermos, no segundo membro da igualdade, o fator 𝑎² − 𝑏², devemos adicionar − 2𝑏²
em ambos os lados desta:
⟹ (𝑎 + 𝑏)2 = 𝑎² + 2𝑎𝑏 + 𝑏² ⟹ (𝑎 + 𝑏)2 − 2𝑏 2 = 𝑎2 + 2𝑎𝑏 + 𝑏 2 − 2𝑏²
⟹ (𝑎 + 𝑏)2 − 2𝑏 2 = 𝒂𝟐 + 2𝑎𝑏 − 𝒃𝟐 ⟹ (𝑎 + 𝑏)2 − 2𝑏 2 − 2𝑎𝑏 = 𝒂𝟐 − 𝒃²
⟹ (𝑎 + 𝑏)2 − 2𝑏(𝑎 + 𝑏) = 𝑎2 − 𝑏 2 ⟹ (𝑎 + 𝑏)((𝑎 + 𝑏) − 2𝑏) = 𝑎2 − 𝑏²
⟹ (𝑎 + 𝑏)(𝑎 − 𝑏) = 𝑎2 − 𝑏 2 ⟹ 𝒂𝟐 − 𝒃𝟐 = (𝒂 + 𝒃)(𝒂 − 𝒃).
995 995
1000² − (𝑘 + 1)² (1000 + (𝑘 + 1)). (1000 − (𝑘 + 1))
𝑏) ∏ =∏
1000² − 𝑘² (1000 + 𝑘)(1000 − 𝑘)
𝑘=1 𝑘=1
995 995 995
(1001 + 𝑘)(999 − 𝑘) 1001 + 𝑘 999 − 𝑘
=∏ = (∏ ) . (∏ )
(1000 + 𝑘)(1000 − 𝑘) 1000 + 𝑘 1000 − 𝑘
𝑘=1 𝑘=1 𝑘=1
995 995
𝑓(𝑘 + 1) 𝑔(𝑘 + 1)
= (∏ ) . (∏ ).
𝑓(𝑘) 𝑔(𝑘 + 1)
𝑘=1 𝑘=1
* 𝑓(𝑘) = 1000 + 𝑘 e 𝑔(𝑘) = 1000 − 𝑘.
995 995
𝑓(𝑘 + 1) 𝑔(𝑘 + 1) 𝑓(995 + 1) 𝑔(995 + 1)
⟹ (∏ ) . (∏ )=( ).( )
𝑓(𝑘) 𝑔(𝑘 + 1) 𝑓(1) 𝑔(1)
𝑘=1 𝑘=1
𝑓(996) 𝑔(996) 1996 4 𝟕𝟗𝟖𝟒 𝑎
=( ).( )=( ).( )= = .
𝑓(1) 𝑔(1) 1001 999 𝟗𝟗𝟗𝟗𝟗𝟗 𝑏
𝑎 + 𝑏 = 999999 + 7984 = 𝟏𝟎𝟎𝟕𝟗𝟖𝟑.
50. 𝑎) Partindo do cubo da diferença de 𝑎 e 𝑏:
⟹ (𝑎 − 𝑏)3 = 𝑎3 − 𝑏 3 − 3𝑎𝑏(𝑎 − 𝑏) ⟹ (𝑎 − 𝑏)3 + 3𝑎𝑏(𝑎 − 𝑏) = 𝑎3 − 𝑏³
⟹ (𝑎 − 𝑏)((𝑎 − 𝑏)2 + 3𝑎𝑏) = 𝑎3 − 𝑏 3
⟹ (𝑎 − 𝑏)(𝑎2 + 𝑏 2 − 2𝑎𝑏 + 3𝑎𝑏) = 𝑎3 − 𝑏³
⟹ 𝒂𝟑 − 𝒃𝟑 = (𝒂 − 𝒃)(𝒂𝟐 + 𝒃𝟐 + 𝒂𝒃).
𝑏) Demonstração.
51.
A função 𝑓 em questão é tal que:
∑3𝑘=0 𝑥 𝑘 1 + 𝑥 + 𝑥² + 𝑥³ (𝑥 + 1) + 𝑥²(𝑥 + 1)
𝑓(𝑥) = √ 2 = √ = √
∑𝑘=1 𝑥 𝑘 𝑥 + 𝑥² 𝑥(𝑥 + 1)

(𝑥 + 1)(1 + 𝑥 2 ) 1 + 𝑥²
=√ =√ .
𝑥(𝑥 + 1) 𝑥
Deste modo, as condições de existência da função 𝑓 são:
1. 𝑥 ≠ 0.
2.
1 + 𝑥2
≥ 0 ⟹ 𝑥 > 0.
𝑥
De 1 e 2, temos:
1 ∩ 2 ⟹ 𝑥 > 0.
Deste modo:
𝐃𝒇 = {𝒙 ∈ ℝ: 𝒙 > 𝟎} ou 𝐃𝒇 = (𝟎, ∞).
* Suporemos que você, nobre leitor, esteja familiarizado com os conceitos e
definições das funções (condições de existência, domínio, etc), pois o regaste
destes não consta em nossa pauta teórica.

52.
A desigualdade em questão é tal que:
7 7

𝑥 4 + 𝑥³ + ∑ 𝑥 𝑘 − ∑ 𝑥 𝑘 < 0
𝑘=1 𝑘=3
⟹ 𝑥 + 𝑥 + (𝒙 + 𝒙 + 𝒙 + 𝒙 + 𝒙 + 𝒙 + 𝒙𝟕 ) − (𝒙𝟑 + 𝒙𝟒 + 𝒙𝟓 + 𝒙𝟔 + 𝒙𝟕 ) < 0
4 3 𝟐 𝟑 𝟒 𝟓 𝟔

⟹ 𝑥4 + 𝑥3 + 𝑥 + 𝑥2 < 0 ⟹ 𝑥4 + 𝑥3 + 𝑥2 + 𝑥 < 0
⟹ 𝑥(𝑥 3 + 𝑥 2 + 𝑥 + 1) < 0 ⟹ 𝑥(𝑥 2 (𝑥 + 1) + (𝑥 + 1)) < 0

⟹ 𝑥((𝑥 + 1)(𝑥 2 + 1)) < 0 ⟹ 𝒙(𝒙 + 𝟏)(𝒙𝟐 + 𝟏) < 𝟎.


Resolvendo esta inequação pelo método do Quadro de Sinais, obtemos:
⟹ 𝑥(𝑥 + 1)(𝑥 2 + 1) < 0 ⟹ 𝑥 ∈ (− 1, 0) ou – 1 < 𝑥 < 0.
Como o intervalo é da forma (𝑎, 𝑏), o valor de 𝑏 − 𝑎 é 0 − (− 1) = 𝟏.
53.
A chave deste exercício é o emprego correto das propriedades e definições dos
Números Complexos, principalmente a Primeira Fórmula de Moivre.
Dito isto, temos que 𝑧 = cos(𝜃) + 𝑖. sen(𝜃) = cis(𝜃).
Uma vez que a parte imaginária de uma soma de complexos é a soma das partes
imaginárias destes complexos, temos:
15 15

∑ Im(𝑧 2𝑟−1 ) = Im (∑ 𝑧 2𝑟−1 ) = Im(𝑧 + 𝑧 3 + 𝑧 5 + ⋯ + 𝑧 29 )


𝑟=1 𝑟=1
30
𝑧(𝑧 − 1) 𝒛𝟑𝟏 − 𝒛
= Im ( 2 ) = 𝐈𝐦 ( ).
𝑧 −1 𝒛² − 𝟏
Deste modo, para 𝜃 = 2°, temos 𝑧 = cis(2°):
31
𝑧 31 − 𝑧 (cis(2°)) − cis(2°)
⟹ Im ( 2 ) = Im ( 2 ).
𝑧 −1 (cis(2°)) − 1
𝒏
Mas, da Primeira Fórmula de Moivre: (𝐜𝐢𝐬(𝜽)) = 𝐜𝐢𝐬(𝒏. 𝜽).
31
(cis(2°)) − cis(2°) cis(62°) − cis(2°)
⟹ Im ( 2 ) = Im ( ).
(cis(2°)) − 1 cis(4°) − 1
Outra propriedade dos números complexos nos informa que:
𝐜𝐢𝐬(𝜶 + 𝜷) = 𝐜𝐢𝐬(𝜶). 𝐜𝐢𝐬(𝜷).

cis(62°) − cis(2°) cis(60°). cis(2°) − cis(2°)


⟹ Im ( ) = Im ( )
cis(4°) − 1 cis(2°)
cis(2°). cis(2°) −
cis(2°)

cis(2°)(cis(60°) − 1) cis(60°) − 1
= Im ( ) = Im ( ).
1 1
cis(2°) (cis(2°) − ) cis(2°) −
cis(2°) cis(2°)
1
Mas é o complexo conjugado de cis(2°):
cis(2°)
1
⟹ cis(2°) − = 2𝑖sen(2°).
cis(2°)

cis(60°) − 1 cis(60°) − 1
⟹ Im ( ) = Im ( )
1 2𝑖sen(2°)
cis(2°) −
cis(2°)
cos(60°) + 𝑖. sen(60°) − 1 cos(60°) − 1 sen(60°)
= Im ( ) = Im ( + )
2𝑖sen(2°) 2𝑖sen(2°) 2sen(2°)
1 1
− 𝑖. + 𝑖 sen(60°) 1−
= Im ( 2 + ) = Im (𝑖 2 + sen(60°))
2sen(2°) 2sen(2°) 2sen(2°) 2sen(2°)

𝟏 𝐬𝐞𝐧(𝟔𝟎°) 𝟏
= 𝐈𝐦 (𝒊 + )= .
𝟒𝐬𝐞𝐧(𝟐°) 𝟐𝐬𝐞𝐧(𝟐°) 𝟒𝐬𝐞𝐧(𝟐°)
54.
Do lado esquerdo, seja S = 1 + 2𝑥 + 3𝑥² + 4𝑥³ + ⋯ .
Multiplicando a soma S por 𝑥:
⟹ { 𝐈: S = 1 + 2𝑥 + 3𝑥² + 4𝑥³ + 4

𝐈𝐈: S. 𝑥 = 𝑥 + 2𝑥² + 3𝑥³ + 4𝑥 + ⋯
Fazendo I – II:
𝐈 − 𝐈𝐈 ⟹ S − S. 𝑥 = 1 + 𝑥 + 𝑥 2 + 𝑥 3 + ⋯ ⟸ Soma dos infinitos termos de
uma P.G.
1
S − S. 𝑥 = 1 + 𝑥 + 𝑥 2 + 𝑥 3 + ⋯ ⟹ S(1 − 𝑥) =
1−𝑥
1
⟹S= .
(1 − 𝑥)2
Para o lado direito:
1 3 5 7 1 3 5 7 3 5 7
+ + + + ⋯ = S2 ⟹ 2. S2 = 2 ( + + + +⋯) = 1+ + + +⋯
2 4 8 16 2 4 8 16 2 4 8
3 5 7 1 3 5 7
2. S2 − S2 = (1 + + + + ⋯ ) − ( + + + + ⋯)
2 4 8 2 4 8 16
3 1 5 3 7 5
=1+( − )+( − )+( − )+⋯
2 2 4 4 8 8
2 2 2
= 1 + ( + + + ⋯)
2 4 8
1
1 1 1
= 1 + 2 ( + + + ⋯ ) = 1 + 2 ( 2 ) = 1 + 2.1 = 3.
2 4 8 1
1−
2
Deste modo, temos a seguinte equação:
1 2
1 √3
= 3 ⟹ (1 − 𝑥) = ⟹ 1 − 𝑥 = ±
(1 − 𝑥)2 3 3
√3 √3
⟹1−𝑥 = ou 1 − 𝑥 = − .
3 3
√3 √3
𝑥=1 − ou 1 + .
3 3
Deste modo:
√𝟑
S = {𝟏 ± } .
𝟑
Gab. C.
55.
Façamos, por pura facilitação algébrica, a seguinte manipulação:
𝜋
= 𝜃, 𝑛 ∈ ℕ > 1.
𝑛
Assim:
2𝜋 4𝜋 6𝜋 2(𝑛 − 1)𝜋
⟹ sen ( ) + sen ( ) + sen ( ) + ⋯ + sen ( )
𝑛 𝑛 𝑛 𝑛
= sen(2𝜃) + sen(4𝜃) + sen(6𝜃) + ⋯ + sen(2(𝑛 − 1)𝜃).
Antes de prosseguirmos com a resolução, façamos as seguintes considerações:
Estudamos, na teoria acerca dos números complexos, a primeira fórmula de
𝑛
Moivre: cos(𝑛. 𝜃) + 𝑖. sen(𝑛. 𝜃) = (cos(𝜃) + 𝑖. sen(𝜃)) .
Vimos, também, que a parte imaginária de um complexo na forma
trigonométrica corresponde ao seno do argumento deste complexo:
Im[cos(𝜃) + 𝑖. sen(𝜃)] = sen(𝜃).
E, de maneira análoga, vimos que a soma das partes imaginárias de vários
complexos corresponde a parte imaginária desta soma (complexo resultante):
Dados os complexos 𝑧𝑖 ∈ ℂ da forma 𝑧𝑖 = Re(𝑧𝑖 ) + 𝑖. Im(𝑧𝑖 ),
com 𝑖 = [1, 2, ..., 𝑛], verifica-se que:
Im(𝑧1 ) + Im(𝑧2 ) + ⋯ + Im(𝑧𝑛 ) = Im(𝑧1 + 𝑧2 + ⋯ + 𝑧𝑛 )
𝑛 𝑛

⟹ ∑ Im(𝑧𝑖 ) = Im (∑ 𝑧𝑖 ) .
𝑖=1 𝑖=1
Perceba que a soma em questão é
𝜋
sen(2𝜃) + sen(4𝜃) + sen(6𝜃) + ⋯ + sen(2(𝑛 − 1)𝜃), onde = 𝜃.
𝑛
𝑖.2𝜃
Desta forma, seja 𝑧 = cos(2𝜃) + 𝑖. sen(2𝜃) = cis(2𝜃) = 𝑒 .
Nestes casos, é preferível o uso da Fórmula de Euler (cis(𝜃) = 𝑒 𝑖.𝜃 ) uma vez
que as operações com exponenciais são deveras simples.
Assim, sejam S1 e S2 tais que:
S1 = sen(2𝜃) + sen(4𝜃) + sen(6𝜃) + ⋯ + sen(2(𝑛 − 1)𝜃);
S2 = 𝑧 + 𝑧² + 𝑧³ + ⋯ + 𝑧 𝑛−1
3 𝑛−1
= cis(2𝜃) + (cis(2𝜃))² + (cis(2𝜃)) + ⋯ + (cis(2𝜃))
= cis(2𝜃) + cis(4𝜃) + cis(6𝜃) + ⋯ + cis(2(𝑛 − 1)𝜃).
= (cos(2𝜃) + 𝑖. sen(2𝜃)) + (cos(4𝜃) + 𝑖. sen(4𝜃)) + (cos(6𝜃) + 𝑖. sen(6𝜃))
+ ⋯ + (cos(2(𝑛 − 1)𝜃) + 𝑖. sen(2(𝑛 − 1)𝜃))
= (cos(2𝜃) + cos(4𝜃) + cos(6𝜃) + ⋯ + cos(2(𝑛 − 1)𝜃))
+ 𝑖(sen(2𝜃) + sen(4𝜃) + sen(6𝜃) + ⋯ + sen(2(𝑛 − 1)𝜃)).
⟹ S2 = (cos(2𝜃) + cos(4𝜃) + cos(6𝜃) + ⋯ + cos(2(𝑛 − 1)𝜃))
+𝑖. S1 .
Veja que se cumpre 𝐒𝟏 = 𝐈𝐦(𝐒𝟐 ).
Desta forma, basta prosseguirmos com o cálculo de S2 e, ao final das operações,
considerarmos apenas sua parte imaginária:
𝑛−1
𝑧(𝑧 𝑛−1 − 1)
S2 = 𝑧 + 𝑧² + 𝑧³ + ⋯ + 𝑧 =
𝑧−1
𝑛
𝑧 −𝑧
= .
𝑧−1
Mas:
𝜋 2𝜋 2𝜋
𝜃 = ⟹ 𝑧 = cis(2𝜃) = cis ( ) = 𝑒 𝑖 𝑛 .
𝑛 𝑛
2𝜋
Uma vez que 𝑧 = 𝑒 𝑖 𝑛 , decorre que:
2𝜋
𝑛
𝑧 = 𝑒 𝑖 𝑛 = √𝑒 𝑖.2𝜋 ⟹ 𝒛𝒏 = 𝒆𝒊.𝟐𝝅 = 𝐜𝐢𝐬(𝟐𝝅) = 𝟏 ⟹ 𝒛𝒏 = 𝟏.
Desta forma:
𝑛
𝑧 −𝑧 1−𝑧 1−𝑧
= =− = − 1.
𝑧−1 𝑧−1 𝑧−1
Disto, segue que:
S2 = − 1 ⟹ S2 = − 1 + 0. 𝑖 ⟹ S1 = Im(S2 )
⟹ 𝐒𝟏 = 𝐈𝐦(− 𝟏 + 𝟎. 𝒊) = 𝟎.
Enfim, a soma pedida é tal que:
2𝜋 4𝜋 6𝜋 2(𝑛 − 1)𝜋
sen ( ) + sen ( ) + sen ( ) + ⋯ + sen ( ) = 𝟎.
𝑛 𝑛 𝑛 𝑛
56.
𝜋
Analogamente ao exercício 55, seja 𝑧 = cis ( ):
𝑛
𝜋 2𝜋 3𝜋 𝑛𝜋
S1 = sen ( ) + sen ( ) + sen ( ) + ⋯ + sen ( );
𝑛 𝑛 𝑛 𝑛
S2 = 𝑧 + 𝑧² + 𝑧³ + ⋯ + 𝑧 𝑛 .
⟹ 𝐒𝟏 = 𝐈𝐦(𝐒𝟐 ).
𝑧(𝑧 𝑛 − 1)
S2 = .
𝑧−1
Mas:
𝜋 𝜋 𝑛
𝑧 = cis ( ) = 𝑒 𝑖𝑛 = √𝑒 𝑖𝜋
𝑛
𝑛
⟹ 𝑧 = √𝑒 𝑖𝜋 ⟹ 𝒛𝒏 = 𝒆𝒊.𝝅 = 𝐜𝐢𝐬(𝝅) = − 𝟏 ⟹ 𝒛𝒏 = − 𝟏.
Assim:
𝑛
𝑧(𝑧 − 1) 𝑧(− 1 − 1) 2𝑧 2𝑧
S2 = ⟹ S2 = =− =
𝑧−1 𝑧−1 𝑧−1 1−𝑧
𝜋
2cis ( )
= 𝑛 .
𝜋
1 − cis (𝑛 )
Veja que:
𝜋 𝜋 𝜋 𝜋 𝜋
1 − cis ( ) = 1 − (cos ( ) + 𝑖. sen ( )) = (1 − cos ( )) − 𝑖. sen ( ) .
𝑛 𝑛 𝑛 𝑛 𝑛
Das relações do arco duplo (ver apêndice), temos que:
𝜋 𝛼 𝛼
𝛼 = ⟹ cos(𝛼) = 1 − 2sen2 ( ) ⟹ cos(𝛼) − 1 = − 2sen2 ( )
𝑛 2 2
𝛼
⟹ 1 − cos(𝛼) = 2sen2 ( )
2
𝜋 𝜋
⟹ 1 − cos ( ) = 2sen2 ( ) .
𝑛 2𝑛
Assim:
𝜋 𝜋 𝜋
1 − cis ( ) = (1 − cos ( )) − 𝑖. sen ( )
𝑛 𝑛 𝑛
𝜋 𝜋
= 2sen2 ( ) − 𝑖. sen ( ) .
2𝑛 𝑛
Novamente do arco duplo:
𝜋 𝜋 𝜋
sen ( ) = 2sen ( ) cos ( ) :
𝑛 2𝑛 2𝑛
𝜋 𝜋 𝜋 𝜋
1 − cis ( ) = 2sen2 ( ) − 𝑖. 2sen ( ) cos ( )
𝑛 2𝑛 2𝑛 2𝑛
𝜋 𝜋 𝜋 𝜋 𝜋
= 2sen ( ) (sen ( ) − 𝑖. cos ( )) = − 2. 𝑖. sen ( ) cis ( ) .
2𝑛 2𝑛 2𝑛 2𝑛 2𝑛
Enfim:
𝜋 𝜋
2cis ( ) 2cis ( ) 𝜋
S2 = 𝑛 = 𝑛 = − 1 + 𝑖. cot ( ).
𝜋 𝜋 𝜋 2𝑛
1 − cis (𝑛 ) − 2. 𝑖. sen (2𝑛) cis (2𝑛)
(Deixaremos o trabalho algébrico omitido acima para o leitor).
𝝅 𝝅
⟹ 𝐒𝟏 = 𝐈𝐦(𝐒𝟐 ) ⟹ 𝐒𝟏 = 𝐈𝐦 ( 𝟏 + 𝒊. 𝐜𝐨𝐭 ( )) = 𝐜𝐨𝐭 ( ) .
𝟐𝒏 𝟐𝒏
57.
𝑎) Sejam S1 = cos(𝑥) + cos(2𝑥) + cos(3𝑥) + ⋯ + cos(𝑛𝑥) e
S2 = cis(𝑥) + cis(2𝑥) + cis(3𝑥) + ⋯ + cis(𝑛𝑥)
2 3 𝑛
= cis(𝑥) + (cis(𝑥)) + (cis(𝑥)) + ⋯ + (cis(𝑥)) .
⟹ S1 = Re(S2 ).
Tomando 𝑧 = cis(𝑥), temos:
2 3 𝑛
𝑧(𝑧 𝑛 − 1) 𝑧 𝑛+1 − 𝑧
S2 = 𝑧 + 𝑧 + 𝑧 + ⋯ + 𝑧 = =
𝑧−1 𝑧−1
𝑛+1
(cis(𝑥)) − cis(𝑥) cis((𝑛 + 1)𝑥) − cis(𝑥)
= =
cis(𝑥) − 1 cis(𝑥) − 1
cis(𝑛𝑥 + 𝑥) − cis(𝑥) cis(𝑛𝑥). cis(𝑥) − cis(𝑥)
= =
cis(𝑥) − 1 cis(𝑥) − 1
cis(𝑥)(cis(𝑛𝑥) − 1) cis(𝑛𝑥) − 1 cis(𝑛𝑥) − 1
= = =
1 1 1 − cis(− 𝑥)
cis(𝑥) (1 − ) 1−
cis(𝑥) cis(𝑥)
cos(𝑛𝑥) + 𝑖. sen(𝑛𝑥) − 1 cos(𝑛𝑥) − 1 + 𝑖. sen(𝑛𝑥)
= =
1 − (cos(𝑥) − 𝑖. sen(𝑥)) 1 − cos(𝑥) + 𝑖. sen(𝑥)
𝑛𝑥 𝑛𝑥 𝑛𝑥
− 2sen2 ( ) + 2. 𝑖. sen ( ) cos ( )
= 2 2 2
𝑥 𝑥 𝑥
2sen2 (2) + 2. 𝑖. sen (2) cos (2)
𝑛𝑥 𝑛𝑥 𝑛𝑥
2. 𝑖. sen ( ) (cos ( ) + 𝑖. sen ( )) sen (𝑛𝑥 ) cis (𝑛𝑥 )
2 2 2 2 2
= = 𝑥 𝑥
𝑥 𝑥 𝑥 sen (2) cis (− 2)
2. 𝑖. sen ( ) (cos ( ) − 𝑖. sen ( ))
2 2 2
𝑛𝑥 𝑛𝑥 𝑥
sen ( ) cis ( − (− )) sen (𝑛𝑥 ) cis (𝑛𝑥 + 𝑥 )
2 2 2 2 2
= 𝑥 = 𝑥
sen (2) sen (2)
𝑛𝑥 𝑥(𝑛 + 1) 𝑥(𝑛 + 1)
sen ( ) (cos ( ) + 𝑖. sen ( ))
2 2 2
= 𝑥
sen (2)
𝑛𝑥 𝑥(𝑛 + 1) 𝑛𝑥 𝑥(𝑛 + 1)
sen ( ) cos ( ) sen ( ) sen ( )
2 2 2 2
= 𝑥 + 𝑖. 𝑥 .
sen (2) sen (2)
Enfim:
𝑛𝑥 𝑥(𝑛 + 1) 𝑥 𝑛𝑥 𝑥(𝑛 + 1) 𝑥
S2 = sen ( ) cos ( ) csc ( ) + 𝑖. sen ( ) sen ( ) csc ( ) .
2 2 2 2 2 2
𝒏𝒙 𝒙(𝒏 + 𝟏) 𝒙
⟹ 𝐒𝟏 = 𝐑𝐞(𝐒𝟐 ) = 𝐬𝐞𝐧 ( ) 𝐜𝐨𝐬 ( ) 𝐜𝐬𝐜 ( ) .
𝟐 𝟐 𝟐
𝑏) Basta considerarmos a parte imaginária da soma S2 feita no item 𝑎:
sen(𝑥) + sen(2𝑥) + sen(3𝑥) + ⋯ + sen(𝑛𝑥)
𝒏𝒙 𝒙(𝒏 + 𝟏) 𝒙
= 𝐈𝐦(𝐒𝟐 ) = 𝐬𝐞𝐧 ( ) 𝐬𝐞𝐧 ( ) 𝐜𝐬𝐜 ( ) .
𝟐 𝟐 𝟐
58. Como os argumentos dos senos estão em progressão aritmética de razão 𝑟 e
primeiro termo 𝛼, basta repetirmos o mesmo procedimento do exercício
anterior, sejam:
S1 = cos(𝛼) + cos(𝛼 + 𝑟) + cos(𝛼 + 2𝑟) + ⋯ + cos(𝛼 + 𝑛𝑟)
e S2 = cis(𝛼) + cis(𝛼 + 𝑟) + cis(𝛼 + 2𝑟) + ⋯ + cis(𝛼 + 𝑛𝑟).
Assim, verifica-se que S1 = Re(S2 ), uma vez que:
S2 = cis(𝛼) + cis(𝛼 + 𝑟) + cis(𝛼 + 2𝑟) + ⋯ + cis(𝛼 + 𝑛𝑟)
= (cos(𝛼) + 𝑖. sen(𝛼)) + (cos(𝛼 + 𝑟) + 𝑖. sen(𝛼 + 𝑟)) + (cos(𝛼 + 2𝑟) + 𝑖. sen(𝛼 + 2𝑟))
+ ⋯ + (cos(𝛼 + 𝑛𝑟) + 𝑖. sen(𝛼 + 𝑛𝑟))
= 𝐜𝐨𝐬(𝜶) + 𝐜𝐨𝐬(𝜶 + 𝒓) + 𝐜𝐨𝐬(𝜶 + 𝟐𝒓) + ⋯ + 𝐜𝐨𝐬(𝜶 + 𝒏𝒓)
+ 𝑖(sen(𝛼) + sen(𝛼 + 𝑟) + sen(𝛼 + 2𝑟) + ⋯ + sen(𝛼 + 𝑛𝑟))
= 𝐒𝟏 + 𝑖(sen(𝛼) + sen(𝛼 + 𝑟) + sen(𝛼 + 2𝑟) + ⋯ + sen(𝛼 + 𝑛𝑟)).
De acordo com a seguinte propriedade dos complexos:
𝐜𝐢𝐬(𝜶 + 𝜷) = 𝒆𝒊(𝜶+𝜷) = 𝒆𝒊.𝜶 . 𝒆𝒊.𝜷 = 𝐜𝐢𝐬(𝜶). 𝐜𝐢𝐬(𝜷), temos que:
S2 = cis(𝛼) + cis(𝛼). cis(𝑟) + cis(𝛼). cis(2𝑟) + ⋯ + cis(𝛼). cis(𝑛𝑟)
= cis(𝛼)(1 + cis(𝑟) + cis(2𝑟) + ⋯ + cis(𝑛𝑟)).
Sendo 𝑧 = cis(𝑟), decorre, de acordo com a Primeira Fórmula de Moivre, que:
2 𝑛)
𝑧 𝑛+1 − 1
S2 = cis(𝛼)(1 + 𝑧 + 𝑧 + ⋯ + 𝑧 = cis(𝛼) [ ].
𝑧−1
cis((𝑛 + 1)𝑟) − 1
⟹ S2 = cis(𝛼) [ ]
cis(𝑟) − 1
cos((𝑛 + 1)𝑟) + 𝑖. sen((𝑛 + 1)𝑟) − 1
= cis(𝛼) [ ]
cos(𝑟) + 𝑖. sen(𝑟) − 1
cos((𝑛 + 1)𝑟) − 1 + 𝑖. sen((𝑛 + 1)𝑟)
= cis(𝛼) [ ]
cos(𝑟) − 1 + 𝑖. sen(𝑟)
(𝑛 + 1)𝑟 (𝑛 + 1)𝑟 (𝑛 + 1)𝑟
− 2sen2 ( ) + 2. 𝑖. sen ( ) cos ( )
2 2 2
= cis(𝛼) [ 𝑟 𝑟 𝑟 ]
2
− 2sen (2) + 2. 𝑖. sen (2) cos (2)

(𝑛 + 1)𝑟 (𝑛 + 1)𝑟 (𝑛 + 1)𝑟


2. 𝑖. sen ( ) (cos ( ) + 𝑖. sen ( ))
2 2 2
= cis(𝛼)
𝑟 𝑟 𝑟
2. 𝑖. sen ( ) (cos ( ) + 𝑖. sen ( ))
2 2 2
[ ]
(𝑛 + 1)𝑟 (𝑛 + 1)𝑟
sen ( ) cis ( )
2 2
= cis(𝛼) [ 𝑟 𝑟 ]
sen (2) cis (2)
(𝑛 + 1)𝑟 (𝑛 + 1)𝑟 𝑟
sen ( ) cis ( − )
2 2 2
= cis(𝛼) [ 𝑟 ]
sen (2)
(𝑛 + 1)𝑟 𝑟
sen ( ) cis ( ((𝑛 + 1) − 1))
2 2
= cis(𝛼) [ 𝑟 ]
sen (2)
(𝑛 + 1)𝑟 𝑛𝑟 (𝑛 + 1)𝑟 𝑛𝑟
sen ( ) cis ( ) sen ( ) . cis(𝛼). cis ( )
2 2 2 2
= cis(𝛼) [ 𝑟 ] = 𝑟
sen (2) sen (2)
(𝑛 + 1)𝑟 𝑛𝑟
sen ( ) cis (𝛼 + )
2 2
= 𝑟
sen (2)
(𝑛 + 1)𝑟
sen ( ) 𝑛𝑟 𝑛𝑟
2
= 𝑟 (cos (𝛼 + ) + 𝑖. sen (𝛼 + ))
sen (2) 2 2
(𝑛 + 1)𝑟 𝑛𝑟 (𝑛 + 1)𝑟 𝑛𝑟
sen ( ) cos (𝛼 + ) sen ( ) sen (𝛼 + )
2 2 2 2
= 𝑟 + 𝑖 𝑟 .
sen (2) sen (2)
Enfim:
(𝑛 + 1)𝑟 𝑛𝑟 𝑟 (𝑛 + 1)𝑟 𝑛𝑟 𝑟
S2 = sen ( ) cos (𝛼 + ) csc ( ) + 𝑖. sen ( ) sen (𝛼 + ) csc ( ) .
2 2 2 2 2 2
(𝒏 + 𝟏)𝒓 𝒏𝒓 𝒓
⟹ 𝐒𝟏 = 𝐑𝐞(𝐒𝟐 ) = 𝐬𝐞𝐧 ( ) 𝐜𝐨𝐬 (𝜶 + ) 𝐜𝐬𝐜 ( ) .
𝟐 𝟐 𝟐
𝑏) Basta considerarmos a parte imaginária da soma S2 feita no item 𝑎:
sen(𝛼) + sen(𝛼 + 𝑟) + sen(𝛼 + 2𝑟) + ⋯ + sen(𝛼 + 𝑛𝑟)
(𝒏 + 𝟏)𝒓 𝒏𝒓 𝒓
= 𝐈𝐦(𝐒𝟐 ) = 𝐬𝐞𝐧 ( ) 𝐬𝐞𝐧 (𝜶 + ) 𝐜𝐬𝐜 ( ) .
𝟐 𝟐 𝟐
59.
2016
𝜋 𝜋
Seja S1 = ∑ cos ( + 𝑘 ) e
𝑘=0 5 7
𝜋 𝜋 𝜋 𝜋 2𝜋 𝜋 𝑛𝜋
S2 = cis ( ) + cis ( + ) + cis ( + ) + ⋯ + cis ( 5 + )=
5 5 7 5 7 7
2016
𝜋 𝜋
∑ cis ( + 𝑘 ).
𝑘=0 5 7

Como nos exemplos anteriores, verifica-se que S1 = Re(S2 ).


É fácil perceber que os argumentos dos cossenos da soma S1 estão variando em
𝜋
progressão aritmética de razão 𝑟 = e 𝑛 = 2016.
7
𝜋 𝜋 𝜋 𝜋 2𝜋 𝜋 2016𝜋
Os termos , + , + , ..., + constituem uma P.A cujo primeiro
5 5 7 5 7 5 7
𝜋 𝜋
termo 𝛼 é , razão 𝑟 = e último termo .
5 7
Deste modo, verifica-se, novamente, que S1 = Re(S2 ):
2016 2016
𝜋 𝜋 𝜋 𝜋
∑ cos ( + 𝑘 ) = Re [ ∑ cis ( + 𝑘 )] .
5 7 5 7
𝑘=0 𝑘=0
E a fórmula obtida no exercício 58 (a) pode ser utilizada:
cos(𝛼) + cos(𝛼 + 𝑟) + cos(𝛼 + 2𝑟) + ⋯ + cos(𝛼 + 𝑛𝑟)
(𝑛 + 1)𝑟 𝑛𝑟 𝑟
= sen ( ) cos (𝛼 + ) csc ( ) .
2 2 2
2016
𝜋 𝜋 2016𝜋 𝜋 2015𝜋 𝜋
⟹ ∑ cos ( + 𝑘 ) = sen ( ) cos ( + ) csc ( )
5 7 14 5 14 14
𝑘=0
𝜋 1 1
= cos ( ) = + √5.
5 4 4
1 1
𝑎 = , 𝑏 = , 𝑐 = 5.
4 4
11
𝑎+𝑏+𝑐 = .
2
60. Demonstração análoga ao exercício 59.
Somatórios e Produtórios
Este documento não deve ser comercializado sob hipótese alguma.

Agradecimento:
Capa – Igor Barbosa.

Fontes e Referências bibliográficas:


GUIDORIZZI, Hamilton Luiz – Um curso de Cálculo – Vol. 1.
IEZZI, Gelson e MURAKAMI, Carlos – Fundamentos de Matemática
Elementar.
GUIMARÃES, Caio Santos – Matemática em nível IME/ITA,
Números Complexos e Polinômios.
APOSTOL, Tom Mike – One-Variable Calculus – Vol. 1.
LIDSKI, V.B – Problems in Elementary Mathematics.
Matemática Sercomtel – http://www.pessoal.sercomtel.com.br/matematica.
Canal Física & Matemática – https://www.youtube.com/user/RodrigoRenjic.

Potrebbero piacerti anche